You are on page 1of 761
SOLUTIONS TO THE PROBLEMS in TRANSPORT PHEOMENA Second Edition (2002) by R. Byron Bird Warren E. Stewart Edwin N. Lightfoot Department of Chemical Engineering University of Wisconsin Madison, Wisconsin 53706 USA This solutions manual has been prepared by the authors of the textbook for use by professors teaching courses in transport phenomena. It contains the solutions to 500 of the unsolved problems in the textbook. No part of this material may be reproduced in any form, electronic, mechanical, photocopying, recording, scanning, or otherwise. JOHN WILEY & SONS, Inc. New York, New York 1A.1 Estimation of dense-gas viscosity. a. Table E.1 gives T, = 126.2 K, p, = 33.5 atm, and pre = 180 x 10-® g/cm-s for Nz. The reduced conditions for the viscosity estimation are then: Pr = p/Pe = (1000 + 14.7)/33.5 x 14.7 = 2.06 T, =T/Te = (273.15 + (68 ~ 32)/1.8)/126.2 = 2.32 At this reduced state, Fig. 1.3-1 gives jt; = 1.15. Hence, the predicted viscosity is = pp/ pte = 1.15 180x10-* = 2.07x 10-4 g/em-s. This result is then converted into the requested units by use of Table F.3-4: = 2.07 x 10-4 x 6.7197 x 107? = 1.4 x 10 Ibm/ft-s 1A.2 Estimation of the viscosity of methyl fluoride. a. CHsF has M = 16.04—1.008+ 19.00 = 34.03 g/g-mole, T, = 4.55+273.15 277.70 K, pe = 58.0 atm, and V, = 34.03/0.300 = 113.4 m*/g-mole. The critical viscosity is then estimated as He = 61.6(34.03 x 277.70)'/?(113.4)-?/? = 255.6 micropoise from Eq. 1.3-la, and Me = 7.70(34.03)"/?(58.0)?/° (277.7) -1/* = 263.5 micropoise from Eq. 1.3-1b. ‘The reduced conditions for the viscosity estimate are T = (370 -+273.15)/277.70 = 2.32, pr = 120/58.0 = 2.07, and the predicted y, from Fig. 1.3-1 is 1.1. The resulting predicted viscosity is H = Mle =1.1 x 255.6 x 107° = 2.8 x 10-4 g/cm:s via Eq.1.3-1a, or 1.1 x 263.5 x 10-* = 2.9 x 10~4g/em-s via Eq.1.3-1b. 1A.3 Computation of the viscosities of gases at low density. Equation 1.4-14, with molecular parameters from Table E.1 and collision integrals from Table E.2, gives the following results: For Oz: M = 32.00, ¢ = 3.433A, ¢/x = 113 K. Then at 20°C, xT/e = 293.15/113 = 2.594 and ©, = 1.086. Equation 1.4-14 then gives = 2.6693 x 107°. (3-433) x 1.086 = 2.02 x 10~* g/cm:s = 2.02 x 10° Pas = 2.02 x 107? mPas. ‘The reported value in Table 1.1-3 is 2.04 x 10-? mPa-s. For No: M = 28.01, 0 = 3.667A, ¢/k = 99.8 K. Then at 20°C, xT/e = 293.15/99.8 = 2.937 and , = 1.0447. Equation 1.4-14 then gives = 2.6693 x 10-° (3.667? x 1.0447 = 1.72 x 10~ g/ems = 172x107 Pas = 172x107? mPas. ‘The reported value in Table 1.1-3 is 1.75 x 10-? mPas. For CHs, M = 16.04, o = 3.780A, ¢/x = 154 K. Then at 20°C, xP/e = 293.15/154 = 1.904 and 9, = 1.197. Equation 14-14 then gives /16.04 x 293.15 (8780)? x 1.197 = 1.07 x 1074 g/cms = 107 x 1075 Pas = 107 x 107? mPass. fe = 2.6693 x 1075 ‘The reported value in Table 1.1-3 is 1.09 x 10-? mPa-s. I-3 1A.4 Gas-mixture viscosities at low density. ‘The data for this problem are as follows: Component M. Ht, poise x 10° 1(H2) 2.016 88.4 2CChF2) 120.92 124.0 Insertion of these data into Eq. 1.4-16 gives the folowing coefficients for mixtures of Hz and Freon-12 at this temperature: Oy = O2 = 10 1 2. os an (& 4)? /120.92\"4]* Sy= (1+ 1+( sa va \* 120.92 ») 124.0, 2016 = 3.934 4 120. 22) ap 14 ( (282) 2.016 \"/*]* =F +m) 120.92, .0920 Equation 1.4-15 then gives the predicted mixture viscosities: m= n= Ye = A= B A+B= l-x SepGig Laeheg rryif Dy t2H2/ Dy pmix X 10° pobs,poise x 10° 0.00 3.934 1.000 0.0 124.0 (124.0) 124.0 0.25 3.200 0.773 6.9 1203 1272 128.1 0.50 2467 0.546 = 18.1 136 0 «1317 131.9 0.75 1734 0.319 38.2 97.2 135.4 135.1 1.00 1.000 0.092 88.4 0.0 (88.4) 88.4 1A.5 Viscosities of chlorine-air mixtures at low density. Equation 14-14 and Tables E.1, E.2 give the following viscosities at 75°F(= 273.15 + (75 — 32)/1.8 = 297.03 K) and 1 atm: For component 1, (Cle), My = 70.91, 01 = 4.115A, e1/K = 357K; hence, KT Je, = 297.03/357 = 0.832 and Qy,1 = 1.754, and #1 = 2.6693 x 107° = 1.304 x 1074 g/cm-s = 0.01304 ep. (4.115)? 1-754 For component 2, (air), Mz = 28.97, ¢ = 3.617A, e1/K« = 97.0K; hence, KT/e1 = 297.03/97.0 = 3.062 and Qq,1 = 1.033, and ia = 2.6603 x 19-8 VEIT X 207.08 = 1.832 x 10~* g/em-s = 0.01832 ep. (S617) x 1.033 Eq. 1.4-16 then gives the following coefficients for Eq. 1.4-15 at this temperature: Oy = Fy = 10 by = (14 OM =) als (ase) 4 728.97) V4]? V8 28.97. 7) +( 0. i833) 70.91 = 0.5339 on =e (14 mat) “8 1+ ( (ase 2 79.91\ 4]? aS VEN 70.91 a) 0.01304 38.97 = 1.8360 Equation 1.4-15 then gives the predicted mixture viscosities: a Me Ye As Bs A+B= 1-2 LepSig Lepdee cin/ Ly 22H2/ Ly Mmixp. x 10° 0.00 0.5339 1.000 0.0 0.01832 0.0183 0.25 0.6504 1.2090 0.005012 0.011365 0.0164 0.50 0.7670 1.4180 0.008501 0.006460 0.0150 0.75 0.8835 1.6270 0.011070 0.002815 0.0139 1.00 1.000 1.8360 0.01304 0.0 0.0130 1A.6 Estimation of liquid viscosity. a. The calculated values for Eq. 1.5-9 at 0°C and 100°C are as follows: TK 273.15 373.15 py g/em? 0.9998 0.9584 _ V=M/p, em*/g-mole 18.01 18.80 AGvray.1y, cal/g-mole= 897.5 x 18.016 x 252.16/453.59 8989. 8989, AU vap,74/ RT = 8989/1.98721/T 16.560 12.120 exp 0.40840 vap,75 /RT 859.6 140.5 Wh/V, g/ems 2.22x10-* 2.12 x 10-¢ Predicted liquid viscosity, g/em-s 0.19 0.0298 8. The predicted values for Eq. 1.5-11 at 0°C and 100°C are: _ 2K 273.15 373.15 Wh/V, gems 2.22% 10-* 2.12 x 10-4 exp(3.815/T) 179.7 44.70 Predicted liquid viscosity, g/em-s 0.0398 0.0095 Summary of results: ‘Temperature, °C 0 100 Observed viscosity, centipoise[=Jg/em-s100 1.787 0.2821 Prediction of Eq. 1.5-9 19. 2.98 Prediction of Eq. 1.5-11 3.98 0.95 Both equations give poor predictions. This is not surprising, since the empirical formulas in Eqs. 1.5-8 et seq. are inaccurate for water and for other associated liquids. 1A.7 Molecular velocity and mean free path. From eq. 1.4-1, the mean molecular velocity in O2 at 273.2 K is a, BRE _ [SxS x10 D732 _ A =Var=V azo = 4.25 x 10 cm/s From eq. 14-3, the mean free path in Oz at 1 atm and 273.2 K is RT 82.0578 x 273.2 ETT = 9.8 x 107% VardpN V2n(3 x 10-*)? x 1 x 6.02214 x 10° age em. Hence, the ratio of the mean free path to the molecular diameter is (9.3 x 10-4/3 x 10) = 3.1 x 10* under these conditions. At liquid states, on the other hand, the corresponding ratio would be on the order of unity or even less. 1B.1 Velocity profiles and stress components @. Ty = Ty = -Mb, and all other ty are zero. /2,v, = pb’y’, and all other pv,v, are zero. b. Ty = Tye =—2qb, and all other 1 are zero. 0,0, = pb?y, pd,Vy = PUyd, = pb*xy, po,d, = pb?x?, and all other po,v, are zero. c.All t) are zero 0,0, = pby’, pv,Vy = pv,v, =~pb?xy, poyv, = pbx? and all other po,0, are zero. d. Ty = Tyy = Ub, T., =~2ub, and all others are zero. the components of pvv may be given in the matrix: 0,0, = pbx” pv,vy=tpb’xy pv,v, = -}pb?xz pyv=| poyv,=tpb?xy poyvy=4pb y? pv,v, =—}pb? yz pv,0, =—}pb’xz pv,v,=-hpb*yz — pv,v, = pb?z? 1B.2 A fluid ina state of rigid rotation a. A particle within a rigid body rotating with an angular velocity vector w has a velocity given by v=[wxr]. If the angular velocity vector is in the +z-direction, then there are two nonzero velocity components given by v, =-w,y and v, = +w,x. Hence the magnitude of the angular velocity vector is b in Problem 1B.1(c). b, For the velocity components of Problem 1B.1(c), My Wy a, 0 and Sy % 95 ox” dy ox dy c. In Eq. 1.2-4, we selected only the linear symmetric combinations of derivatives of the velocity, so that in pure rotation there would be no viscous forces present. In (b) we see that the antisymmetric combination is nonzero in a purely rotational motion. 1B.3 Viscosity of suspensions Expanding the Mooney expression, we get (with € = ¢/¢,) eee) al) =14§g(l+ere? Nee era. =142 949? 2 alts o+9 (2 The first two terms match exactly with the first two terms in Eq. 1B.3-1. We can make the third term match exactly, by setting 25 25) 82 _ 7.17 whence 9, = 0.618 0 and the coefficient of ¢° becomes 225,25 1 1 got 1 = 20.26 48 40.618 20.382 If we try $y = 0.70, the coefficients of ¢? and $° become 6.70 and 17.6 respectively. This gives a somewhat better find of Vand's data. \-10 1C.1 Some consequences of the Maxwell-Boltzmann equation a. The mean speed is em hau mu? PRT ary ORF T eee dg &e a xT its [xT [peer Pee gy m [eee dé Vm Na Vim b. First rewrite Eq. 1C.1-4 as = oman, [MO Ay [om dy, [em ay, . fer du, . mr mi PRT gy The integral over u, in the numerator of the first factor is zero because the integrand is the product of a factor "u," (an odd function of the integration variable about 1, = 0) and an exponential function (an even function), and the range of integration extends equally far in the positive and negative directions. c. The mean kinetic energy per molecule is cuted ner ete Pas (pure mT gy? om (ee P de NR and is thus }xT for each degree of freedom. I- 1C.2 The wall collision frequency When we change to dimensionless variables in the second line of Eq. 1C.2-1, we get 2a smez) beat) (Geta) Pr (eee) moan) C3 PE 9) EE) \-12 1C.3 The pressure in an ideal gas. a. The dimensions of the quantities in Eq. 1C.3-1 are Using these units, one finds that the expression on the right of Eq. 1C.3-1 has units of M/Lt? (which are the same as the units of force per area). b. Combining Eqs. 1C.1-1 and 1C.3-1 we get 2” p= 2nn{ 5) pute ay aud, m_\P rm me 0 muy P2KT ee yom, =2an{ A) Cacermtl a, fem, erm, 2: 2 m( 222)" [ee Pag [mean [ied = 2nnf *2)(2)" (F)(o2)(va)= ner 1-13 1D.1 Uniform rotation of a fluid a, For the special case that w =3,1, we get = [wx r]= 2,226 45,00/% = (BE 2Y +8 .€ 05% —8,y+5,x) Then using Eqs. A.6-1, 2, 13 and 14, we can get the velocity components in cylindrical coordinates ((—8.y+8,x)-8,)=w(-ycos6 + xsin8) —rsin @cos 6 + xcos Osin 8) = 0 (-3.y +8,x)-89)= w((-y)(-sin @) + xcos 0) rsin sin 0 + rcos@cos0) = wr Therefore, the angular velocity of every point in the fluid is v4 /r = w, which is a constant, and there is no radial velocity. This is the way a rigid body rotates at constant angular velocity. b. The vector operations are (using the abbreviated notation of §A.9 and the Einstein summation convention) (VV) = 9,2; = 8; nn @n%n = indi Enmn@; £¥v} = {VLw xt], = Emu n = Ej On Sin = € jin =-{Vv},=-{Vv}} and from this last result we see that Vv +(Vv)* =0. c. The results above indicate that for a fluid is a state of pure rotation, the tensor + is identically zero. That is, there are no viscous stresses present in the fluid. This was the assertion made just before Eq. 1.2-4. I-14 1D.2 Force on a surface of arbitrary orientation. a. We can specify the surface area and the orientation of the surface of AOBC as ndS. To project this surface onto the yz-plane, we take the dot product with 8,, so that the area of AOBC is (n-3,)ds. b. The force per unit area on three triangles perpendicular to the three coordinate axes are Force on AOBC =, 7,, +8 yy +372 Force on AOCA =8 ty, +8 y Ty +3,Mye Force on AOAB=8, 71. +8 fxy +8,T.. c. Force balance on the volume OABC is then 14S =(8 7, +8 yy +8, ,)(n-8,)dS +(8, tye +8 yhy +8, |(n-By)dS + (8,5 +3yty +8,7,,)(n -8,)dS or 1-8,5 Ay] + [10-88 yy ]+[n-5,8, 7] +[n-8,8, 7, ]+[n-3,8,%y]+[n-8,8,7,.] +{n-8,8,7..]+[n-5,8,7.]+[n-3,5,7,.] =2E[P88))]=[-#] i-15 2A.1 Thickness of a falling film. a. The volume flow rate w/p per unit wall width W is obtained from Eq. 2.2-25: _w__ vRe _ (1.0037 x 107?)(10) pw 4 4 Here the kinematic viscosity v for liquid water at 20°C was obtained from Table 1.1- 2. Since 1 ft=12x2.54 cm, 1 hr =3600 s, and 1 gal=231.00 in? x (2.54em/in)*=3785.4 cm (see Appendix F), the result in the requested units is = 2.509 x 107? em?/s — 2 Z piv = 9.02509 cm? /s x 57 = 0.727 U.S. gal/he-tt gal/em? x 30.48 cm/ft x 3600 s/hr 4. The film thickness is calculated from Eqs. 2.2-25 and 2.2-22 as ow \NS 5-( geos B pW = (3 vRey'" ~ \geosB 4 2 1 = (Sara —500 x 10) = 0.009167 cm = 0.00361 in. 2A.2 Determination of capillary radius by flow measurement. Assuming the flow to be laminar, we solve Eq. 2.3-21 for the capillary radius: Rea o[ Sele _ «/Subw ~ \xpap ~ V aR Insertion of the data in mks units gives fn V (8.1416)(4.829 x 105) = 9/3186 x 10-8 7.51 x10 m .51 x 107? em ‘As a check on this result, we calculate the corresponding Reynolds number: Re = Dive _ Aw _ _2w # xDa =Rvp 2 (2.997 x 10-8) “(7.51 x 10-4)(4.03 x 10-5)(0.9552 x 103) 66.0 This value supports our assumption of laminar flow. Since the entrance length, Le = 0.035DRe = 0.35 cm is less than L, the entrance-effect correction to R is at most of the order of [[1 —(L-/L)]!/4—1], or 0.2 percent of R in the present example. Difficulties with this method include: (1) Inability to account for departures from a straight, circular cylindrical wall geometry. (2) Inability to account for in- advertent spatial and temporal variations of temperature, hence of the fluid density and viscosity. A simpler method is to measure the length L and mass m of a small slug of liquid mercury (or another liquid of known density) injected into the tube, and calculate the mean radius R of the slug as (m/[pxZ])"/?, on the assumption that the slug is a right circular cylinder. This method allows comparisons of mean R values for various intervals of the tube length. 22 2A.3 Volume rate of flow thrugh an annulus. Assuming the flow to be laminar, we use Eq. 2.4-17 to calculate the volume flow rate w/p, with the specifications = 0.495/1.1 = 0.45 4 = 136.8 (Ibm /ft-hr)(1hr/3600s) = 3.80 x 107? Ibm /ft-s (Po — Px) = (5.89 psi)(4.6330 x 10 poundals/ft?/psi) = 2.497 x 10! Ibm /ft- R=1lin.=11/12 ft Here Appendix F has been used for the conversions of units. With these specifica- tions, Eq. 2.4-17 gives w _ (x)(2.497 x 10*)(1.1/12)* ee [a - (045) - (= war] P_ (8)(8.80 x 10-2)(27) In(1/0.45) = (0.49242) [a — 0.04101) ~ (1 = 0.2025)? 1n(1/0.495) = (0.6748)[0.1625] = 0.110 ft/s As a check on our assumption of laminar flow, we calculate the Reynolds num- 2RQ ~ n)(ve)p Qw w Rul +) _ 2(0.110)(80.3) > BIA6)(.1/12)3.80 x 10-70-45) ber: =1110 This value is well within the laminar range, so our assumption of laminar flow is confirmed. 2A.4 Loss of catalyst particles in stack gas. a, Rearrangement of Eq. 2.6-17 gives the terminal velocity = D'(p, — p)g/18p in which D is the sphere diameter. Particles settling at ve greater than the centerline gas velocity will not go up the stack. Hence, the value of D that corresponds to v¢ = 1.0 ft/s will be the maximum diameter of particles that can be lost in the stack gas of the present system. Conversions of data to cgs units give vy = (1 ft/s)(12 x 2.54 em/ft) = 30.48 cm/s p = (0.045 Ibya/ft?)(453.59 g/lbm)((12 x 2.54)~* £3 fem*) =7.2x 10"! g/em? Baws (18)(0.00026)(30.48) mx Vp, —p)g i .2—7.2 x 10-4)(080.7), 1.21 x 10-4 = 1.1 x 107? em = 110 microns Hence, b, Equation 2.6-17 was derived for Re<< 1, but holds approximately up to Re=1. For the system at hand, Dup _ (1-1 x 10~?)(30.5)(7.2 x 10~ # (0.00026) Re 0.93 Hence, the result in «. is approximately correct. Methods are given in Chapter 6 for solving problems of this type without the creeping-flow assumption. 2-4 2B.1 Different choice of coordinates for the falling film problem Set up a momentum balance as before, and obtain the differential equation ary, “ae 7 PRCOSB Since no momentum is transferred at ¥=6, then at that plane T,=0. This boundary condition enables us to find that C, =-pg6cosf, and the momentum flux distribution is te, = —P6 cosp{ 1 - Note that the momentum flux is in the negative X-direction. Insertion of Newton's law of viscosity t;, = (dv, /dz) into the foregoing equation gives the differential equation for the velocity distribution: a -) This first-order differential equation can be integrated to give 0 (A(5-3)] The constant C, is zero, because v, = 0 at ¥=0. We note that ¥ and x are related by X/5 = 1-(x/5). When this is substituted into the velocity distribution above, we get eeeslaseel) which can be rearranged to give Eq. 2.2-18. 2-5 2B.2 Alternate procedure for solving flow problems Substituting Eq. 2.2-14 into Eq. 2.2-10 gives d(_dv,)_ #v, __ pgcosB 4 uw Bs) = pgcosp ec eR A Integrate twice with respect to x (see Eq. C.1-10) and get = PBB 2 Cree, 2u Then use the no-slip boundary condition that v, =0 at x= 6, and the zero momentum flux boundary condition that dv, /dx=0 at x=0. The second gives C,=0, and the first gives C, = (pgcosB/2)5*. Substitution of these constants into the general solution and rearranging then gives Eq. 2.2-18. 2B.3 Laminar flow in a narrow slit a. The momentum balance leads to (Po-P.) L x+C Substitution of Newton's law 1, =—1 & into the above gives do, __(P P,) __(Po- Pi)" | G dx uL H 2uL raaed Use of the no-slip boundary conditions at x= +B gives the expres- sions in Eq. 2B.3-1 and 2. One can also see that C, = 0 directly, since we know that the velocity distribution must be symmetrical about the plane x =0. b. The maximum velocity is at the middle of the slit and is The ratio of the average to the maximum velocity is then =(1-4)= Pe ax fr fedxdy ae (8) c. The mass rate of flow is w= p(2BW\v,) = p(2BW)(3) Po= PL) _ 2 (Po- Pi )oBW 2yL 3 uL d. In Eq. 2.5-22, set both viscosities equal to j1. set b equal to B, and multiply by BWp. 2B.4 Laminar slit flow with a moving wall ("plane Couette flow") Start with the velocity distribution from part (a) of Problem 28.3 (in terms of the integration constants). Determine C, and C, from the boundary conditions that v,=0 at x=-B, and v, =V at x= B. This leads to ere Car) This expression can be differentiated with respect to x and then Newton's law of viscosity t,, =-1(dv,/dx) can be used to get the expression for the stress tensor. Notice that the velocity distribution is no longer symmetric about the midplane, so that C, #0. 2B.5 Interrelation of slit and annulus formulas From Eq. 2.4-17 we get By (1-@-e? y age)“ ae (1-1+26-6)* =(1-1+4e-6¢? +469 —e*)+ 7€ (4e7-4e3 +64) eve tie tie. =(4e-6e? + 4e9 -e*)-(4e-6e? + Se? -Let+ =(4e-Ge? + 46° -e4)- This gives, finally, a result in agreement with Eq. 2B.5-1 _P)R! 2(Py-P,)R Paes Bets. Oe BL 2B.6 Flow of a film on the outside of a circular tube a. A momentum balance on the film gives d(rt,.) _ d(_dv, _ dr +pgr=0 or 1S (fis) + pgr=0 The latter may be integrated to give 2 =P Cyl tCy Next use the boundary conditions that at r=R, v, =0 (no slip) and that at r=aR, dv,/dr =0. When the integration constants have been found, we get for the velocity distribution 2 2 o, = PR -(2) +2a?In= 4u R R b. The mass rate of flow in the film is then w= fp" fp pv.rdrd0 = 2nR*pf v,Ed€ in which a dimensionless radial coordinate €=r/R has been introduced. Then PERS ca PR f(a e? +20? ne) Ede 2u = TER (4g get 20 de set nel) 2, ZoRS = ER (14 4a? ~3a + 4a Ina) 8u c. If we set a=1+€ (where € is small) and expand in powers of € using §C.2, we get 2-lo w= TPR (i669 -0(e*)) 22 8u Su This is in agreement with Eq. 2.2-21 if we make the identifications W =2aR and 5=eR (and furthermore consider only the case that cosB = 1, 2-11 2B.7 Annular flow with inner cylinder moving axially a. The momentum balance is the same as that in Eq. 2.3-11 or Eq. 2.4-2, but with the pressure-difference term omitted. We can substitute Newton's law of viscosity into this equation to get do, _C, ¢ 2, r wwe te whence 9, =—SLnr tC, or wea Ding +, That is, we select new integration constants, so that they are dimensionless. These integration constants are determined from the no-slip conditions at the cylindrical surfaces: v,(KR)=v) and v,(R)=0. The constants of integration are D, =0 and D, =~l/Inx. This leads then directly to the result given in the book. b. The mass rate of flow is : oR fn ga =2np eR (yg2 Ing-4E*)f = 2ap%®-(_4x?Ine—2(1- x2) w= [" [j.00.rdrd0 = 2np. which is equivalent to the answer in the text. c. The force ona length L of the rod Le2n| dv, F=fi"(+u which gives the expression in the book. d. When we replace x by 1-€ and expand in a Taylor series, we get = (1/xR) |g Rate = 21 1 Tealeaie.) (c+fe?+der tte’ F=2nL(-1)0 = 2ALM)% (4_ 1p aoe (1-}e-fe To get this last result one has to do a long division involving the polynomial in the next-to-last step. 2B.8 Analysis of a capillary flowmeter Designate the water by fluid "I" and the carbon tetrachloride by "II". Label the distance from B to C as "J". The mass rate of flow in the tube section "AB" is given by a(P,-P,)R%o, _ (Ps Pa) + Pit ]R*p, Bul BuL Since the fluid in the manometer is not moving, the pressures at D and E must be equal; hence Pa + Pigh + pig] + SH = Pp + P18) + PSH from which we get Pa-Ps + Pigh=(by —P,)gH Insertion of this into the first equation above gives the expression for the mass rate of flow in terms of the difference in the densities of the two fluids, the acceleration of gravity, and the height H. 213 2B.9 Low-density phenomena in compressible tube flow When we replace no-slip boundary condition of Eq. 2.3-17 by Eq, 2B.9-1, we get c, = (PoxPu)R® , (Po=PL)RE a 4uL 2uL so that the velocity distribution in the tube is (Po- Pi )R® [+2] =PL)RE 4uL R 2uL Next we write the expression for w, but consider only the flow through a length dz of the tube: od 1M. w= J" [*p(z)0.(r,2)rdrdo = aan'{ 2) fo.gas where we have introduced the ideal gas law, with R, being the gas constant (we use a subscript g here to distinguish the gas constant from the tube radius). We have also introduced a dimensionless radial coordinate. When we introduce the velocity distribution above, we get e2mt!( Moga) ha oe (#5 }el P a 5°) 2 eas a AR'( M\_, dP), Abo “Bu (#5) rfl sa) This is now integrated over the length of the tube, keeping mind that the mass flow rate w is constant over the entire length tnie=—28'( M) p(y Se) Le 2 Move Ske) 2-14 aR M _\(po-pi Ao, an (#5) Zt R (Po - PL) This gives » Airs (Me | mse. «te BuL RT) 2 R = (Po~PL)R*( PavgM(,, 40 8uL R,T RPsvg which leads then to Eq. 2B.9-2. 2-15 2B.10 Incompressible flow in a slightly tapered tube a. The radius at any downstream distance is R(z) = Ry +(Ry ~ Ry)(2/L) b. Changing the independent variable proceeds as follows: _AR'p a ar ae) _ aR'o( dP) R Fe) Bn VC aR)\dz) Bu aR OL c. First we rearrange the equation in (b) to get ae (2 a a R, Ry) R* Then we integrate this equation to get Pup _(8Hw)(__L pnd —fPap =| Se aR toa ( 7 a =x) Rt whence we can get the pressure difference in terms of the mass rate of flow SuwL)( Rj? - Re? 3mp — R, Next we solve to get the mass flow rate -( 3%(Po-P.)P\(_Ro-R,_)_( #(Po-Pi)RoP | 3_ 8uL RE-RS 8uL This is the result, with the first factor being the solution for a straight tube, the second factor being a correction factor. It would be better to write the correction factor as "1- X ", so that the quantity X gives the deviation from straight-tube behavior. The quantity X is then Po-P, 2-Ib x=1-2. -R, =1- 3fi- (Ri/Ro Ney _ 3[1-(Ri/Ro)|(R./Ro)” RE RP =RS (Ro/Ri) =1 1=(R:/Ry) _ 3(R:/Ro)’ ___1+(Ri/Ry) +(Ri/Ro)” =3(Ri/Ro)° 14+(R,/Ro) +(Ri/Ro)” 1+(R,/Ro)+(Ri/Ro)” which then leads to the desired result in Eq. 2B.10-3. 2-11 2B.11 The cone-and-plate viscometer a. In a parallel-plate system with rectilinear flow, the velocity distribution is just 0, /v) = y/b, where b is the plate spacing and % is the velocity of the upper plate. We now make the following correspondences between the parallel-plate system and the cone- plate system (using @ as the usual variable in spherical coordinates measured downward from the z-axis, and y as the variable measured upward from the plate surface): Py Oy; Vy QP; berrsinyy; yorsiny=ry= n-0) When this correspondence is made, Eq. 2B.11-1 results. b. From Eq. B.1-19, we get for the force per unit area in the ¢-direction on a face perpendicular to the 0-direction a 0 B96 | sind Here we have used the fact that the angle between the cone and plate is so tiny that @ is very nearly }7 so that sin @ is very close to unity. c. The torque is obtained by integrating the force times lever arm over the entire plate area: 2m FR, _ of Ha HQ) RE T.= "I (too Degatiriv==2a{ M2) Pa -aa{ 22) Ey } which leads to Eq. 2B.11-3. 2-18 2B.12 Flow of a fluid in a network of tubes At A the pipe splits into three pipes, and at the next set of junctions the fluid flows equally in six pipes, and then at the next set of junctions the fluid flows back into three pipes, and finally at B the fluid is all returned to a single pipe. Call the modified pressure at the junctions where three pipes split into six pipes P_,,, and that where six pipes join to form three pipes P,_,5. Then in each of the first set of three pipes w _ mPa -Ps,6)Rip _ Sulw 3) Bul or Pa Pans = aarp In each of the batch of six pipes 8uLw or P345— Pog = Saki and in each of the final batch of 3 pipes w _ 2(Po3—Ps)Rip _p . Sulw 3) SuL or Poss Pa a ERip When all the pressure differences are added together, the unknown quantities P, ,, and P,,,, cancel out, and we get 8uLw (5 3a(P,-P,)R4p Pa-Pa= eal 6) o ee on id 2C.1 Performance of an electric dust collector a, First we solve the problem of the vertical motion of the particle as it falls under the action of the electromagnetic field. The equation of motion for the particle (without gravitational acceleration or Stokes drag) is This equation may be integrated with the initial conditions that x=x, and dx/dt =0 at £=0, to give _e6P 2m X=Xp From this we can get the time f, required for the particle to fall to Next we look at the horizontal motion. From Eq. 2B.3-2 and the expression for x(t), we find that (with v,, = (Po — p,)B?/2uL sofa} [ 329) This may be integrated to give L y 1 xeSe? | 674 Laftemegft[t-ge( 2-2" SE jt e6t; Ste 3m 20m? neo -x)y+ Next, square this expression and then insert the expression for ty above to get 2(Po - 2m(B + x, ira B Bs 0)(3B-214)(B +29) Then, in order to remove the radical, we square this, thereby getting — 8(Po— PL) 225y7e6. (3B-2x))'(B+x))° Next, we set dL'/dx) equal to zero, and this yields 4 values for x9: 4B, 3B,B, and B. It is only the first of these that is physically acceptable. When that value is put back into the expression for L, we get finally [2 (Po- pu)? mB° T Lin =| 357 oe 35 ates 9-2\ 2C.2 Residence-time distribution in tube flow a. A fluid element at a radius r within the tube will require a time f to reach the tube outlet Li L 2) Oneal (O° tant CIR) All the fluid with a radius less than r will have left the tube at this time. Hence the fraction of the flow that will have left the tube is r= forardo _ (ty (2 FP foraraa “\R) WR When the first equation is solved for (r/R) and substituted into the expression for F(t), we get 2 — 2(0,)t rordiess) ests) ra) b, The mean residence time is then obtained by solving the last equation in (a) for #, and substituting into the Eq. 2C.2-1: L Vmax 2L L 1p p_aF hid? => hae a-1L 2C3 Velocity distribution in a tube The derivation in §2.3 is valid up through Eq. 2.3-15. If the viscosity is dependent on the radial coordinate, however, Eq. 2.3-16 is inappropriate. Instead we get Application of the no-slip boundary condition at the tube wall gives (Po-P,) pe Fo o =o Fu) Tapa c ae bate This may be solved for the integration constant, and the velocity distribution is then ppalPamPi)i Fp ggg a(Po“PUR GA Tg 2L* u(r) 2L u(y) This is the same as Eq. 2.3-18 if the viscosity is a constant. Next we get an expression for the average velocity ofl v.rdrdo ( be ne) (».) “wh v,rdr = 2[)0.ydy re eo yay = = (Po Fie “PR Pre 7 ony, Then we find the dimensionless ratio 2-23 2C.4 Falling-cylinder viscometer a. Equation 2.4-2 is valid for this problem, but the pressure difference is not known. When Newton's law of viscosity is substituted into Eq. 2.4-2 we get Bi-P, .=-{ SF} -cuinrsc, The two constants of integration and the (unknown) pressure difference can be obtained from two boundary conditions and a mass-conservation condition: At r=«R, 0, =—0p; at r=R, v, =0; and J." J¥,0.rdrd0 = m(KR)? v9. This states that the fluid displaced by the falling cylinder must be compensated for by a net motion upward through the annular slit. These three unknown constants may be obtained from these conditions (lengthy!) and the result is v, __(1-€?)-(1+«?)in(/é) b. The force acting downward on the cylindrical slug of height H is (py — p)g- (KR)? H. The difference in the pressures acting on the top and bottom of the slug is an upward force (Bo —P,)-m(KR)? = —[)! (dB /dz)z-n(KRY __ 49H - (KRY R?|(1- x?) (1+ «?)in(/x)] In addition, there is an upward force associated with the frictional drag by the fluid 2n(kR)H(-r,,)] aK = antaryiin( 2) Ir=KR - 2x -(1+«?)(1/x) ge When these are equated and the result solved for the viscosity, the expression in Eq. 22C.4-2 is obtained. c. Next put x=1-e, and expand in powers of €, keeping terms up to €°. Use §C.2, and obtain Eq. 2C.4-3. 9°25 2C.5 Falling film on a conical surface a. A mass balance on a ring of liquid contained between s and s+ As gives (213(sin B)5(s){v))), — (2(sinB)4(s)(v)),,,, = 0 Letting As— 0 then gives 4 (s5(0)) =0 — whence, from Eq. 2.2-20 4(s5)=0 ds . ds Equation 2.2-20 is valid strictly for a flat plate with constant film thickness, but we apply it here approximately to a different geometry. b. When the equation in (a) is integrated, we get sd° =C, in which C is an integration constant. This constant is determined by requiring that the mass flow down the conical surface be the same as that flowing up the central tube (ie., w). We hence write (width of film) x (thickness of film) x (mass flow rate), and then use Eq. 2.2-20: : «= (2rssinp)-6-p(e) = (2assinp){S) * eaclsreese From this we get C; _ 3yw _ 3a (2msinB)-(p*gcosB) mp*gsin28 The film thickness as a function of the distance down the cone from the apex is thus 20% gssin 2p 2C.6 Rotating cone pump a. Inner cone not rotating. For sufficiently small values of B, the flow will resemble very much that for a thin slit (see Problem 2B.3), for which the mass rate of flow is given as the answer to part (b). This formula may be adapted to the flow in the annular space of height dz. as follows, if the inner cone is not rotating and if the gravitational force is not included: -}{-2) B3(2azsin0)p “3 L where we have made the identification (py —p,)/L—>-dp/dz and W > 2ar=2nzsin@. Across any plane z = constant, the mass flow rate will be a constant. Hence the above equation can be integrated to give 2 gy = 3 MD _ pad 3 wo i = -p = 2 Yn pena Bosinoln Z Pi-P2 = 47 Bpsind "L, b. Effect of the gravitational force and the centrifugal force The result in (a) may be modified to include the effect of gravitational acceleration g and the angular velocity Q of the inner cone. The gravity force in the z-direction (per unit volume) is given by Fyv.x =—PC0SB. The centrifugal force (per unit volume) acting in the middle of the slit will be, approximately, Fey = ($2r)*/r =4p°zsinB, where r is the distance from the centerline of the cones to the middle of the slit. The component of this force in the 2- direction is then F,.,,; = 4Q?zsin? B. Then the first equation in (a) can be modified to give B3(2nzsinB)p L w 2( £2 + 4parzsin®B-agcos8) This equation can be integrated to give 2-24 lla LU Pa) (Res si? BE -F8)-(05088\ LL] Many assumptions have been used to get this solution: (1) laminar flow (turbulent flow analog is not difficult to work out); (2) curvature effects have been neglected (correction for this is easy to do); (3) entrance effects have been ignored (this can probably be handled approximately by introducing an "equivalent length"); (4) instanta- neous accommodation of velocity profiles to the changing cross- section (it would be difficult to correct for this in a simple way). 7-28 2C7 A simple rate-of-climb indicator a. Consider two planes of area $ parallel to the earth's surface at heights z and z+ Az. The pressure force in the z-direction acting on the plane at height z and that acting at the plane at height z+ Az will be just the mass of air in the layer of height Az: SPI, ~ SPlesas = P8SAz Division by SAz and then letting Az—0 gives the differential equation - ap __| pM. PS OF ik fa which describes the decrease in the atmospheric pressure with increased elevation; here R, is the gas constant. b. Let p; be the pressure inside the Bourdon element and p, be the pressure outside (i.e., the ambient atmospheric pressure), We now write an equation of conservation of mass for the entire instrument: =Po)R* BuL i = Po)R* Sa Pe vipe Mey =~ Wei Vie d a Pavgi VP = Here m,,, is the total mass of air within the system (Bourdon element plus capillary tube), w, is the mass rate of flow of the air exiting to the outside, p; is the density of the air inside the Bourdon elements, and ,,, is the arithmetic average of the inlet and outlet densities within the capillary tube (see Eq. 2.3-29). The third form of the mass balance written above has made use of the ideal gas law, p=pR,T/M. If we neglect changes in the arithmetic average pressure Pag and use the abbreviation B= 2R*p,.,/8uLV, we can integrate the mass balance above and get pi =e" ({Bpe™dt +C) To get p,(t) we make use of the fact that there is a constant upward velocity, so that AP. _ Apo dz __| PM) dt dz dt (eet se: ‘Ap, whence Then the mass-balance equation becomes p. =e ({ Bpte-*elat +C) = Bps ett CoB ‘ * B-A Determine the constant of integration, C, from the initial condition that p? =p? att =0. Then Be“ — Ac Pi=Pi—z—a— and P In the limit that t > ©, we get fir B>>A 8VuL P Mg 25 Bo ORT aR Pag Hence for p, ~ Pavg, the gauge pressure is _ _., (8uL) Mg m, nooo Sat) Hence the pressure difference approaches an asymptotic value that varies only slightly with altitude. c. To get the relaxation time, note that gy (1-er@-a)_ Bea, BoA P.(i-e 3) whence 2-30 “8 sels so that — ty =t=—SH#LV Bo AR "Pavg It is necessary to have tj <<100 to insure the plausibility of the assumption that B>>A. 2-31 2D.1 Rolling-ball viscometer The rolling-ball viscometer consists of an inclined tube containing a sphere whose diameter is but slightly smaller than the internal diameter of the tube. The fluid viscosity is determined by observing the speed with which the ball rolls down the tube, when the latter is filled with liquid. We want to interrelate the viscosity and the terminal velocity of the rolling ball. The flow between the sphere and the cylinder can be treated locally as slit flow (see Problem 2B.3) and hence the only hydro- dynamic result we need is dp _12m(o,) eo) But we must allow the slit width o to vary with @ and z. From the figure we see that R? =(R-r) +(r’ +0)? -2(R-r)(r’ + 6) cosO where r’= Vr? —2?. Solving for o we get o=-r'+(R-r)cos0 +Ry1+[(R-r)/R} (-sin? 6) The second term under the square-root sign will be very small for the tightly fitting sphere-cylinder system and will hence be neglected. Furthermore we replace V7? — 2? by VR? — 2 and add compensating terms o=(R-r)| cos0+ R-VR? 2? +(R-1r)-3(R-1)(2/R) R-r ed =(R- nfo =(R- nfioma +1)+ =2(R -rfoc! 8, RovRP= a ae The omission of the term containing (z/R)’ and the higher-order terms is possible, since the greatest contribution to the viscous drag occurs at the plane z = 0, and hence less accuracy is required for regions of larger z. Note that the above result gives correctly o=0 atz=0, @=2,and o=2(R-r) atz=0, 0=0. Next we assert that dp/dz will be independent of @, which is probably a good approximation. Then according to (*) (v,) must have the form (v,)=Biz)o* — (**) Next, the volume rate of flow across any plane z will be = ["%0,}o(0,z)RAO= RB(2){""[o(0,2)] 40 =8RB(z)\(R—r)'f'"Joos? 40+ a] d= 8RB(2)(R-7)°I(a) in which a= (R-VR?-2”)/2(R-r). The volume rate of flow Q at all cross-sections will be the same, and its value will be, to a very good approximation Q = 7R?vp, where vj is the translational speed of the rolling ball. Equating the two expressions for Q gives RU B@)= TF iR-) (a) (Hy Combining (*), (**), and (**) we get dp ___ 3mRoy dz 2(R-r)*I(a) The total pressure drop across the slit is then 2-33 +rdp = dp de 4 Creo 2b ie da into which we have to insert dz/da. Virtually no error is introduced by making the upper limit infinite. From the definition of a 2 =-4(R-r) a? +4R(R-r)a The first term on the right is smaller than the second, at least for small z. Then dz=.R(R-r)da/ Vo, and the pressure drop expression becomes (with £? = @) Ap= 2yRR= NP da= a Ne Pas 3/2 =4R(R-r Jag gers 1G Te)" Se J where _4 1 yo) _ J= apt Te\® $1 y2-Fg(v0+2) Jr osat The pressure drop multiplied by the tube cross-section must, according to an overall force balance, be equal to the net force acting, on the sphere by gravity and buoyancy 42R°(p, - p)gsinB = nR?Ap where p, and p are the densities of the sphere and fluid respectively. Combining the last three results gives the equation for the viscosity -4 Rios eigen R=)" ~ On 0 R 234 2D.2 Drainage of liquids a. The unsteady mass balance is a 2 (paw) =(ole.)W8), -(ol0.)W8).,, Divide by pWaz and take the limit as Az— 0, to get Eq. 2D.2-1. b. Then use Eq. 2.2-22 to get Eq. 2D.2-1: 05 __ pg d5> __pgd? 95 ‘oO 3nd pp a which is a first-order partial differential equation. c. First let A = g/t, so that the equation in (b) becomes: OA _ dA AA _ poh a Inspection of the equation suggests that A =./z/t, which can be seen to satisfy the differential equation exactly. Therefore Eq. 2D.2-3 follows at once. This equation has a reasonable form, since for long times the boundary layer is thin, whereas for short times the boundary layer is thick. 9-35 3A.1 Torque required to turn a friction bearing. Equation 3.6-31 describes the torque required to turn an outer cylinder at an angular velocity Q,. The corresponding expression for the torque required to turn an inner rotating cylinder at an angular velocity 9; is given by a formally similar expression, 2 = Arp QGRL ( ) 1-5? derivable in like manner from the corresponding velocity profile in Eq. 3.6-32. ‘The specifications for this problem (converted into SI units via Appendix F) 0.998004; x? = 0.996012 k 0.996012 (; = =) = 0.003088 ~ 7498 200 ep)(10~* kg/m-s/ep = 0.200 kg/m-s 200 rpm)(1 min/60 s)(2x radians/revolution) = 20/3 radians/s ')(1 m/39.37 in)? = 0.000645 m? LE =2 in = 2/39.37 m = 0.0508 m P= (50 Ibm /ft®)(0.45359 kg/Ibm(39.370/12 ft/m Hence, the required torque is = 800.9 kg/m? Tz = (4)(0.200 kg/m-s)(20x /3radians/s)(0.000645m?)(0.0508 m)(249.8) = 0.431 kg-m?/s? = 0.32 ft-lby and the power required is P = TM; = (0.32 Iby-£t)(20x/3 s-*)(3600 s/hr)(5.0505 x 10-7 hp-hr/Iby-ft) = 0.012 hp In these calculations we have tacitly assumed the flow to be stable and laminar. To test this assumption, we formulate a transition criterion based on the critical angular velocity expression given under Fig. 3.6-2: DipR2(A — Ki? # Insertion of numerical values for the present system gives Re = (20/3 radians/s)(800.9 kg/m*)( 0.000645 m?)(1 ~ 0.998004)*/? _ 4 gous ~ (0.200 kg/m-s a“ Re < about 41.3 for «= 1 ‘This Re value is well below the transition value of 41.3 for this geometry; therefore, the foregoing predictions of T; and P are realistic. 3- 3A.2 Friction loss in bearings. ‘The power expended to overcome the bearing friction is 2 P=T.O; = sry RL G a) in which L is the total bearing length of 2 x 20 x 1 = 40 ft for the two shafts. The specifications for this problem (converted to SI units via Appendix F) are: ke 16 _ ~ 16+2 x 0.005 — wt 0.998751 G = =) = pooizag = 199° = (5000 cp)(10~* kg/m-s/cp) = 5 kg/m-s Q; = (50/60 rev/s)(2x radians/rev) = 57/3 radians/s R? = (8/39.37 m)* = 0.04129 m? L = 40 ft = (40 x 12/39.37 m) = 12.2m 999375; x? = 0.998751 With these values, the calculated power requirement is, P = (4n)(5 kg/m-s)(5x/3 rad/s)?(0.04129 x 12.2 m*)(799.6) = 6.938 x 10° kg-m?/s? ‘This result is then expressed in horsepower by use of Table F.3-3: P = (6.938 x 10° kg-m?/s*)(3.7251 x 1077 hp-hr-[kg-m?/s?]-")(3600 s/hr) = 930 hp Thus, the fraction of the available power that is lost in bearing friction is 930/(4000+ 4000) = 0.116. 3A.3 Effect of altitude on air pressure. For a stationary atmosphere (i.e., no wind currents), the vertical component of the equation of motion gives dp _ ened The air is treated as an ideal gas, pM. RT with M ~ 29, and with temperature in °R given by T(z) 30 — 0.0032 at elevation z ft above Lake Superior. The pressure pz at 22 = 2023 ~ 602 = 1421 ft above lake level is to be calculated, given that p: = 750 mm Hg at 2 = 0. The foregoing equations give ding ____Mg__ dz — R(530 — 0.0032) Integration gives Mi In(po (pi) = Fe Mg 1 sunt == — 0.003: FH d.003 (530 — 0.0032] . . — Ma_,, [525.737 ~~ 0.003R 530 Insertion of numerical values in Ibm-ft-s units gives (29 Hhbm/Ib-mol)(32.17 R/s") (C003 R/ft)(4.9686 x 108 Tb,,-t? /s™-Ib-mol-R) —0.0505 In(p2/p1) In [525.737/over530) Hence, 2 = pr exp(—0.0505) = 750 x 0.9507 = 713 mm Hg Since the fractional change in P is small, one gets a good approximation (and a quicker solution) by neglecting it. That method gives p2 = 712 mm Hg. BAA cosity determnation with a rotating-cylinder viscometer. Here it is desirable to use a sufficiently high torque that the precision of viscosity determinations is limited mainly by that of the measurement of angular velocity. A torque of 104 dyn-cm, corresponding to a torque uncertainty of 1%, appears reasonable if the resulting Reynolds number is in the stable laminar range. ‘The geometric specifications of the viscometer are: R=22%5 cm; KR=2.00cm & = 2.00/2.25 = 0.888889; «? = 0.790123 1-4? = 0.209877; (wR)? = 4.00 cm? R? = 5.0625 cm? ‘The angular velocity corresponding to this torque value is: T=?) __(10* geem?/s?)(0.209877) _ Mo = Fru(RR)PL ~ Tx(0.57 g/em-s)(4 em?)(4 em) = 18.3 radians/s ‘The Reynolds number at this condition is: QR? p _ (18.3)(5.0625)(1.29) _ Re= = 210 eo 037 Accordng to Fig. 3.6-2, this Re value is well within the stable laminar range; there- fore, a torque of 10 dyn-em is acceptable. 3A.5 Fabrication of a parabolic mirror. Equation 3.6-44 gives the shape of the free surface as 2) , sax (Z)e ‘The required derivatives of this function at the axis of rotation are dz az cies Setting the desired focal length equal to half the radius of curvature of the mirror surface at r = 0, and using Eq. 3A.5-1, we obtain 1 ig? g9/@ ‘Thus, the required angular velocity to produce a mirror with focal length f = 100 em at standard terrestrial gravity is 214 radians/s which corresponds to 602/2n = 21.1 revolutions per minute. 3-5 3A.6 Scale-up of an agitated tank. ‘The specifications for the operation in the large tank (Tank I) are Ny=120rpm; y= 13.5 ep; pp = 0.9 g/cm and the tank is to be operated with an uncovered liquid surface. To allow direct prediction of the operation of Tank I from experiments in the smaller system (Tank IT), the systems must be geometrically similar and must run at the same values of Re and Fr. To meet the latter requirement, Eqs. 3.7-40,41 must be satisfied. Equation 3.7-41 requires DuN} = Din? when, as usual, the gravitational fields for the two systems are equal. Then the model must operate at Nu = NVDi/Dy = 120V10 = 380 rpm and Eq. 3.7-40 requires man (2) (Nw)? "="\D.) Mi = (13.5/0.9)(0.1)*(V10) = 0.474 ep From Table 1.1-1, we see that this value of v1 corresponds closely to the value for liquid water at 60°C, Thus, the model should opeerate at 380 rpm, with liquid water at very nearly 60°C. 3A.7 Air entrainment in a draining tank. As this system is too complex for analytic treatment, we use dimensional analy- sis. We must establish operating conditions such that both systems satisfy the same dimensionless differential equations and boundary conditions. This means that the large and small systems must be geometrically similar, and that the Froude and Reynolds numbers must be respectively the same for each. Choose D (tank diameter) as characteristic length, and (4Q/D*) as charac- teristic velocity, where Q is the volumetric draw-off rate. Then 16Q? Re= sp, and Fr= aps Subscripts L and $ will be used to identify quantities associated with the large and small tanks, respectively. We take the gravitational field g to be the same for both. ‘Then the requirement of equal Reynolds numbers gives (2) (B= (2) (2) = (Rosanne, and the requirement of equal Froude numbers gives (&)- (2) Combining these requirements, we obtain (32) = (0.02277)"/* = 0.080 Di : Hence, Ds = (0.080)(60 ft) = 4.8 ft Qs = (0.080)°/?(800 gal/min) = 1.46 gal/min ‘Therefore: a. The model tank should be 4.8 ft in diameter. b. Its draw-off tube should be 0.080 ft in diameter and 0.080 ft hi ¢. Its draw-off tube should have its axis 0.32 ft from the wall of the tank. Furthermore, if water at 68°F (20°C) is withdrawn from the model tank at 1.46 gal/min, air entrainment will begin when the liquid level is (4.8/60) of the level at which entrainment would begin in the large tank at its withdrawal rate of 800 gal/min. 3-7 3B.1 Flow between concentric cylinders and spheres a. The derivation proceeds as in Example 3.6-3 up to Eq. 3.6- 26, which we choose to rewrite as U9 ay % =p +d{® pape? The boundary conditions are that v9(KR)=Q,xR and v9(R)= Q,R. Putting these boundary conditions into the above equation for the angular velocity gives =D, +d, and Q,=D,+D, These equations can be solved for the integration constants Hence the solution to the differential equation is (2-2?) (21-2) r 1-K* 1-« Ur The z-components of the torques on the outer and inner cylinders are Teel ghia or 2(8)] fr = ant un So) xy (2 (2 z) T= ufo" (+t KR), _.g KRdOdz = retort] (wry = +4muL(Q, -Q. oo ge ae b. In Example 3.6-5 it is shown how to get Eq. 3.6-53 for the velocity distribution. The boundary conditions are : v,(kR)= kRQ,sin@ and 04(R) = RQ, sin 8. Equation 3.6-53 can be written in the form > . st =D, +D,(8) rsin@ r The constants can be obtained according to the method of (a) and the final expression is % (2 =2,x°) (Q, =B.}( sk) rsin@ 1-K* 1-e Ur The torques at the outer and inner cylinders are then To) jag (Rin O)R? sin adodp ehh =f “lew s(2 | (Rsin 0)R? sin odod6 =-8mu (2, -2. oe (ey T.= O71 (+o), gg (RRSin 8)(KR)? sin AdOdG = +824(2, -2,): 3-4 3B.2 Laminar flow in a triangular duct a. It is clear that the boundary conditions that v, =0 at y=H and at y=+V3x. Therefore the no-slip boundary conditions are satisfied. Next it has to be shown that the equation of motion -P a 42 0 P SF] is satisfied. Substituting the solution into the second-derivative terms, we get Po-PL Yl Pe f tut \é +a (3x°y-y? - 3Hx? + Hy’) -(PaxPe \ey- 6H -6y+2H) and this just exactly cancels the pressure-difference term. b. To get the mass rate of flow we integrate over half the cross-section and multiply by 2: VNB alee P-P, M8 = of FaSEe) hy He? yall ay Py-P, af a to 3 _ ~ Fe -P,\(_H®)_ (3(P)-P,)H* =P\ 39% uty) 20)" 180uL The average velocity is then the volume rate of flow (w/p), divided by the cross-sectional area H?/4/3 so that we aof FacFe (y-H)(3x? -y?)dxdy _(Po-P,)H* 60uL 3-10 The maximum velocity will be at the tube center, or at x = 0 and y = y=2H/3, so that o -Po=PL)H? me ee =F.) 3-11 3B.3 Laminar flow in a square duct a. The boundary conditions at x= +B and y=+B are seen to be satisfied by direct substitution into Eq. 3B.3-1. Next we have to see whether the differential equation op -(Po=F.) + Se a) is satisfied. Substituting the derivatives from Eq. 3B.3-1 into this differential equation gives ote -a(etafe (3-9) Hence the differential equation is not satisfied. b. The expression for the mass flow rate from Eq. 3B.3-1 is given by 4 times the flow rate for one quadrant: wna tela) (8) =P PP Pesan w(e=PaP tee) asl _ (Po ~ PB ee 0.444(Po - P, Bip HL 5 HL 3B.4 Creeping flow between two concentric spheres a. From Eq. B.4-3, there is only surviving term on the left side 1 =) Slr sino) = 0 whence v9 sin@ = u(r) b. From Eq, B.6-8 (omitting the left side for creeping flow) the only surviving terms are Aa fn te) or on 24, [1 (a a) Pa” ar do "| 7sinddr\” dr c. When the equation in (b) is multiplied by rsin@, the left side is a function of @ alone, and the right side contains only r. This means that both sides must be equal to some constant, which we call B. This gives Eqs. 3B.4-2 and 3. Integration of the pressure equation proceeds as follows: aE 4 frap-p" eo, ~P, =Bin HE). Bin Sze Py © sin@ tan} tante From this we get the constant B -P, _ P,-P, Incot?te 2Incot}e Next we integrate the velocity equation (ad) me BR(#_GR.c,) f(r ar rr or ua mR or +C, where we have selected the constants of integration in such as way that they will be dimensionless: C,=-K and C, =-x~1 (from the no-slip condition at the walls). When this solution is combined with the expression for B we get: aa mare ala] which leads to Eq. 3B.4-5. d. The mass rate of flow must be the same through any cross- section. It is easiest to get w at 0=42 where v,=u(r)/sind= u(r)/sin} 2 =u(r): w= fo" fPPel,..,74rdd = 2nR pf uedE - aarp EP 0- a+ +(3-F) fa 4uincothe —oo. This emphasizes that kinetic energy is not conserved. e. Eq. 3B.7-1 clearly satisfies the equation of continuity, since for incompressible flow (dv, /3x) + (dv, /dy) + (9v,/3z) = 0. When the derivatives are calculated from Eqs. 3B.7-2, 3, and 4, it is found that these expressions also satisfy the incompressible equation of continuity as well. f. From Eqs. 3B.7-2 and B.1-1 we get uf 2) 3x2 axt ___ 40 nWp. (@+¥) (e+y) 0 mWpx a, - af 22) 3x2 xt =0 exo mW) (x2 +y2) (x? +y?)? Lo Tq = 2p Ox The second of these is an illustration of Example 3.1-1. g. From Eqs. 3B.7-2, 3B.7-3, and B.1-4, we get, after evaluating the derivatives 3B.8 Velocity distribution for creeping flow toward a slot a. For the given postulates, the equation of continuity gives from which it follows that 0, = i f(0) Since the flow is symmetric about @=0, df/d0=0 at 0=0; and since the fluid velocity is zero at @ = +47, it follows that f=0 at @=+47. b. The components of the equation of motion given in Eqs. B.6-4 and 5, appropriately simplified are = 98 Qu of and O50 FF ad c. When the first equation is differentiated with respect to and the second with respect to r and the two results subtracted we get Eq. 3B.8-1. d. The equation in Eq. 3B.8-1 can be integrated once to get a Wer s +4f=C, A particular integral is f»; = }C,, and the complementary function is (according to Eq. C.1-3) foe =C,c0s20+C;sin20 . The complete solution is then the sum of these two functions. e. The integration constants are determined from the boundary conditions. It is found that C, = 0, and that $C, =C,. Then from w= —W|'""0,rd0 = -WpJ™, f= -2WpC, f°") s+n/2 cos? 640 =-WpC,2 we get C, =—w/Wpz and the velocity distribution is given by Eq. 3B8-2. jf. From the velocity distribution and the equations obtained in (b) we can get 320 (cost @-sin? 0) +F(6) (*) Furthermore oP du ff 2p) 22. 30 * 72 do or P = | awp cos? @+G(r) Here F and G are arbitrary functions of their arguments. The second expression for the modified pressure can be rewritten as Pe-8 #(3e cos? 0+(1-sin? 9) +G(r) = eG 6-sin?6)+H(r) (*) By comparing the two (*) expressions for the modified pressure, we see that they are the same except for the functions F and H. Since the first is a function of @ alone and the second a function of r alone, they must both be equal to a constant, which we call P... This is the value of the modified pressure at r =<. g: The total normal stress exerted on the wall at @= 7/2 is (when one uses the result of Example 3.1-1) Teoloan = (P+ To Nooaya= (P -pgh)),.. [2 _ 2uw = 2uw = Bat prt PBR = Pos + oe z h. From Eq, B.1-11 3-2! Tel =u 2 (22) 428 aati arr) r 88 Nona 2w . _ 5 -ifo-( Ata \-acoesino)| =0 pana The first term is zero since vy/r=f since vj =0 was one of the postulates. The second term is zero, as can be seen by using Eq. 3B.8- 2, and the fact that cos$=0. This is agreement with the result in Problem 3B.7(g). i. Since the z-component of the velocity is zero, we can expand the velocity vector in either the cylindrical coordinate system or the Cartesian system thus v=5,0, +540) =5,0, +5 vy Since vg =0 was one of the postulates, when we take the dot product of this equation with 3,, we get the x-component of the velocity qw os % =(6,°8,)=0,c080=- Fe 2, », sino = -—2_ cos? gsing =- 272 ¥ aWor mWor xy W(x? +) These results are in agreement with Eqs. 3B.7-2 and 3. Boe 3B.9 Slow transverse flow around a cylinder a. At the cylinder surface we get by using Eq. B.1-11 Pag =P MeaSO88 _pyrsind 2 fo) .22 Cuv.. sin 8 alr) *r ao), = Also from Example 3.1-1 we know that 7,,|_, =0. b. If n is the outwardly directed unit normal vector for the cylinder, then the force per unit area acting on the surface is —[n-1] evaluated at the surface. But n=5,, so that the x-component of the force per unit area at every point is Bs 18, =-8e [3 (03+2))),, The pressure and stress terms are evaluated thus (using Eq. A.6-13): ~(5s [5,75], =—(8x-3.P).g = Phan 089 ~(8.18,-*))g =-Be 15, -(-45,857%,+~))) =~. 50) Tol a = Tol ag SiO These expressions lead to Eq. 3B.9-5. c. The total force on a length L of the cylinder is then F,=[30"(-Pl,., 080+ tl... Sin 0)RdOdz Cuo,, cos? 108° in? 5. pgRsin 0coso + He= Sin” 6 R — py ( CHOm) p2x = RI AEP \ft" do = 2nC yo = refe(-. cos0 + a The first and third integrals in the next-to-last line are zero since the integrands are odd. 3-25 3B.10 Radial flow between parallel disks a. The continuity equation is, for v, = v,(r,2) , from Eq. B.4-2 ee) ‘ = H(%)=0 from which 10, = 9(2) The equation of motion is obtained from Eq. B.6-4 da, fa(1a/,,)), Pero Eee StS 0))+ | b. When the results for the equation of continuity in (a) are used in the equation of motion, we get Eq. 3B.10-1. c. With the creeping flow assumption, Eq. 3B.10-1 gives ded . dP Po rap tH gge fromwhich =r =B and wo S= since the left side is a function of r alone and the right side a function of z alone, and therefore both sides must be equal to a constant, B. When the pressure equation is integrated from r, to r2, we get Pp pie ar _P,-Py Jp, a = BI? Whence Bian) d. When this result is substituted into the equation we get ao _ -P,-P, : P,-P, 2 = =-—1—,_~Ss fromwhich g=-—1 2 = + 2+, dz? win(r,/r,) qln(r,/r,) 20 7? The integration constants are obtained from applying two boundary conditions. We could require that ¢=0 at z=+b, and thereby determine the integration constants. Another method is to recognize that the flow is symmetric about z = 0, and use as one of the boundary conditions dé/dz=0 at z = 0. Either method will give C, =0, and then C, is easily obtained. The final result is gat Peel zy eel 2uin(re/r)| \b Division by r then gives Eq. 3B.10-3. e. The mass rate of flow at any cylindrical surface in the system must be the same. Select the surface at r =r, and obtain 2a pb _ op AP1= Pa)? cays gr OTP rlran, d2rdO = 2mpb-2 2uln(ra/n) Si(a-&? ag The integral gives 2/3, so that Eq. 3B.10-4 is obtained. 35 3B.11 Radial flow between two coaxial cylinders a. From Eq. B.4-2 we get for this flow, with 2,(r), 14(7,)=0 whence v,=© where Cis a constant 7 Atr=R, 0,(R)=C/R so that C = Ro,(R). b. The relations in Eq. 3B.11-1 follow immediately from Eqs. B.6-4, 5, and 6 for the velocity profile v,(r) in (a). c. Integration from r to R gives fe -tnc(d-3) P(R)- Pir) =0C*(- Fy] =? This gives, making use of the meaning of C obtained in (a) : Pie) POR) = take, IP Fo) dole wy |1-( 4) | d, The only nonzero components are (from Eqs. B.1-8 to 13) 1 2c. “Ht = air 1 =4+2uC3; +2uC a 3-26 3B.12 Pressure distribution in incompressible fluids The equation of motion method to get the pressure distribution is correct. On the other hand, the second method gives nonsense, as one can see from Fig. 3.5-1. For an incompressible fluid (the vertical straight line), specifying the density does not give any information about the pressure. 3-21 3B.13 Flow of a fluid through a sudden contraction a. For an incompressible fluid, Eq. 3.5-12 becomes 40(v3- v7) +(P2- pi) + g(a -m)=0 or 4(v3 -v7)+(P,-F,)=0 If "1" is the large tube and "2" the small one, then the fluid velocity in "2" must be greater than in "1." Then the modified pressure in "2" must be less than that in "1." Thus the modified pressure decreases as the fluid moves from the large cross-section region to the small cross-section region, in agreement with experimental observations. b. For an ideal gas, Eq. 3.5-12 becomes RT, Pp. 1 2 _ y2 em -h,)= $o(v3 a) + 08(Iy -h,)=0 The pressure and elevation terms may no longer be combined. If the elevation does not change, the pressure decreases as the fluid moves into the contracted part of the tube. 3-28 3B.14 Torricelli's equation for efflux from a tank From Eq. 3.5-12 we get 1 4 (Binux ~ 0) + (Pam ~Patm) + (0h) =0 Here it has been assumed that the velocity at the surface is virtually zero, that the pressure is atmospheric at both "1" and "2", and that the datum plane for the height is at the exit tube. When the above equation is solved for the efflux velocity we get Torricelli's equation. 374 3B.15 Shape of free surface in tangential annular flow a. The velocity distribution is given by Eq. 3.6-32, and the equations corresponding to Eqs. 3.6-38 and 39 are: 2-422) 10) Gl] ow Bon Integration of these equations gives (see Eqs. 3.6-40, 41, and 42) a aun (8) (J | Pp if SS =} wAlnr+{ 2] [eget Now let p=Paym at r= R and z=zp, where zp is the height of the liquid at the outer-cylinder surface. Then we can write at r= R and Z=Zp Pain = (5) [-1-4InR+1]-pgzp +C which is the equation that determines C. When we subtract the last equation from the equation for p, we get P~Patn 19f 22 =| (-2 Feo aing +8 *) -aete~2n) The equation of the liquid surface is then the locus of all points for which p = Pjtm, OF nena BES) [Beane-e) b. When the outer cylinder is rotating, we can use Eq. 3.6-29 for the velocity distribution to get 330 Q,xR)*[ 1_r 1/«R\? =p =(—] -2inr-=( =] |-. P rare) [&) " 7) ca Then, we select r = R and z =z, as the point to determine C. Q KRY 1.1)? 1) 2 Pam = PS) :@ ~2InR~2( x) |-pgze +C Subtracting, we get 2B) Cee oper From this equation we can get Eq. 3B.15-2 by setting the left side of the equation equal to zero and solving for za ~z- 333! 3B.16 Flow in a slit with uniform cross flow From Eq. B.6-1, for this problem we have do, _(Po-P 1), dv, ao do Poy ay? oa an +1=0 in which n=y/B, @=0,/[(P)-P,)B’/uL], and A=Bogp/p. This equation has to be solved with the no-slip condition at n= 0,1. We write the solution as the sum of a complementary function and a particular integral. The equation for the complementary function is Poe _ 44 2 dn dn with solution — cp = Sean +Qy By inspection, the particular integral is ¢p, = n/A. Application of the boundary conditions then gives the constants of integration. The final solution is then (with A= Buop/tt) —(Po-Pi)B fy _ wL OA\B efuis b. The mass rate of flow in the x-direction is then Py -P, BW, w= oogiyde= (PELE gin =P Fi Pel, 1 ) - A\2 A’ e4-1 ¢ By making a Taylor-series expansion about A = 0, from (a) we get ¢=4(n—1?)+0(A). When A— 0, this result can be shown to be equivalent to Eq. 2B.3-2. Similarly, A Taylor-series expansion about A = 0 yields from the result in (b) w cafa_a, 2 ) [(Po—P,)WB*p/uL] le Ate 3-32 ware) But the "B" in this problem is twice the "B" of problem 2B.3. If we switch to the "B" of Problem 2B.3, it is found that the answers agree exactly. d. For the coordinate system here, we select as the dimensionless quantities 2 _ bop yo4, ve ae (8 Pat ;V= b Then the differential equation and boundary conditions are af 21, 4¥ ay” aoe with V(4l)=0 The solution is then the sum of a complementary function and a particular integral (as before in (c)) Vv Det ec+¥ Application of the boundary conditions then leads to *Y +cosha+Ysinha asinha Vv Then the average value of this over the cross-section of flow is fvay _1[i(-e% +cosha+Ysinha)dY _ -(1/a)sinha + cosha V)= a= 5 ; fia 2 asinha asinha Then we can form the ratio given in Eq. 3B.16-3: V__ ef ~Ysinha-cosha (V)~ (W/a)sinha—cosha Asa check on this we can go directly from Eqs. 3B.16-1 and 2 to Eq. 3B.16-3. From the first two equations we get efv/s . _ 2A((e4 -1)y-B(e**? -1)] @) ww 1,1 B[(A-2)(e* - 1) +24] 2 A e*-1 ¥ vy Oy B 1 Next, we make the connections between the notations in the two different approaches: y=ztb; B=2b; A=2a (the "y” of part (c) is called "z" here, and {= z/b). Then 0, _ af(e% -1)(z+b)-20(e% —-1)] _(e*-1)(S+1)-2(e%e*-1)_ (e*-e°*)(F +1)-2(e% -e°*) © (Yorf(a=1(e* -1)+2a] (a)[(a-1)(e* -e°*) + 20e-*] = Sle -6°*) +(e" +e°*) - 2088 _ §sinha:+ cosha—-e% (Yonfa(e* +e*)-(e%-e*)] coshar-(1/a)sinh 3C.1 Parallel-disk compression viscometer a. The equation of continuity of Eq. B.4-2 for incompressible fluids, taking into account the symmetry about the z-axis is just Eq. 3C.1-6. The equation of motion in Eq. 3C.1-7 comes from Eq. B.6-4 ignoring the hydrostatic pressure, the inertial force terms, and omitting the terms that are small. b. Equation 3C.1-7 can be integrated with respect to z to give wag bat +Cz+C, The constant C, is found to be zero from the boundary condition in Eq. 3C.1-8, and C, is found from Eq. 3C.1-9. c. Integrating of Eq, 3C.1-6 with respect to z from 0 to H H1O( 1 dp 0 h i3fr Late m) emf do, "Or 12y r dr d. Integration of the equation in (c) then yields ~ Baer Fenn, The integration constant C, must be zero, since the pressure is finite at the center of the disks, and C, is determined from Eq. 3C.1-10. Equation 3C.1-13 is thus obtained. e. The force on the upper plate is then FW)={ CO 1 -(§) ic d= 2nR? peek -& de The integral is 1/4, and this leads to the result in Eq. 3C.1-14. 3-35 f-In this situation, the radius of the glob of liquid R(#) and the instantaneous disk separation H(t) are related to the sample volume Vby V = a[R()]' H(t). Then the force acting on the upper disk is _ pee pr Suro [R()T r\ FO=f"f sell (5) bdrdo _ Sav [R(H)}* __ 3H0,V? AHOP — 2a{HOF g- If, in Eq. 3C.1-14 we replace vy by — dH/dt, we then have an ordinary, separable differential equation for H(t). Integration gives 2Fy tq, __ px dH Saunt t= Ho HE whence ee er (H@®P Ho 3zpur* 3-3t 3C.2 Normal stresses at solid surfaces for compressible fluids First write the equation of continuity for a compressible fluid as 9 inp=-(V-v)-21(v- ane =-(V-v) a” Vp) The normal stress on a surface perpendicular to the z-axis is Tealswo = (-» oe +(3u- xv-v)} le=0 a =($n+n( Zino] leeo The terms containing v drop out by the no-slip condition at the surface, and their derivatives with respect to x and y drop out on the surface as explained in Example 3.1-1. This result shows that the normal stresses at surfaces are zero for compressible flow if the flow is at steady state. 337 3C.3 Deformation of a fluid line The curve at any time t is (r,t) =(v,/r)t, which in tangential annular flow is (from Eq. 3.6-32) _( (Rr -1 __AR*/r?) Qt aen-( R= lat and do = “Wea dr The differential element of length along the curve is given by (at)? = (an? + (rdo)? = (ar) | 1 MRA) (vx) - 1) The total length of the curve is then refla=(* 4(R/r)' aN) 5 _Rf! 16m N? aie Ce To get a rough, order-of-magnitude estimate assume that N is large and then the "1" can be neglected and the integral performed analytically 1 _4nN« R l+k (limit of large N) 3-38 3C.4 Alternative methods of solving the Couette viscometer problem by use of angular momentum concepts a. By making an angular momentum balance (actually the z~ component of the angular momentum balance) over an annular region of thickness Ar and height L we obtain (2arL)-(rt,9)|, — (2a(r + Ar)L)-(rt,9)} 0 near Dividing through by 2aLAr and rearranging we get (P20) Laan 7(%0) a) Ar whence d al? te)=0 the second form resulting from taking the limit as Ar + 0. Then using Eq. B.1-11 for the stress-tensor component, we have Af 3 4(2))_ f(r (2 ae whence ¢ 2 =-h4Gr From this Eq 3.6-20 follows. b. Here we start with Eq. 3.4-1, which simplifies to the following for the symmetric stress tensor [¥-{rxe}"]=0 The z-component of this equation is 334 nia rdr (riext})=0 or 14 ({rx¢},.)=0 where, in cylindrical coordinates, r=5,r+5,z. We now work out the cross product, which is {ext}, = DD eay(87 +8,2); t= Eval Foy HE so/(O)Frr ry Hence the equation of change for angular momentum simplifies to 1dya ql te) =0 and the development proceeds further as in (a). 3C.5 Two-phase interfacial boundary conditions a. This result follows at once from Eq. 3C.5-1, when the viscous-stress-tensor terms are omitted. b. To get the right side of Eq. 3C.5-3, it is evident that Eq. 3C.5-1 had to be multiplied by 1/p'v3. The interfacial-tension term in Eq. 3C.5-3 is then (oh Vio (1,1 o 24 I) len] — += || —_ “(ie Re JL ood" |)" RR Lebo" The terms involving the viscous stress tensor are Lg Fl Hm FJ pat gt [ B | ploy Ip l ] Iyvop" Sa bg ba eer lel] And finally, the pressure terms are converted to modified pressure terms plus terms involving the gravitational acceleration a(2 =Po+ O'g(hh=he) —Eicpaspialtc ta) Ethel 2) opr pr To? Tot pip pig piv els fel We see that the Reynolds numbers for the two phases, the Weber number (Eq, 3.7-12), and the Froude number (Eq. 3.7-11) appear as well as the density ratios for the two phases. 3-4 3D.1 Derivation of the equation of motion from Newton's second law of motion a. Equation 3D.1-1 is the statement that the time rate-of- change of momentum is equal to the sum of the surface forces and the gravity forces acting on a small blob of fluid. When the Leibniz formula (Eq. A.5-5) is applied to the left side of Eq. 3D.1-1, we get a fev J Sa Spd + Jlenne- n)dS= J Spvav + ie pvv]ds = =f, Sit fv. pyv]dv (osing Eq. A.5-3) vit) The term containing the stress tensor in Eq. 3D.1-1 can also be rewritten as a volume integral using Eq. A.5-3 to give a J aevdv = — J[V-evv]av - f[V-m]av+ Jogav vit) vi) vit) vay Since the choice of the blob volume was arbitrary, all the volume integral operations may be removed, and we obtain the equation of motion of Eq, 3.2-9. b. If the blob is fixed, then we can write a momentum balance over the blob as follows: £ | pvav =-[[n-pwv]as— f[n-x]dS + | pga v 5 3 v This states that the rate of increase of momentum within the fixed volume equals the rate of increase because of convective transport, the rate of increase because of molecular momentum transport, and the force acting on the system by gravity. The time derivative can be taken inside, since the volume is fixed, and the surface integrals can be converted to volume integrals. The result is an equation containing only volume integrals over the fixed volume: [Spevav =-{[9-pww]av —f(¥ mk + fogiv ade Since the volume was chosen arbitrarily, the volume integrals can be removed, and, once again, the result in Eq. 3.2-9 is obtained. 3-43 3D.2 The equation of change for vorticity Method I: Start with the Navier-Stokes equation in the D/Dt-form, but rearranged thus: W]-Svp+ wWvtg =-Viv? +[vxdvxv]]-Evp + wWv+g Next we take the curl and introduce the vorticity w =[V xv] ow 3 =[Vx[vxw]]+ Ww or SE <[v-wy]-[¥-vw]+ Ww Then using Eq, A.4-24 and the fact that (V-v)=0 for incompressible fluids and (V-w) = 0 always (since the divergence of a curl is always zero, we get Eq. 3D.2-1. Method II: Start now with the Navier-Stokes equation in d/dt-form ov 1 > Da_y-w]-= my [ ] p Vp+wWvt+g Take the curl of this equation and introduce the vorticity to get oe =-[¥x[V-w]]+ vw or e:{Vv- Vv}]-[v-Vw]+ vWW?w Details of the manipulations involved in this last step are given here using the abbreviated notation of §A.9 with the Einstein summation convention: AV x[V-w]], = -€4.9;(0;0;24)=—€ 9; (0,410, —2,9;0,) but 3,0, =0 =~€al(3;2,)(Ar1) +(29;,2%)] = €5j(9j21)(4%x)-(14(E9;%)) =[e:{vv-Vv}], -[v-V[V xv], in which € = SEZCq,5,,5,5, is a third-order tensor. AS 3D.3 Alternate form of the equation of motion Take the divergence of the equation of motion for an incompressible fluid in the form of Eq. 3.2-9, but with the stress- tensor term written in terms of the viscosity and the Laplacian of the velocity. This gives (8-19 -w))-2¥% or 0=-(vv:(Vv)")- av? Then use the definitions in Eq. 3D.3-2 to get Eq. 3D.3-1. 3-4b 4A.1 Time for attainment of steady state in tube flow. a, In Figure 4D.2, the centerline velocity comes within 10% of its final value when vt/R? = 0.45, giving t = 0.45R?/v = (0.45)(0.49 x 10~* m?)/(3.45 x 10~* m?/s) = 0.064 s 4. If water at (68°F =20°C) is used, with v = 1.0037 x 10-° m?/s from Tables 1.1-1 and F.3-6, the time required is t = (0.45)(0.49 x 107* m?)/(1.0037 x 10-® m?/s) = 22s 4A.2 Velocity near a moving sphere. From Eq. 4.2-14 at @ = 7/2, the fluid velocity relative to the approach velocity falls to 1% of veo at ve = —0.99va0 relative to the sphere, giving ob) with R/r <1. If R/r << 1, the cubic term will be unimportant, giving 3(R oor 2(8) r 300 Roa? Clearly, the neglect of the cubic term at this distance is justifiable. or 4A.3 Construction of streamlines for the potential flow around a cylinder. In the following drawing we show the construction of the streamline Y = method described in the problem statement. by the 4A.4 Comparison of exact and aproximate profiles for flow along a flat plate. Let My = v,/veo and ¥ = yx/vq/va; then Eq. 4.4-18 gives the approximation 3/3, 1/13\%?_, m= 35" 3 (Gr) us = 0.323209Y — 0.005002Y* and Fig. 4.4-3 shows Blasius’ “exact” I, vs. Y. These two velocity representations will now be compared. Location, Approx. lp, “Exact” Ty, Approx. IIy/Exact Ty Y Eq. 44-18 Fig. 443 a 15 0.468 0.49 0.96 6. 3.0 0.835 0.84 0.99 «4.0 0.973 0.96 1.01 4-4 4A.5 Numerical demonstration of the von Karman momentum balance. a. The integrals in Eq, 4.4-13 are Ae [rete -vedy and h= ft pve — ve)dy Figure 4.4.3 give ff = ve/te0 28 function of dimensionless coordinate, ¥ =yvVou/(va) and v, = Uso in this geometry. Thus, vz = voof! and pdy = \/puz/vadY, so that these integrals take the forms = van fil - f'dY and b = van f (-f)dy 0 0 Numerical evaluation of the integrals over ¥ then gives [ f'(l = f')dY = 0.664 and [ (1 = f)d¥ = 1.73 if an accurate table of the solution is used. The following calculation was made from Fig. 4.4-3: Y 0 1.0 20 3.0 40 5.0 6.0 f 0 0.34 0.63 0.844 (0.955 (0.983 1.00 a-f) 1 066 037 0.156 0.045 + 0.017 _—0.000 sf) 0.2244 0.2331 (0.1309 0.0430 0.0167 0.000 Application of the trapezoidal rule gives the values Ty = (1/2 + 0.66 + 0.37 +.0.156 + 0.045 + 0.017 + 0/2] x 1.0 = 1.74 J, = (0/2 + 0.2244 + 0.2331 + 0.1309 + 0.043 + 0.0167 + 0/2] x 1.0 = 0.65 which agree, within their uncertainty, with the accepted values 1.73 and 0.664. b. Use of Eq. 1.1-2 and the results of a. in Eq. 4.4-18 gives di; Telyeo = Ge + Ok an = 0.65)/pn03,— a = 0.325 / pv /= c. The force in the x-direction on a plate of width W and length L, wetted on both sides, according to the result in 6, is 1 = ow [ Tyz| yao? = 2W(0.325/ ppv) fo =1.30/puv3,LW?, ‘The recommended coefficient is 1.328. L dr 4A.6 Use of boundary-layer formulas. ‘The data for this problem are: W=l0f L=3tt Veo = 20 ft/s From Table 1.1-2 and Appendix F: = (0.1505 cm?/s)/(12 x 2.54 cm/ft)* 1.62 x 10~* ft?/s # = (0.01813 mPas)(10~* Pa/mPa)(6.7197 x 107 Ibm/ft-s/Pa) 1.218 x 107° lbp, /ft-s = p/v = 1.218 x 107° /1.62 x 10-4 = 5107? Ibm /f® a. The local Reynolds number at the trailing edge (x = L = 20 ft) is: Re = Lvso/v = (3ft)(20 ft/s/(1.62 x 10°* ft?/s) 7 x 10° 4. According to Eq. 4.4-17, the boundary layer thickness at the trailing edge is az) =4.04/%% — gpg, [(1-62 x 10-4 1 /s)(3 fe) = 404 2 i]s = 24x 1075 ft ¢. According to Eq. 4.4-30, the total drag force of the fluid on both sides of the plate is Fr = 1328/ppL Wk, = 1.328 y/(0.075 Ibm /ft*)(1.22 x 10- Ibyq/ft-s)(3 £4)(10 ft)?(20 ft/s) = 0.62 Iyyft/s? = 0.019 Iby A-G 4A.7 Entrance flow in conduits. a. With the indicated substitutions, Eq. 4.4-17 gives 1 T, 5D =4.64)/ 2 OAV ae Setting vmax = 2(v) at the end of the entrance region, we obtain the following estimate of Le: vk. (DY? ev) ~ (928 2 D*v) oF vy = 0.023DRe or which is similar to the expression given in §2.3, except that the coefficient is about 2/3 as large. 6. At the typical transition locus 2v./v = 3 x 10° for flow along a flat plate, Eq. 4.4-17 gives 5/2 = 4.64,/ = 4.64(3.5 x 10°)-1/? = 0.00847 and the transition Reynolds number based on the characteristic length 5 is = (3 x 10°)(0.00847) = 2542 For flow in tubes, with transition occurring when 6 = D/2 and with va = 2(v), the latter result gives D(v)/v=2542 as the minimum transition Reynolds number, in fair agreement with the reported value of 2100. . For laminar flow between parallel planes, the method in Problem 4.C gives 6 = B and vmax = (3/2)(vz) at the end of the entrance region. Insertion of these results into Eq. 4.4-17 gives whence 15 Bus) Le Guay = 0.070B?(v-)/v = 0.070BRe with Re=B(v)/v 4B.1 Flow of a fluid with a suddenly applied constant wall stress a. Differentiation of Eq. 4.1-1 with respect to y gives Then using Newton's law of viscosity, we get or, ar, ue 7 Tw ot oy? b, This equation is to be solved with the initial condition that Ty, =0 for £<0, and the boundary conditions that ty, =t% at y=0, and that 1, =0 at f=. c. The solution is exactly as in Example 4.1-1 with appropriate changes of notation, and the solution is given in Eq. 4B. d, To get the velocity profile, we integrate Newton's law of viscosity: or 2, 2 1-er ne Changing variables we get 1 = Dy = eM Nj erty % 1 a-vi/owe = ial Levin H [ae i) The velocity at y = 0 is then T) |4vt t 0,t)= 0 JAM =o, Jt (Ort) = BE = 200 4-3 4B.2 Flow near a wall suddenly set in motion (approximate solution) a, Integration of Eq. 4.1-1 over y gives a, ho Since the velocity gradient is zero at infinite distance from the plate, we end up with Eq. 4B.2-2. b. We introduce the variable’ = y/8(t). Then when Eqs. 4B.2-3 and 4 are substituted in Eq. 4B.2-2, we find -» BD, 00 dpm ef eda= uf Ge whence 4 [Foo.dz=n 1 d gp SIgee=(1-3n+ 4m? )an= -wen(—3407)), soy Then after dividing by pv., and evaluating of the integral, we get ae aw=3y ea (see Eq. 4B.2-5) c. Eq. 4B.2-5 when integrated gives ads=avfidt or 6(t)=V8vE Then the velocity profile is given by _3(_v_),1f_y vi (6) +16) for 08vt 4-9 4B.3. Creeping flow around a spherical bubble a. According to Eq. B.1-18, the vanishing of the shear stress is A(v_) 100, | a(1f_1 dy)\\,1a 1 ay\_ FH) 557 oot ane et) tae mamo 96 foo In the second form, we have inserted the expressions for the velocity components in terms of the stream function from the last line in Table 4.2-1. Next we insert y = f(r)sin? @ and obtain Eq. 4B.3-1. b, Equations 4.2-7 through 10 are still valid for this problem, as well as the values of C;=-4v,, and C,=0. Hence we have to determine the remaining constants in f(r) = C,r“ + Cyr by requiring that Eq. 4B.3-1 be valid at r = R, as well as f = 0 atr = R (Eq. 4.2-3). These boundary conditions lead to C, =0 and C, =4v,,R. Then Eqs. 4B.3-2 and 3 follow directly. c. When the velocity distributions in Eq. 4B.3-2 and 3 are substituted into the equations of motion (Eqs. B.6-7 and 8), we get OP _ of Hes \(R) ae wea) B)' a ae a ) cos and (i ) sino r Integration of these two equations gives P= (=) cos0+F(8) and P= {#28 coso+ air) In order for the solution to be unique, F(@) and G(r) must be equal to a constant. If we require that the modified pressure be equal to py at z= infinitely far from the sphere, we then get RY P=Po ~092-(42\(4) cos d. The z-component of the force acting on the sphere is O"I5(8.-[8, -{P5+z}))|_ R’sinadade 4-10 = -2nR? ["(pcos0+ 7, cos@- 7,9sin8)| _, sin@de where Eqs. A.6-28 and 29 have been used for getting the dot products of the unit vectors. Then we use the result of (c) and Eqs. B.1-15 and 18 for the components of the stress tensor (along with Eqs. 4B.3-2 and 3) to get the three contributions to the force: 2 Fp =~2aR’[f (re-aee-(4=) 8) cw] cos Osin 640 ler = 4aR°pg-+4auRo., Fg, = Ba? -24%e cos) __snawo = muro, [{[ 4 cos" 6) sin @d0=$ mpRo., R ~2nn?fs(-n lr 2 (22) +2e) (-sin 8)sin a40=0 R When these are added together, we get Eq. 4B.3-5. 4B.4 Use of the vorticity equation a. For the postulate v, = v,(x) the vorticity w=[Vx-v] has but one component w, =—dv,/dx. Then at steady state, the y- component of Eq. 3D.2-1 is (v v2) = vw Bs) +(w-V)e, But 2, is postulated to be zero so that the last term drops out. Also the first term drops out, because v, and v, are zero, and v, is postulated to have no z-dependence. Consequently the equation simplifies to d°v, /dx* = 0. Integration gives Wy =4Cx2+Cx+Cy OF 0, /Pemay =D, 2 + Dye +Dy where we have redefined the integration constants. From the three boundary conditions it is found that D, =-1, D, =0, and D; =1, Eq. 4B.4-1 results. Then the z-component of the equation of motion becomes at steady state @o, ax? plv-v)o,=-E suv, or 0 +u dz Knowing the velocity distribution we can evaluate the second derivative of the velocity and get the pressure distribution thus 0 HO, ma —7 dz B ) and For z= L, this gives 2, max = (Po — P,)B”/2uL in agreement with Eq. 2B.3-2. b. Since it is postulated that v, =v,(r), the only nonzero component of of the vorticity vector is w, =—dv,/dr. Then the 6- component of the steady-state vorticity equation is: 4-12 2 (12 (roy) h2ae Fite, 2 dee arr ars)" ae? "az?" 7? 00 {x ap w( 22 + 22) oo a) or PT Ge TO ae This simplifies to a(14/de,)) . 1c? a(t4(r 7 ))-o or 0, =4C\r? +C)Inr+C, This is the same solution found in the soution to Problem 3B.6. When the two boundary conditions and the conservation-of-mass condition are used, we finally get the solution in Eq. 3B.6-2. To get the pressure distribution we use the z-component of the equation of motion, which is o--28, 14 (4) arr ar\” ar When the velocity distribution of Eq. 3B.6-2 is inserted, we get dP suo, (I=n°) +24 In(n) dr R2(1— x?) (1- x?) -2(1+ x?)In(/x) This may then be integrated to get the pressure distribution. 4B.5 Steady potential flow around a stationary sphere a. The boundary conditions are: (i) as r>&, v0.8, or by using Eqs. A.6-28 and 29 as roo, ¥, =0,,cos0 and Uy =-v,, sind (ii) at r=R, v, =0 (iii) at z=0 as ra, P> Py b. Since v=-V@, we have as r— -09/dr = 0.,cos@ and -(1/r)39/00 = -v,, sin® When these equations are integrated we find that ¢=-v.rcos0, Thus we feel that ¢ = f(r)cos@ may be an appropriate trial function. c. We next write the 3-dimensional Laplace equation in spherical coordinates (for a system with symmetry about the z-axis) a 2). 1 (« 2) aa" ar) * Fain "7 36) =° Into this equation we substitute the trial velocity potential and get 1 d( .df ral" £)-o4 72=0 which as the solution faCr+Cyr? since this is an “equidimensional” equation of the form of Eq. C.1-14. d. Application of the boundary conditions then gives soo] goa(z) me o--een ge) fs e. Then from the components of v =—V9. we get Eqs. 4B.5-2 and 3 by differentiation. f. Then from the equation of motion for steady potential flow ov 4 v)= -VP (see Eq. 4.3-2) By integrating the components of this equation we get 4-14 _P =e [1-(8) Joo @ +: 35) oe osc, The integration constant is then obtained from the boundary condition given in (a): C;=-P,-pv2. Then when the modified pressure is evaluated at the surface of the sphere, Eq. 4B.5-4 is obtained directly. 4-15 4B.6 Potential flow near a stagnation point a. At the origin of coordinates (z = 0) the complex velocity dw/dz = ~2092 is zero, which is a stagnation point. b. By taking the real and imaginary parts of the complex velocity, we get from Eq. 4.3-12: v, = 2v9x and v, = -2vpy. c. When % is positive, the fluid is flowing toward the surface y =0in the upper half plane. The magnitude of v9 specifies the speed with which the fluid is flowing: v= Jv? +03 = 2vgr. 4-16 4B.7 Vortex flow a, By using Eq. 4.3-12, we find that aaa) dz Anz 2n\ zz in which the overbar indicates the complex conjugate. Equating the real and imaginary parts gives v,=-L{¥ -- ES) 4 -E a -£(%) oa aetay) tale Soa ay) dale The components in cylindrical coordinates are 2, =0,c080 +0, sin8=0; 09 =-2,sin0 +2, cos0= (4) b. The forced vortex is given by v9 = Qr in Eq. 3.6-37. 4-17 4B.8 The flow field around a line source a. For this purely radial flow v, =v,(r) and the other two components are zero. Then V7¢ = 0 simplifies at once to Eq. 4B.8-1. b. Integration of Eq. 4B.8-1 gives $ =C,Inr +C,. Then, since v, =-do/dr, we find v, =-C,/r. Next we calculate the volumetric flow rate per unit length thus r T={}"0,rd0=-2nC, whence », = = The pressure distribution is then obtained from the radial component of the equation of motion pun oe (LY Tala HP dr dr 2a) Dat) ae Integration then gives (raw=f£) (Ar or p.-P=8(L) AB 4B.9 Checking solutions to unsteady flow problems a. Substituting the solution of Eq. 4.1-14 (or 15) into Eq. 4.1-5, we have to verify that fo gera) oo (e- ter) We have to use the chain rule of partial differentiation along with the Leibniz formula d (pn qty) An_. da? (on ge an) gleam) Sh vale an)( 2 or (I dae) When the definition of 7 is used, the above is found to be an identity. 4C.1 Laminar entrance flow in a duct. @. Calculation of the mass flow rate according to Eqs. 4C.1-1 and 4C.1-2 gives w= plve)W) = paw [ (2) 2) ere omaw [ney = puW [3] + pveW[B— = poe | 5 +3] Equating the first and last expressions for w, we get (with 6 < B and A= 6/B), ve(z) = ‘The following related equations will prove useful in part b: dog _ 3 aA = ")G—ay ae _yy2_9BA dd Ge = (2) G-AP de 2 sonic z) aa] aay dA = Biv.) a3] aa +b. The boundary layer in this system lies between y = 0 and y = 6, so those limits suffice for the integrals in Eq. 4.4-13. Evaluation of the terms in Eq. 44-13 (divided by p) according to the results in a then gives: _ 2vve __Gu(vz) “é ~ BAGB—A) se | valve —ve)dy = ite ft —u?)(1— 2u + u?))du with u = y/6 = £03 of [2u — du? + 2u? — uw? + 2u? — udu = Leon — (4/3) + (2/4) — (1/3) + (2/4) - (1/5)] = Bl.) [zte4] 4 44 ons] 4-1o dee dee (ve — ve)dy = veb = [ic-240m = ves en -1+ i A = 9(vs)?B = ee Bg A SUal With these substitutions, Eq. 4.4-13 gives ove a pe da 2p = ya — Bo? [FAS] Sens seg S09 _ _Blvz)* a 54+ 18A + 454) ~ (3— A)e dr 15 _ Blvs)? dA [544634 ~ (=A)? de Multiplication of both members by (5/3)A(3 — A)/(B(vz)? gives 10 (—¥_) - 84474? aa (o2)B?) ~ (3— A)? de in agreement with Eq. 4C.1-4. c. Integration of the last equation with the initial condition that A = 0 at 7 =0 gives ve 1 [4 6s +73? we ~ 10 J, Bs From integral tables we get the formulas ds lector F [meri + 2] +0 Iwi Fs [e+ oe - 20100 +0) — which yield the definite integrals 4A and the solution oti 5 [in (254) +545] 47[e-2)-0m (254) - 5° =7A+48In (54) +4 4n-% 3 3-4 3-A _ 3-A) | GA~21(3~ A)~63 = 7a +4sin (254 j Sears 3-A\ 27a = 144480 ( +75 in agreement with Eq. 4C.1-5. a. Setting A= 1 and « = L; in the last equation gives 1 27 fa [r +481n(2/3) + #| = (0.1) (7 — 19.462325 + 13.5] = 0.104 in agreement with answer (4). ¢. Application of Eq. 4.3-5 to the region 6(z) < y < B, with vy neglected so that v =v. there, gives Sok +P = constant Insertion of the result of part @ gives 1 2 3 = doe) (5 ) +P = constant Evaluating the constant at © = 0, where A = 0 and P = P,, we get 3 ony (3) p= pte? prtoel (52g) +P = golve)? +P P Po = solve)? [ - (23) or Aw 4C.2 Torsional oscillatory viscometer a, The equation for the rotating bob in a vacuum is just "moment-of-inertia times angular acceleration equals the the sum of the torques." In mathematical terms this is #9, 1a = kOe in which —K@, is the "restoring torque." The solution to this equation is 6n=C, cos ff +C,sin Kt =C,cos gt + C, sinwgt This states that the bob is oscillating with a frequency @, = \K/I, which is called the natural frequency. b. If there is a fluid in the gap, the equation of motion of the bob must contain both the "restoring torque” and the torque exerted by the fluid in the @ direction on the solid surface that is perpendicular to the r direction: Ox ae IB = -kO, —(20RL)(R)(-1, 70 Here (2RL)(—1,9|,..) is the force, and R is the lever arm. Next one uses Eq. B.1-11 to replace the shear stress by the appropriate product of viscosity and velocity gradient. This gives Eq. 4C.2-1. c. Equation 4C.2-3 is obtained from simplifying Eq. B.6-5. d, The choice of the variable x is convenient, since x = 0 at the surface of the bob, and x = 1 is the inner surface of the cup. There are many ways to select the other dimensionless variables, but the choices we have made allow the viscosity to appear only in the parameter called M, Equations 4C.2-11 and 12 follow immediately. e. From Eq. 4C.2-14 we get for the bob angular displacement 6, = R{ 98 exp(i@r)} = (02, + 108, )(coswr + isinwr) = Of, cosMr— 68, sinDt 423 Here 68, and 6%; are the real and imaginary parts of the complex amplitude 6g. The angular displacement of the bob can also be written in terms of an amplitude and a phase shift: 0, = Acos(@r- a) == Acos@rcosa— Asin@rsina When the two expressions for the bob displacement are equated, we find that Acos@r = 0%, and Asin@r = 08, from which we get oR + = for the amplitude and the phase angle respectively. The ratio of the amplitudes of the cup and bob is then |@2|/ 62. ig fi. ig? = d (im) 08 = ~o3 + mae") g. The differential equation for ° is then ep? dx? ° = 0 with 9° (0) = Agia and 9°(1)= AO%,ia The solution to this equation with the boundary conditions is then = sinh ia] ° = Ai cosh [+ - 0 .cosh fe) See Differentiation of this with respect to x gives Eq. 4C.2-16. h, From Egs. 4C.2-12, 14, and 16 it follows that MAID [ID{ go _ 9° cosh 2 * sinh jiayM VM x This equation may be solved for 6%, and Eq. 4C.2-17 results. 4-24 i, When the hyperbolic functions are expanded in Taylor series and the terms arranged in powers of 1/M, we get From this, Eq. 4C.2-18 follows. The results of part (e) can then be used to get the amplitude and phase angle. 4-25 4C.3 Darcy’s equation for flow through porous media. (a) In Case 1, the equation of state is p = po. Hence, Eq. (4C.3-1) gives (Vew)=0 Insertion of Eq. (4C.3-2) for vo, with p = py and g = —V4, gives (v *[¥p + p0V4)) =0 or VP =0 in which P = p+ po® is the “modified pressure” defined in Chapters 2 and 3 for systems with constant p. (b) In Case 2, the equation of state is p = poe#?, Hence, Vp = poBe*?Vp = pBVp, so that pVp = 3 Expressing the divergence term of Eq. (4C.3-1) via Darcy’s equation, we get ~(V # p00) = +2 (V © plVp— eg) K ply © VP — Pa) Kil = =([SV2p— (Ve p? plpy'e—(V *e*9)) Inserting this result, multiplied by 48/x, into the smoothed continuity equation, we get Or HB. op HV 099) (©) In Case 3, the equation of state is p = pop. Hence, Vp Vp = (1/po)Vp. ‘Then Eqs. (4C.3-1,2), with the pg term neglected, give op eR = (8 ol-Z vel) poVp and k =+H(Vep—yv, +( Soars p) = (Vepy, pa ee k 1 = AV WS? aaa (5) Kya =;V’ up’? 4 Hence, 2epupo Op _ 22 nT (A) In Case 4, the equation of state is p = pop”, giving p = (p/po)!/™. Thus, vp a dpetle plasma P= Po Pl Ve whence a Bim ting pm =— 7 Po ee so that fa =(V # pvo) = +e opl™Vp) & eB gMm eg g [ag glmtnyyin Po [av With this result, Eq. 4C-1 gives 2 glmta) im 42 4C.4 Radial flow through a porous medium a. For the radial flow (in cylindrical coordinates) of an incompressible fluid Eqs. 4C.3-1 and 2 become ld 0=-2 7 (%,) and Integrating the first of these equations gives vp, = C,/r. Substituting this into the second equation and integrating gives Ciinr+G, =-FP or D,Inr+D,=-? where new constants of integration have been introduced. These constants are determined from the boundary conditions: B.C.1: D,InR, +D. B.C.2: D,InR, +D, P, P, When the integration constants have been determined, the pressure distribution is found to be P-P, __InG/R,) P,-P, In(R,/R,) b. The velocity distribution is then given by using Darcy's law KdP K Por Sarr 2-P,) 1 __1__« (P.-F) in(R,/R)r ur n(R,/R,) c. The mass rate of flow through the system is 2axh(P, -P,)o w= phoo,(R,)2aR\h= aint TR 4-28 4D.1. Flow near an oscillating wall The problem is to solve Eq. 4.1-1, with the initial condition that v, =0 at f=0, and the boundary conditions that v, =0 at y=o and v, =) coswt at y=0. When we take the Laplace transform of Eq. 4.1-1 and the boundary and initial conditions, we get po, = vee with 9, This ordinary differential equation is easily solved with the boundary conditions to give B, = 0)? exp(\p/vy) p+? This may be inverted by using the convolution theorem, or else by consulting a table of transforms (see, for example, Formula #11, on p. 246 of Vol. 1 of A. Erdélyi, et al., Table of Integral Transforms, McGraw-Hill, New York (1954)). The use of the table leads directly to Eq. 4D.1-1. 4D.2 Start-up of laminar flow in a circular tube a. The partial differential equation, initial condition, and boundary conditions are am PaPs yl 2,90) Oa OL erst with v,(r,0)=0, 0, (0,t)= finite, and v,(R,t)=0. We now introduce the following dimensionless variables HE pR? g= Uy . © (Po—P,)R?/4nL’ Then Eq. 4D.2-1 is obtained, along with the initial and boundary conditions: ¢(E,0)=0, 9(0,1)= finite, and 9(R,t)=0. b. The asymptotic solution is obtained by setting the time derivative equal to zero and solving the ordinary differential equation with the boundary conditions. Then the partial differential equation for ¢,(E,7) is 1 1 Of 5 AH ar EAE? OE with ancillary conditions: ¢,(E,0)=.(€), 9,(0,t)= finite, and ¢,(1,7)=0. We now try a solution of the form @,(&,7) =(&)T(r). This leads to two ordinary differential equations Tala which have as their solutions aT ep a and g T=Cyexp(-a*z) and E=CyJo (aE) + Ca¥ o(aE) in which the Cs are constants, and J, and Y, are zero-order Bessel functions. Since Yo is not finite at the tube axis, we must set C, equal to zero. Since = must be zero at the tube wall, this will occur only if 4-20 Jo(@) =0.This will happen only at o%,,0,,0r3,---, that is, at the zeroes of Jy. Hence there are many solutions to the E-equation that will satisfy the boundary conditions: =, =C,,Jo(@,£)- Therefore, the general solution to the partial differential equation must be $,= DB, exp(-a2)Jo(a,2) mL The constants B,, are to be determined from the initial condition, (1-€")= EB ,Jo(au8) This is done by multiplying both sides of the last equation by Jo(Om€) and integrating from 0 to 1: [plolain8)6(1- £2) = 2B, [TENT mS )EAE Because of the orthogonality properties of the Bessel functions, the only term on the right side that contributes is the term for m = n. The integrals may then be evaluated using some standard relations for Bessel functions. This gives 8 SiGe) =B,-3[i(@,)) from which Bn = 7 (aa) and this leads directly to Eq, 4D.2-2. 4-3) 4D.3 Flows in the disk-and-tube system a. If the tangential component of the velocity depends only on rand z, then the equation of motion simplifies to # (720) )+ 32 =0 We try a solution of the product form: v9(r,) = f(r)g(z). When this is substituted into the partial differential equation above, and the resulting equation divided by f(r)g(z) we get 1d(1d/)__ 10g padre a) aa The left side is a function of r only and the right side a function of z, only and therefore both sides must equal a constant; we call the constant ~c?. Then we have two ordinary differential equations to solve: d(id a EL) +er=0 and Si-ee=0 The second equation has the solution g = Acoshcz + Bsinhez, where A and B are constants. The requirement that the velocity be zero at z = L, gives sinh{cL(1-[z/L])] coshcL g where B is a constant. The first equation can also be written as af id 1 etree which is now recognized as a Bessel equation of first order, with the solution f = MJ,(cr)+NY;(cr), where M and N are constants. But since Y, becomes infinite at r = 0, only the J, term is needed, Since f = 4-32 0 at at r= R, we must have J,(c,)=0. There are infinitely many c, that satisfy this equation. Hence the solution is of the form sinhe,L(1-¢) coshe,L 29(6,6)= DB h(cn8) The coefficients B,, are determined from the initial condition RQE= ZEN (r5) This is done by multiplying the equation by J,(c,£)é and integrating over € from 0 to 1 and making use of the orthogonality relation. This gives ROPT (enG)E2dE = 3B, Jian i(CmE EAE or Jalen) _p 2 ___2RQ ROM = Bm Aland) OF Ba = TT Thus the steady-state velocity distribution is vplé.t)=2ROS Led) sinhe,L(1~<) naiy]a(C,) — coshe,L 453 4D.4 Unsteady annular flows For the tangential annular flow (part (b)), the equation of motion for v9(r,t) is pray 2f12 Pan # 3} al” )| with 09(r,0)=0, 09(kR,t) = KRQ,, v9(R,t) = RQ,. We introduce these dimensionless variables: Then the partial differential equation for $(£,7) becomes a SE x00] with 6(€,0)=0, ¢(K,t)=-Ka, o(1,t)=1- a. For the steady state (i.e., at infinite time), we have the solution 1-a(1- x’) 2 yy 1 . (Hea) peas- at Hence the time-dependent solution is 9(8,7)= 0.()- 6(6,7)- Here ¢,(&,t) is the transient contribution, which satisfies the partial differential equation for 9(E,7) , but with boundary and initial conditions: 9,(x,t) = 9,(1,7)=0 and ¢,(E,0)= 9.(€) Application of the method of separation of variables with (6,7) = f(€)3(z) gives 434 1, pd (1d gat . ral ge) where -b* is the separation constant. Thus we arrive at two ordinary differential equations which can be solved (the second- order equation is a Bessel function ). Since this is a Sturm-Liouville equation, the complete solution is a sum of eigenfunctions multiplied by the appropriate exponential function: 06:1) = BC,2(b8)e% in which Z,(b,£) is the superposition of the two solutions to the Bessel equation: Z, (OnE) =Jr(Bn$)¥s (On) — Jan) (ba$) These functions satisfy the boundary conditions in that Z,(b,£)=0 at € =. The conditions that Z,(b,¢)=0 at€=1 determine the eigenvalues b,. The C, can then be determined from the initial condition, which is 9-(6)= EC.2a(b6) When this equation is multiplied by Z,(b,)édé and then integrated over the domain of interest, we get * flZa(by5)) Ga 3[Z5(b,)- 725 (0,)] The integrals are performed by making use of a mathematical handbook. The expression above may be simplified by using 4-35 i. The expressions for A and B ii. The defining equation for the b, iii, The relation Jo(x)Y, (x)= J,(x)¥(x) = -2/ax Then it may be shown that 2_Ii(bnk) 21 Zo(b,) =— pa o(be) = Talbs) and Zolbok) =~ and also my(b, )[(L- aI (On) + 2X, (b,,)] [FG.«)-F7(,)] in which or is the dimensionless angular velocity. Then the complete expression for the transient behavior in tangential annular flow of a Newtonian fluid is 1-a(1- x? we-n-{ HS) (i ai (bn )[(1= @)Ja(byke) + Is (bn)]Za (bu) ete [i G.")- J? Gn)] Complete tables are given in the original reference. as well as some typical velocity profiles. Also, a Laplace-transform solution is given for small times, for which the expression above converges too slowly to be of much value. C,(K, a2) 4-36 4D.5 Stream functions for three-dimensional flow. (a) The divergence of the curl of A is 5,(0/0z;)[V =A]; which gives, according to Eq. A.4-10, DW0/ae1) DY ess4(0/025)Ae = OY Deise a 7 7 Er i oj ok " PA Ag =v Len ast ~ eal rr Thus, the mass flux function pu = [Vx] satisfies the continuity equation for steady flow (or for unsteady flow with constant density). Here use has been made of the relations eijx = —cyik and eiik = eine = 0 contained in Eq. A.2-15. ‘The divergence of the product expression for pv is (¥ ((V1)(Vv2))) = © (ok «fragt “eet vy Ov: = z= rd . (6;x6) (@ oe) 8 (dry dy, = LL DG nbirgy, (3) = VY Vein ds(Gjvr deve) = DO Weise OiitrOeve + jp diev2) cron = €123(A2vrOsv2 + Arr Aso) + €231(Aosth1 Oi Ho + Asi 92142) + €312(Os1 hi Povo + Ort 522) + €132(AistrOep2 + Asti Dior) + €321(As2vh1 ho + O21 9s1}2) + €218(Gor 1 Osho + Orth I2sib2) = Or2rb1 Os ho(Er2s + e213) + Oey isthe (Erzs + e321) + dost 91 Y2(€ns1 + e321) + Osth1 Der Ho(E2s1 + €132) + Ositrdova(Eai2 + e132) + Orr Os2va(Esr2 + €213) =0 Thus, the product expression for pv is divergenceless. Here Eq. A.2-15 and the symmetry relations 3j; = 9: have been used. Stream functions of this form have been used by several authors; see footnote 1 of §4.2. 437 (b) In each of the coordinate systems shown in Table 4.2-1, the two nontrivial velocity components for incompressible flow are the corresponding components of curl(—534s)/p = curl(—63%)/hs), as may be verified by use of Eqs. G, H and I in Tables A.7-1,2,3. Thus, for Cartesian coordinates with v, = 0 and no z-dependence, hy =h, = 1, and Eq. A.7-18 (with v read as 63y/hs) gives the velocity components and vy = curly(—A)/p = +22 ve = eurle(—A)/p x For cylindrical coordinates with v; = 0 and no z-dependence, hs = A.7-18 (with v read as 55%/hs) gives the velocity components = Land Eq. 1 v, = curl,(—A)/p = 1% and vp = curle(—A)/p = +58 For cylindrical coordinates with vg = 0 and no 6-dependence, hs = he =r and Eq. A.7-18 (with v read as s—)/hs) gives the velocity components v, = curl,(—A)/p = -128 and v, =curl,(—A)/p = 41% For spherical coordinates with vg = 0 and no ¢-dependence, hy = hg = rsin@ and Eq. A.7-18 (with v read as 634/h3) gives the velocity components = =--1_% eo) vr seurl(-A)/p=——G 5p and ve =curl A)/p = G5 (c) Consider two surfaces, th (#1,r2,23) = Cy and o(r1,72,23) = C2, which intersect along a line £. At each point on £, the vectors Vi; and Viz are normal to both surfaces, and the velocity vector » = [(Vi1)x(V¥2)] is consequently tangent to £. Thus, the intersection of any such pair of surfaces is a streamline. In Fig. 4.3-1, we may choose y1 = V(r, 8) and pz = 2; the resulting streamlines in a plane of constant tz are shown in the figure. (A) Read v in Bq. A.5-4 as the vector A whose curl is the local mass flux pu. Then the net mass flow through S is [om [Vxa)as = fe * pv)dc Is lc for steady flow, or for unsteady incompressible flow. A no-slip condition » = 0 on C requires [Vx A] = 0 there, but this derivative condition does not require the vanishing of A nor of the net mass flow. 438 5A.1 Pressure drop needed for laminar-turbulent transition. ‘The minimum value of Re= 4w/(Dp) needed to produce turbulent flow in a long, smooth tube is about 2100. Poiseuille’s law, Eq. 2.3-21, holds until this critical Re value, giving Pr)RYp a for Re or an rte «(Hy 1d°F a( #) 1a°F an? an dn) 2dn? f. The final result in (d) can be integrated at once to give d 2 Cc or —F? =fF yoc ary : 7 ant ae But at 7 =0, both F and and its second derivative are zero so that C’ g. Integration of the result iin (f) gives 5-10 dF dF pete “. an +C or an h, Integrating this last result gives F_dE 1 E li peaee= Sian or ~ arctan == 0 or F=-CtanhCn Then the time-smoothed axial velocity is - a nr SB [ie aF1__ [J ari ~ pa ae _[fre [t_ 2 ci ~ cnr i. The mass rate of flow at plane z is wf pty Poms aa aa “sech*Cndn-4Az =Wpz wee 20h ale 2CWpz pWz = 248A JoWz Thus we see that the mass rate of flow increases with the distance from the exit of the slit. This is a result of the fact that additional fluid is dragged along by the jet. Sl 5C.2 Axial turbulent flow in an annulus a. We divide the annular region into two parts, depending on whether or not r is less than or greater than bR: Region <: aR: bR. | _ |(Po-Pi)R = 2 Region >: way iF “1 aIp (1-0?) =0.V1-0 where v,, = \(P)—®,)R/2Lp. Then the velocity distribution in the two regions may be obtained from Eq. 5.3-3: Region <: oe Jove ae ie Ju{ 2) +4 oe K v my Region >: et wf Se, pet n( =) +P eK v ey y being defined as y =r —aR in Region . b. The continuity of the velocity profile at r=bR gives a relation between A‘ and 4”: 5-12 Jul(®=)o-m } =m @)o-miF Jer K v a rs v c. The mass rate of flow through the annulus is w= 2npfedsrdr + 2npfy Ozrdr The first integral is: 2npws iz: (2 afar = 2npvs gle 1( 2} vay any ~2npoi( Z) pretest ainsea| x +t = 2now'{ 2) [2 Sine) (M2 ina x) poet = ar*po;| 2 [(b-) +2a(b- aie af 225) +4-[(0-a) +20(0-0)]-2] 0-0) +20(0-.)]] say eft ohn The second integral in the expression for the mass flow rate is then: 2npv? fal em( 2) a 5-B = 2190? ge eu) + re ~y)dy ~anme 2) ge ms 2a = ano 4) Fas ye vw Bea be - aoe | 2 0-mma-»f )-c-»| -}0-wrma- of 22)-50-0] 424 (1-b)- 4 (1-b) ] = aR%po2 (1-0? [mao 2) +e (3 til We now combine the two integrals and use the result in (b) to eliminate A< in favor of A”; furthermore, we use the result in (2) to introduce »,,. This leads to: w= nR°po., {af Sm(S0-eyi-e*) +x |-a} in which (1-0)( Ree /v) Aza? (0? a2)” +(1-b2)” are -o” 2(Lagt of AE) The expression for A does not agree with that of Meter and Bird, and we conclude that their expression is incorrect. SV 5C.3 Instability in a simple mechanical system a. The centrifugal force acting on the mass is mQ?r= mQ?Lsin@. The gravitational force acting downward is mg. These forces must have a resultant in the direction of the rod, and therefore ne tano=22USING ggg mg QL If the angular velocity goes to zero, it would appear that cos @ would 1 b. However, this formula describes the relation among the various quantities when the system is rotating and @>0. When as Q decreases, the right side of the equation attains a value of 1 when Q=Q,,,, and then cos@=1 and @ is zero. For for value of Q less than its "threshold value", the value of @ remains at 0. When one starts up the system from rest, @ will always be zero. However, if Q>Q,4, and there is any disturbance on the system, then the system will move up to the stable curve (given by the equation in part (a). It has to be understood that the graph we have given is only for the steady state, and that to understand the system fully, it is necessary to examine the full unsteady-state equation. c. According to p. 12 of L. D. Landau and E. M. Lifshitz, Mechanics (Pergamon, 1990) the Lagrangian for this problem is L=iml?(6? + Q? sin? 6) +mgLcos0 Then Lagrange's equation of motion da a_ ata6 00 gives for the system we are considering the following equation of motion PO are . mL = mQ"Lsin Ocos 6 —mgsin GAS d. Consider the lower (unstable) branch in the diagram. For very small perturbations (6,) to the steady state (6), we have then sin 9 = sin 8, = 0, and cos@ ~1. Then the equation of motion becomes #0, _(o2_8 av -(2 a Now we try a small perturbation of the form 0, = AR{e} When we substitute this function into the differential equation we get o, = ti/Q? ~ (g/L). If Q? < g/L, then both roots @, are real and 6, oscillates. If Q?>g/L, @, is positive imaginary and e“ will increase indefinitely with time. Hence the branch @ = 0 is unstable with respect to infinitestimal disturbances. e. Next we consider the upper branch for which 0s 6) = g/Q7L and sin = /1-(g/Q°L)*. Then the equation of motion becomes 2, mo = mQ?L(sin 8) + 8, cos 4 )(cos 8 ~ 8, sin 8) —mg(sin 6) + 8, cos 6) =-mQ?L6, sin? 05 where we have neglected terms quadratic in 0,. Hence the equation of motion becomes ; Et + do(O? + (@/L) (0-9/0), We now try a solution of the form 0; = AR{e~} and get (2? + (g/L))(2? -(g/L)) For the upper branch, Q? > (g/L), and hence both quantities @, are real. Hence the system is stable to small perturbations. 5-16 5-17 5D.1 Derivation of the equations of change for the Reynolds stress Multiplication of the ith component of Eq. 5.25 by 0; and time-smoothing gives for constant p gives (in the Cartesian tensor notation of §A.9, with the Einstein summation convention and with the shorthand notation d, = 0/dt): 2019.0, = 070 p" —p(07,5,0) + 019,010, + 070,0(07) + 1079,0,0) ( (2) @) @) 6) (6) Then we write the same equation with i and j interchanged. When the two equations are added we get, term by term: (1) of9,07 + 970,27) = pa, (a70; F O77) -p( 370,07 + 79,07) The second term on the right side is the negative of the left side. The first term on the right side can also be written as 20,0107. Therefore the left side is just pd,777) @) (ap AP") 8) -0( 79,5 D,0} + V; ID; =-1(5,0,0707) = ~(870,9,07 + 978,0,07) (used 5.2-10) 4) P(A MT, +7; I,O7, Ufo, HG, + 070,9,0,) (used 5.2-11) ©) ~0f 79,0f07 + 05,0707) = —p( 9.070707 - 070,01) -p( 270, 9,0; + 070,0,07) Note that the third term is zero by Eq. 5.2-11, and that the second and fourth terms cancel giving -p(0,0;0;27) ©) — +n (070,9,05 + 079,9,07) Combining the above gives: O0,p" + 0,0,p") -p(5,0,0,01) -p( 110,9,5, + 070;,0,0, ) —p( 9,0(0707) +u(0f0,9,07 + 079,9,07) or pA, 070; +0(B, 9.0707) = -p( 070, 9.5, + 07,947, )-o(9,7,0777) ~(0aip" + Fp’) +u(07,9,07 + 779,0,07) The two terms on the left side are the substantial derivative term in Eq. 5D.1-1. The remainder of the terms are set out in the same order as in Eq. 5D.1-1. Bi 5D.2_ Kinetic energy of turbulence Taking the trace of Eq. 5D.1-1, we get p2.Ww)=-2(7v:V9)-p[V-W-wv") Dt -2(v’- Vp’) + 2n(v’-V2v/) Modify the third term on the right as follows: -2(0- Wp) = 210-9) +209 (9-0) The last term on the right is zero according to Eq. 5.2-11. Then divide the first equation above by 2 to get Pipe? =-p(vv-v0) -(V-Gpo7)v') -(V-pv)+u(v’- vv’) which is Eq. 5D.2-1. For an interpretation of this equation, see pp. 63 et seq. in the book by Tennekes and Lumley cited on p. 176. 520 GA.1 Pressure drop required for a pipe with fittings. ‘The average velocity at the given conditions is (o) = Mule) _ (097 m*/s) “gD? ~ (x)(0.25 m)? 40.1 m/s and the Reynolds number is Div) .99 x 10° (0.25 m(40.1 m/s)/(1.0037 x 10-° m?/: Thus, the flow is turbulent, and Fig. 6.2-1 gives £=0.0020 for hydraulically smooth pipe. The total equivalent length of the pipe and fittings is L, = 1234 m of pipe + (4)(32)(0.25) m equivalent for 4 90° elbows + (2)(15)(0, 25) m equivalent for 2 90° elbows = 1274.5 m ‘The required pressure drop, according to Eq. 6.1-4 with L replaced by Le, is then (po ~ px) = 225. tuys a 908 kg/m*)(40.1 m/s)?(0.0020) =3.3x 107 Pa=4.7 x 10° psi GA.2 Pressure difference required for flow in pipe with elevation change. In this problem the pipe diameter is (3.068 in.)/(39.37 in./m) = 0.07793 m, the mass flow rate is w = (18/60 gal/s)(3.7853 lit /gal)(0.9982 kg/lit) = 1.13 kg/s, the average velocity is 0.237 m/s, and the Reynolds number is _ Dlv)p _ (0.07793 m)(0.237 m/s)(998.2 kg/m*) _ ‘ LOST; (1.002 x 10-5 kg/m-s) SEH From Fig. 6.2-2 we find that for this Re value, f = 0.0066 for smooth tubes. Hence, Le Po pL = ~pa(ho ~ hn) +258 plo)f = ~(998.2 kg/m)(9.807 m/s*)((—50 x 0.3048/V2) m) + 2{(50 x 12/3.068) + (2 x 15)] - (998.2 kg/m*)(0.237 m/s)*(0.0066) 1,055 x 10° Pa+ 167 Pa 055 x 10° Pa = 15.3 psi ‘The corresponding calculation in terms of Ibm, ft and s, neglecting the small friction term, is Po — pL = (62.4-x 0.9982 Ibm /ft*)(32.174 ft/s?)((50/ V2) ft) = 7.09 x 10* Ibm /ft/s? = 15.3 psia G2 6A.3 Flow rate for a given pressure drop. ‘The quantities needed for this calculation are as follows, in units of Ibm, ft, and s: (0.25 Ths /in®)(144 in? /ft?)(32.174 Tom ft/s?-Iby) 16 x 10° Ibm /fts?; D=05 ft; = 62.4 Ib /it?; L = 1320 ft; = 6.73 x 10 lbp, /ft-s Po- PL Hence, the solution must lie on the locus ney f= Be |= 00P 2Lp _ (0.5)(624)_[(.15 x 10-8)(0.5) ~ 673 x 10-7) 2(1320)(62.4) = (4.64 x 10*)\/3.52 x 10-3 = 2.74 x 10° Method B: The last equation gives a straight line on the logarithmic plot of f vs. Re, passing through f = 1 at Re= 2.74 x 10° and through f = 0.01 at Re= 2.74 x 10, and intersecting the f curve for smooth tubes at Re= 3.6 x 10. Hence, the average velocity is = 3.6 x 104 "= Dein) 464x108 and the volume rate of flow is .78 ft/s 2 qu _ (3.1416) (0.57) —_— = 68 U.S. gal/hr ).78) = 0.152 ft/s ‘Method A gives the same result if the plot of f vs. Re/f is accurately drawn, 6A.4 Motion of a sphere in a liquid. The force of gravity on the sphere is Feray = mg = (0.0500 g)(980.665 cm/s?) = 49.03 dynes ‘The buoyant force of the fluid on the sphere is Fhuoy = (4/3) "Rpg = (4/3)(0.25 cm)$(0.900 g/em*)(980.665 cm/s*) = 57.77 dynes a, The resultant upward force is Fruoy — Farav = 57.77 — 49.03 = 8.74 dynes and is balanced, at steady state, by an equal and opposite drag force Fk = 8.74 dynes. b. The friction factor is defined by Bex (@R\()p02)F ‘Thus, for this system, A __ 8Fe P) Gre) Dine 8(8.74 dynes) * 57(0.500 cm)?(0.900 g/em*)(0.500 cm/s)? = 3.95 x 107 c. From Fig, 6.31 we see that f is very close to its creeping-flow asymptote, 24/Re. To the same approximation, Dvoop 24/f = 0.061 Hence, _ Dvsopf _ (0.5 cm)(0.5 em/s)(0.9g/em*)(3.95 x 10?) 8 24 = 3.7 g/em-s = 3.7 x 10° cp 4 GA.5 Sphere diameter for a given terminal velocity. a. Method A: Replot the f-curve of Fig. 6.3-1 as f/Re (which does not contain D) vs. Re. Then from this curve we can find the value of Re for any calculated value of f/Re, and determine D as Rep/ pvc. >. 4, Method B: On the log-log plot of f = f(Re), plot also the locus f (f/Re)Re, which will be a line of slope 1, and find the desired Re at the intersection of the two loci. This method avoids any need to prepare an auxiliary plot. b. The data of Problem 2A.4 give Yoo = (1 ft/s)(12 x 2.54 em/ft) = 30.48 m/s p= 0.045 Ibm /ft3)(453.59 g/Ibm)(12 x 2.54 cm/ft)-* 2x 107 g/em=? 2 g/em* B= 26 x 10~* g/ems 180.7 cm/s? from which we calculate Ht (Pa=P 1 re= 308 (854) '980.7)(0.00026) “3 1.203, ( = 278 7.2 10~ 72x 10-4 We therefore draw a line of slope 1 through f = 27.8, Re= 1 on Fig. 6.3-1. This line intersects the f vs. Re curve at Re= 0.95. The particle diameter is then calculated as “Rew _ (0.95)(2.6 x 10~*) a Voop (30.48)(7.2 x 10-4) (0.0112 cm)(10* microns/cm) = 112 microns ¢. Here vce is 10 times larger, giving f/Re= 27.8 x 10%. This locus intersects the f(Re) curve at Re= 75. Hence, at this gas velocity the diameter of the largest particle that can be lost is _ _(75)(2.6 x 10-4 ~ (30.48)(7.2 x 10-4 D (0.89 cm)(10* microns/cm) = 0.89 x 10* microns GA.6 Estimation of void fraction of a packed column. ‘The superficial velocity is _ (244 Ib/min)(1_min/60 s)(453.59 g/1b) _ “(1.2865 g/em*)(146 in?)(2.54 em/in)? v% 1.522 em/s According to Eq. 6.4-9 (the Blake-Kozeny equation, developed for laminar flows), 150p.Lv9 (=e? ~ Daa 150(0.565 g/cm-s)(73 x 2.54 cm)(1.522 cm/s) = ~(0.2 ein)?(158 psi)(68947 dynes/cm™-psi) = 0.0549 Solving this equation for ¢, we find ¢ = 0.30; hence, 0.2 em)(1.522 em/s)(1.287 g/em®) 1 _ 4 og (0.565 g/em-s) (2= 0.30) ~ which indicates (see Fig. 6.4-2) that it was appropriate to use the Blake-Kozeny equation here. 6A.7 Estimation of pressure drops in annular flow. 4. The definition of f in Eq. 6.1-1 gives Fe rR = 1?)(Po-Pr) _ R= )(Po = Px) f 2QnRL(1 + K)p(z)2/2 2eRL(1 + w)p(vz)?/2 Lp)? Equations 6A.7-1 and 3 give 6» ~ f DA =«)(o:)p Insertion of the previous result for f gives K Whe)? BL) ~ RO =*)(Po- Pr) DO=«):)p R21 —«)?(Po— Pr) Equation 2.4-16 gives K .) Rfl-«6t 1-.' (Po-Pr) Bul li =r ~ in(i/n) Combining the last two results, we get the relation 1 [lect 1-0? x [ES all ar needed to make Eqs. 6A.7-1 and 3 consistent with Eq. 2.4-16 for laminar flow. The K values given by this formula are in excellent agreement with those tabulated on page 194 and recommended for turbulent flow as well. . The data for this operation are 1 = 1.139 ep(=mPa's) = 7.66 x 10~* Ibm, /ft-s; 62.2 Ibm /ft; D=15in=125f; «=Gin/ISin=04; G =3.801; H=0131; “K =06759; — w/p = 1500 ft°/s; 4(w/) 4(1600 ft/s) @.) = =[D® — (nD) ~ 7.25? — 057 2] 388 ft/s; Re, = KO Ne # = 0.6759 1:25 £)(1 = 0.4)(988 ft/s)(62.3 Ibm /f*) 4g ag? (7.66 x 10-4) Equation GA.7-2 then gives: SH = 3.801 logyo(1.6 x 10°F) — 0.131 “ Solving by iteration, we get f = 0.00204. The longitudinal pressure gradient in a horizontal flow is then (po = pr) _ 2fp(@:)? TL ~ D(i=n) _ 2(0.00204)(62.3 Ibm /ft*)(388 ft/s)? (2.25 ft)(1 — 0.4) = 5.06 x 10" poundals/ft® = 10.9 psi/ft ¢. The mean hydraulic radius for this system is S_7D(1-.)/4 _ Dlr) R=7-—Spa-n) 7 4 and the correspnding Reynolds number is _ ARa(@.)e _ DO = n)(os)e = Re/K = 2.37 x 10" # # Equation 6.2-15 then gives f = 0.00181, which is 0.885 times the value found in part (b). The predicted pressure gradient is reduced correspondingly, giving (Po =P) _ 4.48 poundals/R? = 9.7 psi/fe 4-8 6A.8 Force on a water tower in a gale. ‘The data for this problem are: D = 40 ft for spherical tank; vo = (100 mi/hr)(5280/3600) = 147 ft/s; p= 0.08 Ibm /ft3; = 0.017 ep = 1.14 x10 Ibm /ft-s ‘The Reynolds number for the tank is Dvzop _ (40 ft)(147 ft/s)(0.08 bm /ft*) Rem = Ad x 10-5 Ib /ft-s) -1 x 107 ‘The friction factor is approximated as 0.5 by extrapolation of Fig. 6.3-1. The horizontal force of the wind on the tank is then A= PIG = ES") (0 08 Ib, /ft2)(147 fs)) (05) = 5.4 x 10° poundals = 1.7 x 10* Iby = 7.5 x 10* Newtons GA.9 Flow of gas through a packed column. For this compressible-flow problem, we write Eq. 6.4-12 in differential form with v9 = Go/p: 2 = x y59HGe A= 2) dz pd, Inserting the ideal gas formula 1G 4D, é ) pM RT to z= L, we get the following implicit expression for p and integrating from 2 = the superficial mass flux Go: wGo(i—e? | 7TEA—)) 7 _ Moa _ip 0 tap, [EO app Pi) ‘The terms are then calculated for this system in egs units: #Go (1—e)* woo aE _ yep (1-495 x 1074 g/em-s)(Gp g/em?-s) (1 — 0.41)? SO SEC one eae (05 8048 crm) = (753.4Go) g?/em*-s? 7G} (1-6) ap, @ © _ 7(Go g/cm*-s)? (1 —0.41) 4 (254/16cem 041° = (15820G3) g?/cm*.s? Mp. 2 44.01 g/g-mol app? — Pb) = &3qaBI <0? x 300 gem? /s*g-mol) x [(25 x 1.0133 x 10°)? — (3 x 1.0133 x 10°)? g?/em?-s*] = (1.116 x 10°) g?/cmt-s* Combining these results, we get the following quadratic equation for Go, 15820} + 753.4Go = 1.116 x 10° (5.5 x 30.48 cm) which has the roots Gq = BAG VTHB-AG? F ATSS2OV(T- TIE x 10°) y= 2 x 15820 53.46 + 265746.2 2 x 15820 ‘The positive root is 8.375 g/cm?/s. the mass flow rate is then w = (n/4)D*Go = (0.7854)(4 x 2.54 em)?(8.375 g/em?/s = 679 g/s An identical result for w is obtained if p is assumed constant at the value p = (Po + py)M/RT. 6-l0 6B.1 Effect of error in friction factor calculations The Blasius formula for the friction factor for turbulent flow in circular tubes is Eq. 6.2-12: _ 0.0791 ~ Re¥# ii We now form the differential of f, thus 10.0791 4 Re¥# dRe af= and regard the differentials as the errors in the relevant quantities. This result may be rearranged thus 0.0791 1 1 = —1_dRe=-f-—-dRi a= —Rete GRe R= STR tke From this we find af __1dRe f 4Re The quantity df/f represents the fractional error in f, and dRe/Re is the fractional error in Re. Therefore, if the Reynolds number is too low by 4%, then the friction factor will be too high by 1%. g-ll 6B.2 Friction factor for flow along a flat plate a. Equation 4.4-30 gives the kinetic force acting on both sides of the flat plate for laminar flow: 1.328)/ouLW?03, and an appropriate Reynolds number for a plate of length L is b. For turbulent flow, the force acting on the plate is given by Eq. 6B.2-1 as F, =0.074p02WL(Lv_p/n) Then the friction factor is _F, _0. O74pvEWL(Le.p/st) AK Here we have used the same definition of Reynolds number as in part (a). 6-12 6B.3 Friction factor for laminar flow in a slit a. The mass flow rate through a slit of width W, thickness 2B, and length Lis, for laminar flow 2(Po-P,)B Wo _ 3 HL (0, )(2BW) and the kinetic force acting on the walls by the fluid is F, =(P)-P,)(2BW) Then the friction factor is FL _(Po-P,)(2BW) AK” @WiV(to(0.)) We now replace one of the (v,) in the denominator of this expression by using the above expression for the mass flow rate. This gives fe (Po-P,)(2BW) 3k - 2B _2 (WL Ee(e.)) (Po-Fi)B 2BKe,)o/u Re b Next we try using the mean hydraulic radius empiricism suggested on p. 183. For the plane slit, R, is S__2BW Z 2W+4B- = In the last step we have made the assumption (consistent with the derivation of the velocity profile for the slit) that B< 1 (Ro — Ri) ‘The mass flow rate is w = (241/60 U.S. gal/s)(231/1728 ft /gal)(62.4 Ibm, /ft?) = 33.5 Ibm/s and the average flow velocity is =-¥__vle () = 55 = a(RE — RB) __ (83.5 Ibm /s)/(62.4 Ibm /ft*) = aT Gray ne) = 0015 ls (ii) The Reynolds number is (see Eq. 6.2-18): Rew Rave _ (Ro Raw ___ dw pw (RR RB (Ro + Ride _ 2(33.5 lbm/s) © x((7-+3)/12 ft)(1.14 x 0.000672 1b, /ft-s) = 3.34 x 10% Hence, the flow is turbulent and f = 0.0059 according to either Eq. 6.2-15 or Fig. 6.22. (i pound From Eq. 7.5-10 we calculate the work required W in ft-poundals per ft? /s?: W = o(ta—m) + ROPES = aha — ha) + (v)? f L (Re — Ri) = (82.2 ft/s?)(5 ft) + (0.615 ft/s)? (yn) (0.0059) 32 = (161 + 0.175) = 161.1 ft?/s? The power output required from the pump is then wW = (33.5 Ibm/s)(161.1 ft?/s? = 5.4 x 10° ft-poundals/s = 1.9 x 10? ft-poundals/hr = 0.31 hp = 0.23 kw TA.8 Force on a U-bend. According to Eq. 7.2-3, if the «-axis is taken in the direction of the downstream unit vector u; at S1, which is the negative of the downstream unit vector wz at Sz and perpendicular to the gravity vector g, then the force exerted by the water on the U-bend has the z-component (52 © Fy—s) = (viw + prS1) (52 © u1) — (vow + prS2) (62 # Uz) + meor(5z 9) when the velocity profiles at Planes 1 and 2 are approximated as flat. With (6, ur) = 1, (6: © 42) = -1, (6, © g) = 0, S; = Sp, and vy = v2 = w/p = Q/S, this gives Feige = (v1 + v2)w + (pr + p2)S (2Q/S)(Qp) + (p1 + P2)S = 2Q?p/S + (pi + P2)S with Q = 3 ft?/s. Thus, the horizontal force of the water on the U-bend is Fy,ge = 2(3 £0? /s)? (62.4 Ibm /ft*) /(4/144 ft?) + (21+ 19 Iby/in?)(4x in?)(32.174 Tbmft/s?-1by) = (12871 + 16172 Ibmft/s*) = 29043 Ibmft/s* = 903 Iby 4-5 7A.6 Flow-rate calculation. As our system for the mechanical energy balance, we consider the water between plane 1 (the upper liquid surface in the constant-head tank) and plane 2 (just inside the outlet of the exit piping). We neglect the velocity at plane 1, set p = Patm at planes 1 and 2, and treat the water as incompressible. Then Equation 7.5-10 takes the form in which the kinetic energy and friction terms all are calculated with the same average velocity v. Collecting the coefficients of v?, we get 2[,.2 v |L+ FF +045 +04 + 0.4) = 29(21 ~ 22) h in which the coefficients 0.45, 0.4, and 0.4 are the ey; values for the sudden contrae- tion at the pipe inlet and for two smooth 90° elbows. There is no enlargement loss in the system considered, since plane 2 is just inside the exit of the piping. Setting Ry = D/4 and solving for v, we get va, | 2@an2) “V2.5 +4fL/D This result is explicit except for the friction factor f, which depends on Re and thus onv. Multiplying the last equation by v/v and evaluating all the dimensional con- stants in cgs, we get the working formula Re = 20 _ 2D, | 20(«1- 22) c= = 7 \ 225 -+4L/DF ___12.7¢m _ [2(980.7 em/s*)(38 x 30.48 cm’ ~ 0.010037 cm?/s 25 + 355.2f 6 1.907 x 10 (4) /2.25 + 355.2 The last equation is solvable rapidly by iteration, A first trial value 1x10® for Re gives f = 0.0045 from Fig. 6.2.2, and a new value Re= 9.72 x 10° from Eq. (A). At this new trial Re value, we get F = 0.00285 from Fig. 6.2-2, and Eq. (A) then gives Re= 1.056 x 10°. Returning to Fig, 6.2-2 with this new trial Re value, we get F = 0.0028, and Eq. (A) then gives the value Re= 1.059 x 10°, which we accept as final. 40 ‘The mass flow rate can then be calculated as w = RetDy/4, or the volume flow rate as _w_ RerDu as (1.056 x 10°)(7)(12.7 em)(0.010037 em? /s —_—_—T—_aseTooererov 1.1 x 10° em?/s = 0.11 m3/s Calibration is necessary to get accurate measurements of flow rates in such an apparatus, because of the uncertainties in the geomietric details and thus in the energy loss factors (see Table 7.5-1). 7A.7 Evaluation of various velocity averages from Pitot tube data. 1.07—— — I I+; 5 0.9 = f 08 — 07 \ 0.6 . \ Us 05 De,max \ 04 0.3 0.2 1 0 01 02 03 04 05 06 07 08 09 10 u=r/R This figure shows velocity data from the C. E. thesis of B. Bird (University of Wisconsin, 1915). Each measurement is labelled with its position number from the table in Problem 7A.7. The flow is evidently turbulent, but not fully developed; if it were, the measured velocities at positions 1, 2, and 3 would show better agreement with those at positions 7, 8, and 9 for the same r values. The solid curve in the figure is a curve-fit of the data, and the dashed curve represents the 1/7th-power model given in Eq, 5.1-4, which is based on extensive experiments in fully developed flow. TS Application of Simpson’s rule to 11 uniformly spaced points on the solid curve ives the following velocity averages. A finer grid would give more accurate approx- mations to the integrals. aCe f * udu Umax Jy Umax = [(1 x 1.0 x 0) + (4 x 0.995 x 0.1) + (2 x 0.986 x 0.2) + (4 x 0.98 x 0.3) + (2 x 0.975 x 0.4) + (4 x 0.96 x 0.5) + (2 x 0.94 x 0.6) + (4 x 0.92 x 0.7) + (2 x 0.87 x 0.8) + (4 x 0.755 x 0.9) + (1 x 0 x 1.0)] x 2/30 = 0.832 2 udu 0 Umax = [(1 x 1.0? x 0) + (4 x 0.995? x 0.1) + (2 x 0.986? x 0.2) + (4 x 0.98? x 0.3) + (2 x 0.975? x 0.4) + (4 x 0.96? x 0.5) + (2 x 0.94? x 0.6) + (4 x 0.92? x 0.7) + (2 x 0.87? x 0.8) + (4 x 0.755? x 0.9) + (1 x 0? x 1.0)] x 2/30 749 w) Lf i 2udu max Jo Umax = [(1 x 1.0% x 0) + (4 x 0.995% x 0.1) + (2 x 0.986" x 0.2) + (4 x 0.98" x 0.3) + (2 x 0.975" x 0.4) + (4 x 0.96% x 0.5) + (2 x 0.94" x 0.6) + (4 x 0.92% x 0.7) + (2 x 0.87% x 0.8) + (4 x 0.755% x 0.9) + (1 x 0° x 1.0)] x 2/30 = 0.680 ‘These integrals give the ratios (v?)/(v)? = 1.08 and (v*)/(v)® = 1.18. As one might expect from inspection of the plotted curves, these ratios differ significantly from the values 1.02 and 1.06 calculated for a 1/7th-power velocity profile. +4 7B.1 Velocity averages from the }-power law a. Average of the velocity 5. 2* Fa (r/R)]” rar 7 7 2) BER) rite a oe = af 6)!” eae = 2fx(1-x7)(7x°)dx=14(8-4)=8 b. Average of the square of the velocity (e 2) Xk f (o/R)]" rarao a [," [i rdrao = 2fpx(1- x7? )(x?)ax = 7(3- so that (02)/(9.)° = (8)(B) = 8 c. Average of the cube of the velocity = 2f(1- 6)” ede sy © _ LM L(y)” rarao [." [i rdrao =af{t-2 jx )oco ie so that (32)/(s.)° =(2)(9)° == 2f(1- £)"” dé si e 7B.2 Relation between force and viscous loss for flow in conduits of variable cross section For the variable cross-section, the macroscopic balances in Eqs. 7.5-5 and 6 have to be restated (when the gravitational forces can be neglected) thus: (momentum) Fy = (01-02) + (P15; — P2S2) (mechanical energy) E, (ei -e8)+ 5 (e1-a) To prove that Eq. 7B.2-1 is correct, we multiply the second equation above by S,, (defined in the problem statement) to get PSE, = 4PS,(07 ~ 03) + S(P: ~ Pa) = 40S (01 —P)(01 +2) + Su(P1 — Pa) If Eq. 7B.2-1 is to be correct, then in the term containing the velocities, we must have w=}pS,,(v, +0). That this is indeed true can be seen as follows: 408 af, 8). (1, 1 SS). zPSa(01+%2) asa] S. ols Ss +5)°° Next, we have to verify that (p,$, — P25) = Sq (Pi —P2) + Pm(S1 - $2)- Substituting the definitions of S,, and p,, gives for the right side: 25:5) \0 PiS1 +P2S2 ig _ (2 +5,)(! r)+( 345, 7%) = 25:S2P1 ~25;52P2 + SiP1 — S3P2 ~ SySoPr + SiSoP2 S$, +S, _ PrSi(S1 + S2)~ P2S2(S1 + $2) = — pS. $45, PSs ~ P2S2 (Ref: R. B. Bird, CEP Symposium Ser. #58, Vol. 61 (1965), pp. 14-15. +l 7B.3 Flow through a sudden enlargement The pressure rise is given by Eq. 7.6-5, and, in Eq. 7.6-6, in terms of the downstream velocity. We need a similar expression written in terms of the upstream velocity: P2~ Pr = (60, )[2 ~ (60,)] = ev7B(1-B) Then, to get the extremum, we set the first derivative equal to zero: d(p, - Ps) ap Hence, we get = pv} (1-2B)=0 Dit D, stot 22 a or D v2 We still have to verify that this is a maximum for the velocity. To do that, we need the second derivative: #(p,-p1) ap? Since this is negative, we have found the location of the maximum. The maximum pressure difference is then =-2p0? <0 Pa Pr = Pv;B(1- B)= po} 2 7B.A Flow between two tanks The flow in a tube can be described by the steady-state macroscopic energy balance of Eq. 7.4-7, which for this problem simplifies to PePa=-£, or P,P =Pe, Then using Eqs. 7.5-8, 6.2-16, and 6.2-12 we get Pa-Po= dole") S= to(v*) 4L »\4L 0.0791 pint) 5 Ret (Note that we could just as well have started with Eqs. 6.1-4 and 6.2- 12.) The last equation can now be written for each of the two systems, and we omit the factors involving numerical constants, tube length, and the physical properties (since they are the same for the two systems): (Pa-Ps), -(2) 2 pe)" -( (Pa-Pa)n (2) 2(Pas y" . [e -(2) as(} In the last step, we have used $, = 4, and w,, = 4w,. Hence beth-g(g aaron ’4 — Pa) eae 7B.5 Revised design of an air duct a. The flow in a tube can be described by the steady-state macroscopic energy balance of Eq. 7.4-7, which for this problem simplifies to PoP __é > or pi P2=pb, Then using Eqs. 7.5-8, 6.2-16, and 6.2-12 we get Pi-P2= ole?) f= 400?) SE p= 4p(or) OO (Note that we could also have started with Eqs. 6.1-4 and 6.2-12.) b. Label the two systems--the original and the revised--I and Il. Then the pressure drop in the two systems will be (,~p,), =_—2 300?) L (aR loyp/ny Baa ee ee Jo(v*) ,L 1 — Pa yp ge ™(4Rin()ye/H) Row Since the numerical constant, the length L, and the values of the physical constants are exactly the same, we now omit those quantities from consideration and concentrate on the quantities that are different. Then we have (with W and H being used for the width and height of the ducts): helt (wisi 25" _[2(Wi +H)" Re (S/z)" SF (WP (r.-P), OMY (w/Su)" 29 _[2(Wa + Hy)] Pi P; (Po Pala Rae (Su/Zn)" Si [Wau 7-14 We have used the fact that w is the same in both systems. Next we make use of the the fact that the pressure drop is the same in both systems: (20. +H )P" _[2Wy + Hw)” Wil Witt c. Next we put in numerical values [a(4+a)]" _ [2 +2))"* [4-4P aw) Taking the fourth power of both sides gives [16 _ [20a +2)F fas]? [2w,]}” Thus the equation for 2W,, =x becomes This equation can be solved by trial and error, and the solution is x = 18.4 ft or Wy = 9.2 ft. q-19 7B.6 Multiple discharge into a common conduit a. Application of the macroscopic mass balance gives: w,=w%, or _(v4)S,=(v»)S__ (for incompressible flow) Therefore, if the ratio of cross-sectional areas = S,/S, is specified, the relation between the average velocities is 22, ! B 1 We now define, for later use, a set of quantities K!) as ty _ (2h) ay where the index i indicates inlet (1) and (2), and j is the power to which the velocities are raised. For j = 2 and 3, these quantities are (for the 1/7th power law of turbulent flow) # and 42% respectively (see Problem 7B.1 b. Application of the macroscopic momentum balance in the direction of flow gives (with no external forces): (27 )S1 — (03 )Sp + PuS1 — P2Se — Fy =0 The force of the fluid on the solid will consist of the viscous force acting tangentially on the walls (which we neglect) and the normal force p,(S;—$,) acting in the direction opposite to the direction of the flow. Thus, when the quantity 8 = S/S, is introduced, we have ota{e- Eh etn ~P2)=0 Hence the pressure rise is given by P2~ Pr = P(t4) (BK? -B°K) c. Finally we apply the steady-state mechanical energy balance (Eq. 7.4-7) for incompressible fluids to get The pressure difference is evaluated from part (b), so that we get B =4(0,)7 {hes 0, )? (BK — p2K® E,=5( (8 we oF (21) (BK? -6°KY?) 1 x?) = Hox (1-0 ~(0,)*(pK?! ~ 62K) f 2) 2) Ke) 2 (3) 42g Ki” 4 gef 9K2 01)" Kf ( 2B +B (2% KO For turbulent flow, the ratios of the K!) in these expressions are of the order of unity. 7-17 7B.7 Inventory variations in a gas reservoir a, The maximum, minimum, and average values of w, are (2), = A+B (®2)min = A~B (w,),. = Jer (A + Beos wt)d(ot) . 2 avg fr ,d(ot) b. A mass balance over a 24-hour period is obtained by integrating the unsteady mass balance over the 24-hour period: Pl dm, 24 f{ es = Boyde [ya If the amount of gas in the reservoir is to have the same average value over a 24-hour period, then 0=24w, -24A from which w, = A. c. The total mass of gas in the reservoir as a function of time can be found by integrating the unsteady macroscopic mass balance: g( Tr i= [wid - [lwpat to get my (t) = m2y + f(A- A - Bost) dt = mo, ~2 sino d. In this problem we are assuming that the density of the fluid does not appreciably change with time, and so we assume that it is constant. However, the volume of the gas. V(t), will be a function of time and 7m. (#)= pV(t), so that pV(t)=m - Beno o The criterion for uninterrupted operation of the system is that V(t) must never go below zero. In this limiting situation, the quantity m,(t)=PV(t) must then oscillate between the minimum value, m®, ~(B/@) and the maximum value, m?, +(B/@). Therefore, the minimum total mass in the system that can accommodate this kind of oscillation is that PV ain = [jon + (B/C0)]~ [re — (B/o0)] or _ 2B ___(2)(2000) = =_ AA = 3.48105 ft? min = Gap ~ (0.044)(2n/24) = 248% 1° 3. Add a three-day supply to the amount found above (72)(5000) (0.044) = (3.48 10° ) + (8.18 10°)= 8.53 x 10° Ft? Vipin = 22+ (8%24A)=(3.48%105 ) + po 7B.8 Change in liquid height with time a. We want to calculate the volume between the liquid level at h and the part of the sphere below the liquid level. The sphere is visualized as being generated by a circle in the xz plane, with its bottom at the origin and its center at x = 0,2 = R. Such a circle has the equation x? +(z-R) =R? or x? =2Rz-2? Next we visualize the liquid volume as being made up of thin circular disks of thickness dz, each disk with a volume dV = nx*dz = n(2Rz—z")dz Then the total volume of the liquid is V=afi(2Re-2?)dz =n(R2? 42°) = (Rh? -4h°) v= aRie(1 - 34) This may be checked by verifying that when the tank is full, h=2R, and the liquid volume is V=47R°; that when the tank is half full, h=R, and the liquid volume is V = 3R°; and that when the tank is empty, i= 0 , the liquid volume is V=0 (Note: This method of obtaining the liquid vplume was suggested by Professor L. E, Wedgewood, University of Illinois at Chicago) 7-20 b. To get the liquid height as a function of the time, we start with the differential equation in Eq. 7.1-7 which may be rearranged to give: =A (H-20-+ ny HORE) at dt This separable, first-order equation can be integrated to give 4H? -2(L+R)H+L(2R+L)InH = At+C Att=0,h=2R, and H = 2R + L. Therefore 4(2R+L) -2(L+R)(2R+L)+L(2R+L)n(2R+L)=C Subtraction and elimination of the integration constant then gives 4[H? - (QR +L) ]-2(L + RH -(2R+L)]+L(2R+ Hint zat or 3f(n+ Ly -(2R +1) ]-2(L-+ R)(n—2R) + LAR + Hin ete =At When h = 2R, this equation gives t = 0, and when h = 0, it gives #= tamu exactly as in Eq. 7.1-8. We now introduce dimensionless variables: =h/2R (dimensionless liquid height) and 4 =L/2R (dimensionless tube length). Then the above equation becomes: 1 2 2] _ nt+A_ At al(n+ay -(+a)]-(2a +10 1)+A(Qeamatt =o, c. The parameter A =L/2R is fixed by the geometry of the system. Choose values of = /2R from 0 to 1 and calculate ¢. from the above equation. These may be plotted to give the curve of the dimensionless liquid height versus the dimensionless time. 7-21 7B9 Draining of a cylindrical tank with exit pipe a. First we write the unsteady-state mass balance for the tank. The mass of fluid in the tank at any time is 2R*hp, and hence the mass balance is (since there is no inflow stream) d oR? —aR*hp = -w: ane The quantity w, is the mass rate of flow out of the tank, and this is equal to the mass rate of flow in the tube. The latter is given by the Hagen-Poiseuille formula: a(Po-P,)D'p _ n(ogh+ pgl)D*p 1284 128 W, = Here pgh is the pressure py exerted by the fluid above the tube entrance, and pgL is the "pgh-term' in the expression for P in fn. 1 on p. 50 and discussed after Eq. 3.5-7 on p. 84. (It is unfortunate that we are dealing with two h's here: the ht in the expression for P and the h which is the height of the fluid in the cylindrical tank; thé‘must not be confused with each other.) The mass balance is then 2 dh __ mpg(h+L)D*p dh __ g(h+L)D*p RG Bu at 5 it 12BuLR This first-order, separable equation can be integrated thus: o dh _ hel 2 Bar b, One has many mass-flow-rate vs. pressure-difference relations to choose from in turbulent flow. For purposes of illustration we use Eq. 5.1-6, which can be shown to be identical to the Blasius formula given in Eq. 6.2-12. Thus we replace the second equation in part (a) by w= ee "ati yy” ~ 2\ 0.19807 Then the mass balance becomes dh 1 V4 RIA 47 ah (Ser) (n+ 1)” =-B(h+ 1)” This equation can be integrated to give Z(h+L)"” =-Bt+C Application of the initial condition gives Z(H+L)" =C Subtraction then gives the expression for the instantaneous liquid level 3[(H +L)” - (n+ 4)” ]= Be The efflux time is then 3[(H +1) - 27] = Bleue or t, efflux = yay 47 198 *) [erey” -27] ERT 7-23 7B.10 Efflux time for draining a conical tank a. From the unsteady mass balance we get, for the truncated cone system with no input stream ~ 42132.) =—p0, (773) From Fig. 7B.10 we see that r/r, = z/z, so that the mass balance may be rewritten as 4 (nz*o(ra/z2)?)=-p0,(m?) or 4 (42*)=-0,23 Therefore we get or up =-2 Hz ee dt b. We simplify the unsteady mechanical energy balance by (i) omitting the kinetic energy contribution on the left side, because it was shown in Example 7.7-1 to be unimportant, (ii) neglecting the viscous dissipation term E, because it is believed to be small, (iii) assuming incompressibility (which causes the E, term to drop out, (iv) omitting the work term, since there are no moving parts, (v) omitting the pressure terms on the right side, since the pressure is atmospheric at both ends, and (vi) assuming no vortex motion of te fluid. All potential energies are with respect to z = 0 as the datum plane. Hence we get: Loy =-(doh + 822) First we get the total potential energy in the fluid. This is done by integrating, gz over the volume (regarded as a truncated cone): Gq = febdV = J pgzdV = pg | z-nr2dz vo vo 24 2 2 2 =pel’z-n 2z| az= th) (7 z3qz = 98" I) (2424 esi, (2 ?) @ ese(22) i? @ 4 (2) & 4) Then the unsteady mechanical energy balance becomes pgn(r,\ d 2 sat (1) a? ~7) ,) 2 2)2 oo(2) Then, using the result of the mass balance to eliminate the time derivatve, we get 2 fo) 29| 72. ala) e(3 After dividing through by - par3v, we get Torricelli's law: 82=402 49% or 0, = 2g(z-2) c. Using the results of (a) and (b) we get a - -$-(2) ae) z (402+ g22)(p0,m3)_ or (303 + g22)(po,mr5) 2 ~($05 + 822)(e0.m?) By making the indicated simplification, we can perform the inte- gration, along with the initial condition that at t = 0, z= zo, to get 2 22-25? =} /2ge2t and ‘aan =¥{2) Pe 2 To get the efflux time, we have assumed that z=z, =0 when the container is empty. 7-25 7B.11 Disintegration of wood chips We start with Eq. 7.5-10, taking plane 1 at the top of the slurry dispersion and plane 2 right at the outlet to the digester; we also take plane 2 to be the datum plane for the calculation of the potential energy: (03-0) +(0-22)+2(p2-py)=0 Therefore the exit velocity is a-fie-ni-e] (200 Iby /in? (uss in?/t2)(30.2 poundals/tb) 65 Ibyy/ ft? = 15550 = 124 ft/s Therefore the mass rate of flow at the exit is + (22.2 tr/s)(20 tt) w, = pv,S, = (65 b,,/£8 Jez ft/ 5)(8 nf? = 2910 Ib,,/s Next, we apply Eq. 7.2-3 (the momentum balance) to get the impact force (assuming that the pressure and external force terms are quite negligible): F 4, = 0,1, =(124 ft /s)(2910 Ib,,/s)(1/32 Ib, /poundals) = 10,900 Ib, 7-26 7B.12 Criterion for Vapor-Free Flow in a Piping System. In the piping system of Fig. 7.5-1, the criterion p > pyap might be violated either (j) in the pump, or (ii) at a plane “A” just downstream of the final elbow. Lacking data on the NPSH (net positive suction head) requirements of the pump at the given operating condition, we can only test for condition (ii); further information on (i) is available in Perry’s Chemical Engineers’ Handbook and in unit operations texts. Applying Eq. 7.5-10 from plane “A” to just above plane “2”, we find (since »y and no fittings or enlargement loss occurs in this vertical section), P2—PA vi Laney lea = za) + Be which indicates the minimum pressure to be v} La: pas pete a(er—sa)+ Beats] Using values from page 208 and setting pa = 1 atm = 6.8087 x 10* poundals/ft?, we find the pressure at plane “A” to be 20 5 +300 + 100 + 120 38087 — 40186 + 3 = 27905poundals/ft? .41 atm PA = 68087 + 62.4 [222-20 + (s0| poundals/ft? Since the minimum pressure is well above the vapor pressure of water at the system conditions, the pipe will run full if the pump does. For mixtures, one must use the bubble-point pressure rather than the vapor pressure. 7-21 7C.1 End corrections in tube viscometers. We apply the steady-state mechanical energy balance, in the form of Eq. 7.4-7, with the assumptions of incompressible fluid and no mechanical work: {2 (22) 3 he) modi -r where "1" and "2" are general labels for the input and output streams. We label the plane at the bottom of the tanks as Plane 5, and designate by p41, the pressure at the outlet plane. Then for Run A, we apply the balance to the region between Plane 5 and Plane 2: (3 ( 1 . al (e) {oat +5 (Pa + 8la ~ Patm) = Ey(5— 2) 3 Note that the kinetic energy term is nonzero, because the velocity distribution at the inlet and outlet are not the same. For Run B, we apply the balance to the region between Plane 5 and Plane 0 to get: fhe sat +3 (Pe +pgls — Po) = E,(5— 0) We now subtract the second equation from the first, noting that the kinetic energy terms will exactly cancel, as will the viscous dissipation terms, since the flow rates (and hence the velocity profiles) in the two systems are equal. This gives llea—pe)+28lla-b)*(Pe-Pam))=9 Next we apply the mechanical energy balance to the region between Plane 0 and Plane 4 in Run B: 71-28 ale —La)+ 5 (Pa Pan) =E(0- 4) Here the kinetic energy terms do cancel, because the flow is fully developed at the inlet and outlet planes. The dissipation term can now be calculated by using Eq. 7.5-7 together with Eq. 2.3-22: B09 T= BM 0~ Pam) alla ba] But, according to Fn. 1 on p 50, Bo ~ Py = (Po — Pa) +8 (Mo — hs) = (Po ~ Patm) + 28 (Le ~ La) Therefore, by combining the last three equations we get alle-La)* (Po -F) 4) Now Eg. (**) can be rearranged and then combined with Eq. (*) to give: a Po Pain = Tol, 8+ i la = 08-7 (Pa —Po) +080 -14)] =Pa-Pa tants mPa Ip-La = PB PA 4 pol 14 la— Lg-La Iy-La in agreement with Eq. 7C.1-1. 7-24 7D.1 Derivation of the macroscopic balances from the equations of change a. Derivation of the macroscopic mass balance We start by integrating the equation of continuity, Eq.3.1-4 over the macroscopic flow system of Fig. 7.0-1: a J Sav =— J(v-pvyav vio vit) We write V(t) to remind ourselves that the volume may be changing with time because of the presence of moving parts within the system. We now apply the Leibniz formula (Eq, A.5-5) to the left side and the Gauss divergence theorem (Eq. A.5-2) to the right side to get d ase - { o(n-v.)dS=— f(n-pv)as vit) S(t) s(t) We now combine the two surface integrals thus: d aio = J(n-p(v-vs))as Ss) Next we divide the surface up into four parts as indicated on p. 221, right after Eq. 7.8-2. We also introduce the assumptions (i) and (ii) listed. in §§7.1 and 7.2. Then we divide the surface into four nonoverlapping parts, $=, +S, +S, +S,,, and write the right side of the above equation as the sum of four contributions: Sg =-J(n-p(v-v,))dS - f(n-p(v-vs))dS— f(n-p(v—vs))45 & & 5 5) - f(n-p(v—vs))dS Sm We now evaluate seriatim the terms on the right side: The surface S, is the inlet plane, which is not moving so that v, = 0, and the outwardly directed unit normal vector n is the negative of the vector u,, which indicates the direction of the flow. Since v= v, at this surface is assumed (assumption (i)) to be exactly in the direction of flow, we may say that v=u,0,. The evaluation of the integral proceeds as follows: J(n-p(v—vs))d5 = + { (uy -uy0,p)dS = + f pr,dS = p,(24)S, = w, 3 3 5 where it has been assumed that the density is constant over the cross section (assumption (ii)). The integral over the exit plane is evaluated in the same way, the only difference being that n is the same as u, SO that ~J(2-p(v~vs))dS = - [(u -u,0,p)dS = + [ pv,dS = p,(v,)S = w, Sy Sp Sy On the fixed surfaces both v and vg are zero, so that S, = 0. Also, on the moving surfaces, v = vg, with the result that S,,. Therefore, Eq. 7.1-1 follows at once: d Sr = 03(01)Ss~ Pal) and the definitions of w, and w, lead to Eq. 7.1-2. b. Derivation of the macroscopic momentum balance First we integrate Eq. 3.2-9 over the volume of the system: S(Gerv=- fc J(V-pvv pV - f(vpyV- f[V-tHV + fogav ve) ve) vit) We now manipulate seriatim the terms in this equation so that they can be interpreted. In so doing we make use of assumptions i-iv in §§7.1 and 2, and follow the procedures in (a) above. The first term is rearranged by using the Leibniz formula (as applied to a vector function), and then P,,. is introduced as the definition of the integral over the momentum per unit volume, and the second term is rearranged, thus: 7-31 a ye d da 1 and my, we have to use L'H6pital’s rule tim So oi SEINE In lem 2B. an = lime =1 eK'lnk Ink (see Problem 2B.7) c. The mass flow rate is then (Yn) w= 2a", pv,rdr = 2AR* poo) (Samat ay 2) _ 2nR? pg (1= x2 _1- x? ~ Kr —1\"3-(n) 2 d. When n=4 use of L'Hépital's rule gives 1-K 2 (m4) ge e. When L'Hépital’s rule is used we get 1-K? w= ron Zin( « 8B.4 Flow of a polymeric liquid in a tapered tube We consider a small region of the tapered tube to be a straight tube over a short distance dz; then we can write “locally” __AR°p [- ae 2f (Yn)+3L" dz 2m Take the nth power of both sides to get dP _2m{_w (2 y 22m) _@ (lis dz R| aR*p\n in which R is a function of z: R,-R R=R, Lo + AB) It is easier to integrate the differential equation if we rewrite it as dB AR _ --(R =f) _2m|_« (2+3)] aRdz dR OL R | aR°p' Then when this equation is integrated with respect to R, we get 7 maw -( om) 2 2 a(4s sf [Regard Therefore retell) |S) “SG (SE) This is the power-law analog of Eq. 2B.10-3. 8B.5 Slit flow of a Bingham fluid a. For |x|< x9 (ie, in the region where the yield stress is not exceeded), 11 = (according to the upper equation of Eq. 8B.5-1. But the expression for the shear stress is, from Eq. 8.3-2: T,, =—ndv,/dx. Since the shear stress is finite, the velocity gradient must be equal to zero. This is the plug-flow region. For |x/2x, (ie., in the region where the yield stress is exceeded), the lower equation of Eq. 8B.5-1 has to be used. This means that in the region where x2 Xp, the velocity will be decreasing in the positive x direction, so that 7 =—dv,/dx is required so that 7 will be positive. Similarly, when x<-x), the velocity will be increasing in the positive x direction, so that y = +do, /dx is needed in order to guarantee that 7 be positive. Hence we have: di Ta = Hy FE for -BSx<-x, a “Hy E+ ty for +x)1 and m—y; we make use of Lhopital's rule to get the result of Problem 7B.9(a): aut) ar? | (d/dn){(H+ Ly") — 2") ta =( es) | (ayant Cn} -( 2uL 2u.) AE (H +L) [in(H + L)|(1/n?) - 2" fin L(1/n?) pgRo) Ry ™! (v*) Eben £18 8B.10 The Giesekus model a. We have to start by expanding the 7’ in Eq. (E) of Table 8.5-1 as a function of (A7)?: 2 _1+4[l6a(1- a) (Ay)? - f[16a(1- @)P ay) + * Ba(l- aay? =1-4a(1-a)(Aj)?+-- and then, the series expansion of the square root of this will be = fl 4a(1- aaj)? +- =1-2a(1- aay) Next we get f to the same order (fis defined in Eq. (D) of Table 8.5-1) _ 1-[1-2a(1- aay) +] ae "TG 2ajfi-2atl-aayer] w Then the viscosity expression (of Eq. (A) in Table 8.5-1) becomes 1 (1- aay? +--)P St = 1-2 V2 pee mm 1+(1-2a)a(apy?+ AYN + Pl antso Similarly, the first normal stress coefficient (Eq. (B) of Table 8.5-1) becomes vy, _ (are 1 4 2mA —af1-(aAHP+)] ANP ano and the second normal stress coefficient (Eq. (C) of Table 8.5-1) is ¥, =-anA =-}a¥, which shows (correctly) that the second normal stress coefficient is smaller than the first normal stress coefficient and has the opposite sign. &14 b. We begin by dividing numerator and denominator of the expression for 7? by (Aj)? and then replacing 1/47 by € (a small quantity): -(16ae(1= ce) 4/1 +[1/16a(1 - a) Je? Ba(1—a) We now expand this last expression in powers of €: 2 __&(t+[y32a(1-a)Je*+-)-e? _ e(1-e+--) 2Ja(1-@) © 2a(t-a) Then we take the square root and get = elinter) v2fa(t- a)" Then fis given by fe _=(1-y[-(1-20)+-]=1-2x(1- 0) “T+(1-2a)x Then the viscosity and normal-stress coefficients are an (inf? _2x(-a)+-) No 1+(1-2a)f — 2(1-a@)+ (20-a)'e fi=o 1 ~ pa-a))-2Jat—a) a Ay % AA 1) (1-a)+--- (2) 2A a(i-f)\Ay) — af2z(1-@)+-- Jay §-20 a (3) mA Ay These expressions show (correctly) that the normal stress coefficient has a steeper slope at high shear rates than the viscosity. They also give a second normal stress coefficient that is smaller than the first normal stress coefficient, and that the two coefficients have opposite signs (this is in agreement with the experimental data for flexible polymers). c. For elongational flow, we first get the limit as 2é 0 (please note that this é is not related to the ¢ in part (b)). In this limit we can expand the square root signs that appear in Eq. (H) of Table 8.5-1 thus: ze “alae 2(1-20)Ae+----1-(1-20)Aé--- 5} Ale+ Fea-2a)aer) fea In the limit that 2é becomes infinite, the quadratic terms under the square root signs dominate and we get 71 2 gi -1642-1-2 3m alt? Y= aq Thus, the‘elongational viscosity remains finite, unlike the Oldroyd model for which the elongational viscosity becomes infinite. $2) 8C.1 The cone-and-plate viscometer a. According to Eq. 2B.11-1, the velocity distribution in the cone-and-plate system is given approximately by -of 62-2) r Wo The shear rate is given by the relation two lines above Eq. 8.3-2. In this problem the only nonzero components of the rate of strain tensor are gy and 749. This can be seen in one of two ways: (a) Look at the right side of Eqs. B.1-15 to 21, but without the factor — 1 and with the div v terms set equal to zero; these are the spherical components of the rate of strain tensor given in Eq. 8.3-1 or Eq. 8.4-1; or (b) add to the components of grad v in Eqs. (S) to (AA) in Table A.7-3 the corresponding transposes in order to construct the rate-of-strain tensor according to Eq. 8.3-1. Therefore, from Eq. B.1-19 ap = Von = OD zs) 216 Yoo = Yoo = 96| sind)” 7 30 since the angle between the cone and plate is extremely small, which means that sin@~sin}=1. When the velocity distribution of Eq. 2B.11-1 is inserted in to the above approximate expression for the nonzero components of the rate-of-strain tensor we get . _, .9(%)_ 2 Yoo = Yoo = 36| = We Hence the shear rate is 3 (Feot 0 + Voot'oy) = #V 09 = We now have to choose the proper sign. Both Q and yo are positive quantities. Therefore the plus sign must be selected. b. The non-Newtonian viscosity is obtained from the ratio 1= (top /—Yop)- Hence we have to find a way to get the shear stress g-20 from the measured quantities. As pointed out in (a), since the shear rate is constant throughout the gap, all stress components are also constant. This means that the torque can be calculated from the shear stress by integrating shear-stress times lever arm over the surface of the plate from zero to R: 3T, Tf het To = 5 ape rdrdg =37R*t,, whence lo=n/2 Then the non-Newtonian viscosity is given by c. Equation 8C.1-2 follows directly from Eq. B.5-7, after dropping the terms on the left side and the 7, and t,, terms on the right side; the terms Ty and t,, must, however, be retained, since we know that normal stresses are nonzero in shear flow. Then Eq. 8C.1-3 follows from ,,=p+,, and some minor rearranging, and Eq. 8C.1-4 follows from the definitions of the normal stress coefficients and from the results of (a). Next we integrate Eq. 8C.1-4 from the outer rim of the cone- plate system to some arbitrary position r: Sp4t00 =-[(¥1 +2¥ 2) 77 |fpdinr The normal-stress coefficients are constants, because of the constancy of the shear rate over the gap. Therefore we get eo(?)= oo (R)—[(¥1 +2¥2)7? Jin = My R)+[%59(R)~ My (R)]-[(¥1 +242)? Jin = Pa Yai? —[(¥1 +2¥2)7? Jin $23 Here we have used the boundary condition that the normal stress at the rim is equal to the atmospheric pressure, and we have also used the definition of the second normal stress difference. d. The force exerted by the fluid in the z-direction on the cone is then obtained from the result in (c) as follows: F,= Jp" Jp 00(ryrdrdo - nR°p, = anf]. -¥,7? -[(¥, 42,7? Jin par —2R?p, =-ARW,j? — 2nR?[(Y, +245) 7? |f (In E)EdE =~ AR'Y 7? —2aR?[(Y, +22)7? (PEP INE -4E")) = AR? +407? + ARV 77 = LAR? 1 lo Here we made use of the fact that limé Ing =0. Solving for the first normal stress difference, we get finally 2F. aR?y? W(y)= e. If one measures my(r)—p, as a function of r/R using flush-mounted pressure transducers, then, knowing ¥’, from (4), the second normal-stress coefficient can be calculated from Eq. 8C.1-5. G4 8C.2 Squeezing flow between parallel disks This problem is solved by a quasi-steady-state method. Conservation of mass states that, for an incompressible fluid, the mass rate at which the fluid crosses the cylindrical surface at r should equal the rate at which the mass between the plates within the cylindrical surface at r decreases. The rate of mass displacement caused by the disk motion is: w(r) = ar?p(-H) where (H = dh/dt) To get the mass rate of flow between the two disks, we adapt the result in Eq. 8.3-14 by making the following correspondences locally: W-2ar, 2B-H, (P)-P,)/L—>-dp/dr, and w— w(r). Then we get for the mass rate of flow emerging from between the disks: . aeeaey dp any dr m (Un)+2 Car Equating the two expressions for w(r) we get a differential equation for for the pressure distribution p(r) between the disks gol ae) This may be integrated to give 2m(-H)"[(1/n) +2)" =f dp 2d page or PPaten = et n+l R which is the power-law equivalent of the Newtonian result in Eq. 3C.1-13. G25 When a constant force F, is applied to the upper disk, this force must be resisted by the pressure in the fluid at the upper disk, integrated over the disk (we include here also the normal stress T.., even though we know that it will not contribute for a generalized Newtonian fluid--the proof for this is similar to that given in Example 3.1-1 for Newtonian fluids): Fo=f0"[) (P-Po + tard 2m(-H)"[(1/n) +2] won ET be He _ 2nm(-H)"[(1/n)+2]" Rm OPT This is now a differential equation for the motion of the upper plate as a function of time: __ A _(_(n+3) "(1 in Huy? -(;3,) (was) This can be integrated to give: pt aH _(_(n+3) \"(_ 1) ant Su Hone -( 235) Wy Fo" fod or 14 -(233)' ‘(ase Hoa Her nmr) | (yn)+2 This simplifies properly to Eq. 3C.1-16 for the Newtonian fluid. e26 8C.3 Verification of Giesekus viscosity function a. Eq. 8.5-4 is the same as Eq. 8.5-3 with 4,=4,=A and Ay = My = Mp =0, but contains in addition a term —(A/no)a{t-t}. Therefore, for steady shear flow, we may take over Eqs. 8.5-5 to 8 by making the appropriate changes in the constants, and by adding the extra term. To do the latter we need to calculate the components of the product {t-7}; this is most easily done by matrix multiplication: x Te «(OT Tm 0 Bett tlt tty) 0 Ty ty Olt ty 0 [=| tlt tty) thtth 0 0 0 zo 0 t 0 0 a When we use this result, Eqs. 8.5-5 to 8 become modified as follows for the Giesekus model: Ty ~ 2A — (2/9 )O( 7% + 73.) =O Ty —(A/ Ng OTe + The =O 1, ~(A/M)at2, =0 Ty ~ AFT yy —(A/ Mo) ye (Tax + yy =—NoY When these equations are multiplied by /my and dimensionless variables are introduced, we obtain Eqs. 8C.3-1 plus an equation that gives T,,=0 (this equation gives another solutions which is physically unacceptable). b. When these dimensionless equations are written in terms of the dimensionless normal stresses we get N,[1-@(N, +2N,)]=2T, 1° N, = a(T2, +3) Ty [1- a(N, +2N,)]=-(1- NP c. The second equation in (b) can be solved at once for the dimensionless shear stress: $-27 72,=Na (a2) @) Division of the first of the equations in (b) by the third gives a relation for the dimensionless first normal stress difference in terms of the dimensionless second normal stress difference and the dimensionless shear stress; into that we can substitute the equation just obtained above and get _2N,(1- aN.) N,= ' a@(1-N,) Cm) Next, we square the third equation in (b), and then insert the expression for T?, from (*) and the expression for N, from (*), to get N,(1- aN, )[1+(1-20)N,[° a(1-N,)* (Hs) d. To solve the final equation in (c) Giesekus (see p 87 of Ref. 5 on p. 262) suggested making the following change of variable: ata _ No= Ty 20x Then the various factors in the final equation in (c) are: = 2x(1- a) 1 N= To aaly _(+a)(1-a) . ON. TT 2ajy’ 2(1- a) 1+(1-2a)N, “Ty-2a)g 8-28 When the last four equations are substituted into (***) we get pe 1-2? 4a(1-a)x* This is a quadratic equation for z? which may be readily solved: = =1-4a(1-a@)f? + 8a(1-a)r? Then this result along with Eq. 8C.3-7 gives the second normal stress difference as a function of the shear rate. Having this, the shear stress and first normal stress difference can be obtained from Eqs. 8C.3-4 and 5. As the shear rate goes to zero, one gets 1— 1, ‘¥; > 2mA, and ¥, > -amyd. The functions given in Table 8.5-1 are given in Giesekus's paper as well as in R. B. Bird, R. C. Armstrong, and O.Hassager, Dynamics of Polymeric Liquids, Vol. 1 (1987). pp. 361- 368. $4 8C.4 Tube flow for the Oldroyd 6-constant model The non-Newtonian viscosity for the Oldroyd model is given in Eq. 8.5-9. Therefore for tube flow MY 2 = mi From Eq. 2.3-13, we have the expression valid for any kind of liquid (Po-P.)r 7 OL Combination of these two results gives noi (2s) =P, (1+ 0,77 The mass rate of flow through the tube is given by w= phe" [*v,rdrdo = 2npf v.rdr= ano{ tor - fpr Be ar] The first term in the last result is zero at both limits. We next integrate by parts again tonne pal | ofa 3 ae oa re baeRe a4 4 apf [r(v)P ar in which 7, is ~ do, /dr evaluated at the tube wall. This result is good for any non-Newtonian fluid. We now specialize to the Oldroyd model: 3 3 , 2m)’ pin{(1+07)] 9340 =1 3g —1 fo R)) OTE 2T 3, Wah mOR R370 | 0 (ze vay 3 3 ; 2noL peat) 1 = 42pR° i o ct YaY— SMR TR 374 #5) o a+ 207 & 30 in which Y = 0,7? and n=0,/0,. The expression for r given earlier may be evaluated at the tube wall, and this gives na Bal (reat) ( 2ngl 1inX) 1 ra . -P, 140,72) (P- x ) a 1+X J Jo, in which X = 0,72. Next, multiply the equation for w. by 3/0, /mpR° and eliminate P, - P, by using the expression for R just above to get g- 3a 1 (1+xy we mpR3 = VR -yea( gay) f%) ) in which f= ney) yay =49°X? -3n?(n—-1)X + 3n(n-1)(2n-1)In(1+ X) 2 (2) [6n+(7n-1)x] () This gives, in dimensionless form, the mass rate of flow in terms of the wall shear rate. The latter may be eliminated in favor of the pressure drop by using the expression for R above. That is, to get Q in terms of the pressure drop, the asterisked equation have to be combined. The curves thus obtained may be found in the original publications (refs. 6 and 7). 8-31 8C.5 Chain models with rigid connectors a. The paper by M. Gottlieb is the first example of a molecu- lar dynamics calculation for polymer chain with rigid connectors. He followed the motions of a three-bead-two-rod model as it moves around in a liquid made up of 47 "solvent beads." The solution is presumed to be at macroscopic equilibrium. He showed that one does not get a Gaussian distribution of orientations for the polymer model, in agreement with the previously published theory of H. A. Kramers, Physica, 11, 1-19 (1944). See also, R. B. Bird, C. F. Curtiss, R. C. Armstrong, and O. Hassager, Dynamics of Polymeric Liquids (Vol. 2), Wiley, New York, 2nd edition (1987), pp. 40-41. This book will hereinafter be referred to as "DPL." b. The publication by O. Hassager showed how to get the viscosity and normal stress coefficien‘s for a three-bead-two-rod chain in a flow situation. In a subsequent publication [C. F. Curtiss and R. B. Bird, J. Non-Newtonian Fluid Mechanics, 2, 392-396 (1977)] it was shown how to extend the calculations to models with 4 and 5 beads. Hassager showed that the viscosity and first normal stress coefficient for models with infinitely stiffened Fraenkel springs are different from those of the rigid rod models. This perplexing situation was what led Gottlieb to investigate the same problem by using molecular dynamics. See also DPL (§16.5). c. The paper by X. J. Fan and T. W. Liu dealt with Kirkwood- Riseman chains of 3 to 8 beads. This model has fixed bond lengths and fixed bond angles, that is, two kinds of constraints. They were able to evaluate the equilibrium configurational distribution, and they found that it differs from the classical "Gaussian distribution.” This calculation enabled the authors to relate rheological properties to "chain stiffness," by comparing their results with those for a Kramers freely jointed chain. See also DPL (§16.6). d. The paper by T. W. Liu is a landmark contribution to the theory of Kramers chains. He showed how one can study the configurations and rheological properties of freely-jointed Kramers chains by the use of Brownian dynamics. By this technique he was able to generate movies showing the change of the configurations with respect to time. He also succeeded in taking appropriate averages over the configurations, in order to get the viscosity and first normal-stress coefficient as a function of shear rate, for chains with 3, 5, 10, and 20 beads. He also calculated the elongational viscosity in steady elongational flow and got information on how the $32. polymer molecules “unravel” under the influence of the elongational flow. e. The paper by H. H. Saab, et al., is an extensive comparison between the Curtiss-Bird phase-space theory for polymer melts and the available experimental data on a host of rheological properties. An extensive comparison of the results with those of the theory of Doi and Edwards confirms that in almost every instance, the Curtiss-Bird theory is to be preferred over the Doi-Edwards theory. In the Curtiss-Bird theory, the polymer molecules are modeled as Kramers freely jointed bead-rod chains, whereas in the Doi- Edwards theory, mixed modeling is used. See also DPL Chapter 19. f, The papers by J. D. Schieber show how the extension of the Curtiss-Bird theory to polydisperse polymer melts can be implement- ed. Both the log-normal (Wesslau) distribution of molecular weights and the Flory-Schultz distribution are used. The curves for the viscosity, first normal-stress coefficient, elongational stress growth viscosity, and steady-state elongational viscosity were obtained. These curves often differed appreciably from those for monodisperse samples. Comparisons with experimental data are given in the second of the two papers. See DPL, Example 19.6-1. i. The rodlike connectors are generally much more difficult to deal with than the springlike connectors, because it becomes necessary to use nonorthogonal coordinates to describe the chain space. For Hookean springs, one can perform many of the kinetic theory derivations with relative ease. For non-Hookean springs, it is possible to make some assumptions that enable analytical results to be obtained. But even for rigid dumbbells and three-bead-two-rod models, analytical calculations become prohibitively time consuming. ii. To overcome the problems associated with the use of rigid connectors in modeling, molecular dynamics and Brownian dynamics have proven to be useful. Also, Brownian dynamics can be useful when considering the flow in constrained channels, where the interaction with the containing walls have to be considered. Brownian dynamics proves to be particularly helpful in getting information about the actual motions of the polymer molecules during various types of flow. 8-33 9A.1 Prediction of thermal conductivities of gases at low density. a. Since Argon is monatomic, we use Eq. 9.3-13 to predict its k in the low- density gas region: = + VT /M k= 1.9801 x 10-5 Here T = 100 + 273.15 = 373.15K, and Table E.1 gives M = 39.948, = 3.432A, ¢/K = 122.4K for Argon. Then KT/e = 373.15/122.4 = 3.049, and interpolation in Table E.2 gives 2, = 1.0344. Equation 9.3-13 then gives k = 1.9891 x 107* 499 x 1077 cal/s-cm-K 3.432? x 1.0344 which is within 1.5 percent of the observed value. }, Equation 9.3-15, Bucken’'s formula, gives k (6 + 1.25R/M) w= (Cy +125R)y/M with R = 1.987 cal/g-moleK. Insertion of the data for CG, and p, and for M from Table E.1, gives 1929 x 10-7 forNO, k= (7.15 + 1.25 ¥ 1.987) o> = 620 x 1077 cal/s-cm-K = 0.02595 W/m-K vs. 0.02590 W/mK from Table 9.1-2. 1116 x 10-7 for CHy, = (8.55 + 1.25 x 1.987) = 0.03212 W/m-K vs. 0.03427 W/m-K from Table 9.1-2. = 768 x 1077 cal/s-cm-K 9A.2 Computation of the Prandtl number for gases at low density. Use of Eq. 9.3-16, with molar heat. capacities G, = MG, calculated from the given values G, and molecular weights M from Table B.2, along with R = 8.31451 x 10° J/kg-mol-K from Appendix F, gives the predictions in column (a) of the following table. Computation of Pr from its definition, Eq. 9.1-9, and the tabulated Cp, 1, and k, gives the values in column (8). The predictions are closely confirmed for He and Ar, but are less successful for the polyatomic compounds. (2) Gas Pr from Eq. 9.3-16 He 0.667 Ar 0.667 Hy 0.735 Air 0.736 CO, 0.782 H,0 (M = 18.016) 0.764 (0) . Pr from Cp, p, and 0.670 0.665 0.714 0.710 0.769 0.862 9A.3 Estimation of the thermal conductivity of a dense gas. a, Table E.1 gives the following critical constants for methane (CHy): Te = 191.1 K, pe = 45.8 atm, and ke = 158 x 10~® cal/em-s-K. The reduced conditions for the prediction are then T, = (459.7 + 127)/(1.8 x 191.1) = 1.71 and p, = 110.4/45.8 = 2.41. From Fig. 9.2-1 we find k, = 0.77 at that state, giving k= ky ke = 0.77 x 158 x 107° .22 x 107 cal/em-s-K = 0.0294 Btu/hrft-F. which is about 4% above the observed value. 6. For this calculation, we need to predict the viscosity of methane at 127 F (325.9 K) and low pressure from Eq. 14-18. We find « = 3.780 and KT/e = 325.9/154 = 2.116 and 9, = 1.153 by use of Tables E.1 and E.2. Hence, 1171 x 1077 g/em-s Next we use the Eucken formula, Eq, 9.3-15, to estimate the thermal condue- tivity k° at low pressure and 127 F, where Cy = 37.119 J/g-mol-K according to the heat capacity polynomial given for methane in Reid, Prausnitz and Poling (1987): 1171 x 1077 16.04 Ke (37.119 + 1.25 x 8.31451): = 0.000347 W/em-K 0.0347 W/m-K .0200 Btu/hr-ft-F Finally, we multiply &° by the ratio of k, at 110.4 atm to the asymptote ky = 0.52 at py = 0 in Fig. 9.2-1. The resulting predicted k at (110.4 atm, 127 F) is k = 0.0200 x 0.77/0.52 = 0.0297 Btu/hr-ft-F and is just 1% above the measured value. This is unusually good agreement. 9A.4 Prediction of the thermal conductivity of a gas mixture. ‘The data for this problem are as follows: Component M Hx10°, Pas — , W/mK Mole fraction 1(H2) 2.016 0.8944 0.1789 0.80 2(CO2) 44.01 1.506 0.01661 0.20 Insertion of these data into Eq. 1.4-16 gives the dimensionless coefficients Oy = O2 =10 2 1 aoe) ae € 3044)? 744.01) 1/* $= (1+ 1 +( av vB 44 :) T.506 , 2.016 = 2.457 1 so) aaa (3) 1 72.916\"4]" On = (1+ 1 +( ae V8 \' * 2076. a) 08944, ou) a0 = 0.1819 Substitution of these results into Eq. 9.3-17 gives k 0.8 x 0.1789 0.2 x 0.01661 ms 0.8 x 1.04 0.2 x 2.457 " 0.8 x 0.1819 + 0.2 x 1.0 1.1204 W/m-K G4 9A.5 Estimation of the thermal conductivity of a pure liquid. We first calculate the derivative (2p/Op)r required for Eq. 9.3-4: (p/Op)r = p~*/ p*(O/Ap)r] = (1/0.9938)[38 x 10~°]-! = 2.648 x 10* megabar cm*/g = 2.648 x 10" cm?/s? Inserting this result into Eq. 9.3-4 and setting Cp © Cy, we obtain vy = 2.648 x 101 = 627 x 10° cm/s Equation 9.4-3 then gives the following estimate of the thermal conductivity: k =2.80(Np/MP Kv, 6.02214 x 10% x 0.9938]/* 18.02 50 x 10* g cm/s*-K .650 W/m-K 1.375 Btu/hr-ft-F = 2.80 x 1.38066 x 1071 x 1.627 x 10° q-5 9A.6 Calculation of the Lorenz number. a. When K and ¢ in Eq. 9A.6-1 are expressed in terms of the gas constant R and Faraday constant F, the Lorenz number takes the form we (RY 3 ( F ) Insertion of numerical values for R and F from Appendix E gives pa@ (831451)? ~ 3 \ 968853, 44 x 107 volt? /K? b. Insertion of the result just found, and the given k, and T, into Bq. 9.5-1 gives the thermal conductivity estimate kok? _ 2.44 x 107 volt?/K? x 293,15K 1.72 x 10-Sohm-em, = 4.16 volt?/K-ohm-em = 416 Win-K for copper at 20°C. 4-6 9A.7. Corroboration of the Wiedemann-Franz-Lorenz Law. Conversion of the tabulated data into SI units and insertion into Eq. 9.5-1 gives the following results at T=293.15K: Metal 1/ke, ohm-m —-k, W/m-K Lorenz number, L = k/keT, volt?/K? Na 4.6x104 133 2.11078 Ni 6.9108 59 1.4x10-% Cu 1.69x10* 385 2.2x10-% Al 2.62108 209 1.9x10-* ‘The approximate agreement of L for these metals illustrates the Wiedemann-Franz- Lorenz law, 9A.8 Thermal conductivity and Prandtl number of a polyatomic gas. a. To calculate k for a polyatomic gas at moderate pressure we use Eq. 9.3-15, k= (6, +1.25R/M)u (Cp +1.25R)n/M along with the viscosity expression in Eq. 14-18: vMT 2M, f= 2.6693 x 107°. From Table E.1 we find, for CHy, the values M = 16.04, ¢ = 3.780, ¢/x = 154K, and from Table E.2 at KT/e = 1500/154 = 9.740 we find Q, = 0.8280. Equation 14-18 then gives the predicted viscosity = 2.6693 x 107%. (3.780)? x 0.8280 = 3500 x 1077 g/cm-s and Eq. 9.315 gives the predicted k value k = (20.71 + 1.25 x 1.987) x 3500 x 10-7/16.04 = 5.06 x 10~* cal/em-s-K = 2.12 W/mK 4. The predicted Prandtl number according to Eq. 9.3-16 is 20.71 P= SoTL e125 x 1.98T = 0.89, dimensionless 4-3 9A.9 Thermal conductivity of gaseous chlorine. For Ch, Table E.1 gives M = 70.91, ¢ = 4.115A and ¢/x = 357K. Equation 14-14 then gives = 2.6693 x 10~ 70, = 1.34274 x 10-° VT with p[=]g/em-s and T[=]K, and Eq. 9.3-15 gives k = (G, + 1.25R)u/M. The calculated results follow, with 1, G, and k in the units of the problem statement: TK 198 275 276 276 363 363 395 453 453 495 553 583 583 676 676 T/387 0.5546, 0.7703 0.7731 0.7731 1.017 1.017 1.106 1.269 1.269 1.3866 1.549 1.633 1.633 1.884 1.884 Qy 2.0915, 1.8217 1.8239 1.8239 1.5799 1.5799 1.5138 1.4172 1.4172 1.3609 1.2974 1.2694 1.2694 1.1994 1.1994 10" 0.8931 1.2044 1.2091 1.2091 1.6008 1.6008 1.7427 1.9935 1.9935, 2.1701 2.406 2.5249 2.5249 2.8775 2.8775 (Gp +1.25R) 10%kprea 10.54 10.59 10.59 10.59 10.81 10.81 10.91 11.02 11.02 11.095 1.17 112 112 11.32 11.32 1.33 1.80 1.81 181 244 244 2.68 3.10 3.10 3.40 3.79 3.99 3.99 4.59 4.59 Ave.= Kobs/ Kprea 0.985 1.056 1.066 1.061 1.074 1.070 1.13 1a 1.10 1.09 1.09 lll 112 1.10 1.07 1.084 ‘The predicted k values exceed the observed values by an average of 8.4% in this temperature range. 9A.10 Thermal conductivity of chlorine-air mixtures. Numbering chlorine and air as components 1 and 2, respectively, and inserting their given properties into Eq. 1.4-16, we obtain the following coefficients for Eq. 93-17: $n =1; y" [1351 (3% wy? k L sal 70.91 cee! 28.97\~1/? Te ( 70.91 Hs 70. iat) 1351 (Ba 97 ee Equation 9.3-17 gives, for binary mixtures: ik tak, Kusie = —— tt _ 5, —_ 79 __ xP + 22912 2b + o2o2 Insertion of the coefficients and compositions for this problem gives At 21 = 0.25, 0.25 x 0.0896 0.75 x 0.02614 Fore = G5 + 0.78 x 0.59800 + 0.95 x 18105 + 0.75 ~ 0-0506 cal/ems:K At 2 = 0.5, a 05 x 0.02614 9.0675 cal/em-s-K At =0.75, 0.75 x 0.0896 0.25 x 0.02614 Kmix = 0775 + 0.25 x 0.59800 * 0:75 x 1.8105 + 0.05 ~ 09800 cal/emsK 9A.11 Thermal conductivity of quartz sand. a. For spheres (9g; = g2 = 93 = 1/3), Eq. 9A.11-2 reduces to aj = 3/(2 + ky/ho) ‘The resulting a; values from Eq. 9A.11-2 for the water-saturated sand are 3 3 3 =p, a = 0.183; a = 2+ (o/h) “T+ Caio) “35 Uhl ho) and Eq. 9A-11-1 then gives a = 0.433 (1)(0.427)(0.00142) + (0.183)(0.510)(0.0204) + (0.433)(0.063)(0.0070) bd (1)(0-427) + (0-183)(0.510) + (0.433)(0.063) which predicts ke = 6.3 x 1078 cal/em-sK, vs. 6.2 x 10~* observed. For the same sand when completely dry (k1/ky = 332, ko/ky = 114), Eq. 9A-11-2 for spheres gives 3 3 3 a= appa h a= ap yyy = 0.00898; a2 = TG = 0.0.0259 and Eq. 9A.11-1 with de Vries’ correction factor of 1.25 for dry sand gives Kem _ (1)(0.427)(0.0000615) + 0.00898)(0.510)(0.0204) + (0.0259)(0.063)(0.0070) 1.25 ~ (1)(0-427) + (0.00898)(0.510) + (0.0259)(0.063) predicting ke = 0.38 x 10% cal/em-sK, vs. 0.58 x 10-? observed. For the same sand when water-saturated at 20°C, de Vries’ recommended 9; values give al 2-000 o= 3 [14 0.125[(1.42/1.42) — 1] © 1+0.750((1.42/1.42)—1]] 1 2 1 a3 F + 0.125((20.4/1.42) — 1] * T+ 0.780(20.4/1.42) — al eee =i{—__? yd sg = 3 |i+0.125[7.0/142) 1] T+ 0.7807.0/14)—1]] ~"™ and Eq. 9A.11-1 gives __ (2)(0.427)(0.00142) + (0.280)(0.510)(0.0204) + (0.582)(0.063)(0.0070) oe (1)(0-427) + (0.280)(0.510) + (0.532)(0.063) predicting ke = 6.2 x 10~> cal/cm-s-K, in still better agreement with the observed value 6.2 x 107%, 9-N For the same sand when completely dry (ki /ko = 332, ke/ko = 114), Ea. 9A-11-2 with de Vries’ recommended gj values gives 1 2 1 mon 3 F + 0.125[1 — 1] ~ 1+ 0.750f1 — al 1.000 1 2 1 a3 [; + 0.125892 — 1] * T+ 0.750;332— iil = 00171 1 2 1 moet loam rosa a] meee and Eq. 9A.11-1 with de Vries’ correction factor for dry sand gives keg _ (1)(0.477)(0.00006) + (0.0171)(0.510)(0.0204) + (0.0048)(0.063)(0.0070) 1.25 — (1)(0.477) + (0.0171)(0.510) + (0.0048)(0.063) predicting ke = 0.54 x 10? cal/em-s-K, vs. 0.58 x 107? observed. (b) Equation 9.6-1 gives Ken 36 ko | * Thr Oko Iho Insertion of ¢ = 0.573 and ky = 0.0189 cal/em-s-K for the solids gives, for the water-saturated sand, 3(0.573) 189 + (2) he 142) _ 0.0189 — 0.00142 : predicting ke = 5.1 x 10~* cal/em-s-K, vs. 6.2 x 10~* observed. This is not as good as the prediction in (a) from Eq. 9A.6-11 with de Vries’ g; values. For the completely dry sand, insertion of the k value for air as ky into Eq. 9.6-1 gives Ken 3(0.573) _ Fo ~ ) * 70.0189 3(0.0000615) oe ae 0.0189 — 0.0000615 ° predicting ke = 0.30 x 10~* cal/em-s-K, vs. 0.58 x 10-* observed. The result in (a), from Eq. 9A.6-11 with de Vries’ g;, is better. Predictions of ke are more difficult for dry sand than for water-saturated sand. An oblate-spheroidal model gives little advantage according to the present data. 4-12 9A.12 Calculation of molecular diameters from transport properti a, Equation 1.4-9 and the viscosity value from Problem 9A.2 yield the following molecular diameter calculation for Argon in cgs units: 1 d= fap (3) Na -18 4 = VU3/228 x 10-4) (228% 1.38066 x 10 xm) 6.02214 x 10% x 73 = 2.95 x 107 cm b, Equation 9..3-12 and the k value from Problem 9.4.2 give the following molecular diameter calculation for Argon in egs units: 1 Ei orn k\ PM _ T (2:38066 x 10718)8 x 300 x 6.02214 x 1028/4 ~ V dor7E4 x 10° #9 x 39.048 86 x 10-* cm c. Equation 1.4-14, Tables E.1 and E.2 and the viscosity from Problem 9A.2 give for Argon in cgs units, ou, |B (Mery © Vi 1642, Ne _ 3 30.948 x 1.38066 x 10-18 x 300) "/* ~ V 16 x 2278 x 10-* x 1.1000 6.02214 x 10" xx = 3.415 x 107% em Equation 9.3-15, Tables E.1 and B.2 and the k value from Problem 9A.2 give for Argon in cgs units, 75 (xrNv\'* aK, \ aM _f 75 (1.38066 x 10-18) x 300 x 6.02214 x 102° "/* ~~ \V 64 x 1784 x 1.100 a x 39.948 = 3.409 x 107° em d. The excellent agreement between the results for ¢, and the poor agreement for d, show that the data are represented much better by the Chapman-Enskog theory than by the simple hard-sphere kinetic theory. 4-13 9C.1 Enskog theory for dense gases a. Equation 9C.1-4 can be written with pressure and temperature as the independent variables, thus: We may now use V = ZRT/p and rewrite the derivatives appearing above as (5) Ae"), Fl) )- FEA) (5), Ce), A) Cae, When these expressions are substituted into the expression for y, we get [Monza | | LaRaaER} y= -1 or y=Z| | 1-(aInZ/dinp), 1-(dnZ/Inp, ),, b. First one would differentiate the Hougen-Watson Z chart to get the derivatives appearing in the result in (a) and hence y(p,/T,)- Then for a given reduced temperature and pressure, one would calculate the right sides of Eqs. 9C.1-1 and 2; call these quantities f,(p,,T,) and f,(p,,T,)- Then KLM fePrT,) Row fuPrT) One can then read off values of from the Uyehara-Watson chart and construct a chart for the thermal conductivity. This procedure is not recommended for polyatomic gases. 4-\4 10A.1 Heat loss from an insulated pipe. ‘We use the notation of Fig. 10.6-2. When the temperatures at the inner and outer surfaces are known, Eq. 10.6-29 can be reduced to Q L 2nL(To — Ts) inal) + ine) + pee ] ‘The r; for this problem are: ro = 2.067/2 = 1.0335 in .0335 + 0.154 = 1.19 in 194+2=3.19 19+ 2= 5.19 nm "2 rs Insertion of numerical values into the above formula gives: Q 2n(250 ~ 90 F) L OR ERCTTED TPH), WEIL) , WEIN py Bt] 3207 0054 + 24.7 + 16.2 24 Btu/hr per foot of pipe {o-i 10A.2 Heat loss from a rectangular fin. From Eq. 10.7-14 we obtain the heat loss expression Q=2WLWTe Ts) +1 in which 7 is given by Bq. 10.7-16: at n= an with N= - For the conditions of this problem, _ fe _ [120 Beufhr-t?-FY(0.2 fF N=\V ip = (60 Btu/hr-ft-F)(0.08/12 ft) ~ Vi2 = 8.4641 1) = tanh(3.4641)/3.4641 = 0.2881 and ‘The foregoing heat loss expression then gives: Q = 2WLK(T —T.)-9 (1.0 f¢)(0.2 ft)(120 Btu/he-ft-F)(500 — 350 F)(0.2881) = 2074 Btu/hr \o-2 10A.3 Maximum temperature in a lubricant. ‘The parallel-plate approximation in §10.4 is used here to estimate the temper- ature rise; more accurate results will be presented in Chapter 11. Multiplication of Eq. 10.4-9 by (T, — To), and setting T, = To, gives the temperature profile T-T% Te #0 (2/6) — (2/2) ‘The maximum temperature occurs at z = 6/2; hence, with vj = QR, 1p? R? “8k 3 g/cxn-s)(7908 x 2/60 radians/s)?(5.06 cm)? .0055 cal/s-cm-C)(4.1840 x 107 g-em?/s* /cal) =88C=16F Tmax — 1 wet Bk ‘Thus, the maximum temperature in the oil is Tmax = 158 + 16 = 174°F. 0-3 10A.4 Current-carrying capacity of a wire. Consider a straight cylindrical wire of radius rz = 0.040 in, tightly covered with plastic insulation of outer radius rs = 0.12/2 = 0.06 in. Since the heat gener- ated by electrical dissipation in the wire must flow radially outward, the maximum. temperature in the plastic will occur at the wire-plastic interface r =r. The rate of heat loss from the wire reaches its largest permitted value when T = 200°F at r = rz while the ambient temperature T, is 100°F, From Eq, 10.6-29 we calculate this rate as oe = On Tamar To) [ERED rehs _ _ eae 1 = 2n(200— 100 F)/ [ 020 * (0.060/12)(1.5) = 200n/ [5.5 + 133] = 4.54 Btu/hr-ft = 0.0436 watt/em In(rs/r2) | 1 | oes brft-F/Btu Next, we equate this rate of heat loss to the electrical energy dissipation: Qmax/L = tinaeRe/L in which R./L =(xrjk.)"' = wire resistance R, per unit length ‘r(0.02 x 2.54 em)?(5.1 x 10° ohm~?em=))* 42 x 10~* ohm/em imax = maximum permitted current The current-carrying capacity of the wire is then tmax = V(Qmax/L)/(Re/L) V(W0435 watts /em)/(242 x 10-* ohm/em) = 134 amperes \o-4 10A.5 Free convection velocity. a, The solution for v, in Eq. 10.9-15 has the form ] in which A = easane and u=(y/B), v= A [ul and its average over the upward-moving stream is (of?) = A in agreement with Eq. 10.9-16. 4. For the conditions of this problem, T = <(T, + Tr) = 60 °C = 333.2 K; /T I 3x10* KK. c. Insertion of known values into Eq. 10.9-16 (with 4/ averaged as 4(T)/p(T) = (PT) = 0.1886 cm?/s) then give (o(00)y = (28047 emm/s*)((B x 10"? K-1}(80 C(0.3 em)? a (48)(0.1886 cm?/s) =23 cm/s 10-5 10A.6 Insulating power of a wall. a, Application of Eq. 10.6-9 to the data for the plastic panel gives Fya(T1 = 72) _ (0.075 Btu/hr-ft-F)(69 — 61 F) (2-1) (0.502/12 ft) = 14.3 Btu/hr-ft? 4% 6. The thermal resistance of the wall is therefore _(2-T) (61-0 F) a. 143 Btu/hr-ft? F ~ 42a Ra 10-& 10A.7 Viscous heating in a ball-point pen. ‘The paralel-plate approximation in §10.4 is used here to estimate the viscous heating of the fluid; more accurate results will be presented in Chapter 11. Mul- tiplication of Eq. 10.4-9 by (T} — To), and setting T, = To, gives the temperature profile in the ink, T-T 1 wee 5 Ee a/ON (2/8) valid when both adjoining surfaces are at temperature Tp. At x = 6/2 the temperature rise attains its maximum value BY} (L-Te)mae = 5 Insertion of the data for this problem gives (1 Te)nag = 1A! X Hg fems)(100 x 2.54/60 om? Sime x 4.1840 x 107 g-em/s*-K) as the maximum dissipative temperature rise in the ink. Thus, the warming of the ink by viscous dissipation will be negligible compared with the warming of the pen by contact with the hand of the user. 10A.8 Temperature rise in an electrical wire. ‘The maximum temperature in the wire occurs at the centerline. Equation 10.2-23 gives this value as SR? | SR ak Dh P_(keE\? 1 _ (Eke ROVE) ROVE ‘The Wiedemann-Franz-Lorenz relation of Eq. 9.5-1, with the Lorenz constant for copper, gives Trnax = Tair + in which k kT and k for copper ranges from 384.1 W/m-K at 25°C to 379.9 W/m-K at 100°C. Assuming the temperature rise to be small, we evaluate the conductivity values at 25°C=298.15K: = 223 x 1078 volt?/K? k= 384.1 W/mK _ k '« (Lorenz constant)(T) _ (384.1 W/m-K (223 x 10-8 volt? /K)(298.15 K) = 5.78 x 10° ohm=!m=!) ‘We can then calculate S, as k, (0.6 volt)?(5.78 x 10° ohm~'m7") (15 ft x 0.3048 m/ft)? = 9.95 x 10° W/m and the maximum temperature elevation in the wire as Dares — Tyg = (9:98. 10° W/m*)(0.005 mm)? moe” 4(384.1 W/m-K) = 16x 10" 40.77 0.77 K (9.95 x 10* W/m*)(0.005 m) 2((5.7 x 5.6782] W/m™-K) b. The temperature difference across the wire is 4 orders of magnitude smaller than that in the surrounding air, as shown by the terms in the last line of the calculation. 10-3 10B.1 Heat conduction from a sphere to a stagnant fluid a. The heat is being conducted in the r direction only. Therefore, we select a shell of thickness Ar over which we make the energy balance: Arg, 4mrq,| -4n(r+Ar)"q, . = 0 or 4n(r?q,)), ~4n(r°4, } near We now divide by Ar and then take the limit as Ar goes to zero nl Haas (PI), 490 Ar - We then use the definition of the first derivative to get dye \_ a(.aT al q,)=0 and 4(n @)- 0 In the second equation we have inserted Fourier's law of heat conduction with constant thermal conductivity. b. Integration of this equation twice with respect to r gives aT ¢ rO-=C, and T=-1+C, The boundary conditions then gives C, =-R(T -T..), C) =T., and c. The heat flux at the surface is aT| 1 KT, -T. leg =k =+kR(Tp - ~ Misa te h(T,-T. ar ler Ter (Px -T.) so that h=k/R=2k/D and Nu =2. d. Bi contains k of the solid; Nu contains k of the fluid. 10-4 10B.2 Viscous heating in slit flow Equation s 10.4-5 and 10.4-6 are still valid for this problem. Since at x=b, g, = -k(dT/dx) = 0, we get from Eq. 10.4-5: 2 2 a, 2 -ni{ 2) =C, or G =-Hh And from Eq, 10.4-6, we get C, = Ty. Then substitution of these expressions for the integration constants into Eq. 10.4-6 yields 242 yy? x? pe ) Xora Ty When this is rearranged in dimensionless form we have (9-38) Jo-10 10B.3 Heat conduction in a nuclear fuel rod assembly The differential equation may be set up following the procedure in §10.2, by replacing, S, by 5, in Eq. 10.2-6. Then, when Fourier's law with constant thermal conductivity is substituted into the thus modified Eq. 10.2-6, we get ne (rte) = ; be Ar ar salt x) for the heat conduction equation in the fuel rod. In the cladding a similar equation, without the source term, is appropriate: d(_aTe heal” on ° The boundary conditions in this problem are B.C. 1: T; is finite B.C. 2: iSite B.C. 3: ~k, (dT; /dr) =—k, (dT -/dr) B.C. 4: ke (dT¢/dr) =, (Te -T1) Integrating the above differential equations twice gives aT, Spot(y, BP) LG ae Ss dr 2k RR 2) arr Syot? | T,pe- Sah (1 ge eCnr cy Tc =Cylnr+Cy 4k, The constant is zero by B. C. 1, since the temperature is not infinite at the axis of the fuel rod. From B. C. 3, we can find C;: She 4) =—20he (142 Ca (tg lo-ul From B.C. 4, we get C,: 2 c=t.-( Ke inne Jeet. o[ ke sink. 9h (143) c Reh, Roh, ‘And finally C, can be obtained from B. C. 2: SaoR? (4, B) , SwRE(4, bY R C=, +500 (1 4) 0 H(t 4) inke a ast on Tat, Then we can get the maximum temperature at the axis of the fuel rod: 2 2 Tr =, + SRE (142) SBE Bink ke } 4k 4)” Oke Re Roh, \O-12 10B.4 Heat conduction in an annulus a. The energy balance on a cylindrical shell of thickness Ar and length L is 2arLq,|, -2n(r + Ar)Lq,| 0 or 2aL(rq,)| -2aL(rq,)),,,, = near Ina When this equation is divided by 2a and the limit is taken as Ar goes to zero, we get d lh) = which may be integrated to give gC, or -KE-G The thermal conductivity varies linearly with temperature, so that T-T) k=ky +(k, -(Z Fe a +(b,fo)® ° Then aT =o do = —[ko + (ky - ~k OE or ~(T,-Ty)[ko + (ki ko jel This first-order, separable differential equation may be integrated: -(T, -To)[kp + (ky — ky JOJO = C, Inr +C, The constants of integration may be found from the boundary conditions: @(r)=0 and @(r,)=1. 0=C\Inry+C, and -(T,-Tpy)[ky +4(ki -ko)]=CiInr, +C, When these relations are subtracted, and equation for C, is obtained: \o-13 (T, c= fT a +) and C, may also be obtained if desired. The heat flow through the wall may then be obtained: Q=2argL4,|,., _~2aln{ S = 2a ae eee (ko +k) b. Let the ratio of the outer to the inner radius be written as 1,/r9 =1+ €, where € is very small. Then use the Taylor series for the logarithm as given in Eq. C.2-3: In(1+e)=e-4e? +te°—---. If we keep just one term of the series, then this corresponds to €=(14/t9)-1=(%1 -T0)/ro When this is substituted into the expression for Q we get Q=2nbro[ (ko +h) PE ; This is just: area times average thermal conductivity times a temperature gradient. 1O-14 10B.5 Viscous heat generation in a polymer melt We can start with a modification of Eq. 10.4-4: [""* do, aT . 1Zao[nm =< Since the velocity gradient is positive in this problem, the absolute value operation is not needed: av (do,)"_ «Emo i) = te, dx When the linear velocity profile is inserted, this becomes Fp ORC -« Ema 3) =<, Integration with respect to x gives 1 fo -kT = Cat Sme'( 2) +Cy The constants are then determined by the boundary conditions given in Eqs. 104-7 and 8, and final result is Bebe) (-0] or T-T) x1 + 2) =*41pr,2(1-* T-T) b 2 "bb which should be compared to the Newtonian result in Eq. 10.4-9. io-15 10B.6 Insulation thickness for a furnace wall Let the regions be labeled as follows: Refractory brick "ol" Insulating brick "12" Steel "23" and we may used the formulas given in Eqs. 10.6-8, 9, and 10. ‘The minimum wall thickness will occur when T, = 2000°F. If for the sake of being on the safe side, let Ty = 2500°F. Then for the region "01" the thickness must be koi(To-T) _ 3(4.1+3.6)(2500- 2000) _ 0.39ft 1% 5000 X—Xp = Here we have taken the thermal conductivity of the refractory brick to be the arithmetic average of the values the thermal conductivity at 2000°F and 2500°F (the latter estimated by linear extrapolation from the given data). For the remaining two regions, we may add Eqs. 10.6-9 and 10 t get XX X3-%, ,-T,=a{ Ba =4) 12 28 or, taking the steel temperature to be 100, - J 2000-100 = 5000] —*2=*1_ , (0.25)i2 4(0.9418) 26.1 This gives x, — x, =0.51ft. 10-16 10B.7 Forced-convection heat transfer in flow beween parallel plates a, Since the temperature depends on both x and z, we make an energy balance over a region of volume WAxAz, in which W is the dimension of the slit in the y direction. The various contributions to the energy balance are: Total energy in at x: e,|,WAz Total energy out at x + Ax: Cele gar WAZ Total energy in at z: e,|, WAx Total energy out at z+ Az: Colese WAX Work done on fluid by gravity: pv,g,WAxAz When these terms are added together and divided by WAxAz, we get since gravity is acting in the -2 direction. Now we use Eqs. 9.8-6 and 9.8-8 to write out the x and z components of the combined energy flux: or ox ey = 1,2, +a={u Be), -k e, =(}002)0, + piv, + 4... +4, 4002)0, +(p—p°)o, +0¢,(T-T°)o, -(2" we), “KZ Substituting these expressions into the energy balance, and making use of the fact that v, depends only on x gives Jo-"] C0, f PT, 2), (2) + Py Pe Pens ae ont at) Max dz Bae 8 The term in the last parentheses is zero by the equation of motion, the term just before that is the viscous heating (which we neglect), and in the first parentheses we neglect the heat conduction in the z~ direction. b. Hence we get A xy lar _ aT 2 #0 orl 1-(3) eno or (lo) 5E= GF with the boundary and initial conditions: at o=+1, +(90/d0)=1, and at =0, @=0. c. For large z we propose the solution @(0,6)=Cy¢ + ¥(o). Then ‘¥(c) has to satisfy the ordinary differential equation 2 SE =c(1-07) which is easily integrated. The expression for @(a,¢) is then. (0,6) = Co +Cy(}0? -ho*) + Co +C, Application of the boundary conditions at o=+1 gives C,=0 and Cy =}. The remaining constant has to be obtained from an integral condition: $= [0(0,0)(1- 0 )ao This gives C, = - 3. Combining these results we get @(0,6)=36+3($0? -b0")- 2% which is in accordance with Eq. 10B.7-4. \o-18 10B.8 Electrical heating of a pipe For the assumptions made in the problem statement we may assume that temperature is a function only of r and that or ‘Orla Glen =0 therefore that all generated heat must leave from the surface at R by Newton’s law of cooling Vs, = Ah(T, -T,) or Tp = T, = T, + RU-K)S, /2h ‘These relations provide a basis for setting up a differential equation as well as giving the boundary conditions for integrating it. The next step is a shell balance over the region between any radial position “r” and a nearby position “r + Ar 2nrLg,|, + 2erLS.Ar = InrL.q,| near 1] 74a a or A) Mirlesar “Urb | 2 Ong = r Ar ror It follows, just as for Ex, 10.2, that or _ 1S, or 2 r However, the above boundary condition now gives 2 o, = ERPS 2 a and 1 (RPS, 2k ERPS. «6 2k Therefore T=T+ FSA =(r/ RY ]- 22 In(r/n)} and T, is related to the surrounding temperature T, by the above equation. \o-\4 10B.9 Plug flow with forced-convection heat transfer a. We can adapt the solution in §10.8, by replacing 0, max by Vp, and omitting the factor [1—(r/R)*] in Eq. 10.8-12. Then we can give the results corresponding to those in §10.8 as follows: (10.8-25) = Ole, OEE 1d(,d¥)_ (10.8-26) ra(«#) =Cy (10.8-27) O(E,£) = CoS +4Cp6? +C, INE +C, (10.8-28, 29, 30) C,=0; Cy=2; C,=-$ With these coefficients, Eq. 10B.9-1 follows. b. The solution of Problem 10B.7 can be adapted by replacing V.max by Up, and omitting the factor [1-(x/B)*] in the energy equation. The results, analogous to those in part (a) are (10.8-25) $= [elo Dodo (10.8-26) (10.8-27) (6, L)=Cy6 +41 Gyo? +C\o + C, (10.8-28, 29, 30) C,=0; Cy=1; C,=-4 With these coefficients, one obtains Eq. 10B.9-2. io-20 10B.10 Free convection in an annulus of finite height a. The appropriate simplification of the energy equation is ld(_ dT ons) which has the solution T=C;Inr+C;. When the constants are evaluated using the boundary conditions we get Eq. 10B.10-1. b, We make use of the linear approximation for the density as a function of temperature: P=, p,B,(T-T;) This leads to the equation of motion =plA(,4:\_(% - o=nt dr i) (2+ 0.8) +o86(r T,) Subtsitution of the expression for the temperature distribution from Eq. 10B.10-1 into the equation of motion, and multiplication by R?/u gives 1 d(,do,)_R?(dp 8B, (T, ~T,)R® baa 6%)-E (Brn) [AeA TE Ing which is just Eq. 10B.10-2, along with the definitions of A and B. c, Integration of Eq, 10B.10.2 then yields $AG? +4B(E?Iné-E*)+C,InE+C, The constants of integration are _(A=B)(1- «?)- Bk? Ink C,=-}(A-B) and en AES The velocity distribution can then be written as \o-2) 40,=(8-Al(1-@)-(1- BE |an(e? - "ng (*) This can easily be shown to satisfy the boundary conditions for the velocity at the inner and outer walls of the annular region. It remains to evaluate the constant A (as we did in §10.9) by equating the total mass flow through a cross-section to zero: figl00.)rdrdo=0 or f'[o,-p,B,(T-T,)]o, Ede =0 Thus we have to evaluate A from fear. -T, OEE]: Ink {e-a[(1-e)-(1-« BE + 0(@ «Ying e460 First we note that Bx AT =T, —T, and therefore that A« AT. To be consistent with the Boussinesq approximation, we should therefore neglect terms that are proportional to (AT)*. This eliminates the need for doing many of the integrations in the above equation. When the necessary integrations are performed we get, after a modest amount of algebra (B-A)= tee foe ase se] 1-K?) +(1- «Jing When this is substituted into (*) along with the expression for B given just after Eq. 10B.10-2, we get Eq. 10B.10-3. |o-22 10B.11 Free convection with temperature-dependent viscosity a. It is instructive and helpful to begin by going back and re- solving the problem of §9.8 using the notation of the present problem. Equations 10.9-9 to 11 in dimensionless form are 6, =P+iGry with 3,(41)=0 ay? This may be integrated to give 8, = Cy + Cy +4 PI? + $Gry® When the boundary conditions are applied, we get C,=-Gr and C, =-4P. Thus the velocity distribution is #Gr(y? - 7) +4P(y* -1) which is equivalent to Eq. 10.9-12. Next we apply Eq. 10.9-13, which, in dimensionless form is Mya a Pe yi : [ia-br9)[ Gry - 7) +4P(y? -1)]ay =0 in which b; = 4BAT; this equation has to be solved for P. In doing the integrations, the integrals over odd powers of give zero, and we get Gr(-$-3+-3)bp + P(-4-244-3)=0 from which P = 3Grb, When this value for P is substituted into the dimensionless velocity distribution we get 8, = hGr(y? - 7) + 5Grb, (7? -1) On the right side, the first term contains AT and the second term contains (AT)?. In §9.8 we chose to neglect the term in (AT)*, which is 10-23 all right if AT is quite small. This led us to the result in Eq. 10.9-17, which is just the first term in the above equation. b. Now we address the temperature-dependent viscosity problem in Problem 10B.11. The equation of motion is then: {ary )-B +00 We now introduce the temperature dependence of the viscosity (Eq.10B.1-1) and density (Eq. 10.9-6) to get do, dy 4(__ pte.) 5 pear Login ts) «ag PsB(T -T) We now multiply this equation by B°7/71” in order to get the equation in dimensionless form (using quantities defined in Eqs. 10B.11-2 and 3): with ,(#1)=0 Integration of this equation gives 8, = Cy + Cy + 3(P-Cyb, Jy? +4H(EGr- Pb, iy - hGrb,y* Application of the boundary conditions then gives C,=-4(}Gr-Pb,) and C, =-3P+4Grb, +4 Pb; In (a) we found that P was second order in AT Therefore the dashed underlined terms in C, and C, can be expected to be of the third and fourth order respectively--thus two orders of magnitude higher than the remainder of the terms. Therefore we drop the dashed- underlined terms at this point. Then the dimensionless velocity distribution is io-24 (-4P + Grb,)-bGry+4(P + kGrb, )y? + Gri? - Grd, 7! Next we deterimine P from the requirement that there be no net mass movement upward in the region between the plates (i.e., Eq. 10.9-13): 0=2(-4P +4Grb, )+3b;(4Gr) +3-4(P-+4Grb,)- 3br(4Gr)-24Grb, When this is solved for P we get P=4Grb, + 4Grb, and this is second order in AT as we had anticipated, and the dropping of the dashed underlined terms is fully justified. When this expression for P is substituted into the dimensionless velocity distribution we finally obtain: 4Gr(9? - 7) + 4Grb, (7? -1)-- Grd, (7? -1)(57? -1) The first term is the "basic solution" in Eq, 10.9-17, the second term is the deviation from the basic solution when terms second order in AT are accounted for, and the third term is the second order term that enters in when the temperature dependence of the viscosity is taken into account. The basic solution contains only odd powers of the coordinate, whereas the second order terms contain only even powers. It is clear that this equation satisfies the boundary conditions. [0-25 10B.12 Heat conduction with temperature-dependent thermal conductivity The thermal conductivity is a function of the temperature: Ty-T k=ky-(kg-k, L = ae ARs) a. Since T is a function of @ alone, we make a shell energy balance thus: 0 ~ (ko ~ ky )O Golo(t2-")E-Aolosao("a ~N)L=0 Dividing by (r, —r, )LA@ and taking the limit as A@ > 0 gives Mo _ do” Then inserting Fourier's law with a temperature-dependent thermal conductivity, we get Tate aol? gl “ho (Ho-ee0) SF Integrating once we get aes 0 (ko - (ko - kn )®). 7 and a second integration gives kyQ -4(k —k,)O? =C,0+C, When the constants are determined from the boundary conditions, ©(0)=0 and ©(z)=1, we get C,=0 and C, =(ky +k,)/2m, so that the temperature distribution in the solid is lo-26 kO-3(ky-k,)O* _@ Hky+k,) Then the total heat flow through the surface at 0 =0 is then ki aT abi), Ler (1d0 -T. nj a To-TeNGI°(258)],_ art -1, ff? (A a nr foes drdz \O-27 10B.13 Flow reactor with exponentially temperature-dependent source For this problem we may take Ty to be zero, since there is no particular need to do otherwise. Then the temperature is made dimensionless by dividing by the inlet temperature: @=T/T,. Also, the quantity S,, may be identified with K of Eq. 10B.13-1. Therefore the quantity $.,F(@) in Eq. 10.5-7 becomes SaF(®)= Kew{-5) = Kexp{-4) where A= ra We may then proceed to Eqs. 10.b-21 to 23: Zonel: @'=T'/T;=1 or T'=T, ot 1 ot A Zonet: fy Fe =f on(+4}0 =NZ To do the integral we make a change of variables A/@ =x so that d@ = -—Ax dx. Then the integral becomes Ale I, etx dx =NZ xen WAG etx tde=+al] A h This last integral can be written as a power series. We then get: fe" =NZ or ox" Aexp(4/8")- Ae- alins +55] h (a/exy" nin exp(4/0")-e- ST +In(a/0")+ 5, \0-28 for 6" as a function of Z. However it is easier to calculate Z as a function of ©", since that does not in- volve any trial and error. Zonelll: @"=0"|, Another way to treat Zone II: If the temperature rise is not too great, we may expand e*x in a power series about x = 1: efx? = ef -(x-1) + $(x- 1) -A(x—1) + H(x-1)* +] Then ~ As etx tady =—Aef"[1- (1-2) +312) =e (Oy ay lay A yA_,’,3(4_,)P 4 (Se-1)-(B-9) +(e) + for ©" as a function of Z. Also, by this method it is easier to calculate Z asa function of ©", since that does not involve any trial and error. 10-24 10B.14 Evaporation loss from an oxygen tank a. The fact that the thermal conductivity varies linearly with temperature can be written as k-ky _ T-Tp =0 kk T,-Tp which also serves to define the dimensionless temperature. From an energy balance on a solid spherical shell of thickness Ar gives =0 In+ar (427°4,)| ~(427°a) Division by Ar and then taking the limit as Ar goes to zero gives 4 (29 \= ala.) =o Then, introduction of Fourier’s law gives A, @) 4 2 ®). aS or aK ar)=° We now integrate once with respect to r to get do C, Gq or rag 82. dr Inserting the expression for the thermal conductivity as a function of temperature, and integrating again, we find {lo + (kiko) ]t0=-2 +c, ee FO +3(ky ~ky)O? S46, 10-30 Now we apply the boundary conditions: B.C. at ro: B.C.at 1: Subtracting the first equation from the second eliminates C, (which we are not going to need anyway) and gives an equation for C,: 2(k + koro (1-7) The heat flow inward through the spherical shell at rg is now aT de Q= smu) = Aara{ +o (t,-T9)2) "0 Ir=ro The derivative of the dimensionless temperature can be found thus: GO _3(Ky+ko)rors 1 dr (n=%m) Evaluating this expression at r = rp gives Putting this expression into the formula for Q, gives (0-3) = 4g op ee 2 = 4mi( +4} ky +k, \{ T, -T, mama *2 Al uta) be : =o b Since most of the quantities are given in the c.g.s. system, we will convert all quantities into that system. The quantity Qy may be evaluated thus: _ (16.2x10*)(4.136x10") 183 Qy = 4m (36 x 2.54)(48 a (a) = 2.8182 cal/s The factor 4.136 x10™ needed for converting the units of the thermal conductivity is obtained from Table F.3-5. The rate of evaporation of oxygen is then: 2.8182 R= 600) = 6.201 g-ml, (# Je ) = 6.201 geml/hr or ew (6:201)(32) F000 0-198 kg/hr 10-32 10B.15 Radial temperature gradients in an annular chemical reactor a. Consider a cylindrical shell of thickness Ar and length L. We make an energy balance over this shell, by paralleling the derivation in Eqs. 10.2-2 to 6, replacing the electrical heat source by the chemical heat source S,. Hence we have (cf. Eq. 10.2-6): d qth) = Sr Into this we substitute Fourier's law for heat conduction in the r- direction to get d _ aT + {- et Gp aes or babel? a provided that the effective thermal conductivity does not vary with position. The boundary conditions are: B.C. 1: at r=1, T=Ty aT B.C. 2: tron, 2-0 at rar, b. A natural choice for the dimensionless radial coordinate involves division of r by either the inner or outer radius; we choose the inner radius and write £=1/r9. Then the differential equation becomes: Keg 1 df aT ned eta) From this, it is evident that k.(T-Tp)/S.r3. is dimensionless. By inserting a factor of 4, we get © = 4kyg(T-Ty)/5,13. The insertion of the factor of 4 is arbitrary, but it makes the final dimensionless answer somewhat simpler. In terms of these dimensionless quantities the partial differential equation becomes: keg 1d ( ,a(T-To) or ‘tral? & ac) = [0-33 with boundary conditions B.C: até=1, @=0 B.C.2: at€=1, d0/dé=0 c. Integration twice leads to @=-£+C,InE+C, Application of the boundary conditions gives C,=2 and C, =1, so that @=1-€74+2Iné d, The dimensionless temperature at the outer wall is then @(a)=1-a? +2Ina (a=n/1) The volume averaged reduced temperature is: PT g? +2ing)eagao _[3e?-36* +6?Ing-367] ff; agao pel (a? +1)+ aes (@)= e. The temperature at the outer wall is =T, +22 T=To+ Gh 2 (4800 Ss s)(3.97x10° be )( 2.5412 =\(28) =900+ hr-cm’ cal ft A 12 40.3) S.r6 (1-4? +2Ina) ql [0-34 x(1- (1.11)? +2in(.11)) = 900 + (0.681)(1-1.23+0.21) = 900 + (63.3)(1-1.23 + 0.21) = 89°F f- If the inner and outer radii were doubled, the temperature difference between the walls would be four times as great. 10-35 10B.16 Temperature distribution in a hot-wire anemometer a. We start by writing a shell balance over a segment Az of the wire: P Gel.(42D?)— delesge($2D?) + yy (t7? ee A(T -T, )mDAz=0 Division by }2D?Az and taking the limit as Az — 0 gives — Felesae ~4el, ,2_An(T-T1) a0 Az k Dp Taking the limit and using the definition of the first derivative gives: _dg, _4h(T-T,) Pr CT 4n(T-T,)__ 2 dz D kK “Ee D k, Let us now define the dimensionless quantities _I-T, H 4h? Pe L iT kD KAT, Now when the differential equation is multiplied by L?/kT, it becomes, in dimensionless form, ee dc? This is a nonhomogeneous, second-order differential equation, the solution of which is the sum of a complementary function and a particular integral: -H@=-J with @=Oat (=+1 Ocx, = C, cosh VHE +C, sinh VHC; >, =J/H Since the solution must be symmetric about ¢=0, the constant C, must be zero. Therefore, the complete solution has the form 10-36 J J/H = nvHt+L =- ©=C, cosh VHE where C,=-— where the boundary condition that @=0 at ¢=1 has been used. Therefore the solution to the problem is i 2 hh [4h/kD o= L{1_soshvHe or T-7, = L2(1-soshysiikDz H\ coshVH 4hk,\" cosh./4h/kD L c. First we evaluate the cosh-function in the denominator, creating dimensionless ratios with internally consistent units: =cosh [Ate = (#\ (4)- - kD D. 4(100)((0.5)/(2. an = cosh 2.535 = 6.348 Then, at z = 0, the quantity in the large parentheses is 1 1-—————_ |= 0.1 ( cosh. imi | 088 Then the volume rate of heat production is P _(T-7,)(4h) __(30)(4)(100x5.678) k, (0.842)D — (0.842)(0.127 x 0.001) = 6.371 10? J/cm?s= 6.37110" amp? -ohm/cm?* = 6.371x 10° J/m?s since 1 amp: volt =1 amp? ohm =1J/s. Then current (amp) = = [6.371x10? x 10° (}2(0.0127)") = ¥63.71x108 (1.267 x10“) = 1.01 amp 10-37 10B.17 Non-Newtonian flow with forced-convection heat transfer a. This problem can be solved by modifying §10.8 as follows: First replace (1- £*) by (1- **1) in Eqs. 10.8-19, 25, and 26. Then the power-law analog of Eq. 10.8-27 becomes ees a6.g=c6ro/ £- eS) +C,né+C, in which the constants of integration are determined to be C,=0 Cy = 2As+3)/(2+1) Cy =[(s+3*)-8/4(s + 1(s+3)(s+5) With these values for the constants, we are led to Eq. 1B.17-1. b. This problem may be solved by modifying Problem 10B.7, part (b), by replacing (1-0) by (1- 0°**) in the differential equation for ¥ as well as in the integral condition. This leads to the following equation for @(0,£): 2 33 O10, = CL +l FS ]¥ Gor, with the constants of integration given as C,=0 st+2 a= 4 _ _8+2( (s+2)(s+3)(2s+5)-6 2 s+ 1 6(s+3)(s+4)(28+5) Then we are led to Eq. 10B.17-2. 10-38 10B.18 Reactor temperature profiles with axial heat flux a. The differential equations in Eqs. 10.5-6 to 8, and the boundary conditions in Eqs. 10.5-9 to 14 are still valid, but with a linear form for the function F: in which S,, and Ty are constants describing the linear dependence of the reaction rate on the temperature. Then, if we use the dimensionless downstream coordinate Z=2/L, a dimensionless quantity B= pC, 0L/k.,.., and a dimensionless chemical heat source N=S,,L/pC,v9(T,-To), the differential equations for the three regions become: ide! _de! 16" ao" ign, 120" _ aot BdZ dZ’ Bdz dz Bdz dz The boundary conditions are: B.C.1: at Z=-00, el=1 B.C.2: at Z=0, oe! =o" B.C.3: at Z=0, do'/dZ = do" /dZ B.C.4: at Z=1, et =e" B.C.5: at Z=1, do" /dZ = do" /dz B.C.6: at Z=0, @" = finite That is, we specify the condition far upstream from the reaction region, and we require that there be continuity of the temperature and the heat flux where the regions join. The solutions of these linear, homogeneous, second-order differential equations are then Region I: @1(Z)=C, +C,e™* Region II: @"(Z)=Cye™” +Cye™ for m, #m_ Region II: ©" (Z)=C, +Cce™ 10-39 where m, = 4B( + 1-(4N7B)). When the integration constants are determined, we get: Region I: @'(z)=14; MA) DJetmemiz egion (2) -( mg em | m, mgm Region II: oN(z)={ meen mere egion (2)-( a im, +m.) i my ~ m2 Region III: o"(z) =| egion (Z= (= rma |e These results correspond to Eqs. 10.5-21 to 23. b. In preparation for taking the limit for B going to infinity, we first find the following Taylor series expansions m, = 4B(1+(1-4(4N/B)+-- m_=4B(1-(1-4(4N/B)+-- =B+N+0(B") N+0(B") It is important to note that the limiting value of m_ for infinite B is not zero, but N. Then, the limiting expressions for the temperature profiles in the three regions are lim@!(Z)= pn) (a Be Bol * ave m,(m+-Je™e"™2 — m_(m,tJe™e™”) yz E rn ve i 3 pmo 2) =f Enon These results are consistent with Eqs. 10.5-21, 22, and 23. To get the second equation above from Eq. 10.5-22, we have to substitute F(@)=© and integrate. Je mye | =1 (since Z is negative) lime" (Z) 10-40 10C.1 Heating of an electric wire with temperature-dependent electrical and thermal conductivity a. Rewrite Eq. 10C.1-3 as kT lela Leta] This is easily transformed into Eq. 10C.1-4, with B= k,gR?E*/k,Tol? . Use of Eqs. 10C.1-1 ane 2 leads directly to Eq. 10C.1-5. b. When Eq. 1C.1-8 is substituted into Eq. 1C.1-5 we get -3 elt- a {4B(1- ¢7)(1+B0,+--)} ~ea{]8(1-2 (14 80,49] oI6-Z (001 E\t+ 80,4) = B(1- 6, {4B(1- (14 BO, +} ~Br{tB(1- 2 )(1+ Be, +) +) Equating terms containing B', we get Faledn-e) which, when the differentiations are performed, gives an identity, B=B, as it should. Then we equate the terms containing B”, which gives 1d[ed yan 1 Sf ] 44) ef p(1- 4Bé“4B(1- raslag? (1-680, |e jel ade et) = ~,B-4.B(1- £*) Division by 4B? and performing the differentiation in the a, term then gives the differential equation for ©, \o-Al d d 2 2 Fal ege-# es --ast-28") 00-22) Performing two integrations then gives (1-€?)0, =~}a,(2€? - &*) + £B,(48? - 4) +C,Iné+C, The constant C, must be zero in order to satisfy the boundary condition at the tube axis, and the application of the boundary condition at the tube wall gives 0=-}a,+ $8, +C, Taking the difference between the last two equations gives (after dividing through by (1- &*)) ©, =}ay(1-é)-48,(3-2") in agreement with Eq. 10C.1-9. c. Rearranging Eq. 10.C-10 we have Into this equation we substitute Eqs. 10C.1-1 and 2, to get (1-2,0-@,07- =(1-B,0-B,0?----)(1+0) =1-B,0 - B,0?---+0 ~ ,0? - f,0° =1-(B, -1) (6 - B,)@?—-- Equating the coefficients of equal powers of @then gives @, = B,-1, Gt = By ~ By, and so on. Then Eq. 10C.1-12 follows directly: © =4B(1- €*){1- 43 [(6, +2)+(B, ~2)87}e} \o- 42 10C.2 Viscous heating with temperature-dependent viscosity and thermal conductivity a. The shell momentum and energy balances lead to d( aT dv, _ ate) (G) =° Mulitply the first equation by b?/1190, and the second by 67/1197 to get Eqs. 10C.2-3 and 4. b. When Eqs. 10C.2-5, 6, 7, and 8 are used in the equation for temperature, and the coefficients of like powers of Br are equated, we get for the terms containing the first power of Br This equation has the solution @,=-4}C}é?+C,€+C,. The constants of integration are determined from the boundary conditions that ©,=0 at €=0,1. Thus the following result is obtained: =4C3,(é-€?) We now tur to the ¢ equation (along with Eq. 10C.2-2) which is e, $E=Gi(1+ 8,0 +8,0*+--) The expansions in Eqs. 10C.2-6, 7, and 8 then give d el + Brg, +--+) = (Cio + BrC,, + Br°C, +) (1+B,(BrO, +Br?0, +---)+--) Equating the coefficients of the zeroth power of Br, we get J0- 43 He=Cn Integration then gives ¢)=Cjé+C). The boundary conditions (0) =0 and ¢(1)=1 lead to %=§ — andhencealso ©, =4(£-&?) These are just the results obtained in §10.4. We next go back to the energy equation and equate the terms containing Br?: d dO, dO, a (ae. “E* Se) 20+, Into this we insert the expression for ©, to obtain ; Geb =— (1-68 +68") 2, -F0(E-24) Integration twice gives 1 0,=-S(3e-e+de)-cue-da(2e—5 &) $C, +Cy Then we use the boundary conditions ©, = 0 at € =0/1 leads to a1 1 1 o,=-S(Re ede) (-6428-8)+Gu(E-2) The velocity distribution that goes along with this approximation is ¢,. Equating the terms linear in Br gives the differential equation Fa =Cy +B,0, =Cy +48,(E-27) Integration gives lo-44 10C.3 Viscous heating in a cone-and-plate viscometer a. First we make a table of "translation" to get from the plane Couette flow to the cone-and-plate flow: Plane Couette Flow Cone-and- yw. (Fig. 10.4-2) (Fig. 2B.11) S=2/b g=V/Vo 9 =0,/2% G =0,/1Q 2 /b Yo Br=y0}/k.Ty Br(r/R)” = (uR?Q?/kyTo (r/R) The torque on the cone-and-plate viscometer is given by the force times the lever arm integrated over the entire plate: T, = 2m {0 opp ap Par But sind 9%), 413% sin@ Toy =~ | 09 B96 1 Oy is a suitable approximation for the cone-plate system, with 0 ~ 1/2. Therefore the torque expression becomes for a fluid at temperature Ty and viscosity 14g rdr(*) v=o an, T, = 2a Hos When there is no viscous heating, the tangential velocity is vy =12(v/Vo) Combining the last two equations gives, for no viscous heating: 10-45 = 2H QR® T, 20° 3M co) When (*) is divided by ("), we get Te 498] gp Too bag.” b. We now consider the system with viscous heating and use the cone-and-plate variables in the correspondence table. This gives fa = 3f,(1-4BrE*B,+~E%dE where we have used the dimensionless velocity expression from part (cin Problem 10C.2. When the integration is performed we get T, rs =1-45%6, +0(Br’) To get the higher-order terms shown in Eq. 10C.3-1, one would have to go back and get the higher order terms in part (c in Problem 10C.2. 10-46 10D.1 Heat loss from a circular fin a. A heat balance over a ring of thickness Ar gives (2n(2B)rq, |, -(22(2B)rq, )|,,,, -2-2ardr-h(T-T,)=0 Divide by 27(2B)Ar and take the limit as Ar goes to zero to get (r4.),—(°4eVae 1h lim gir-T.)=0 or 1390 Ar from which we get the differential equation a th (dt) _ rh —-“(rq,)- “(7 -T,)= £(,@)_ir_t je a()~BT-T)=0 or wa) pe T~T)=0 In the second form, Fourier's law has been introduced. Next we rewrite the equation in terms of dimensionless variables: € = 1/Ry and © =(T-T,)/(To-T.): 1d{,d0\ (hR> ea(*@)-(e- This equation has the following solution (with B? = hR3/Bk): ©(£)=C,1(BE) + C.K, (BE) in which I, and Ky are zero-order Bessel functions. The constants are determined by use of the boundary conditions: @(1)=1 and dO /dé,_ gg, = 0- This leads to 1=Cylg(B)+C,Ky(B) and 0=C,Bl,(BR,/Ry)- C2BK,(BR,/Ro) These two equations can be solved simultaneously to give BK,(BR,/Ro) * T(BYBK (BR, Ro) + BI(BR,/Ra)Ko(B) 10-47 cs Bl,(BR,/Ro) ° To(B)BK,(BR,/Ro) + Bl, (BR:/Ry )Ko(B) Hence the dimensionless temperature profile is (g) = (PEK (BRR) (08, Ro BE) 1o(B)Ki(BRi/Ro) + 11(BR,/Ry )Ko(B) b. The total heat loss is then: do| de Ee ane 1,(BR,/Ro)Ko(B) + 1o(B)K; (BRi/Ro) Q= 2aRy(2B)kaE = 4nBk(T) -T,): leaky = 4nBkB(T, nf lo- 48 10D.2 Duct flow with constant wall heat flux and arbitrary velocity distribution a. The analogs of Eqs. 10.8-19 26 are 3 _10(,00 qa, wees fa) ER After multiplication by & the second of these can be integrated once to give tee 55 = Cole EOE)HE +, Then division by & and further integration gives Eq. 10D.2-1 (along, with the defintion in Eq. 10D.2-2). The constant C, has to be zero, since the temperature must be finite at the tube axis (B.C. 1). Then from B.C. 2 we get C, = 1/I(1) using the equation immediately above. To get the remaining constant, we use the integral condition analogous to Eq. 10.8-24, to get C= flo(e, coe )edé = loos +Cafp EU(E)aE +C, poeyeas = CofM(A) + Co fy 0(8)8 [0 E71(E AE f'E + C100) = C+ (rey ole )e[ EE u(E)az ts + +C.1(2) From the last line, Eq. 10D.2-3 follows immediately. b. The wall and bulk temperatures, in dimensionless form, are given by @, = O( =0,2) and Substitution of the result in (a) into these expressions gives 10-43 1,1 Al, @o= me ry" s es fo ofa hae +c, 0,=— Tan ‘Tor Subtracting and then changing the order of integration in the second expression, we obtain 1(1) 4 o= 1m ag pM poe cas} ° irae The inner integral may now be written as the difference of two integrals, the first going from 0 to 1 and the second from 0 to €. But these two integrals are, respectively, just I(1) and I(€). Hence we get finally 1 ply awe sg aes eT Ee 1(1) plOhag o— To Thus, the first two terms cancel each other, and the last term gives Eq. 10D.2-4. c. From the definition of the dimensionless temperature, we get Ty-Ty 9oR/k Taking the reciprocal, and replacing R by D/2, we get Eq. 10D.2-5. d, The quantity I(1) is the ratio of the average to the maximum velocity in the tube. @,-9, = 1(M) = 9EdE = (1/0. max) fo 22848 = (02) / sae (0-50 11A.1 Temperature in a friction bearing. ‘The method given in the solution of Problem 10A.3, based on Eq. 10.4-9, gives the maximum temperature rise in the lubricant as, 1 pO? R? Tmax — Th = 5 _ 1(2.0 g/em:s)(8000x/60 rad/s)?(2.54 em)? © 8 [4.0 x 10-4 x 4.1840 x 107] g-cm/s?-K) = 16.9C and the maximum temperature Tinax as 217C. Next, we consider the analysis given in §11.4, which includes the curvature effects. Since T, and T, are equal, Eq. 11.4-14 is applicable and gives —_ [ 21nG/x) meV (G/s? =) as the location where the temperature Tmax occurs. For this problem, « = 1/1.002; hence, __ [21nG.002)_ . Sax = 4 TgoajF = Ty = 0.999001165 which location is essentially in the middle of the gap. To evaluate Tmax, we multiply Eq. 114-13 by (Ty — T,) to make each term finite; the result (after division by «* in the numerator and denominator) is ron OE ore ap l(t-@)- (1-2) Bal With € = max, We obtain the maximum temperature rise in this system as Doag — 7, = (20.g/em's)(8000n /60 rad/s)*(2.54 cm)? | 1 max 8 "(4.0 x 10-4 x 4.1840 x 107] g-cm/s?-K) © (1.002? — 1)? [-0.002000667 + 0.002002667] = 8.438 x 10° - [2.00 x 10°] = 16.9C in agreement with the previous calculation. This good agreement is attributed to the narrowness of the gap relative to the cylinder radii in this problem. 11A.2 Viscosity variation and velocity gradients in a nonisothermal film. a, We begin by determining the temperature at which the logarthmic discrep- ancy, A, between the two viscosity representations in Eq. 11.4-18 is largest. The discrepancy is expressed as follows: A= In(First p(2) function) — In(Approximation to first (z) funetion) = [eta (| ~ [2 ae @ Use of Eq. 11.4-1 to express (2/8) in terms of temperatures then gives a= lr (Faw) |- [em -™ (Zam)| __BIT-h T-h Fall T Ts ] The T-derivative of the logarithmic discrepancy is dA BT 1 at T% |T? Ts. Setting this derivative equal to zero, we get ace rT as the temperature of maximum discrepancy between the two expressions for In y(z). b. For the conditions given, u(To) = 4(80°C) = 0.3548 x 10-? g/cem-s; (Ts) = 4(100°C) = 0.2821 x 107* g/ems; Ts = (273.15 + 80)(273.15 + 100) = 363.01 K or 89.86°C Eqs. 114-14 and 114-19 then give the viscosity at Ty as wesemen| (2) (2-2) 0.2821 89.86 — 80 = (0.3545 e) — = (0.3548 ep) exp [(m (3353)) ( 100-80 | = (0.3548 ep) exp {(—0.2293)(0.493)] = 0.3169 ep ‘Three-point Newton interpolation of Inj: in Table 1.1-2 gives = 0.3151 at 89.86°C, so the largest relative discrepancy is A = —0.0057, or —0.6 percent of (2). W-2 11A.3 Transpiration cooling. a, In the absence of transpiration, Eq. 11.4-1 is indeterminate, but its limiting form is obtainable by expressing the exponential functions as first-order Taylor expansions in w, (and thus in Ro): This profile, designated as Oo, is tabulated here for the present geometry: 1, microns 100 200 300 400 500 Oo 1.000 0.375 0.1666... 0.0625 0 In the presence of transpiration with the given rate w, = 1 x 107° g/s, the constant Ro in Eq. 11.4-27 is (1.x 10% g/s)(0.25 cal/g-C ) (4x)(6.13 x 10-F eal/em-s-0) = 0.003245 cm = 32.45 microns Ro Equation 11.14-27 then gives, with r in microns, TT, __(exp(-32.45/r) ~ exp(—32.45/500)) T.—T, — ((exp(—32.45/200) — exp(—32.45/500)) A table of this function, here called ,, follows: r, microns 100 200 300 400 500 Ow 1.000 0.406 = 0.185 0.070 0.000 c. The ratio of the heat conduction to the inner surface r = «R with the latter transpiration rate to that with w, = 0 is, from Eq. 11.4-32, @__¢ Qo expg-1 ___(Ro(1 = r)/sR) exp(Ro(1 — 6)/nR) — _ __(82.45)(0.8)/100) ~~ exp(32.45)(0.8)/100) — 1 0.2596 = expt0.2506) 21 = 876 Thus, this small rate of transpiration reduces the rate of heat conduction to the inner surface by 12.4 percent. 1-3 114.4 Free-convection heat loss from a vertical surface. ‘The physical properties for this problem are as follows, evaluated at an average temperature T = (Ty + T,)/2 = 110°F = 43.33°C = 316.5K: B =1/T =0.00316K~* p= pM/RT = 0.001154g/cm* 1.923 x 107*g/cm-s from Table 1.1-2 1.007 J/g:K from Perry's Handbook, 6th Ed., Table 3-212 0.2407 cal/g/K 0.0276 W/m-K from Perry's Handbook, 6th Ed., Table 3-212 = 6.60 x 10~*cal/scm-K ‘The Prandtl and Grashof numbers are then ig Pr= S (0.2407cal/g‘K)(1.923 x 10-4 g/cm: 6.60 x 10-Scal/s-cm-K) Gr = P9B(To = TH? # _ (0.001154g/cm*)?(980.7em/s?)(0.00316K—*)([(150 ~ 70)/1.8]K)(30cm)* ~ (1.923 x 10-4g/cm-s)? 0.701, and = 1.34 x 10° ‘The heat loss rate from one side of the plate, according to Eq. 11.4-51, is Q = WHaavg = WC - (To — Tr) (GrPr)'/* With Lorenz’ value of C, this gives Q = (50cm)(0.548)(6.60 x 10~Scal/s-cm-s)([80/1.8]k)(1.34 x 10° x 0.701)!/4 7.9 cal/s With the value C= 0.518 recommended by Whitaker for air, the result is Q=75 cal/s However, this value of C is based on a Prandtl number of 0.73 for air, rather than the value 0.701 found at the present conditions from more recent data, Linear extrapolation of the table on page 349 gives a C value of 0.516 at Pr= 0.701, and a corrected heat loss rate of 7.4 cal/s. WW 11A.5 Velocity, temperature and pressure changes in a shock wave. a. From Eq. 11.4-7, the initial air velocity is vy = May V7RT/M 1.4(530 R)(4.9686 x 104 Ibm ft? /s?- Ib-mol-R)(28.97 Ib /Ib-mol) = (2) = 2256 ft/s b. The final velocity veo is found from Eq. 11.4-75 with € + 00, so that y-1, 201 ee aan y+l "741 May 04 21 =aqt ogy 7037 ‘Then Eq. 11.4-69 gives Yoo = $v1 = (0.375)(2256) = 846 ft/s ‘The final temperature is obtained from the energy balance, Eq. 11.4-65: 9) ” (2256? — 846? ft? /s*)/2 1 Too = Ti + 5 1 2 (0.242 Btu/Ib,-R x 2.5036 x 10? Ibp,ft?/s*- Bin) = (530 R) + = 801°R ‘The final pressure is calculated from the foregoing results by use Eq. 11.4-61 and hence the ideal gas law: Pootoo/Too = prvi /Ti3 v1) (Leo mmo (22) (H) Veo 2256) (891 = (0 atm (2%) (3%) = 4.48 atm ¢. The change in specific internal energy is U = GyAT = (p/)AT = ((0.242/1.4] Btu/Ib,_-R)(891 — 530 R) = 62.4 Btu/lbm, and the change in specific kinetic energy is 1__ (846? — 2956? ft/s?) 22.5036 x 104 Ib,, ft? /s?-Btu) = -87.4 Btu/lbm 11A.6 Adiabatic frictionless compression of an ideal gas. ‘The states encountered in such a compression satisfy Eq. 11.4-57, pe-t=C) as well as > ap RIM Combining these relations, we get pT p-* =Tp'-7 = Cy Hence, the initial and final states in such a compression satis Be (2)" Tt \n and the final temperature in the case considered here is Tz = (460 + 100)(10)'*-! = 1407°R = 947°F 1I-G 11A.7 Effect of free convection on the insulating value of a horzontal air space. ‘The relevant properties of air at 1 atm and 100°C= 373.15 K are: = 0.02173 ep from Table 1.1-2 = 2.173 x 10~* g/em-s p= pM/RT = 1/(82.0578 x 373.15) = 0.0009461 g/cm? B= 1/T = 0.002680 K-* Cy = 1.015 J/g:K from CRC Handbook 2000-2001, pp. 6-1, 6-2 k = 31.40 mW/mK from CRC Handbook 2000-2001, p. 6-185 = 31.40 x 107° W/em-K (1.015 3/g-K)(2.173 x 1 31.40 x I Pr= Cpu/k The no-flow state will be stable as long as the Rayleigh number, GrPr, does not exceed its critical value of 1708, given in Ref. 4 of §11.5. This gives the following restriction on the temperature difference: 2 36 #98(T = Ty Lo < 1708 or 1708p? (Ti-To) < PoBhs Pr 1708(2.173 x 10-*)? (0.009461 g/em*)?(980.665 cm/s?)(0.002680 K-*)(2.5 cm)?(0.703) 11°C If a very thin metal sheet is placed midway between the two plates, forming two cylindrical chambers of height h/2, then a corresponding calculation for the total temperature difference across the two chambers gives the stability condition (Ti — To) < (2)(8)(3.1) = 49.6°C for absence of free convection. II-7 11B.1 Adiabatic frictionless processes in an ideal gas a. For adiabatic frictionless processes, the fluxes may be set equal to zero. Then the energy equation becomes pt, PT (3m) or | PoeDT (gue) Be D Dt \dinT M Dt ainT J, Di For an ideal gas p= pM/RT, so that (JInp/dInT), = -1. Then Files or +er*)20 Pp RT Di Dt Hence for a given element of moving fluid (Z+oeer)ar =RP T P Integration from the initial state ;,T, gives answer (a) or T £72 P In +b(T -T,)+=(T* - Rin— ain, #oT-Th)+ (T° TH) = Rin c. For the data given, we have ang 2-=3,204In SO = 3.156 cal / g-mole-K T, OT, - en "3 (800 — 300) = 9.205 cal / g - mole-K 6 £ (1? ~1})=- £8220" (600? 3002) = -1.231cal / g-mole-K Summing these results we get =270 Po _ Po _ 11.124 Inf = 11124 or Pep cay Hence p, = 270 atm W-$ 11B.2 Viscous heating in laminar tube flow (asymptotic solutions) a. From the energy equation we have eB] Into this we substitute the expression for the velocity distribution of a Newtonian fluid in a circular tube: 0, = 0, mac{1~ (r/R) ]- This leads, then, to Eq. 11B.2-1. b, Integration gives (for the isothermal wall) at large z : ‘i =e. r44C,Inr+C, The constants may be evaluated using the boundary conditions given in the problem, and the result is given in Eq. 11B.2-2. c. To get the z dependence of the temperature we perform an average over the cross-section aT =i Tessas fg? EdE CPx fo(1- $7 SUE which leads to the result =T, = (410, mox/PCpR?)z We then make the postulate in Eq. 11B.2-4, and substitute this postulate into Eq, 11B.2-1 to get the following equation for f(r) ‘ Aime KAA { gf), SHO moe Hama) Bae Sas)” me This equation may be integrated to give S(7) = (HO? mox/k)(§? - 38") which leads to Eq. 11B.2-5. 11B.3 Velocity distribution in a nonisothermal film a. When x= 6, the two terms inside the large bracket are equal, but with opposite signs, and hence v, = 0. b, The second term in the bracket, being a constant, will not contribute to the derivative. The factors in front of the bracket are also constant. Therefore the derivative of the bracket is: dy _0+(Y8)In(v45/HMo) _1-(x/8)In(Hs/H40)( Hs)” 5, Hs at] (H5/Ho)” [(us/H0)*] (is) ' (és) ale When we set x equal to zero in this expression we get . (1/8) In(115 /1o)— (1/8) In(145 /H4o) = 0 Hence at x = 0, we have found that do, /dx =0 c. Let M=In(i1s /t4o) and X = x/6. Then Eq, 11.4-20 becomes > =OeP os (1a _ 1M) Ti UME emer _ pgs? con B 1 (2 MX)e™ -(1+ en Sg ME et _ seo rte A el ae = Uo MeeMeAy _ pgcosB(1- xX? ++O(M) 2p (eNO When [ts — Hy (or M- 0), the above result simplifies to pgcosB (x2 -54(,-(3)'] oOo (-%?) tty 6 in agreement with Eq. 2.2-18. \I-10 11B.4- Heat conduction in a spherical shell In this problem, the heat flow is in the @ direction. Then Eq. B.9-3 simplifies to 1 d(, ar ara aal G0) =° The first integration leads to sino o a _o- 49 sin® do The second integration gives T=C,In|tan}6|+C, =C,Intan}0+C, Since the argument of the tangent function is always less than a right angle, the absolute value sign is not needed. The constants of integration are obtainable from the boundary conditions, which give: T, =C,Intan}6, +C,; T =C,Intan}(2-0,)+C, We next form the following differences: tando, tan}(2-0,) tan}@ tan}(m-6,) T, ~T, = C,[In tan $6, — In tan} (2—- 0,)]=C, In T-T, =C,[Intan}-Intan}(n-,)|=C, In Finally we get for the temperature distribution in the shell: T, _ Inftan}6/tan}(2~-6,)] T, In{tan46,/tan}(7-0,)] TY, This solution clearly satisfies the two boundary conditions. Heit 11B.5 Axial heat conduction in a wire a. This problem involves purely axial flow of heat (by conduction and convection) so that the energy equation is a aT _ | @T eCvaT_ &T aT _@T EE gy Poh aT aT -agt OP ag Re ae ae oe AE in which v, =-v, and A= pC,v/k. Integration of the differential equation gives -at=Zee, At z=, we know that T=T. and dT/dz=0; hence C,=-AT.. Hence the first-order differential equation becomes B.--n0 where @=(T-T..)/(Ty-T..) in which (since @(0) = 1) In@=-Az+InC, or This is just Eq. 11B.5-1. b. For temperature-dependent physical properties we have the following energy equation: do_d do_d de -0¢,.L(8)0 = F(x) or -a,4e)42= £( x02) dz dz dz in which A. = pC,,v/k,,. The first integration gives +A, [r1@) Baz -xo) 2 +, Wiz We now use the boundary conditions that at z=, @=0 and d@/dz = 0, to find that C, =0. The above result may then be written as i) +A, L(O)d0 = Kee © \dS do or -A. J, L(8)d0 = Kee This equation may be integrated to give “Ae a This result simplifies to that in (a2) when K and L are both equal to unity. Furthermore, it satisfies the boundary conditions at z=0 and Z=0, c. To show that the last equation in (b) satisfies the differential equation (the first equation in (b)), we differentiate both sides with respect to z (on the right side, we differentiate with repsect to @, using the Leibniz formula, and then multiply by d@/dz): _K@) 4 FYE} or -A. f° L(6)d6 = Key A second differentiation with respect to z (once again using the Leibniz formula) gives and this is the differential equation with which we started. \-13 11B.6 Transpiration cooling in a planar system We start with Eq. J of Table 11.4-1, and assume steady-state, negligible change in pressure with distance, and negligible viscous dissipation. Then for constant thermal conductivity this equation simplifies to ote, afl gp o_ do PP ay age dn dn? in which ©=(T-T,)((To-T,), n=y/L, and ¢=pC,v,L/k, a constant. This equation is to be solved with the boundary conditions that (0) =1and @(1)=0. Set p= d@/dn to get the first-order separable equation dp dp = or —=¢dn ra n P which may be integrated to give Inp=$n+InC, or This is also a first-order separable equation and integrating it gives = on, = Gq on 0=C,femdn+C, “3° +C, The constants of integration are then found from the boundary conditions, and we get finally The heat flux at y=0 is then _k(To-T1) d0 K(T,-To) 9 Ldn L 1-e€ -\4 11B.7 Reduction of evaporation losses by transpiration a. Without transpiration, we have from Eq. 11.4-31 _ 4nwR(T, -T,) ~ 1-K = 82Btu/hr Q 4n(}£t)(0.02Btu /hr- ft-°F)(327°F) OO 2 b. With transpiration, we get from Eq. 11.4-30 _ AnRk(T, -T,) = STRolRR NIA) — Q with R, =w,C, /4nk We do not know R,, but we can get it by using an energy balance in the form vap 2a «| 8% Gq When this is inserted into the left side of Eq. 11.4-30, we get Flap __(Ty=-Te) = iKR)(1-1 6, turn -1 This may be solved for Ry, and then the energy balance may be used to convert the result into an expression for Q _AmkbA a (KR), (C,(Ti-Tx) Q=—e =( eo | _ 4n(0.02)(91.7)( 0.5), { (0.22)(327) 0.22 (23) uf (17) 4] = (104.76)In(1.7845) = (104.76)(0.5792) = 61 Btu/hr 15 11B.8 Temperature distribution in an embedded sphere a. In both regions the partial differential equation is 3.2(12), 19 sno2) 7H ar) * sind a0? a9) =° b. At the surface of the embedded sphere the boundary conditions are that T, = T, and that k,(9T, /dr) = ky(@Tp/dr). c. For the temperature field inside the sphere, substitution of Eq. 11B.8-1 into the terms in the above differential equation gives: 1.9(20T)_[_ 3k bya 7 aa" FP) -[ gt par cos 1 a(, par 3k, , ~£| singZ)=-| 0 \2 Ar cos Fano 300 2) ch cos Outside the sphere we get for the same two terms 10( 20° k, -k a =}=2Ar7 - 0. ya *cos@ 3.3238) saarsono- Hate |) arc 1a, par)_ [,_[ kk y 7 Fain 30 55 )= [r- FE ee los d. The boundary conditions are also satisfied: 3k k-k |AR: 6=|1-|+—® ||ARcos@ Fe “ [ [i +2ky ] “oe 3k, 3k, ky-ky —2_ IA = 9 0+ ko} 3 \Acos@ if eos to [Acoso f= Lace [I-16 11B.9 Heat flow in a solid bounded by two conical surfaces a, The temperature has to satisfy the differential equation Z{sin of) =0 b, Two successive integrations give and T=C,In|tan}6|+C, Since in this problem, @ may go from 0 to 4, and hence the tangent will not have negative values. Therefore absolute-value signs are not needed. c. The integration constants are determined from the following simultaneous equations: T, =C,In(tan$@,)+C, and T, =C;In(tan}6,)+C, On solving them, one gets co-—_ich T,-T, *" Intan}@,-Intan}@, — In(tan}6,/tan}0,) c, - Zzintan} 6, ~T, Intan} 0, 1 = a Mtan 2 ~T Intanz Oy Intan}6, —Intan}@, d. The @ component of the heat-flux vector is obtained from the first equation in (b) above __ Ok k T,-T, rd@ rsin@ sin In(tan} 0,/tan}0,) e. The total heat flow across the conical surface is then _ Rex . _ T,-T, Q=h 5 Fo yao, Sin doar = 2aRKT an} 0,/tan}0,) Wey 11B.10 Freezing of a spherical drop a. The heat conduction equation for the solid phase is 1 d(2aT)_ Sir to (Ry 0 and use is made of the definition of the first derivative, we get 2 (?4,)-78.=0 Insertion of Fourier's law then gives 4 4D) 2g =9 (** Al, | 2g - (rat +7°S.=0 (**) or ke r +S. = for the appropriate equation describing the heat conduction with heat generation by chemical reaction and constant k. b. From Eq. B.9=3, with the time=derivative term set equal to zero, and all velocities set equal to zero, and all derivatives other than r derivatives set equal to zero gives the heat conduction equation in spherical coordinates for a system with no chemical reaction. Therefore, we have to add a term describing the heat production per unit volume: d( .aT yi(e) es =0 which is the same as the result obtained in (a). c. The above differential equation may be integrated in a sequence of steps as follows: Het dfdT)_ S72. (2aT)- 5 at_ sr. al" *) kK? G a) + dr 3k re sr, Page CeO) The constant C, must be zero, because neither the temperature nor its gradient are expected to be infinite. The heat loss to the surroundings provides the second boundary condition needed for getting C,: Definition of heat transfer coefficient: 4,|,_,=1(Tx-T,) From Fourier's law: Glen = Equating these expressions: A(T x-T,) , S.R? Inserting T, from (**): i —"g-+C,-T, = . _§,R? SR Solving for C,: C,= 6k + 3h +T, Thus we finally get the temperature profile within the catalyst pellet: S.R? ry], S.R Pte Ge : (3) | 3h d..When the heat transfer coefficient goes to infinity, the last term in the temperature distribution drops out. e. The maximum temperature in the system is 6k 3h 6k Rh S.R? , S.R sx 24) 4245 1+ f. In Eq. (**) one would have to leave k inside the differential operator, and insert the specific r dependence of both k and 5, . {1-20 11B.12 Stability of an exothermic reaction system a. For the postulated steady-state solution 2. EF + Sexp(A(T~Ta))=0 with T=T, at x=4B b. Using the given dimensionless variables, we may rewrite the problem as F £8 3.9 =0 with @=Oaté=41 ag c. Multiply the differential equation by 2d0/dé to get : d (do de? _ or aaa) t24Ge=° Integration then gives 2 do (2) +226 =C, We now use the fact that, from the symmetry of the problem, at € =0 we must have d@/dé = 0. Then if we let @=@, at € =0, we can get an expression for C, and then write 2 rc) (2) -2A(exp®, - exp®)=0 Note that we have not "evaluated" the integration constant C,, we have merely replaced it by @g, which has a recognizable physical meaning. c. We next take the square root of both sides: al = +21 exp®, —expO {I-21 The minus sign has been selected, since the left side of the differential equation is inherently negative, and the quantity under the square- root sign is positive. Then we integrate over half the plate oF do Ope, wpe PIM d. The integral can be done analytically by making the change of variable y? = exp(@- @,): © do ° fi-exp(@-©,) =exp(-4 ©) 390) yt —y? _ dy _ 1-y¥ = 2exp(-}@, )arccosh(exp}@p) The integral over y can be found in an integral table. Combining these last two results we get exp(-40, )arccosh(exp}@,) = (32 e. For A > 0.88 .no value of @, can be found. This means that when S,9 is too large, or B is too large, or k is too small, then the heat cannot be dissipated fast enough. This is an important example, because it illustrates that it is not always possible to get a steady-state solution to a physical problem. To do a complete analysis of this problem, it would be necessary to solve the problem with the time-derivative term included. \-20 11B.13 Laminar annular flow with constant wall heat flux The equation analogous to Eq. 10.8-12 for the annular flow is A ry _i-x? (RR) lar _,10(_ aT 1-[=) - eee 06m ) mys) Es WS) where Eq. 2.4-14 has been used. We now introduce dimensionless variables: €=1/R, £=|z/pC,v, maeR?, and © = k(T-T;)/qoR- Then the above energy equation becomes [0-#)-0- ine ae ra aE This is to be solved with the boundary conditions ar Atr=kR, ~k a= or at é=k, Atr=R, Zao or atg=1, -2 Atz=0, T=T, or atf=0, @=0 We seek an asymptotic solution for large downstream distances of the form @(E,2) = Cyf + ¥(E). The function ‘¥(E) has to satisfy 1a(,a¥ _ ying rag(!ge) "OCF The first and second integrations lead to (cf. Eq. 10.8-27) Ww _o|(E_s Sing-S) [4S -o(E-S)-SeGee DS aa : Ye ol E -£) 15 Pa 158 (Eing-L)}rcaingecs 1-23 The constants Cy and C, are determined from the boundary conditions at € = « and € =1: 1 Cy)=——*" ana er) 5G") 9 te) (1-K*)+ fe (1-K*)+ ae Then the equation for the radial distribution of the temperature is arora Ink Ws The constant C, can be obtained by using an integral condition, similar to that in Eq. 10.8-24 or 25 QnKRz-qy =f?" | 0C,(T-T,)v.rdrd0 or c= fleosas where $= 0,/0. max This gives Efecuneca}- -£)}rcmeecs} é [0-2)-(-) BS leas \- 2 since the terms containing ¢ just cancel. This last equation has to be solved for C. This is in principle an easy problem, but extremely time-consuming and unrewarding. It can, however, be solved by using Mathematica, and the result is* -9(1- x?) (1143x?) —2(1- x?)° (88 + 88x? + 25x*)In x K +3(-25- 24x? +184 +24K° +7K8)(InK)? -72K4 (Ink)? 72(1— x?) ((1- x?) +(1+ x?)(Ink))" = “The authors wish to thank Mr. Richard M. Jendrejack for his help on this problem. \\-25 11B.14 Unsteady-state heating of a sphere a. The time-dependent heat conduction equation is ar 53(rZ) or 2 _1 Of 529 ao rar or ar PAE? OE In the second form, we have introduced the dimensionless variables: @=(T-T)/(T;-T), €=7/R, and t=at/R*. The solution in Eq. 11B.14-1 is, in terms of the same dimensionless variables S/_qyn SiNNE a? 21425) (-1)" SREB yee O=1+ > ( aE b. We do the differentiations; then it is clear that the first and last expression below are the same. Therefore Eq. 11B.14-1 satisfies the differential equation. 8 _ 8 /_4)"( 2p?) SHNMTE nee Be 23( 1)" (-n? x?) ne 20 _nSy_yn, [EcosnmE _ sinnné |, 29%2 oe OO) al nt (nny nt nt 25 naf sna nag sinnné _cosnae |, 29% n=l 1 95290) 8) ayn [_nasinnné | are Ll 2 @) 25 (=) nal -MS Bae) 2k na ng c. When & = 1, we get from Eq, 11B.14-1 = 2 SIN NT 922, ve ap @= 14251)" ae since sinnz = 0 for all integral values of n. d. Inasmuch as [26 sinx x90 x =1 the solution for = 0 is certainly finite: O=1425(-1)'e"** nal e. In dimensionless form Eq. 11B.14-2 is 4. 25(- 1)" sina Then, multiplying by sin mé and integrating gives ~figsinmnkde = 23)" 4 fisin mnég sinnnédé The left side can be evaluated as follows: ~f[sinmngdg = - 1 Tag hssinate=— aia tm wan (sinx - xcosx)|}” ann) ewme (ay mn mn =t (m The right side may be evaluated thus: ¥(-1)" 1 psi is Sal pea 24-1) sa sin ng sinmnSaé = 25 (-1) iff sine sironxd Say (2 Lm =2>(-1)"— | =6,,, ]=—(-1 2 » wa wm) ma) Thus the two sides are equal, and it is proven that the initial condition is indeed satisfied. W-2] 11B.15 Dimensionless variables for free convection a. When the proposed dimensionless quantities are intro- duced, Eqs. 11B.15-1 to 3 follow directly. Nothing more needs to be said. b. We can convert Eq. 11B.15-1 into Eq. 11.4-44 if we require that 20 oo ) DoH 1 Then, when (1) is substituted into Eq. 11B.15.2, then the latter may be converted into Eq. 11.4-45 if we also require that a Hop, PY0%yo T; am ¥e8B(To-T) _ py VyoVz0 Next we substitute (1) into Eq. 11B.15-3 and further require that — PC, Yoyo av) then Eq. 11B.15-3 becomes Eq. 11.4-35. We thus have four equations from which to determine the three "scale factors" yo, Vy, and V,9. However, it can be seen that Eqs. (II )and (IV) are not independent, since multiplication of Eq. (IV) by Pr just gives Eq. (I). Thus we have three independent equations from which to determine three unknowns. We now eliminate y, by multiplying Eqs. (I) and (II) to get Eq. (V), and by dividing Eq. (III) by Eq. (1) to get Eq. (VI): Veoh oH _p, “ vegHp r V-210. SB(T-T.)H _p, 2 Vx (v1) Introducing the abbreviation B= pgB(T. -T;), Eq. (VI) can be solved for Vzo to give Eq. (VII), and then 0,9 is obtained from Eq. (V) to give Eq. (VII): BH _ [oBH 1) enemas (WI) %.0= Vopr Fi _ |_#_ [@BH _ Jo3B cat) 29= foes un \nH Finally yp may be obtained from Eqs. (IV) and (VIII): k HH _ faut TX) =—— = aS = gf (PM PC, 2 yo aB B The reciprocals of these last three quantities appear in Eqs. 11.B-41 to 43, c. If all the dimensionless groups were set equal to unit, then combination of Eqs. (Il) and (IV) would give Pr = 1, thereby severely restricting the applicability of the results. 11-28 11C.1 The speed of propagation of sound waves a, Equation 11C.1-1 can also be written as (2) _ (HIF), ( ®) (27/28), _ (AH/9T), (pat), av), (au/ar),\ av), (avjas), (au/aT), (av/ar), This may be rearranged to give (35) (30) (Ze), G3) (33) (Fe) as),\ar),ar), \ar),\ as), ar), Next we apply the Maxwell relations to the first two factors on the left and right sides, and we apply two of the four "fundamental relations" for pure fluids to modify the third factor (ALFA HAA FS) Then we apply the relation (BGG) to the first and third factors, to obtain (5/9), _ (95/2V), (av/9S),. (ap/2S),, On cross-multipiclations, this gives an identity. b. The equations of continuity and motion are 5 (e0+0')=-(9(b0+0'Kv0 +0"): $0 +P va +?) =—V( 00 +1) Since py and py are constants and vy is zero, we get (when the products of the primed quantities are neglected) i) ov 2 _ —py(V- id -V) Bp 7 POV v) and Py—-=—Vp c. Since the momentum and energy fluxes have been taken to be zero), the flow is isentropic. This enables us to rewrite the equation of motion thus: oo a (val $) =o) To get the second expression, Eq. 11C.1-1 has been used. Use of the definition of the speed of sound then gives Eq. 11C.1-4. d. Next we take the time derivative of the equation of continuity to get Hp _ a ov a T= -2lo(v-v)]=-(¥-99 4] or eee To get the second form, the equation of motion of Eq. 111C.1-4 has been used. e. From Eq. 11C.1-6 we get &p_ 2n\ gee) 20p_ a (2) (34 ] ae 02 Py ‘7 sin| z (z-,t) Therefore, Eq. 11C.1-6 is satisfied. \{-30 11C.2 Free convection in a slot a. The equations of change are simplified as follows: Continuity: 2 9, -{2 ci av, HPP: $0.0.) - fe peel 3 242 ay | eB T) or Po, ae . ~ O-u Se +PBs(T-T) since p= —pgz+ constant Energy; 2 onkr whence T=T+Ay b. The boundary conditions are; v,(tB,y)=0 0, isanodd functionofy —_T(x,0)-T =0. c. To get the velocity profiles we rearrange the equation of motion as follows: ao, ox? This may be integrated to give: PBgA PBgAB? x/ v,(x,y)=— Ba Py +CyetCy or 2,(x,y)= aca ( -(3) ly d. For water at 20°C , 7 = 0.9982 g/cc and = 0.00021 (°C). The maximum velocity will occur at x= 0 and y = W, so that 1-31 3 gAB"I Qu Hence the corresponding temperature gradient will be HO, ne 2(1.0019x 107 g/cm: of CORO) em au =) ~ (0.99823 g/em?)(0.00021 °C*)(980.7 cm/s*)(0.01 em)*(0.2 cm) =0.271 °‘C/em 11-32 11C.3 Tangential annular flow of a highly viscous liquid First, rewrite Eq. 11.4-13 to eliminate N in favor of the Brinkman number Br, which appears in Eq. 10.4-9: ond mag a) Oa] To show that this reduces to Eq. 10.4-9 in the limit of a very thin annulus, begin by taking the limit of the term that does not contain Br and then we treat the term containing Br. -e and €=1-e(1-n), where 7 is the x/b of §10.4. Then using Eq. C.3-2 we find -fe(i=n)+4e2(-n? -[e+de?+-] Ing _ 1-fee =1-(1- )+O(e)= 1+ Oe) In the limit of vanishing ¢, this leads to the last term in Eq. 10.4-9. Term with Br: The coefficient of Br is now ella (2e-e?)" {1-e(a— {,-1_)eG=m+he* = n)*+ (1-e)* etlert. When everything is expanded in terms of powers of €, we find pay tet Self zett—m+3e a+) : ~(1-[1+ 26 +3e7+--])([1- n]+de(-n+ n?)+-)] Simplification of this expression leads to (1-33 a+ Be +e? +.-)[(-2e(1- n)-3e7(1- 0)’) ~(-2e-3e?--)([1- n]+4e(-n+ 0? )+-->)] Then cancellation of some terms gives qadi-ser-)[-ser(1- n)? ~{3e2(1-0)-€?(-n+ 9? )+--J] =F 3er)[-3(-nen?)+(-n4 a) +] =3 (1-7) This is in agreement with the term in Eq. 10.4-9 that is multiplied by Br. {1-34 11C4 Heat conduction with variable thermal conductivity First we obtain an expression for the gradient of F: VF = V(fkaT + constant) = |(Vk)aT dk =jSvrar Var The V operator can be taken inside the integral, since it involves only differentiation with respect to position coordinates and therefore commutes with the integration over T. Since k depends solely on T (which may in turn depend on the position coordinates), we must differentiate with respect to T and then perform the gradient operation on T, as shown above. Next, since VT depends on position coordinates, and not on T, the VT may be removed from the integral sign dk vF=vT{ Sat Sar When the integration is performed, we get VF =kVT We now form the Laplacian of F V°F =(V-VF)=(V-kVT) But (V-kVT)=0 by the equation of energy. Therefore, we have finally V?F=0 (1-35 11C.5. Effective thermal conductivity of a solid with spherical inclusions a. At very large distances from the region containing the inclusions, the coordinates of the various inclusions (r,8) will not be very different from one another. Furthermore, if the density of the inclusions is small, the effect of the various inclusions will be additive. Therefore to get the temperature field far from the region containing the inclusions, we can write ky ky (R)? -T? =|1-nki= ho (R Tp(r,8) : ne) rome in which 1 is the number of inclusions, each with a thermal conductivity k,. This is the equation that the describes the system in Fig. 11C.5(a). b. For the system in Fig. 11C.5(b), we can apply Eq. 11B.8-2 directly _ 7? =| 4—-Ket = (zy) T,(r,0)-T -|! ical a) Arcos? regarding the shaded sphere as a hypothetical material of thermal conductivity kg. c. Next we relate the volume of the inclusions in the true system to the effective volume of the inclusions in the equivalent system: $aR°n=42R9. Hence R’? =(n/9)R°. d. We can now equate the right sides of the above two equations for the temperature profiles far from the origin BCE) “A(T K+ 2k 7) keg t2ky\ 7) 6 We can now solve for the effective thermal conductivity, and express the ratio ky/ky as 1 + (deviation resulting from the inclusions). This is exactly Eq. 9.6-1. \\-36 11C7 Effect of surface-tension gradients on a falling film a. If we let I be the gas phase and II be the liquid phase, then the contribution from Eq. 11C.6-4 to the stress component f,, will be the z-component of [ee-t]-vo or = 2 Hence, in Eq, 2.2-13 has to be replaced by Tz =(egcosB)x+A When this is combined with Newton's law of viscosity, Eq. 2.2-14, we get do, —HFE = (egcosp)e+ A When this is integrated, and the no-slip boundary condition at the wall is used, we obtain the velocity profile as follows: (3) v,= ye 1 5 + i 1 5 b. The mass rate of flow in the film is w=} [) po,dxdy = pWof,v,dé 5 AS = pie eee ese (1-g)a5+ 48 o~sys] _ P*Wg6? | ApWé* 3u Qu \\-37 11D.4 Equation of change for entropy a. For the volume element AxAyAz we write a balance equation (not a conservation equation) for the entropy: $ (pSaxayaz) = (p80,)| ayaz—(p80,)| | Ayaze--- +8,|, AyAz—s,|,,,, AyAzt---+g,AxAyAz le+ax Here the dots indicate the additional terms associated with the transport in the y and z directions. Then after dividing by AxAyAz and taking the limit as the volume element goes to zero, we get: 8 6§-{ p80, 4-.)-( 25.4 5 Se (Saree which is equivalent to Eq.11D.1-1. b. Along a streamline we can write (using Eq. 11.2-2) Du _ Ds Dt Dt or al-(¥-a)-(¥-v)-(:9)] = rs, re po Dt The use of the equation of continuity and subsequent multiplication by p/T leads to Eq. 11D.1-3. c. Insertion of s = q/T into Eq, 11D.1-3 leads to oS. 1. Mrz vv) =-(V-s)- ( WT) 2 vv) =+(V-5)- 3 1q: vr)-F(e vv) In the last form, the right side of the equation has the same form as Eq. 11D.1-2, so that the last two terms can be identified as the rate of entropy production (as displayed in Eq. 11D.1-4). 1-38 11D.2 Viscous heating in laminar tube flow a. We introduce the following reduced variables: PE KT = T°) MO max Here T° is the wall temperature (and entry temperature) for the isothermal wall problem, and the entry temperature for the adiabatic (insulated) wall problem. Then Eq. 11B.2-1 may be written in dimensionless form as _¢2)90 _1 0 ( 20 2 OE Ea Eze © The boundary conditions are: Isothermal wall: at £=0, at €=0, at €=1, Insulated wall: at ¢=0. at €=0, at €=1, The solution to the differential equation will be written, for both cases, as a sum of two functions: © = ©, + @,. Here the function ©, is that derived in Problem 11B.2: Isothermal wall: ©, =4(1-£*) Insulated wall: e, =40+(é? 464) When these expressions are put into the partial differential equation (*), we then get for both cases 20, 19 (, 00, 9 Isothermal and insulated wall: (1- £”) a \1-39 This problem is solved by the method of separation of variables, and the solution is of the form: ©x(6,£)=-E,B (Eile) in which the B, are constants, and the functions ,(€) and y;(¢) must satisfy the following differential equations: 49, ~ E29, ay Fal )va( a), 0) qe aM The a; are the eigenvalues of the problem, obtained when the solutions to the ¢, equation are required to satisfy the boundary conditions. The solution to the y, are proportional to exp(-a,¢). The solution to the ¢; equation has to be done by a power-series expansion, and we return to that shortly. For the moment we remark that the solutions to the ¢; equation must satisfy an orthogonality relation: fidio,(1- €?) gag =0 for i#j The final temperature distribution expressions are then Isothermal wall: @=4(1-£*)-}.Bo,(E)exp(-af) Insulated wall: @=4¢+(é*-4é4)- EBA )exp(-4;6) From these solutions, it can be seen that the solutions obtained in Problem 11B.2 are just the limiting solutions for large values of the dimensionless axial variable ¢. \I- 40 The constants B, are obtained by writing the above equations for ¢ =0, then multiplying by ¢;(1- &*)é, and next integrating over & from 0 to 1. Doing that we get: Isothermal wall: B= a hlt=6*)oj(1- 5") sae 4 fror(a-e*)éae WG? -48*)0,(1- 5?) eas hor (1-8?) gas The above results were first given by H. C. Brinkman, Appl. Sci. Res., A2, 120-124 (1951). The eigenfunctions 9,(£) that satisfy (**) can be written as a series expansion: Insulated wall: $:(6)= Dbus* 0 The coefficient bjy may be arbitrarily chosen to be unity. The next coefficient, bj, must be chosen to be zero in order to satisfy the boundary condition at the tube axis. Substitution of the above series solution into (*) gives the following recursion formula: b, ~ GE lCine bir) Therefore, all of the b, can be expressed in terms of the eigenvalues a, as follows: bo=1 Baa} byte? p= tba? et By=0 by =0 byte by = Ra? The eigenvalues a, are determined from the boundary conditions at € =1, which require that: il-41 Isothermal wall: 1b, =0 &% Insulated wall: Ykby =0 & The left side of each of these equations is a polynomial in a,. Setting the polynomial expression equal to zero gives an algebraic equation which has to be solved for the infinite set of a;. This is clearly a very tedious process. The first few eigenvalues have been calculated by Brinkman in the reference cited above. Also given there are some sample temperature profiles. b. The power-law fluid can be handled by the same method used for the Newtonian fluid, given in part (a). The reference to Bird, cited on p. 373, gives the details and numerical results. {I-42 11D.3. Derivation of the energy equation using integral theorems a. For an arbitrary volume V fixed in space, we can write the total energy balance as Gel l0 + Ape wv =fen-e)s+ [(w-}V [F(ea + pot }iv=-[(v-eav+f(v-}v To get the second form, the Gauss divergence theorem was used on the right side, and on the left side the time derivative is taken inside the integral (because the volume element is fixed). Then, since the volume V was chosen arbitrarily, the integral signs may be removed to get Eq. 11.1-6. b. For an arbitrary moving volume element V(t) of fluid, we may write the equation of conservation of energy as follows: 4 J(eti+4o0?)av =~ f(n-q)ds- f(n-[m-v])aS+ f(v-g)av vit) S(t) S(t) vo This accounts for the heat transported into V across the surface, and the work done on V by the surface forces. There is no term accounting for convective transport across S, since the surface itself is moving with the fluid velocity. Next, use the Leibnitz formula on the left side to get afl eden v= J S(ol+apet}v + f(pll+ toot)in-vs)is J Fed +4pe? pv + J (n-(nlt+toot)vs}s The surface velocity v, for a moving blob of fluid is identical to the fluid velocity v. When the term containing v; = v is transferred to the right side and the Gauss divergence theorem is used, we get the energy balance in the following form \1-43 J xu hoo! }iv=- J (v-(plt+4o0?)y}av “fe -q)dv - Je +[x-v])dv + Je -g)av Since the element of volume was completely arbitrary, we may now remove the integral signs through the entire equation and obtain Eq. 11.1-7. 44 12A.1 Unsteady-state heat conduction in an iron sphere a. The thermal diffusivity of the sphere is given by Eq. 9.1-8: = 30 2 — WHT O57 / he b. The center temperature is to be 128°F; hence Toy = Ty _ 128-70 _ T,-Ty 270-70 0.29 Then, from Fig. 12.1-3, at/R? = 0.1, and 2 ro) -01{ 29 ]-s0ps0hme-na s a 0.573, c. By equating the dimensionless times, we get Qty _ Ante ROR or a= a(4) = o.s79{ d. The partial differential equation from which Fig. 12.1-3 was constructed is $ooh3(-2) Pa ar 12A.2 Comparison of the two slab solutions for short times According to Figure 12.1-1, at at/b? = 0.01 and y/b=0.9 T-Ty 17T = 0.46 where y is the distance from the mid-plane of the slab. Next we use Fig. 4.1-1, which can be interpreted as a plot of (T-T,)/(T,-To) vs y’/V4at, where y’=b-y is the distance from the wall. We then get yi _1(1-0.9)_1 V4at 2 Jar/y? 2 Then from Fig. 4.1-1_ we find T-T, 177 = 0.48 Hence the use of the combination of variables solution introduces an error of about 4%. Smaller errors occur at smaller values of the dimensionless time at/b? . {2-2 12A.3 Bonding with a thermosetting adhesive The dimensionless temperature at the time of bonding is Tae -To _ 160-20 foe 0 = =0.70 T;-T) 220-20 This occurs very nearly at a dimensionless temperature at/b? =0.6. Hence the time required is _ wy _ (0.77)? _\_ t= col ) = ool O20, zr.) =85s 12-3 12A.4 Quenching of a steel billet The thermal diffusivity of the steel billet is k__(25)(4.1365x10") aA e 9G, (7-7)(0.12) The dimensionless time is then at _(0-112)(5%60) _ 4 145 RR? (6x2.54)" From Fig. 12.1-2, the dimensionless center-line temperature is about 0.31. Therefore Tee =To _ 9.31 1-To We can now solve for the centerline temperature Tey, = 0.31(T; ~ Ty) + Ty = 0.31(200 - 1000) + 1000 ~ 750° 12A.5 Measurement of thermal diffusivity from amplitude of temperature oscillations a, From Eq. 12.1-40, we see that the amplitude of the temperature oscillations is the following function of y: wrt af (8) Then the ratio of amplitudes at two different distances from the plane y= 0is en -fe) The logarithm of this expression is (3) (0 ~wa)= +2 -») This can be solved for the thermal diffusivity to obtain 2 2 2 wrn ) w-W ] =f YY ) In(A,/4z) (4:/42) In(A,/A2) where vis the frequency in cycles per second. b. Inserting the numerical values given: “2 6.15 a= (0.1416)(0.0030 $15, 2 ) =0.110 cm?/s \a-5 12A.6 Forced convection from a sphere in creeping flow a. The dimensions of the quantities in Eq. 12.4-34 can be obtained from the table on pp. 872-876, thus: ; Oe Di RE Re[=] dimensionless Pr[=] dimensionless To check for dimensional consistency: b. We now write Eq. 12.4-34 in terms of the Péclet number: 33 <, Q=(xD?)(T, {5} Es ae 2)" n)°2 | Dv. pC, ~(ont tse] oe I In this form, the viscosity does not appear in the expression. We now fill in the numerical values, using c.g.s. units throughout: 4.462 Coaye.eno.sy.asy)® = n(0.1)(60)(3%10~ lemaenan 3x10 = 0.0240 cal/s \2-6 12B.1 Measurement of thermal diffusivity in an unsteady-state experiment We first tabulate the midplane temperatures as a function of time, and then convert them to the dimensionless midplane temperatures. Having done this, we consult Figure 12.1-1. We find the value of the dimensionless midplane temperature on the ordinate of the graph, and from this we can read off the value of at/b?. This quantity then can be multiplied by b? and divided by ¢ to get the thermal diffusivity. The calculations may be summarized as follows (working to two significant figures): T, Ty FTypine SET Payfo§ 0 20.0 0 120 24.4 0.220 0.20 1.50x 10% 240 30.5 0.525 0.40 1.50x 10% 360 34.2 0.710 0.60 1.50x 10% 480 36.5 0.825 0.80 1.5010 600 37.8 0.890 1.00 1.50x 10 Thus, the experimental data give us @ =1.50x10™ cm? /s. Next, the thermal conductivity can be obtained as follows: k=apC, =(1.5010% cm? /s)(1.50 g /cm*)(0.365 cal /g-C) =8.21x10" cal/s-cm-C or k =(8.21x10™)(2.418 x10”) = 0.20 Btu / hr: ft-F The conversion factor was taken from Table F.3-5. 12-7 12B.2 Two-dimensional forced convection with a line heat source a. The energy equation simplifies to oT _ pias T ax Ry The boundary conditions have the following meanings: Eq. 12B.2-1: Far from the wire, the temperature of the fluid is unchanged from its value, T.., for x <0 Eq. 12B.2-2: The approaching fluid is all at the temperature T..--that is, there is no heat conduction upstream Eq. 12B.2-3: The entering into the fluid from the wire must appear somewhere in the cross-section In addition, we need a statement that T(x,y)=T(x,-y)--that is, symmetry about the plane y = 0, which contains the wire. b. The postulated solution in Eq. 12B.2-4 states that the temperature profile at any value of x will be geometrically similar to the profile at any other value of x. When this expression for T is inserted into Eq. 12B.2-3 we get: C0, — where 0, = 0p, a constant 1C,r0f'f(2)g(m),5(x)dn = Q/L When the integral on 7) is evaluated, it is found that f(x)6(x)=C,, a constant. When the expression in Eq. 12B.2-4 is inserted into the energy equation, we get 22. {peat ny] = alsa] t5Lanats) | SLacelats)} [At 9. de a8) 1g 1 Fax 8 & dn dx Multiplication by 5° then gives Eq. 12B.2-5. \a- c. When the quantity in brackets in Eq. 12B.2-5 is set equal to 2, we get a first-order separable equation for 6(x)--the thermal boundary-layer thickness--which is integrated from 0 to x to give 5(x)=./4ax/0, . The main reason for setting the bracketed quantity equal to 2 is that the solution to the equation for g(7) comes out to be a little simpler. d. The equation for g(7) can be written as d (dg d afd) yd ata an 8) which has the solution ds nn ONE Ca Since g(n) is symmetric about 7=0, we know that dg/dn=0 at n=0; therefore, this last equation tells us that C,. A further integration leads to a Gaussian function g=Cye™ We do not evaluate C; but instead "absorb" into the constant C,. e. We can now evaluate C, by substituting the postulated temperature profile into Eq. 12B.2-3 to get pC,00C,[~e(mdn=Q/L or pC, nC, [edn = Q/L which gives Q/L Oe t Which then completes the determination of the temperature distribution in the wake of the wire. 12-9 12B.3 Heating of a wall (constant heat flux) a. The equation to be solved for q,(y,t) is am ay a ay This is the same mathematical problem that is solved in Example 12.1-1, so that we can write down at once with q,(y,0)=0, 9, (0,t)= qo, and q,(%,t)=0 fy 2 2 a 5 vaca” du To get the temperature we use Eq. 12.1-39 Fy (Gt) mpm oa? T-Ty= Er ~ zh iat dudy va"! ott, ae et dud = fe Fa tT aa Syca®” AVM a ee ee Bae Ph [« ah = 40{ [dat 2y pe aw -4( 2 cae wd Fly” When the first integral is evaluated, we get Eq. 12B.3-1. In the above, to get the second line, we made a change of variables and then interchanged the order of integration; in the third line, we performed the inner integration. b. Some intermediate steps in showing that the partial differential equation is satisfied: We write k(T -Ty)/q9 = F—G. Then (fe, [Tv ) pa, og 2F_f [1 -{= yet Fe anot t)° * “ay? "\Vanat & Vat) we a 12-10 12B.4 Heat transfer from a wall to a falling film (short contact time limit) a, From Eq. 2.2-18, we get Pal (1/3) |= Pm 1-(1-(u/8))"] =P. nae[1=142(y/5) + (y/5)" | 20. ma ¥/3) this last expression is good in the vicinity of the wall, where the quadratic term can be neglected. b. Equation 12B.4-2 presupposes that the heat conduction in the z direction can be neglected relative to the heat convection in the z direction. In addition, laminar, nonrippling flow is assumed. c. The fictitious boundary condition at an infinite distance from the wall may be used instead of the boundary condition at a distance 5 from the wall, since for short contact times the fluid is heated over a very short distance y. Therefore the inifinite boundary condition can be expected to be adequate. d, Equation 12B.4-3 can be written as y(0/dz) = B(0°@/ dy"). Next we have to convert the derivatives to derivatives with respect to the dimensionless variable 7: 90 _ dan _do_y (- 1) ‘a dn dz dnyopz\ 3z, 28 _d0 dn _dO_1 ; $2-4(2 m\e @o| dy dn dy dn Y9Bz" dy? dn\ dn Y9Bz) dy dn When these relations are substituted into the partial differential equation and use is made of the defining equation for 7 we get Eq. 12B.4-7. e. When we set d0/dn =p, we get dp/dn+3n?p =0, which is first-order and separable, and the solution is given in the book. The next integration gives O=C lean +C, \2-1 12B.5 Temperature in a slab with heat production a. This problem is discussed on pp. 130-131 of the 2nd Edition of Carslaw and Jaeger. The solution in dimensionless form can be obtained from Eq. (7) on p. 130. We must first determine the correspondence between their symbols and ours: C&J 1 K ov x/l K iP Ay at BS&L ob k T-T) 7 = t= So Therefore the temperature rise as a function of position and time is K(T-Tp) 1 (=)" weir 9. 7 1-1 - Saye a =cos(n +4) é } b, The center-plane temperature is obtained by setting equal to zero. The maximum of the center-plane temperature is then Sib? Tmax = To +2 max ot 2k c. According to the figure on p. 131 in Carslaw and Jaeger, 90% of the total temperature rise occurs at about 7=1, that is, at about t= b?/a. 12-12 12B.6 Forced convection in slow flow across a cylinder a. First we have to determine the velocity component v, from Eq. 12B.6-1 (here we let €=1/R and measure 9 downstream from the stagnation locus): _ dy _ 0. sin@ ee 09 = Gh a are [iame 1)+2 #| Then let € =1+ n and, for small 7, introduce the Taylor expansions In€ =(€-1)t= nts y/R+- and -1/€? =-1+2n+--. Then we may write _ 2. sind _ 20.0 Sin [4n+ sr Y= Ay Therefore, 6 = 2v,, sin0/SR. b. We identify the boundary-layer coordinates as x= 0, y=r-R, and z=z. Then we recognize that h, = R and set h, =1. Therefore we can get the heat loss from a length L of the cylinder as follows, starting from Eq. 12.4-31: _ 38 K(Ty-T.)L ~ 20 r(§ ) 3% k(Ty -T..)L( 20, 3 re) Cane P10) leony, 1. 2) 2 ys 2ar(4)s a) EP coor ASLPT Comparison of this result with the solution to part (b) given in the text allows the constant C to be evaluated. c. The boundary-layer thickness can be obtained from Eq. 12.4-29 as follows: (2: y" (F sin Rao) 2-3 At the separation locus, = 7, the integral is finite (according to Eq. 12B.6-2) and the denominator in f is zero, so that f is infinite. Near the stagnation locus, @= 0, the numerator and denominator of f may be expanded in Taylor series to give: (2 o-4o°+-d0)" (vo(1- 4e+-)ao)" fs a1 (OL 1 V3. =zRlt + $67 +-)(36% - 3207+...) Then in the limit as 6 > 0, this gives the stagnation value 1p f=) Numerical integration gives for 0= 42 f=1.1981 To get the answers given in the book, we have to recognize that the theta in the problem is being measured from the stagnation point at @= 7 to the separation point at 0 Note that for this flow, just as for the flow around the sphere, the boundary layer thickness increases through finite values to an infinite value at the separation locus. [Note: Eq. 6.2.1 of Abramowitz and Stegun (NBS, Applied Math Series, 55) is a convenient formula from which the integral from zero to 47 can be calculated in terms of gamma functions, which are tabulated.] 12-14 12B.9 Non-Newtonian heat transfer with constant wall heat flux (asymptotic solution for small axial distances) For Newtonian fluids, Example 12.2-2, we wrote ac) wean 2h-(5) ) 2m where Eqs. 2.3-18 and 19 have been used, as well as the definition y=R-r. The analogous procedure for power-law fluids gives vn (6) ena) vel iil) abe ALT “| : vmla : Ka) . (ta) i) + i( (ane Halend oes oR This gives the formula for the vp that has to be used for power-law fluids. The quantity v) , with dimensions of velocity, appears only in the dimensionless parameter A. at Sin + ey = als aN {2-15 The boundary condition at =0 gives C,=1. The boundary condition at 7 =e, gives feran And the complete solution (in dimensionless form) is eran e an i ean fe 7 aq Sevan va eran ra) The integral in the numerator cannot be integrated analytically. ff. The local wall heat flux (i.e. at any position z down the wall) is L- a 4ef9) To=T__k To-T, Tyan dn, V9Bz T(3) Y9B2 In taking the derivative, the Leibniz formula was used. Finally the average heat flux at the wall is _k 1-71 phil. =Fey T() ye ih This is the result that is given in Eq. 12B.4-9. Gavel ett 126.1 Product solutions for unsteady heat conduction in solids a, We begin by defining a dimensionless temperature difference by T, -T(x,y,2,t) A= T,-Tp in which T, is the initial temperature of the solid rectangular parallelepiped, and T, is the imposed temperature on the surfaces of the solid. Then the 3-dimensional heat conduction equation for the solid is aa _{% ee +74, 28 at ty If we now postulate a product solution A(x, y,z,t)= X(x,t)Y(y,t)Z(z,t) then we get (XYZ) _ 5 °(XYZ) , (XYZ) , #002) al oe Division by XYZ then gives _1_a(xyz)_ XYZ at When the product on the left side is differentiated we get 1,11 of LX, HY 19% X a Y a Yop Zar The first term on the left is a function of x and f, as is the first term on the right. Similarly, the functional dependences of the second and (2-17 third terms are the same. Therefore we postulate that these pairs of terms can be equated to give: OK yPK NPY A _y PZ a Oe aye OF That is, we get three one-dimensional heat-conduction equations. These can be solved according to the method of separation of variables given in Ex. 12.1-2, and all of them have the same initial conditions, and the same kinds of boundary conditions. Therefore, when we combine the three solutions in the product form above we eH coms) eli Pet cosine 7 Peril He cost +d rae] or yr 5___¢ poo + 4) n+ 4)(p +4) {cos(m + 4)xx/al[cos(n + 4)ny/b][cos(p + $)x2/c] cg liebe tora [eto 2 ret This can be substituted into the differential equation to verify that the latter can indeed be satisfied. Also, the satisfaction of the boundary and initial conditions can be verified. b. This and many other interesting and important product solutions are discussed in Carslaw and Jaeger on pp. 33, 184, and 227. (2-18 12C.2. Heating of a semi-infinite slab with a variable thermal conductivity a. The heat conduction equation is, for variable k, 6, 7-2 62] or v6, 2-4, 3 1+ p0)%) or In order to use Eq. 12C.2-2, we need to convert the derivatives: 98 _ d® dn _ a- a dn at dn 98 _ dd dn _ do 1 oy dndy dnd b. We now substitute the above derivatives into the heat conduction equation to get (after multiplication by 5*) d® dé d (db d® AO 588) og A (ab | pa d@ a a) ao Ge +b) When this is integrated over 7 from 0 to 1, we get fd d5_ pd (dd, dd L100 fn- 58 = oof A 2 + p02 in Performing the integrations we finally end up with 1 d5_— (d®, dd = |) lod 6—= — — nol, + [oan 57 o( Gee pose | which is equivalent to Eqs. 12C.2-3 to 5. 12-14 c. We now use @=1-31+47°, which is a good choice for the approximate temperature profile, since it gives (0)=1, ®(1)=0, and (1) =0 in agreement with our intuition. We then get M and N by substitution into Eqs. 12C.2-4 and 5: M= [joan =[(1-3n+hn)dn=3 ; db , 2 do w=(Br poe) =((-3+40°)+6(1-3n+4n°)(-3+40?)]) =3(18) This leads to the differential equation for the boundary layer thickness: dé 302 -2(1+B)ao which when integrated gives 5 = 8(1+ B)aot The time-dependent temperature distribution is then 3 T-T _,_3 y 1 y =1-5| 4 _|, 4) __y __ T,-Tp 8{ [8(1+ B)aot } 2| /8(1+ Aart The heat flux at y = 0 is finally oT} K(T, To) do| a) dn _3_K(T,-T) no 2 -(8(1+ B) art tbl," *5y ly-0 in which kis given by Eq. 12C.2-1. (2-20 12C.3 Heat conduction with phase change (the Neumann-Stefan problem) a. Using the definition of the dimensionless temperature differences in Eqs. 12C.3-1 and 2, we may write the heat conduction equations for the solid and liquid phases as follows: id: 208 = 20s og = OO, _ 7, Solid: Fe = a5 Liquid) k= aS) The initial and boundary conditions are: Lec: att B.C. 1 at B.C.2: at z=0, B.C.3: at z=Z(t), AH, B.C.4: at z=Z(t), Ps _ 1 _ PAs dz EA a 1T,-Ty dt b. The assumed forms for the solution are chosen because of their similarity to other one-dimensional heat-flow problems in a semi-infinite region: , z Solid: O5=C, + Cyerf = aa z Liquid: ©, =C, + Cyert => =(Cs +C,)-¢(1-ert =(C; +C,)-Cyerfe Jaca The last form for the liquid-phase temperature equation will turn out to be somewhat more convenient to use. c. When we apply B. C. 1, we get C, = 0. Using B. C. 2, we get C,+C,=1; we not that the initial condition is automatically satisfied. Then B.C. 3 gives Z(t) V4at Cerf Z() 1-Cyerfe 2-21 The only way that this equation can be satisfied, is if Z(t) «VE. We elect to write, then, Z(t)=AV4ad where A is a dimensionless constant that has yet to be determined. Finally we apply B. C. 4 to get te, Real 2) acy) 2 ra Vn pal, 1 - avda TT) aE or 2 pAH Aan ke” (C, -C,) = f * FG -)= a “ Next we apply that part of B. C. 3 that deals with the dimensionless melting temperature: Cyerfl=0, and —-1-Cyerfc = 0, whence On _1-Om + CC SE exkcd a Combining (*) and (**) and rearranging, we get ®, 1-0, - +02 ert erfea = This gives A in terms of A and @,,. Then we get for the temperature 1g _o er(z/4at) 1-7 \Sttelz/4at) Solid: @5=©,— 5 Liquid: ©, =1-(1-,.) Fg Finally, we have Z(t)=A-/4at, where 2 is now a known function of Aand ©,. \e 22 12C.4 Viscous heating in oscillatory flow a. The starting point for this problem is Eq. 4.1-44, and the development up to and including Eq. 4.1-50 can be taken over here. B.C. 1 is also valid, but now B.C. 2 must read: at y=, v° =0. When Eq. 4.1-50 is solved for these boundary conditions, we get v°(6) _ sinhe(1~ 6) % sinhe where c= iwb?/v = ob?/2v(1+i)=a(1+i) and &=x/b. The above expression for v°(£) can now be substituted into Eq. 4.1-48 to get Eq. 12C.4-1. b. Next we get the dissipation function, which for this problem is ®, = (dv, /Ax)*. The derivative of the velocity is 90, _ [0° gion ox dx and its square is obtained by using the relation (which should be proven) K{u}S(v} = [M{uo} + R{uo*}], where u and v are complex numbers and the asterisk indicates a complex conjugate: a, do’ dv? dv’ 2) 1g. zion | 4 | FO” ($) i ek (21) | Next we get the time average of the above function; the first term vanishes because of the exponential (which contains sine and cosine functions) and only the second term survives: __ : av," dv? do do° __v, ccoshe(1-£) (%) - #4 =| =) where “eb sinhe from part (a). Then the time averaged dissipation function is: \2- 23 3) S|) s (ce*)(coshe(1- ))(coshe(1~ €))* (Sinhe)(sinhe)* Cs xy [cosh? a(1- £)cos a(1- €) + sinh? a(1- £)sin? a(1- €)] sinh’ acos*a + cosh? asin? a To get the third line of the above we have used the identity cosha(1+i)(1- )=cosha(1- &)cosa(1-)+isinha(1- )sina(1- &) which should be verified. Then, finally, use sin? 6+ cos? @=1 and cosh? @ ~ sinh? @ = 1 to simplify the above expression for ®,, thus: (2) | cos? a(1- €)+sinh? a(1- €) aa'(2) em b sin? a+sinh? a b the last expression being a limiting expression for very large frequencies. c. To get the time averaged temperature distribution, we use Eq. 12C.4-4, thus oat o( 2) (= a(1- &)+sinh? 0:5)) dé? k sin? a+sinh? a Then introducing a new variable n=1- we write PT __ pvp cos*an+sinh? an dank \. sin?a+sinh>a Integration once then gives aT _ 12 sin2en-sinhen) Be dan) 4k" sin? a+ sinh? 12-24 and a further integration then yields 2 (gin? anh? a _ _ Mvp ( sin’ an+sinh* an 7 = —H%0| sin en +sinh an al sin? a+sinh? a )+Can cs The constants of integration may then be determined, so that we get finally: =r For the high-frequency limit, we get T-T, = (1-e*)-g(1-e**)] (Note: The solution given here for finding the dissipation function does not use Eq. 12C.4-1 and is hence somewhat easier than the method suggested in the text. It does, however, require somewhat more familiarity with doing manipulations with complex variables.) 12-29 12C.5 Solar Heat Penetration The basis for this problem was Eq. 5-40, M. N. Ozisik, Heat Conduction, 2end Ed. , Wiley 1993 -- - p.206. The temperature profile is re-expressed as pew LOx- Te) Tinax (0) — T(~) and should have been written as T* = exp[-y yo /2a]-cos[at — yo /2a] - z (3 expl-1?}- costar — ya / an? Id ‘The thermal properties are taken from (Rohsenow, Hartnett and Cho, Handbook of Heat Transfer, McGraw-Hill, Third Ed., p.2.68, 1998): Density, kgm" ‘Thermal conductivity, ‘Specific hea Wimk kek 0.027 It follows that, a = 0.027 mW -3600joules _ 02-10% m 800-1515 joule-W — hr hr a) For the assumption of a sinusoidal input with a 24 hour period T(0,t) = asin(no t)} ‘Show that the mean temperature, in the absence of day-to-day variation is just ap , and for a periodicity of 24 hrs @ t=nt/12hrs;o=2/12hrs The amplitude of the fluctuations at depth y relative to those at the surface, “A,” is just Ay = exp(-w/"o /2a)exp(-40.4-y/m) ‘Then at a depth of 10 cm the relative amplitude 12-26 A, = exp(-4.04) = 0.0176 as required. b) The transient term is difficult to evaluate, but this is of no practical importance. The periodically steady oscillations are about the initial temperature of zero, and therefore all transients must be of lesser magnitude than these periodic excursions. Since the excursions are already negligible there is no need to examine this problem further. ©) Only the slowest component of the oscillations need be considered as the others damp. out even faster. 1227 12C.6 Heat transfer in a falling non-Newtonian film For the non-Newtonian falling film problem, the velocity distribution has been found in Problem 8B.1: (2) ial") This can be rewritten in terms of the coordinate y, which is the distance from the solid surface (a) When this expression is expanded in a Taylor series, we get Hl) IT > n N6) ala” Nahe For positions very close to the wall, this can be approximated by orl Inserting the expression for the maximum velocity, we get vn oe (22 m y which simplifies to Eq. 12B.4-1 for a Newtonian fluid. Therefore the solution in Problem 12B.4-1 may be taken over for a power-law fluid by replacing = uk/p°C,95 = (k/pC, )(u/ps6) (just below Eq. 12B.4-3) by 6 =(K/pC,)(m/ogd)"". The quantity 6 enters into Eq. 12B.4-8 via the variable 7, and explicitly in Eq. 12B.4- 9. jr-28 12D.1 Unsteady-state heating of a slab (Laplace-transform method) a. We take the Laplace transform of Eq. 12.1-14 along with the initial conditions and boundary conditions given in Eqs. 12.1-15 and 16. This gives: pO-1=5> with O(41)= This second-order, nonhomogeneous, ordinary differential equation can be solved by getting the complementary function (from Eq. C.1- 4a) and the particular integral (by trial and error); this gives B=C,cosh\pn+C, sinh pny When the constants are evaluated using the boundary conditions, we find that ‘osh./pn Pp pcosh./p Taking the inverse Laplace transform, we get ee cosh\pn. pcosh |p. geod exp[(n +4)" wr] The inversion was performed by using Eq. (40) on p. 259 of Erdély, W. Magnus, F. Oberhettinger, and F. G. Tricomi Tables of Integral Transforms, McGraw-Hill, New York (1954), with @ set equal to zero. Unfortunately, this formula has two misprints in it: the x in the denominator of the first term should be /, and the factor 2p in front of the summation sign should be 27. The inversion can also be performed by using the complex inversion theorem, as explained in Carslaw and Jaeger (see reference at the bottom of p. 403). 12-29 b. The expression for the transformed function © (in (a)) can also be written in terms of exponentials. Then we expand part of the denominator in a binomial expansion to get: evn e-in > re Fec¥) > le AP er Fem) 5 1J"e 2p at 1S (gy e-vlanet-ny _ =1)" e-WP@ntt+n) P ae we aX ) The inverse Laplace transform is then T,-T < n 2nt+1-n_ © n 2n+1+n Qs =1- S(-1)"erk -¥(-1)" erfe TT XC )" ert a >I )" erfc ie This can be rewritten in terms of the dimensionless temperature of Example 12.1-1 thus: a ® Sayer 2n+d- ES ~1)" erfe 22414 0 vat vat Here, the dimensionless temperature ® has been introduced to avoid confusion with the dimensionless temperature © used in Example 12.1-2. c. To compare the results of Examples 12.1-1 and 2, we replace the position variable y of Example 12.1-1 by x (so that it will not be confused with the y of Exammple 12.1-2). Then the first few terms of the final result in (b) are, written in terms of x= by, are: T- Te erfet B(x) [1-(/2)] were eB (¥/)] [1-(=/2)] - \4at/b? a cxt/b? et] (x/b)] foot = (Pb) (x/)) \aatfe? —J4an/o the subsequent terms being smaller. (2-30 12D.2 The Graetz-Nusselt Problem a. The partial differential equation to be solved is o6,Pome| (5) Jaf 2(-2)] When this is put in nondimensional form, using the dimensionless quantities defined in Eq. 12D.2-1, the following equation is obtained for the temperature distribution @(2,¢): 9 _1 9/00); = = | oo2-22(¢ 22 with @(E,0)=1, O(1,2)=0, and 55 We now try the method of separation of variables with the dimensionless temperature given by (E,¢)= X(E)Z(¢). This gives, after division by XZ ddZ_i1i ea) Zag xe Eda? ae, Since the left side is a function of ¢ alone, and the right side is a function of & alone, both sides have to equal a constant. We choose this constant to be - B?. Hence we get two ordinary differential equations: dZ we eZ or 2(f) eexp(-B?¢) and ra(«%) +B76X=0 with X(1)=0 and X’(0)=0 This last equation (with boundary conditions) is a boundary-value problem of the Sturm-Liouville type. Therefore, we know that there will be an infinite number of values of eigenvalues f? and an infinite (2-3) number of eigenfunctions X,(£). We also know that the eigenfunctions are orthogonal on the range (0,1) with respect to the weight function . Hence the solution must have the form: 2 SX wedé O(E,)= LAX (E)exr(-6'5) with A, Bz The expression for A, is obtained by using the requirement that @(,0)=1, and making use of the orthogonality relations for the eigenfunctions. b. We write for the Newtonian fluid H£)=?./(02)=Pemoe (I~ $*)/(2,)=2(1- &) so that Eq. 12D.2-3 becomes: 1d(,dX, pone si) +261 1-€?)X,=0 with X(1)=0 and X’(0)=0 zae(*e)7m0-€*) f This second-order equation will have two solutions, but one of them will become infinite at £=0 and is therefore unacceptable. The remaining solution we write as a power series X(6)= Sby6? ) io When this is substituted into the differential equation, we get 3 bj? + 267(1- €)Fd,g! =0 0 jo By collecting equal powers of & we get the recursion relation for the coefficients: by = (28?/i?)(B.j-4-B.-2) “) \2-3u The bj) are arbitrarily chosen to be unity, and the higher coefficients are obtained from the equation immediately above. The values of B? are then obtained from the two equations marked with (*), along with the boundary condition at £ = 1, which requires that by =0. fe This involves solving an infinite order algebraic equation by trial and error-—-clearly a tedious process. Methods are, however, available to get the eigenvalues ? for small and large values of i. For small values there is the method of Stodola and Vianello, and for large values there is the WKB (Wentzel-Kramers-Brioullin) method. The following eigenvalues have been obtained: i 1 2 3 4 5 6 267 7.314 44.61 113.9 215.1 348.4 513.8 See B. C. Lyche and R. B. Bird, Chem. Eng. Sci., 6, 35-41 (1956), for the first three values both for Newtonian and power-law fluids; the higher values are obtained from the WKB method as 26? =(4i- 4)*. Further discussions and literature references for non-Newtonian flow may be found in the 1st edition (Chapter 5) and the 2nd edition (Chapter 6) of R. B. Bird, R. C. Armstrong, and O. Hassager, Dynamics of Polymeric Liquids, Vol. 1, Fluid Mechanics (published by Wiley-Interscience). 12-33 12D.3 The Graetz-Nusselt problem (asymptotic solution for large z) a. Using the definition of the dimensionless temperature as given in Problem 12D.2-2, we can write aT or __K(Ti=To) 20} _K(Ti=To) 0) __K(T,=To) 00 en, RO, Ele 90 = 4rhar = —* b. For very large z, = , 4X; aX, “2Ae he exp(-B76)= A PE PM) Furthermore, from the definition of the bulk temperature in Eq. 10.8- 33 we find using Eqs. 12D.2-2 and 3 fooede = ey. = = = 7 d Pegi 2 goede 2A, ex( BPC){ OX Ede = 234, exp(-6? Neg 4 was ; =-25 4 exp(-#8e)( «2 j lb =-2$4, exp(- BR a 7 =-2Aen(-Bie) ge Fe the very last expression being valid for large z. When these results are substituted into the result in (a), we get wo=-Eer,-1,){-2) 0p ot) This is the same as Eq. 12D.3-3 in the text. (see also p. 217 of R. B. Bird, R. C. Armstrong, and O. Hassager, Dynamics of Polymeric Liquids, Vol. 1, Fluid Dynamics, Wiley-Interscience, New York, 2nd edition (1987)). 12-34 12D.4 The Graetz-Nusselt problem (asymptotic expression for small z) a, When Eq. 10.8-12 is written in terms of the dimensionless variables © =(T,-T)/(T, ~Tp), €=1/R, and ¢ = 2/R, it becomes 200 _a19(,00 romell-F V3 REH €38) Next we introduce the three assumptions used for small z in Example 12.2-2, as well as the dimensionless quantities in Eq. 1214-1. }-D This gives for the partial differential equation for @(o,é) the following problem statement: 20 _3O Nose Gok with @(0,0)=0, O(0,¢)=1, O(%,¢)=0 We use the method of combination of independent variables, by introducing the new independent variable: 1 =(No*/9¢)¥°. Then the transformation of the derivatives proceeds as follows: 2) - (20) = 40(No* (ewe 2e ag), dnlat), dn\ 9 3)” 8 dn (2) -2(4) -2(8 "an aq ( FO -f9(a) do), dn\do), dn\9¢) ~ dno a7), dn? \o, When these substitutions are made into the partial differential equation in dimensionless form, we get (after some simplification) the following ordinary differential equation and boundary conditions for (7) #0 5,240 _ ae ao with @(0)=1and @(~)=0 12-35 To solve this equation, we set (7) = d@/dn, and obtain a separable first-order differential equation for @: do . . do a -3n°@ with solution an” ® =C, exp(-n°) This first-order separable equation can be integrated to give ©=C,f)exp(-7° Hi +C, The lower limit of the integral has been chosen arbitrarily. Application of the boundary conditions enables us to determine the constants of integration. The final solution is then (lexe(-7 tn _ Sexp(-1° yin [exp im where Eq. C.4-3 has been used to write the denominator integral in terms of a gamma function. The integral in the numerator cannot be evaluated analytically. b. To get the wall heat flux, we use Fourier's law of heat conduction: yn or! waa - k N “R-5e) Fil Into this expression we substitute the final result of (a), to obtain (with the help of the Leibniz formula for differentiating an integral, given in Eq. C.3-2) frlen =-Ler-1(2)" [-sigoo-"),} 12-36 _.& 1_(4(v,)R_R\" =H -Rigwg 2) ys k 1_[D(v.)e CH D =+R (Ts TOseta| tie c. In (a) we can write the velocity profile for the Newtonian fluid (or any generalized Newtonian fluid) as v, =(v,)0(€) with 9(1)=0 — (no-slip condition) That is, for the Newtonian fluid 9(€)=2(1-£?). Then when we switched to the dimensionless coordinate ¢ measured from the wall, the velocity profile can been written as =(2,)v(o) with y(0)=0 —(no-slip condition) Then, the velocity profile in the immediate vicinity of the wall is 2, =(0,)w/(O)o+- and, for the Newtonian fluid, =-(0,)I(d/a)2(1- 2}, o-= 4(0, 04 (0.)0/(Qor- Hence, the above solution in (a) can be modified for the generalized Newtonian fluid by replacing N = 4(v,)R/a by _{_4¢| \(@.)R “(ate 12-37 12D.5 The Graetz problem for flow between parallel plates a. Let the flow take place in the positive z direction between plates at x= +B. Then the partial differential boundary equation for T(zx,2) and the boundary conditions are: . a 060,002 ae with T(x,0)=T,, T(£B,z)=To We can, if we wish, replace the boundary condition at x =-B by a boundary condition dT/dx=0 at x=0, because of the fact that we know that the distribution of temperature about the center plane will be symmetric. Then we solve the problem for positive x only, knowing that the full problem must be symmetric about the plane x =0. We now introduce the dimensionless variables 2, (x) ax 9(6)= te.) $5 Then the problem may be reformulated as oe #e .. OS) Sea ger with — O(E,0)=1, O(1,4)=0, 5 We anticipate that the method of separation of variables will be appropriate, and write @(€,£)= X(£)Z(¢). When this is substituted into the equation above, we get (after division by XZ) 1dZ_ 1 PX 2 Zdl Xode? B where we have introduced the separation constant -B?. Hence we get two ordinary differential equations _ ge Gaz whence —_Z(£) exp(-B76) 12-38 is X | p9x =0 with X(1)=0 and X’(0)=0 Since this last equation with its boundary conditions is a boundary- value problem of the Sturm-Liouville type, there will be an infinite number of values of the eigenvalues 67 and an infinite number of eigenfunctions X,(£). Furthermore, it is known that the eigen- functions are orthogonal on the range (0,1) with respect to the weight function 9(€). Therefore, the dimensionless temperature profiles must have the form: O(6.6)= ZAx(sle(-#Re) 0) with oe The expression for the A, is obtained by using the requirement that @(,0)=1 and using the orthogonality relations for the eigen- functions X;(é). It remains to find the eigenfunctions X,(£) and the eigen- values ?. This has to be done by solving the equation for X;(£) by a power-series technique. It is a tedious process. The results are given in the Handbook of Heat Transfer, by Rohsenow, Hartnett, and Cho, cited in a footnote in Table 12D.2 on p. 404. b. In the limit of very large ¢, only one term in the summation in (*) is needed, that is, we need only X,(£) and ?. To get the heat flux into the wall at large distances downstream, we need to find _K(T, To) (9/8 )lens OT) MT, =To) T,-To alae B e qo=+k =T, where, = and T, is the bulk fluid temperature. For very large ¢, \2-39 20) aX, Fels EA ae, cere) Se en(-0) ial The bulk temperature is, for very large ¢ obtained as follows: ; ie = ie = =DAerp(-f7.) fo oxds c ; =-3A exp(-B?¢) [osx ,(0)= 11 aX, PBF ae 4 ax, B dé |, EX: ge =-3A exp(-B?4) Substituting the last two results into the expression for the heat flux, we find K(Ty -T, p= MoT pp In terms of the Nusselt number, this result may be written as Nan a ey ae 7.541 which is the result cited in Eq. (F) in Table 14.2-2. c. For the limiting case of very small values of z, we start with the same equation used in part (a) for T(x,2): or oT OV 5 = OR with T(x,0)=T,, T(£B,z)=Tp Since we are concerned only with the temperature change in the immediate vicinity of the wall, it is convenient to change to a new coordinate y = B-x and to expand the velocity in powers of y thus: 2 2 2 2.=.anl-(2) Jeane 1-(1-¥) Fane 2)-(8) a B. a B. ” B. B y =2i ==3 = Penang = 30s) 5 where we have used the result from 2B.3(b); this truncated expression is valid in the vicinity of the wall for small values of z. The energy equation for T(y,z) can then be written as oT PT . (0. ) FS Spe with T.0)=Ty, M0,2)=To, Te,2)=T, The last boundary condition is sufficiently good for small values of z, since taking the center plane as being at an infinite distance from the wall will not change the temperature distribution significantly. We now introduce the following dimensionless quantities: Then the differential equation for @(7,¢) and the boundary condi- tions are: a vo rar with ©(7,0)=0, O(0,6)=1, O(~e,¢)=0 We now seek a solution by the method of combination of variables, using the combined variable z = (Nn°/9£)¥*. This is possible since the first and third boundary conditions both state that the dimensionless temperature is zero. We are then led to the ordinary differential equation (for the details, see the solution to Problem 12D.4) #0 ,.2.d0 ae ay with @(0)=1 and @(~)=0 for which the solution is \a-41 e= a Srexp(-z°)ez To get the wall heat flux, we use Fourier’s law ~~] MTT) 0) _ (Ts - To) N)” a a a “(z2) Who = MT = To) N ae “(52) Ane jexp(-z')) _, a4 MTo-T)(N "1 Fle) The above result can be put in the form of a Nusselt number as follows: a 4AhB___4qB Ten) Nae ET Ty) 42) aed “4 7 “ara ) This is in agreement with Eq. (C) of Table 14.2-2. The replacement of the bulk temperature by the entrance temperature in the first line is appropriate for the entry region, inasmuch as heat has had the opportunity to penetrate only a negligible amount in that region. (2-42 12D.6 The constant wall heat flux problem for parallel plates a. Application of §10.8 to parallel-plate system For the laminar flow in the z direction between parallel plates at x= +B, the differential equation is én[utaile We now introduce the dimensionless variables x kz K(T-T;) == a o(g,2)= + fee Oe (E.6)= When the differential equation is multiplied by B/qp, the above differential equation can be rewritten as 2 _ 20 “ (-8°)5e- Ge *) with the boundary conditions: B.C.1: at €=0, 370 B.C.2 at E=4, Bet B.C.3: at £=0, e=0 We now seek a solution valid for large downstream distance, and try a solution of the form @(E,6) =Coo + ¥(E) © Since this function cannot satisfy B. C. 3, we replace the latter by B. C4: Condition 4: atany planez, 2zWqy = r [o p0C p(T —Ty)v,dxdy \2-43 or, in dimensionless variables atany plane £, ¢=fto(E,¢)(1- €?)dé When the trial function in (*) is substituted into (**), we get ay (1-&)Co = er Integration of this equation gives ae GenCAE-AE)+G, ) Further integration gives V=Co(28? -E4) +E +C, so that O(E,6)=Col +Co(PE?- HEt) HCE HC B.C. 1 leads to C, =0. Furthermore, B. C. 2, with the help of (**), gives C, = 3. Finally Condition 4 leads to: o=f[pe+9(ge? - be") +c 1-6) ae from which we get C, =-3. Therefore, finally O(E,6)= 35+ 3(8? - 86") - 3 Then the bulk temperature as a function of downstream distance is (2-44 _LUT2)-TyJe.(x)ax 908 LOCE-SMI- EAE _ gu T,-T, fs : fpv.(x)dx ko fi-s?)ag (39) and the wall temperature as a function of downstream distance is .B 7-1-4 B)- FE e+8) Therefore 17 4B 4q.B___ 140 Ty -T, = 2 8 Nu=— 2008 _ 140 oe 35 k or N= ETy-T,) 17 in agreement with Eq. (L) of Table 14.2-2 (note the definition of Nu in the table title!). b. For very small values of z the velocity distribution the linear portion very near the wall is the only part of the distribution that is important; if we let y be the distance from the wall at x =B, we have 2 at oa Boz oy Aside from the replacement of R by B, these equations are identical to Eqs. 12.2-13 and 14, and they have the same boundary conditions. Therefore, the remainder of Example 12.2-2 is valid for the slit flow (with the replacement of R by B). When 7)=0, we also have x =0, and Eq, 12.2-24 gives for the difference between the wall and bulk temperatures 12-45 (aoB/k) (3) T(3) VooB? inasmuch as the bulk temperature is virtually the same as the entrance temperature, the heat having penetrated to only a very small distance into the fluid. Then the Nusselt number is Na = MB = AaB _A19),fBE k KT -T,) V3 V oz in agreement with Eq. (H) of Table 14.2-2. c. The slit analogs of Eqs. 10.8-12 and 19 are rinal (SEAS «(008-38 dz ax? at ak In the second equation we have introduced the dimensionless variables _T-T q0B/k x fp — 9? 2, maxB? [ke From §10.8 we know that the asymptotic solution for large downstream distances is 0..(8,0)= 30+ 3(8? -384)- The complete solution is assumed to be of the following form (6,0) =0.(6,6)-@,(6,¢) Substituting this into the partial differential equation above tells us that the time-decaying function @,(€,£) must satisfy the equation 12-46 20, #0, (8) Ge = et with the following boundary conditions: 00, : = a8 B.C.1: at €=0, ae 7° 2, B.C. 2: =1 ae c at €=1, ae 7° B.C.3: at €=0, 0, =0.(E,0) We try a solution by the method of separation of variables, letting ,(,¢)=X(€)Z(¢). The functions X and Z must then satisfy the following ordinary differential equations az #Xx wen ee gerte (In €)X=0 ) where -c? is the separation constant. We then expect to have a complete solution of the form O(E,5)=0.(E,5)- SB, exp(-c2)Xi(6) where p, = 0-(.1%(E)|1-€")a8 * Klxe(QPO-2)a8 Therefore, the problem is reduced to one of finding the eigenfunctions X,(&) of (**), and then getting the eigenvalues ¢,, by applying the boundary condition at the wall £ =1. The Nusselt number is shown in Fig. 14.2-1 over the entire range, including the limiting cases given in parts (a) and (b). 2-47 12D.7 Asymptotic solution for small z for laminar tube flow with constant heat flux Exchange the order of integration in Eq. 12D.7-1, recognizing that the area of integration is a triangular area bounded by the lines ¥ = y, =, and % =. Then the inner integration can be performed analytically: } (0,4) =99A JP waz = 194 J [Ra Tee rae dz ar ae [f an |ez Tay xe [x -x\de Next rewrite the result as (1,4) == Halse e Pay 3x Re® az - (xe? a] The first integration can be performed analytically, the second integral can be put in the form of a (complete) gamma function, and the third integral can be written as an incomplete gamma function. In the second integral, we set t= Z°, so that dt=377dy and dz = 4¢ dt. Then (5 He Pay =f Pet (he 7 dt = 4 fr 1 etat = 41 (3) A similar derivation can be performed for the indefinite integral. Then we are led directly to Eq. 12.2-24. 12-48 12D,8 Forced conduction heat transfer from a flat plate (thermal boundary layer extends beyond the momentum boundary layer) Evaluate the left side of Eq. 12.4-5 by using Eq. 12.4-8: at| _2k(T..-T)) Z| oy or ly=0 Next, evaluate the the integral on the right side of Eq. 12.445: fF 0C,0,(T. -T)dy = pC,0..(T.. -To)8r gz) The dimensionless integral on the right side can now be split into two parts to perform the integration, since the equation for the velocity profile changes before we get to the upper limit: i (207A -2nf.a? + nfa®)(1- 2; +2n} - nf dn 4 +fjs(D(1- 2m +207 ~ nt ny = (GA - BA? + {a4 -ZA°)+(h- AT 4A? - $04 444°) = B- fA + RA? - at + A = F(A) We can now write Eq. 12.4-5 as 2k(T..-Ty) _ a a or dx pC,0.(T.-To)5;F(A) or ea = F(A), Ge Integration of this equation then gives 5, = «/4(ax/v.)/F(A). The result in Eq. 12.4-12 is valid for A>1, so that the ratio of the two boundary-layer thicknesses is _ 6, _ [A(ox/o.) 37 (2. 37 angst ra F(a) oa Ge =e 315Pr F(A) When this is squared, we get Eq. 12D.8-1. 42-44 13B.1 Wall heat flux for turbulent flow in tubes (approximate) The integral in Eq. 13.3-6 can be evaluated by making a change of variable. Let Then Eq. 13.3-6 becomes (after setting the upper limit equal to infinity): 4 -p(_0» = dx = mPa) bys =Mto-Te) The integral may be found in a table of integrals, where we find °1+a? 3singa 3($V3) 313 r dx a a 2a Hence Eq. 13.3-6 becomes a yx _@ \ 20 _ yp op for (waz0) ays ~MTo-Tr) Then the dimensionless wall heat flux is GoD. =28( v ) ys WTo-T,) 2x Tas)? This now has to be written in terms of dimensionless groups: GD _ N38 ( v. \(Dv.)P), s__3v3_ [ff TEE tn ri ay zRerr® where Eq. 6.1-4a has been used in order to introduce the friction factor. Thus we have obtained Eq. 13.3-7. 3-\ 13B.2 Wall heat flux for turbulent flow in tubes a. The assumptions are, for the asymptotic solution discussed here (the asymptotic form of the solution is introduced in Eq. 13.4- 10): Fully developed turbulent flow Axial heat conduction is presumed to be negligible with respect to axial heat convection iii. The turbulent Prandtl number can be taken to be unity (in developing Eq. 13.4-20) iv. The modified van Driest equaion of Eq. 5.4-7 is used to describe the turbulent velocity profile (in development of Eq. 13.4-20) b. To get the constant C, in Eq. 13.4-10, we use B. C. 2. First we have to get the derivative d0/dé using the Leibniz formula do, 1(é) aE OT aa) Then at €=1 (the tube wall), the turbulent thermal diffusivity vanishes, and the dimensionless temperature gradient is unity, so that we get the result in Eq. 13.4-14: Te Oe The dimensionless wall temperature is obtained from setting € =1in Eq. 134-12 (and setting C, = 0) Oy =OS+ Cf! Cais arena yee The dimensionless bulk temperature is obtained using the definition in Eq. 10.8-33, starting with Eq. 13.4-12 (and setting C, = 0): (oOsds | 1(2) eae 00 corey as ata a) é |édé +C, Then we get the dimensionless temperature difference: oc Org Snell laters Then we exchange the order of integration to get the second line of Eq. 13.4-15: 1(é) @)- ©, =Cy tesa} ae 18) “Hs °E [1+ (a/a 1(8) co p__1é) FW aE ~Coher(a®7a)] “a roa ora ()-1(8 a 1__1(é) (3) Chr (atyayl soos Tay Arla Ihoeae— [f o8de ig E 3) ae “elmer The first two terms cancel, and the third term gives Eq. 13.4-16, when the expression for Cy in Eq. 13.4-14 is used. c. It is not necessary to find the constant C, in this problem because we are interested only in the dimensionless temperature difference. In obtaining this difference, the integration constant C, cancels out. If, however, we want to find the complete temperature profile, then we need an expression for C,. 133 13C.1 Constant wall heat flux for turbulent flow between two parallel walls a. For slit flow, the analog of Eq. 13.4-6 is [73] with the boundary conditions 40/dé'=0 at €=0, 0/é=1 at €=1, and @=0 at £=0. Here the dimensionless variables are defined as: _T-1, 0B/k’ ——— max B? [ke We now try a solution of the form @(E,£) = Cyf + '¥(), which should be asymptotically correct at distances far down the tube. This leads then directly to the following differential equation d al ae coal) A first integration then gives (e) E\yE (12) Ecol oe +, =Coj(€) where we have introduced the abbreviation J(£)= | ¢(E)aF and the constant of integration has been set equal to zero since 00/dE = 0 at t he tube axis. Then, since 40/dé=1 and a =0 at the tube wall, we find that C, = 1/J(1). Thus the dimensionless temperature profile is UG) 1+(a/a) OLE, 0) = Cob + Col E +Cy in which it is understood that a“ is a function of &. Next we form 13-4 1 2 JE) @,-,=C, pO) ae-c betel storey oe er s(a®]ay °° froté)ag Next we exchange the order of integration in the numerator and make use of the definition of J in the denominator to find JI IE) = Ty T3 [a 7a] tale 8)ae ag The inner integral of the second term may be written as J(1)-J(E), and the J(1) contribution to the second term of @)-@, exactly cancels the first term. The final expression for @, — ©, is thus _ 1_» DOL 0 Oo TF b+ (aa) Then, since qoB/k(Ty -T,) = 1/(. - ©), Eq. 13C.1-1 follows. b. For laminar Newtonian flow, ¢=1- €? and a =0; then J(é)= ff (1-E?)aE = €-4£% and J(1)=3. Furthermore SUV ag = fi(G? 364 + 36°) as = Hence, finally, for laminar Newtonian flow (using the mean hydraulic radius in the expression for the Nusselt number) qo(4B) _ 140 Nu=aty-T,) 17 (see the entry in Table 14.2-2) For plug flow, J(£)= fj d = €, J(1)=1, and [I[J(E)) dé = [E°dé = 4 Hence we finally get Nu = 12, which is the entry in Table 14.2-2. bs 13D.1 The temperature profile for turbulent flow in a tube a, Condition #4 is [}O(E,C)6(€)Edé =. Into this, we now substitute the expression for dimensionless temperature of Eq. 13.4- 12, in order to get C,. We do this term by term: Ist term: [}CoCO(E)EdE = Cyl, o(E)EdE=¢ (Eq. 13.4-14 was used) 4th term: {)C,dE =C,I(1) e[i+( at!" for 2nd term: ly ross =G artery (2) 848 Jag =Cal stare MO fh o(eyeas ee HET ag =p) ee Casa Ta) 3 Ta epe( (aa) Substitution into the 4th condition above and solving for C, gives =f (1) ACCT) ee (E10) é no °g[1+(a/a)] Ae (2/a)] which is the same as Eq. 13D.1-1. b. For a Newtonian fluid 1(&)= f*(1- E?)EdE = 34? -46*and 1(1)=4. Then [1(£)/1(1)] = 2? - 4, [1(€)/1()}° = 4€4 — 4€° + 8, and 2= (46? — 485 +27 a - [26 -68)de =—% 13-6 14A.1 Average heat transfer coefficients. The total heat transfer rate is: Q = wl;(Ti2 — Ta) = (10,000 Ibm /hr)(0.6 Btu/lbm-F)(200 ~ 100 F) = 600,000 Btu/hr The total inside surface area of the tubes is: A= wDL4ox = ()(1.00 ~ 2 x 0.065 in)(1/12 ft/in)(300 ft) = 68.3 ft? ‘The various temperature differences between the inner tube surfaces and the oil are: (T> — Th). = 213 — 100 = 113 F (To —Th)a = (113 + 13)/2 = 63 F (Zo — Ty)in = (113 — 13)/1n(113/13) = 46.2 F Insertion of these values into Eqs. 14.1-2,3,4 then gives the heat transfer coefficients: hy = (600, 000)/(68.3 x 113) = 78 Btu/hr-ft?-F hq = (600, 000)/(68.3 x 63) = 139 Btu/hr-ft?-F Tan = (600, 000)/(68.3 x 46.2) = 190 Btu/hr-ft?-F iat 14A.2 Heat transfer in laminar tube flow. (a) The Prandtl number, based on the property values given, is (1.42 Ibm /ft-hr) = 8.43 = Divbe tw H Dp o (4)(200 bp, /hr) ~ (x)(1/12 £t)(1.42 Tb, /ft-hr) = 1076 (c) From Fig. 14.3-2, at L/D = (20 ft)/(1/12 ft) = 240, we read aN (at), (2). Com (2) aa tana (To-Ty)m \4L. k Ho Now, for uniform Tp, » (B22) To — Tre. Ty-Tn - In (FR) = (0.0028)(4 x 240)(8.43)-?/3(1.0) = (Toa = Tn) _ (To-T)n Hence, for this problem, .649, giving = exp(—0.649) = 0.523 Insertion of Ty = 215°F and Ty, = 100°F gives Tho = 215 — 0.523(215 — 100) = 155°F 4-2 14A.3 Effect of flow rate on exit temperature from a heat exchanger. (a) From the solution of Problem 144.2 we find that Re= 10.76w and that er) = exp(-232V) in which w is the mass flow rate in Ibj»/hr and Y is the ordinate of Fig. 14.3-2 at the prevailing Reynolds number. The exit bulk temperature is then Tyo = Ta + (To ~ Tn) [1 — exp(—232¥)] (b) The total heat flow through the tube wall is Q = wly(Te — Tar) Caleulations for (a) and (c) are summarized below: w, Re Y 1-e7?2¥ Typ Th, Tho, Qa Ip /he oF °F Bu/hr 100 1076 0.0028 0.478 54.9 154.9 2690 200 2152 0.00185 0.349 40.1 140.1 3930 400 4304 0.0036 0.566 65.1 165.1 12760 800 8608 0.0040 0.605 69.5 169.5 27260 1600 17216 0.0037 0.576 66.3 166.3 51950 3200 34432 0.0033 0.535 615 161.5 96460 143 14A.4 Local heat transfer coefficient for turbulent forced convection in a tube. Fig. 14.3-2 requires the viscosity values j1y at J, and pio at Tp. Interpolation in Table 1.1-1, or on page 6-3 of CRC Handbook of Chemistry and Physics, 81st Edition (2000-2001) gives jz) = 1.13 mPaS at T, = 60°F= 15.56°C, and uo = 0.398 mPa-s at Ty = 160°F= 71.11°C, whence (114/10) = 2.84. The other phys- ical properties in Fig. 14.3-2, including j in the Reynolds and Prandtl numbers, are evaluated at the “film temperature” Ty = (To + Ts)/2 = 110°F= 43.33°C, giving 1 = 1.489 lbm/hr-t, Cp = 4.1792 J/g-K = 0.99885 Btu/lbm-F, and & = 0.6348 W/m-K = 0.36679 Btu/hr-ft-F. Then the Prandtl number at Ty is Cou _ (0.99885 Btu/lbm-F)(1.489 Btu/heF) _ 4 45 ko 0.36679 Btu/hr-ft-F ~ and the Reynolds number calculated at Ty is DG _ 4w rea Pel # Dp 4 x (15,000 Ibm/hr) = 4 (15,000 Iba /hr) = F{B/AD (LARD Tb, Jey ~ 70 * 10 From Fig. 14.3-2 at this value of Re, we get the ordinate expression A 2/8 0.14 can ( Co a) = 0.0028 (144.4 — 1) GG \ k Ho. in which hj, can be regarded also as hjge according to the analysis in Problem 14B.5, and G = 4w/nD? = (4r)(15, 000 Ibm /hr)/(2/12 ft)? = 6.88 x 10° Ibm /hr-ft?. Insertion of the foregoing results into Eq. 14A.4-1 then gives a \ 8p. 08 ioe = 0.00286,6 { Se! iS k Ho = 0.0028(0.99885 Btu/Ibm-F)(6.88 x 10° Ibm /hr-ft?)(4.05)~?/9(2.84) +14 =78 x 10? Btu/hr-ft?-F as the asymptotic value of the local heat transfer coefficient, and Go = hioc(Ts ~ To) = (7.8 x 10°)(60 — 160) Grlror -7.8 x 108 as the radial heat flux at the inner wall of the pipe. \4-4 14A.5 Heat transfer from condensing vapors. (a) The boundaries of the condensate layer are at Ty = 190°F and Ty = 212°F; thus the film temperature Ty is (190 + 212)/2 = 201°F. The physical properties at this temperature are well approximated by the values at 200°F, given in Ex. 14.7-1: Alfysp = 978 Btu/Ibm k = 0.303 Btu/hr-f-F p= 60.1 lbp /ft® )-738 Ibm /hr-ft ne ‘The resulting abscissa for Fig. 14.7-2 is: p?/3g'/3(Ta — To) 173 AH yap _ (0.393 Btu/hr-ft-F)(60.1 Iby /ft?)?/3(4.17 x 108 ft/hr?)!/3(22 F)(1.0 ft) ~ (0.738 Ib, /ft-hr)*9(978 Btu/Iby.) = 168 Btu!thr-1-2/3+8/3pp1-241/34145/SP-141, dimensionless ‘This value falls in the laminar region of Fig. 14.7-2. Extrapolation of the laminar line with a slope of 3/4, consistent with Eq. 14.7-5, gives T'/p = 170(0.168)"/* = 45 ‘The heat transfer rate, neglecting subcooling, is Q = ADI Afyap = (1/12 £t)(45 x 0.738 Ibm /ft-hr)(978 Btu/lbm) = 8400 Btu/hr. A similar result is obtainable from Eq. 14.7-5, once the flow is known to be laminar. (b) Comparison of Eqs. 14.7-5 and 6 gives, for laminar condensate flow: Qhor. _ 0.725 (L\"* Qvert. 0.943 \D_ Hence, if the tube were horizontal the heat transfer rate would be: Qhor. = (8400)(0.725 /0.943)(12)'/* = 12, 000 Btu/hr ‘The assumption of laminar condensate flow on the horizontal tube is clearly rea- sonable, given the result of (a) and the still smaller value of I/ in (b). 14-5 144.6 Forced-convection heat transfer from an isolated sphere. (a) The physical properties of air at 1 atm and Ty = }(Ty + Tix) = 150°F= 65.56°C= 338.7K are: pM/RTy .042 x 1075 g/em* 0.02023 ep = 2.023 x 10~* g/em-s, from Table 1.1-2 1.008 J/g-K = 1.008 W-s/g-K from CRC Handbook 2000-2001, pp. 6-1,6-2. k = 26.9 x 10-? W/m-K from CRC Handbook 2000-2001, p. 6-185 ‘The Reynolds and Prandtl numbers are Re = Dior # _ (2.54 cm)(100 x 12 x 2.54 em/s)(1.042 x 10~* g/cm*) - 2.023 x 10-4 g/cm-s = 3.99 x 10* _ (2.008 W-s/g-K)(2.023 x 10~* g/em:s) ~ 26.9 x 10-5 W/em-K = 0.703 Substitution of these values into Eq. 14.4-5 gives Num = 2 + 0.60(3.99 x 10*)'/?(0.703)'/* = 108.6 108.6k/D = 108.6(26.9 x 10~* W/em-K)/(2.54 em) 0.01150 W/em?-K and the convective heat loss rate is Q = 1D" hAm(To — Too) = (2.54? em?)(0.01150 W/em?-K)({100/1.8] K) =12.9 W =3.1 eal/s according to Eq. 144-5. The radiative loss is about 1.0 W for a perfectly black sphere in a large enclosure with walls at 100°F, and can be estimated by the methods of §16.5. 14-6 (b) For Eq. 14.4-6, we need io = 0.02144 cp at Ty = 200°F = 93.3°C and the following property values at Ts. = 100°F = 37.8°C= 310.9 K: Hoo = 0.01898 cp = 1.898 x 10~* g/em-s p= pM/RT.. = 1.136 x 10~° g/cm? 'p = 1.007 J/g-K = 1.007 W-s/g-K k= 27.0 x 107° mW/m-K = 27.0 x 10-5 W/cm-K ‘The resulting values of Re and Pr calculated at the upstream state are Re = (2:54 em)(100 x 12 x 2.54 em/s)(1.138 x 10° g/cm’) 1.898 x 10-* g/em-s = 4.63 x 10* Pr = (1.007 W-s/eK)(1.808 x 107% g/em-s) 27.0 x 10-5 W/eam-K = 0.708 Substitution of these values into Eq. 14.4-6 gives tao Num = 2+ (0.4 Re'/? + 0.06 Re?/*) Pr°# (2) o = 2+ 138.1 = 140.1 whence 40.1kgo/D = 140.1(27.0 x 10-° W/em-K)/(2.54 em) .0149 W/em?-K and the convective heat loss rate is Q =D" hm(To — Too) = 7 (2.54 cm)?(0.0149 W/cm?-K)((100/1.8] K) = 16.8 W = 4.0 cal/s according to Eq. 14.4-6. This result is believed to be more accurate than that found in (a). \4-7 144.7 Free-convection heat transfer from an isolated sphere. For the conditions of this problem, the thermal expansion coefficient 8 = 1/Ty is (1/338.7 K), and the other physical properties are the same as in part (a) of Problem 14A.6. (Note that, for the correlations in §14.6, 9 and p are evaluated at Ty rather than T for calculation of Gr.) Then 3 2 Bi GrPr= (meget ae) (%) i E __ (2.54 em)*(0.001042 g/em-s)?(980.7 em/s?)(100/[1.8 x 338.7]) (2.023 x 10-4 g/cm:s)? (0.703) = 4.92 x 10* Eq. 14.6-4 gives = 0.878 x 0.671 Num = Te oa Paper ( GrPr)'/* (0.878)(0.671) aya = Bx (o.a92/0.T0ayerepre (4-92 x 104) =6.73 Hence, hm = 6.73k/D = (6.73)(26.9 x 10-* W/em-K)/(2.54 cm) 0.000712 W/em?-K and the convective heat loss rate is Q =D? hm(To - Too) (2.4 em)?(0.000712 W/em?-K)([100/1.8] K) = 0.80 W = 0.20 cal/s By the methods of §16.5, one can calculate that the rate of heat loss by radiation is of comparable magnitude: 1.0 W for a perfectly black sphere in a large enclosure with walls at 100°F. \4-8 14A.8 Heat loss by free convection from a horizontal pipe immersed in a liquid. From the data provided, we find the following values at Ty = 32.3°C: dp 19463 — 0.99528 p aT 0.99496(33.3 — 31.3) 27 x 10~* K-} = 1,815 x 1074 Fo? p = 0.99496 g/cm*)(12 x 2.54 cm/ft)* /(453.59 g/lbm) = 62.11 Ibm /ft® G, .9986 cal/g-C ~ 0.9986 Btu/lbm-F # = 0.7632 cp = 1.8463 Ibm /hr-ft k = 0.363 Btu/br-ft-F (0.9986 Btu/Ibm-F)(1.8463 Ibm /hr-ft) 0.363 Btu/br iF e = 5.08 Hence, — 05 ft)? (62.11 Ib,» /ft?)?(4.17 x 10° ft/hr?)(1.815 x 10~ 4 20) a (1.8463 Tp, /hr-ft)? (6.08) = 1.088 x 10° ‘Then from Eqs. 14.6-1 to 3 and Table 14.6-1 we get Num = 7 o.e71 ar Cr + aaa) (2.088 x 10°)'/4 0.671 = 0.772 (9) (181.6) = 84.6 ‘The heat transfer coefficient is then 0.363 fig = Bag = 915 (S38 ne ) = 61.4 Btu/br-ft?-F and the rate of convective heat loss per unit length of the pipe is Q _ hmAAT 7 = (61.4 Btu/hr-ft?-F)(3.1415)(0.5 £t)(20°F) = 1930 Btu/hr-ft hy DAT \4-9 14B .1 Limiting local Nusselt numbers for plug flow with constant heat flux (Note: Problem 10B.9-1 should be worked prior do doing this problem) a. For circular tubes with plug flow, the dimensionless temperature distribution, the dimensionless wall temperature, and the dimensionless bulk temperature are obtainable from Eq. 10B.9-1 on p. 325: K(T-T, r o- MTA) ogee (where =F and ¢ Qo =O; =26+4 fs Orage ~ Tose = 2ffogde = 2f'(2¢ +44? - d)edé = 26 0 2, Then the difference between the wall temperature and the bulk temperature is K(T, -T, 1 o,-0,-Mia=B) 1 : and the Nusselt number is —hD__4(2R) Nas Ty -T;) =4-2=8 in agreement with Eq. (J) on p. 430. Note that, by convention, the Nusselt number for tubes is defined using the diameter rather than the radius, and this definition introduces the factor of 2. b. For the plug flow in a slit of width 2B, we have for the dimensionless temperature, wall temperature, and bulk tempera- ture, all obtainable from the results of part (b) of Problem 10B.9: |4-10 K(T=Ty) _ 908 oz @= 0B? C+h0?-2 (where o=e and ¢=-%,) Qo = Olga = S44 [,Or0do firodo » = [edo=[(o+40? -})do=6 Then the difference between the wall temperature and the bulk temperature is k(T) -T, 1 o,-0,-Mtes).2 : The Nusselt number is then —M(4B)__ 90(4B) _ 3 4_ Nus= | “r,-1,) 4? in agreement with Eq. (J) on p. 431. Note that the Nusselt number for slits is defined in terms of 4B, and this is the origin of the factor of 4 which appears here (see heading of Tables 14.2-1 and 2, as well as the caption for Figure 14.2-1 on p. 429. 4-11 14B.2 Local overall heat transfer coefficient. Let 0 and 1 denote the inner and outer surfaces of the tube, and fg and hy denote the local heat transfer coefficients on those surfaces at the cross-section where the oil bulk temperature is 150°F. According to the development in §9.6, the temperature drops within a cross-section have the same ratio as the corresponding resistance terms that sum to 1/(roUo): Be (ae Lal 213-T1 ~ rok rhy ‘The numerator on the right is 1 1n(0.5/0.435) (0.435/12)(190) ~~ 220 = 0.1452 + 0.0006 = 0.1458 hr-ft-F/Btu in which a thernal conductivity of ko, = 220 Btu/hr-ft-F has been used for copper at T = 190°F, based on Tables 9.1-5 and F.3-5. To calculate the denominator, we use Eq. 14.7-3 for the heat transfer coefficient for filmwise condensation on horizontal tubes. Iteration is required, since the temperature difference across the condensate film is unknown. As a first approximation, we choose T, = 190°F, and use the physical properties at 200°F from Example 14.7-1: AB yay = 978 Btu/lbm 393 Btu/hr-ft-F 0.1 Ibm /ft 738 Ibm /hr-ft ‘Then Bq. 14.7-3 gives 3 p?gAB ap ue Ay = hy = 0.725 | — = ‘ [sae -) = 12,500(213 — T,)“1/4 Equating the heat flow through the numerator and denominator resistances gives (Ty — 150)/0.1458 = (213 — T1)ry hi = (213 — T))*/4(0.5/12)(12, 500) or 213 — T, = 0.0000183(7; — 150)*/* Successive substitutions of 7; in the right-hand term give a rapidly converging se~ quence of left-hand values: to the solution: T = 212.9975, 212.9954, 212.9954, Thus, the outer-surface temperature of the tubes at this cross-section is 212.9954 F. The temperature drop through the tube wall is 0.0006/0.1458(212.995 — 150 0.25F. Thus, the thermal resitances of the tube wall and condensate film are unim- portant here, as assumed in Problem 14.1. 4-H 14B.3 The hot-wire anemometer a. The physical properties of interest at p = 1 atm and a film temperature of 335°F are: p=0.0499 Ib,,/ft* ", = 0.242 Btu/Ib,,-°F 4 =0.0594 Ib, /ft-hr =1.64x 10° Ib,,/ft-s (from Eq. 1.4.14) _ 5( 1.986 -5)_ ° k = (0.282 +2( 4288) (1.6410 ) = 5.373 x10~ Btu/ft-s°F (from Eq. 9.3-15) Pr = 0.74 (from Eq. 9.3-16) (0.01/12)(100)(0.0499) _ (1.6410) 2A Also the Reynolds number is Re = Then Eq. 14.4-8 gives Nu,, = (5.99 + 2.29)(0.905) +0.92(-3.54 + 1062)" (6.33)(0.905) = 8.010 Then we get the heat transfer coefficient from 010)(5.373 x10 (0.01/12) = 186 Btu/ft? -hr°F =0.0516 Btu/ft?-s°F Finally, the heat loss from the wire is Q=h,AAT =h,,-2DL-(T) -T..) = 0.01 0.5) 69 _ = (186) 2S (600 70) Btu/hr =10.75 Btu/hr = 3.15 W=10.75 W b. For an approach velocity of 300 ft/s, Re = 762. Equation 14.4-8 gives Nu,, = 14.20, and Q(300)/Q(100) = 14.20/8.010 = 1.77. This is very close to V3 =1.73 from King's relation. 14-12 14B.4 Dimensional analysis. (a) The left-hand member of Eq. 14B.4-1 is expressible in terms of integrals of the dimensionless product function v T as follows: Tee — To To wT = [P= Then) — Ts nD (a) Here the angle brackets denote cross-sectional averages as in Eqs. 10.8-32 and 33, whereas overlines denote long-term time averages. Thus the averages on the right depend only on Re, Pr, and L/D, when viscous dissipation and radiant energy ab- sorption and emission are neglected in the energy equation. With these assumptions, and the further neglect of axial heat conduction, the quotient (Ts2~ Ti1)/(Zo — Ts) is equal to the time-average of Q/(wCy(To — Th). (b) The heat transfer coefficients hy and hy, each differ from hy only by the ratio of the corresponding AT definitions given in §14.1. But AT, and ATig are expressible in terms of AT; and AT», so their ratios depend only on Re, Pr, and L/D according to the result in (a). Hence, Nug and Nuja are functions of the same arguments as Nuj, confirming Eqs. 14.3-12 and 13, Equation 14.3-14 requires extension of Eq. A to a variable upper limit 7, giving Tez) = Tr = a function of (Re,Pr,z) (B) —Th Then, according to Eq. 14.1-18, MocD _ wy din(Tr — Ts) k xDk d(z/D) (i) 2) = dz = 4 RePr Combining this result with Eq. B, we get (for this special case of uniform wall temperature in the heat-exchange section), Nuoc = Nuc (Re, Pr, in agreement with Eq. 14.3-14. \4-13 14B.5 Relation between /,,, and In, a. We relate the rate of heat transfer across the increment of surface 2Ddz to the decrease in the internal energy within the volume element 42D*: Ige(#Ddz)(T;, - To) =-($2D*) pC, (v) AT This is really an application of the d-form of the energy balance discussed in §15.5, and given specifically in Eq. 15.4-4. It is clear from this equation that the kinetic and potential energy changes are being neglected, and that Eq. 15.4-4 has been multiplied through by w. The above equation may now be integrated over a length L of the tube to get, with Ty =a + BT;, 1, ny aT, ap .ety(t) aT, folie = -4D pC, (v oT, -T, = DG MTA pyra = EO ar(1-6)~al = BH A {In[T, (L)(1-B)- @]-In[T, (0)(1-B)- @]} = By {T5(L)~To(U)]-lnf75(0)-T9(0)]} - _4Detse) [Ps(L)* To(L)]-[7(0)-T(0)] (To(L) IT9(0) 1-B (T,-To) A y{t(t)- (a/(1-B))]-[T.(0)-(a/(-8)] =AP0c (0) (Toh = +4006, (oy BW=NO, (To-To)in which agrees with Eq. 15B.5-2. b. Equation 14.1-14 can be written as 1444 _ 4D 0C, (0) [To(L)-7.(0)] he TD (ToT from which Eq. 14B.5-3 follows at once: Lee Ig = 7 Jolncl2 Then, differentiating the integral in Eq. 14B.5-3 using the Leibniz formula in Eq. C.3-2, we get diy, 1 1 Fin = (Lig) + = Mg aE (in) + Til or dh, Miloas = hin + LG which is Eq. 14B.5-4 14-15 14B.6 Heat loss by free convection from a pipe The properties of air at 1 atm and a film temperature of 190°F (or 650° R) are 1 =0.216 cp k=0.0173 Btu/hr-ft°F B=1/650 (°R)" ¢, =0.242 Btu/lb,,:°F Pr=0.727 Since the temperature difference is the same in both the original problem (of Example 14.6-1) and the new problem (Problem 14B.6), it suffices to determine the ratio h,/h,, , where the accent indicates the result for the "new" problem. Next we calculate the ratio of the heat-transfer coefficients: ay [i+(0. aozypryns]” [1+ (0.492/Pr’ yay” ~(00%8 o17e)""( $80.0 .242 0.0190 0190) "(128 129) 0.0152) \ 650 0.241 0.0216) \ 1.299, = (1.102)(0.930)(1.000) = 1.025 Thus, in the "new" problem, the heat-transfer rate is only slightly greater than in the original problem. If the thermal conductivity and viscosity had been assumed to vary with temperature by the simple power-law suggested by the simplified kinetic theory of Chapters 9 and 1, then almost no change would have been predicted. -lo 14D.1 Heat transfer from an oblate ellipsoid of revolution a. From Eq. A.7-13 we get (with Z=sinhé, K=coshé, S=sinn, C=cosn, s=siny, c= cosy) + (28s) +(KC)]=«*[(zS) +(KC)’] [(K? -1)s? +K?(1-S?)]=02(K?-S?) hy =aVK? =? nB(3) =a"[(KCc)? + (KCs)’ + (28)°]=a[(KC) +(z8)'] a?[K?(1~S?)+(K? -1)s*]=a2(K?-S*) sh, =aVK? 5? ia 32) ems? + (Ks Joate’s! vty = aKS These results may be used with the expression for surface elements after Eq. A.7-18 to get Syy =a? (cosh? € - sin? (cosh € sin n)dndy b. From Eq. A.7-17, we get Laplace's equation as ve i 4 =0 a sh. #2)-0 L{2 we aang since heat is flowing in the & direction only. This equation may be integrated to give © = K,arctan(sinh €) + K, with the boundary conditions: @(£,)=0 and @(c)=1. This leads to Eq. 14D.1-7. c.In the limit as &, — 0 (a two-sided disk of radius a = R), the result of part (b), namely Eq. 14D.1-7, simplifies to 14-17 41 arctan(sinhé) @=1- : in-0 = 2 arctan(sinh 6) The normal dimensionless temperature gradient at the surface is then obtained by using Eq. A.7-15, thus: 1 do 1 hig dE|ey RYl-sin® n dE eno 1 2) 1 ) =| £ |} ———-cosh Ry1-sin? n ( (aie 5 eo raat a) (Famers Then the total heat loss through both sides of the disk is 4(2 arctan(sinh §)] Q=2f(n oa n-VT)dS =2k(T, -T..) Oe ? cos nsin ndndy cos a =2k(T) -T..)- 2 atl sin ndn=8kR(Ty -T..) The heat transfer coefficient is then ~Q_ 8kR(Ty -T..) _ 8k "AMT (nR?\(T,-T.) 2K and the Nusselt number is Nu, = fA). 8k 2B = £5.09 14-13 15A.1 Heat transfer in double-pipe heat exchangers. (a) In the absence of heat loss to the surroundings, Eqs. 15.4-7,8 give weCpe(Tea ~ Ter) = —waCpa(Tha — Tir) with each flow rate w expressed from plane 1 toward plane 2. Insertion of the data then gives Qc = (5000)(1.00)(Te2 — 60) = ~(—10, 000)(0.60)(200 — 100) = 600, 000 Biu/hr whence Tey = 60 + 120 = 180°F ‘The log-mean temperature difference is (AT)ia = (20 — 40)/1n(20/40) = 28.85F and the required heat exchange area, from Eq. 15.4-15, is Q_____ (600,000 Btu/br) gy g2 T(AT\n (200 Btu/he-f?-F)(28.85F) Ao (b) Eq. 15B.1-2 gives Q UAT, — UAT, A~ in(@AT,/0,AT,) _ (60 x 20) ~ (350 x 40) ~ In((50 x 20)/350 x 40)) = 4926 Btu/hr-ft? ‘The required heat exchange area is then _ 2 _ 600,000 ~ Q/A~ 4926 A = 122 £? (c) The minimum usable flow rate of water to cool the oil to 100°F in coun- terflow is __ (20, 000)(0.60)(200 — 100) ce (2.00)(200 — 60) whereas the minimum usable flow rate of water in parallel flow is tw — £20:000)(0.60)(200 = 100) _ om (2.00)(100 — 60) ~ (d) If parallel flow is used, with w, =15,500 Ib,,/hr of water, the outlet water temperature will be Tea = 60 + (10, 000)(0.60)(200 — 100)/(15, 500)(1.00) = 98.71°F “Then (AT), = (140—1.29)/In(140/1.29) = 29.6°F and the required heat exchange area is = 4286 Ibm /hr 15,000 Ibm /hr a Q__ _ (10,000)(0.60)(200 ~ 100) = UT ~ (200)(29.6) = 101 #7 \5-1 15A.2 Adiabatic flow of natural gas in a pipeline. (a) The density and mass flux at plane 1 are = 3 RT (1545 x 530 ft-lb, /Ib-mol) = 0.2821 Thm ft G: privy = (0.2821 Iby,/ft*)(40 ft/s) = 11.28 Ibm /ft?-s) Re and f at plane 1 are the same here as in Example 15.4-2, and f is approximated as constant at 0.0025 along the pipeline, giving ev = 4fL/D = (4)(0.0025)(52, 800/2) = 264. Furthermore, Pipi = (100 x 144 Iby/ft?)(32.2 Ibmft/Iby-s*)(0.2821 Ibm /ft?) = 1.308 x 10° 1b?, /ft*-s? Eq. 15B3-7 may be rearranged to give or, for this problem with 7 = 1.3 for methane, 264 —(2.3/2.6]Ins 0.3 _ 1.308 x 10° $ 26” (41.28) 1026 This equation has the solution s = 0.74, corresponding to p/p: = v0.14 = 0.86 and p2 = 0.86 x 0.2821 = 0.243 Ibn /ft?. (b) Eq. 15B.3-8 and the result of (a) give ma mt [4 boule (1-2) [ 1 — 0.8677}(11.28 Ibm/ft?-s)? 0.3 (1.308 x 105 Ib?, /ft* 2.6 = (100 psia)(0.86) ph + = 86 psia (c) The temperature at the compressor inlet is Ts = Ti(p2/Pi)(01/p2) = 530(86/100)/(0.86) = 530°R. (5-2 Eq. 15.4-22 then gives : ler" - | 2b) 4, (49086 x 530 Ibyft2/s?-Tb-mol 3 [(100sn0_y 16.04 Tb /Ib-mnol) 3 | 86 = (-282 + 2.520 x 10° ft2/s?)/(32.2 Ibmit/s*-lby = 7817 ft-lb; /Ibm and the required compressor power output is a nD?) Wn = wWn = (3) GW 2.770 x 10° ft-lby/s = (2) (11.28 Ibyp/ft2-8)(7817 ft-lby /bm) = 504 hp 15-3 15A.3 Mixing of two ideal-gas streams. (a) The right sides of Eqs. 15.3-6, 7 and 10 are w = wie + wi = 1000 + 10,000 = 11,000 Ibm /hr = 3.0556 Ibym/s P= vyet1e + v18015 + PiaSte + PisSis Tia Wie , RT wie M ve M vp = [(1000)(1000/3600) + (100)(10, 000/3600) Ibyy ft/s] (ee x 540 pa ‘#) [scout 4 10,000/3600 = Matia + vw + 28.97 1000 100 = 555.6 + 25983.5 = 26539.1 Ibm-ft/s?] Ibm i] With G, = 6.97 Btu/Ib-mole-F, hence Cy = 6.97/28.97 Btu/Ibm-P, the right side of Eq. 15.3-10 becomes E=C, [urea + wuss] + 5 § [wreofs + wri) 1000 = 5 si 5 2/2. = (6.97 x 540/28.97 Btu/Ibm)(25036 Ibmft?/s?- Btu) [300 Sia ° bn /s| 1 [1000 (549 9)2 4, 225000 99)2 1,2 + [50020007 + Fog (200)? Wmft?/s* = 9.939 x 10° + 1.390 x 10° = 1.0078 x 107 Ibm ft? /s® and 7 = C,/(G, — R) = 6.97/(6.97 — 1.9872) = 1.399. ‘Then Eq. 15.3-13 yields the solutions L 3) 265901 [, é 399? — 1) 3.05556 x 1.0078 x 107 2 = \ 3.399, 359) 3.05556 7309? (26539.1)? = 5065[1 + 0.97838] = 109.5 = ‘and 10020 ft/s the smaller one of which is stable. ‘Then Eq. 15.3-10 gives the temperature of the mixed stream: T= (Bfw 5/6, _ [u.oo7sx107 1 Bae = | gopesg ~~ 3 (109.5)? 22/8 = 546.5°R = 86.5°F 28.97 6.97 x 25036 *R-s?/ft? 5-4 ‘The cross-sectional area of the mixed stream is S2= Stat Si= wre, oe RT, fre, wwe Piatia pistis = PiM | re Ub 49686 x 540 £3 1000/3600 10, 10, 000/3600 = (soos 87 aegt 2) Sic 7000 * 199 Pw/# = 0.3816 ft? and the pressure p2 of the mixed stream is obtained from Eq. 15.3-7: _ P= vw. nS _ 26539.1 — 109.5 x 11,000/3600 4 2 = [ mex | 6.86 x 104 Ibm/fts? = 1.00 atm (b) If the fluid density were treated as constant, Eq. 15.3-6 would give (Sta + Sts)u2 = Stari + Srovre whence Sratra + Sos Stat Sip = Viale + wise ~ wa] pra + wre/prie wie + 18 wre/Y1e + wr8/YB 11,000 = Thioo = 1089 f/s v= (c) Eq. D of Table 15.3-1 gives, for the conditions of (b), 2 2,08 (Wg /t02) BE + (w/oa) "BE — 2 2 = (1000/11000), oe +(10, 000/11000) 290" — 45454.5 + 4545.5 — 5995.1 f02/s* = 44004.8 ft? /s x 3.1081 x 10-? ft-lby / (Ibm ft? /s? = 368 x 10° ft-lby/Ibm. 15-5 15A.4 Flow through a Venturi tube. (a) Eq. 15.5-34 requires the calculated values So = (1 /4)(Do)? = (x/4)(3/12 ft)? = 0.04909 ft?, _ pM 1/y (14.696 X 144 Ib /2)(28.97 Ibm /Ib-mol) oes /1.4 oa = hep, Pals)” = saa 3 520.7 Belg /Ib-mol) (279) = 0.0610 Ibjn /ft3, Pi _ RT, _ (49686 x 520.7 Ibm-ft?/s?-lb-mol\ _ eae a. M ( 28.97 Ib, /Ib-mol ar 1 = (So/$1)*(P2/ pr)?! = 1 — (Do/D1)*(p2/p1)*!" = (3/12)*(0.75)°/"* = 0.99990. Substitution of these and known values into Eq. 15.5-34 gives (0.98)(0.0610)(0.04909) = 2.08 Ibn/s (b) For isothermal flow, _ PaM _ (0.75 x 14.696 x 14 Iby/#t?)(28.97 Ibm /lb-mol) RT, (1544.3 x 529.7 ft-lb ,/Ib-mol) 1 — (p2S0/p1S1)* = 1 — (p2/pi)?(D2/D1)* = 1 — (0.75)? ((3/12)* = 0.9978, = 0.0562 Ibm /ft®, and 2 _ _ At _ (49686 x 529.7 _ aye f[ (1/e)dp = “p- In(pa/p2) = “Fey — n(1/0.75) = 2.614 x 10° £0? /s! Then Eq. 15.5-33 gives tw = (0.98)(0.0562)(0.04900)/ 2 a 10° (c) For constant density at the entering value, PM _ (14.696 x 144 Iby /ft2)(28.97 Ibm /Ib-mol) (1544.3 x 520.7 ft-lb, /Ib-mol) RT, 1 — (p2S0/p1S1)? = 1 — (Do/D1)* = 0.9961 96 Ibm /s. = 0.0750 Ibm /ft*, 2 (1/p)di RT 25) = 52.8 atm ‘The expansion from p, to p2 satisfies p/pT = R/M, hence Th _ Pz po Ty po pr Inserting p2/po = (p2/po)'/’ according to Bq. 15.5-39, we get Tr = To(p2/'p0)-7 2 \Wor) =7|(2- . (F&) n(— 00 (2) = 250K ~ (sa) = (z3) = (b) When pz = pa = 1 atm, p; = 1/0.528 = 1.893 atm. Then the temperature within the main part of the tank is G-Diy O.4/14 T= 3) = 300 Cin) = 300 x 0.5078 = 152 K Geply (c) At the state described in (b), pi/po = (1.893/100)!/1 = 0.0588. The time required to reach this state is computable from Eq. 15.5-46: - Ey 2) femyern_ '* TqRieela + yore 6 ) [@) _ (10/0.1 ft) (&) [(o.0s89)-24/2 3] = 0.58 s \o-7 15A.6 Heating of air in a tube. _ For the air contained in the 20-ft length of tube, we write Eq, 15.1-3 with Wr = 0 (since this system has no moving surfaces); also we use the approximation A(v?/2) of the kinetic energy change from inlet to outlet. This gives the simplified steady-state energy balance A (a + >) 1, . G,aT +5 of |(@B) -1/=6 hr Ty for horizontal flow of an ideal gas in a tube, Proceeding as in Example 15.3-1, we get the algebraic equation 4 for horizontal flow, or aa 39(T2 — Ti) + 8 x 1074(T} — TP) - aK 107 (Z} — T?) Btu/Ibm] +7 Las ft/s)? x 3.9942 x 107° Btu/(Ibm ft?/s?)) 407, 7 -1 2 ° ™ 15T; = (800 x 20/185 = 86.4865 Btu/Ibm) Inserting T; = 5°F, and an initial guess of 800°R for Tz, we get the following Newton iteration sequence, converging to 864°R~ 354°F: OldT, LHS, f=LHS f'=df/dT, AT, New 72, °R Btu/Iby -RHS —Btu/IbR =-f/f' °R 800 82.9245, -3.5620 0.2458 14.49 814.49 814.49 86.5761 +0.0896 0.2474 ~0.36 814.12 15-8 15A.7 Operation of a simple double-pipe heat exchanger. (a) The two exchangers will work most effectively if connected so as to simulate a single double-pipe exchanger operating in countercurrent flow. That is, the stream. to be heated should enter exchanger A through its inner pipe at plane 1, next to the outlet of the twice-cooled hot water stream, and should exit the inner pipe of exchanger B at plane 2, next to the original inlet of the hot-water stream. (b) The Reynolds and Prandtl numbers for the stream being heated are: ) _ ( (4)(5400 Ibm /hr ~ \x(0.0875 ££)(1.09 Ibm /har-ft) ) = 7.21 x 104 — (%) _ C* Btu/Ibm-F)(1.09 Pal) ean 0.376 Biu/hrft-F At these conditions, with jp and 1s not distinguished, Fig. 14.3-2 gives IugD k (Re)“1(Pr)"/* = 0.0028 and the insensitivity of hj, to L/D at Re above 8000 gives hoc = hin according to Eq. 14B.5-4. Hence, the local heat transfer coefficient for the inner pipe wall is hi= 0.0028 %-(Re)(Pr)* 1s (0.376 Beu/hr-ft-F) (0.0875 fry = 1237 Btu/hr-fi?-F 0.0028 (7.21 x 10*)(2.90)'/? and the local overall coefficient U; based on Dj satisfies + Wall and deposit resistance based on Dj 1 0.001 = (oss x 2f(By 1) eee (3) 1 e. From Eqs. 6.1-4 and 15B.2-2, we get for D, =D, =D and 2b 0.0791 ~D (Dop/a) We now have to show that the result in (d) simplifies to this result when-D, =D, =D and 0, = 0; =. Clearly the first term on the right side of Eq. 15B.2-6 vanishes when D, = D, = D. The next term gives 0/0, and hence we have to apply L'Hépital’s rule: me [(@/,)** -3] sin =D pg EM Dred, (D,/D,)-1 =! x-1 ot 1 This, along with the statements that D, =D, =D and 0, completes the proof of equivalence. 5-14 15B.3 Steady flow of ideal gases in ducts of constant cross section a, In the absence of work terms associated with moving mechanical parts and for a duct which is horizontal, the second term. on the left side of 15.4-2 and the first term on the right side can be omitted. For a circular tube, 4 times the mean hydraulic radius equals the tube diameter, we get for a differential length dL of the tube 1 f 1 vdo+—dp+2v°LdL=0 or vdv+ dp +4u%de, =0 pry pvrtt b. Use of the product rule for differentiation, A) -ot-(2 a2) =-dp-| 2 lap (5) oe leads directly from Eq. 15B.3-1 to Eq. 15B.3-2. The d-form of the mass balance is dw=0 or d(vS)=0 or pdv+udp=0 or dp=p@ For an ideal gas, pV = RT or pMV = RT or pM =pRT. Hence for the isothermal flow of an ideal gas, the second term in Eq. 15B.3-2 is zero, and that equation becomes, after multiplication by 2/v? #)-(3) Gre) Then, combining the last two equations, we get Eq. 15B.3-3. c. When the result in Eq. 15B.3-3 is integrated from "1" to "2" encompassing a length L of the pipe, we get 2 2RT (v7? _ 9,7 RT vy 2% = 2RT/ O22) _ oan, -Inv,)= 2 of 1-21 | +n| 2 & are -2 (Inv, — Ino) Mvz\ 02 Dy 2 2 -RTpt 1-(2) vn(2) = Pip) 4Inr M(0,2,)" Pr rn) G ene 15-15 This may be solved for the "mass velocity” G to give d. From Eq. 15.4-4, we find that the right side is zero (for an adiabatic system with no work done by moving parts), and that the ideal gas law makes the fourth term on the left be zero. In addition, since the tube is horizontal, the second term on the left is also zero. Hence we are left with a CG Ry wiveCdT=0 or vive srdT=0 or vive eT oar=0 The third expression comes from the use of the ideal-gas expression C,-Cy = R and the definition y = C, /Cy, which give (6,/é) = é,/Cy)-1 y-1 Next the energy equation can be integrated to give RT y_)p 2 y_)Pa v4) BF const Be oto Bate +(e 3 y-1)M * y-Up ** \y-U py 15-16 where the ideal gas law has been used to get the second form. Next, after solving the integrated form of the energy equation for p/p, we substitute this expression for p/p into the second and third terms of Eq. 15B.3-2 to get (after multiplying the latter by 2/0”) ft 1-1} 0 z(e ee v y Jo vln” x Y p Then we use the d-form of the mass balance (pdo + udp = 0) to get y+1\do_ 2 y jo ca Y 7 Using the macroscopic balance, 2,/2, = p;/P2, we get Fl 2 (He )nds BA ( ota) 3/2) ha) Y Pr (p,2,)' \% 2 y Noy We can now solve for G=p,v, (using s =(p,/p,)") to get PiPy [y+ 1/y}ins_y-1 I-s 2y G= pin = IS-I7 e. The macroscopic energy balance is €,(T, -T,)+4(v3 -v?)=0 Then using the ideal gas law in the first term and the macroscopic mass balance on the second term gives MC, 1a “(2-2 }s40(4-4)=0 Ria ~m Ps pr Then, since MC, /R = 7/(1 y), this equation can be rearranged thus 2 BB-(+-2)6 1-(4] p,m \ 2y Jer] le And this expression can be rearranged further to give Eq. 15B.3-8. Let the right side of the above equation be designated by X and proceed as follows: PoP Pry op Pr Po Pex or B= B( 14213) PiPr Pt Pa oA Pt PP Py Inserting the expression for X will then lead directly to Eq. 15B.3-8. \5-1¢ 15B.4 The Mach number in the mixing of two fluid streams s. Requiring that the radicand in Eq. 15.3-13 be zero is equivalent to the statement that GAO 2 y KP) ow The factor (w/P) can be obtained from Eq. 15.3-11, and the factor (E/w) can be obtained from Eq. 15.3-10. This gives We now move the second factor on the right side to the left side, multiply the entire equation by 03, and replace C, by (R/M)[y/(y -1)] to get 1 RT,\ _(y2-1R (ee) -(Ge alae We next multiply by 2 and rewrite the equation thus: 2rT, RT,)? +1) 2RT. 21 [ot +28f208«(22) He ae (ot lo! Then we collect terms in the same powers of the velocity at plane "2" to get 1) 4 2a Zoe (2) ( = }!2 —- + | — |v. +1 —* ] =0 or 0: (7) M \y)? (mM Y This equation may be solved for the velocity at plane "2" to give 15-19 This is exactly the speed of sound, given in Problem 11C.1(c). b. To describe the behavior of a gas passing through a sudden enlargement, we can set Wy =w, and wy = and also Pu=Pi Ty 1 Sa=S; Sy =0 Then when w, P, and E are defined analogously to the quantities in (a), we find that Eqs. 15.3-11, 12, and 13 remain unchanged. {5-20 15B.5 Limiting discharge rates for Venturi meters a. First, we take the square of Eq. 15.5-34 ws cns(a) oer | Yd 1 pr in which f = (S,/S,)? and r= p,/p,. Next we set dw?/dr =0 to get a[P(1-eyr) dr| 1-pr and, performing the differentiation, we have (2/7)? - (vr) + ar’ - rir (1 - rrr) 7 B(2/y)r Bev (ary =0 or [(2yr)e? (yr) +1) [-6”) =r (1 = lyr eer) ] =0 When this is multiplied out, two terms cancel, and the remaining terms are: 2am whe pr -(3 ajpr ~p2rv =0 Y Y Y y Multiplication by yr” finally yields 2 (ity) part “ir Bly -1)+ =0 This is equivalent to Eq. 15B.5-1. b, First we solve Eq. 15B.5-1 for 6 = 0 and get j5-21 2 yl) r=|—— (Fa) We substitute this expression for r into the first equation above in (2) in for B=0 and then rearrange it as follows: Pif_Y_), 21 (q_ po = CaP 2b aay (1-70) 24(y-1), = 1 \_2_ yee = CaP? Rr amy ly) ( 7) V5 \0-) y_\(_2_ Croll 7) +1 (r=) =CypiSp | A] RT, 73) in agreement with Eq. 15B.5-2. c. For isothermal flow, we get from Eq. 15.5-33 ah (RT/pM)ap (2RT/M)in(p.7P2) Ww = C4P2Sp4]—" = Cy 2804, 1-(P250/PiS:)" 1-(P20/P:51) (2RT, ‘M a = Cyo,syr | CRM Int) in which r= p/p, and B=(59/S;)*. Now for negligibly small 8, we can find the maximum discharge rate by setting dw?/dr = 0 which gives 4(rint) =2rInt-1=0 dr’ rr r This gives the value of r for the maximum discharge rate as 5-22 Then 1 /2RT 1 ‘RT Wanax = CuPr$0 Te Hq = CaP S0y) og M Capo) oer It is interesting to compare the results for air (7 = 1.4) for the two cases. For adiabatic flow, we get Wax ( 2 un CpSoVMRT, "741 = (1.183)(0.5787) = 0.6846 = na) =Vi-4(0.8333)° For isothermal flow, we have a Cyp:SyVM/RT Ve 2.718 The ratio of these two results is 1.13. We estimate that this ratio will amost never be greater than 25%. Thus, the simpler isothermal expressions are frequently useful for preliminary estimates of flow behavior = 0.6065 5-23 15B.6 Flow of a compressible fluid through a convergent-divergent nozzle a, From Eq. 15.2-2 with no work done by moving parts and with viscous heating neglected, we get for the assumption of flat velocity profiles: 3(v3 - 27) + frido=0 Then, we set 0, =0 and, assuming adiabatic flow, we can use Eq. 15.2-5 to evaluate the integral. This results in ny Po (y-air +B -1/= ar eae 7a ) Then the ideal gas law may be used to rewrite this expression as 0) ya -Bh (27 a mee M y- \p, b. We may start with Eq. 15.5-34 with S, considered to be very large, and S, replaced by S, ocos ALSL-E) | We next replace p, by p,(p,/p,)"" and solve for Sy, assuming that C, =1. This gives, with r= p,/p, ols tle ~ rrr) c= us 15-24 The minimum cross-section for given values of w, p,, and p, will occur when 453 94 (7 ries Calerry =(r dry] dr dr (rit — pony When this equation is solved for r, we get Po 2 vi(y-l) PL -( 1+ i) which is Eq. 15B.6-2. c. Combining the results from (a) and (b) we get agp RM _y [2 JLRM_y (y-1)_ RM _y zee""M y-1L) y+) M y-1ly+1)) M y41 Then, from Eq. 11.4-56 and Eq. 15B.6-2, we find Q-1/r =T,{ 2 = 11) a (2) n 2 From the last two equations we get RT, RT; 2520) of Therefore, the velocity at surface "2" is sonic (cf. Eq. 9.4-4). It is clear that this problem is very similar to that in Problem 15B.5. Here, however, we are considering the effect of varying S,, whereas in Problem 15B.5 the reverse procedure was used. d. Since we now want to get the velocity as a function of the local pressure, we have to replace the first equation in (a) by Av? ~0f)+ fap =o I-25 We then use the fact that 2, =0, and perform the integration to get the velocity at any value of the local pressure for an ideal gas in adiabatic flow aap (2 pro yale =p") tp, yi el (r prof 2)" "| a \y-l P. {eer where r=p/p, (1). Then taking the square root and using the ideal gas law, we get Oe ar \e- ron) The cross-section at any value of the local pressure can then be obtained from (with r= p/p,): oo (SIH) = p,Sr" EYE )-ro-m) Then, solving for the cross-section S, we get of se w/o, RT, _y (r-tyh or EBB )-vo) I5-26 Next, for the conditions of the problem given in part (d) of the problem, we have = (1544)(32.17) = ft*lb,, /Ib - mole-s?-R RT,/M= r)- (32.17)(560)/29 = 9.592 x 10° ft?/s? es =, (9.592 x 10°) a ) we le a) (29 Ib, /Tb- a _ PM _ (10 atm)(6.8087 > 10" (ib, / ft-s*) per atm.) ‘RT, (iat Ib,,ft® /s? «Ib - mol-°K)(560°K) =0.710 Ib,, /ft® = 2591 ft/s We may now summarize the calculations of v, T, and S thus: v= 2591V1- 19% ; T =560r°®; 0714 af — 79 p(atm) 70286 2 T s 10 1.0 1.000 0000 560 © 9 09 0.970 449 543 0.977 08 0.938 645 525 0.739 7 07 0.903 807 506 0.650 6 0.6 0.864 956 484 0.613 5.28* 0.528 0.833 1058 466 0.606 5 05 0.820 1099 459 0.607 4 04 0.769 1245 431 0.628 3 03 0.709 1398 397 0.688 2 0.2 0.631 1574 353 0.816 1 01 0518 = 1798 290 1.171 0 0.0 0.000 2591 0 © *Pressure at the minimum cross-section 15-27 15B.7 Transient thermal behavor of a chromatographic device We start with the energy equation of Eq. 15.1-2, which we apply to the gas inside the chromatographic column. We are concerned only with the change in time (and not distance) within the column. Since there are no moving mechanical parts, the equation simplifies to fu =Q=UA(T, -T) in which the total kinetic and potential energy in the system are considered constant, and U is the overall heat transfer coefficient between the external gas stream and the gas within the chromatographic device. From thermodynamics we can write ~ (au a) a op 6 du =| —| dT+| —| dV= =] =| \dv a (=), (FB) 4 var v2), | The quantity inside the brackets is zero for an ideal gas and is neglected here. Then U-U°=C,(T-T°), where the superscripts ° stand for the reference state, and User = J PAV = pUlPV 4, + PCy(T-T? Vion where the density and heat capacity are taken to be constants. The energy balance becomes ay aT_ _ aT__UA t)) oC Via. Ge =UA(T, T) or fs In (1+e) 7] We now introduce the dimensionless temperature © = T/T... and the dimensionless time 7 = t/t) and rewrite the energy balance as do an B[(1+7)-©] with B=UAty/pCyVice 15-28 This differential equation can be put into the form ® 4 B= B(1+2) with @=1at r=0 This is a linear first-order differential equation (see Eq. C.1-2), which with the indicated initial condition has the solution e-(1+ e-H)edem B) B This is probably the simplest solution that can be offered for this problem. We are now in a position to answer the questions in the problem: a. The difference between the external gas temperature and the temperature within the chromatographic device is 1,1 pe 1_1 ee Ty-T=Ty(l~t)-Tye( 142d 420 es )=T [5-40 : ) b. The limiting value of the temperature difference will be 1 (47). =ToF c. The time required for the temperature difference to be within 1% of the limiting value is given by the solution of the equation e“** = 0.01 or t=(2.303)(2)/B. d. Constant physical properties; no dependence of temperature as a function of distance down the column; the neglecting of potential energy and kinetic energy effects. e. All the quantities that are contained in B. 15-29 15B.8 Continuous heating of a slurry in an agitated tank a. The starting point is Eq. 15.1-1 or 2. On the left side the potential energy term ®,,, is omitted because for a full tank the ® i. is a constant. Similarly K,,, is time-independent and can be omitted. The term Uo = f, pUdV will be retained. Since Ui = Hi-(p/p), and p and p are time-independent, we can replace the internal energy by enthalpy, which is given by Eq. 9.8-8, the second term on the right side of that equation being negligible. Furthermore, since the heat capacity is considered constant, Eq. 9.8-8 becomes simply A=A,+6(r-1) where the inlet slurry temperature T; has been chosen to the datum plane for the enthalpy. On the right side of Eq, 15.1-1, the mechanical work term and the kinetic energy and potential energy terms can be disregarded. But the heat addition term Q and the outlet enthalpy term must be retained. Therefore, Eq. 15.1-1 becomes w+Q or ad . eC,v = -w¢,(T-T,)+UA(T, -T) which is Eq. 15B.8-1. b. When the system has attained steady state operation, the left side of the above equation becomes zero, and the slurry temperature has leveled off to T.. This quantity is then defined by the equation 0=-w¢,(T..-T,)+UA(T, -T..) or (with the definition that UA/wC, =@): 15-30 T,-T, T,-T. which is needed presently. Now the differential equation can be rewritten in terms of the dimensionless variables given in Eqs. 15B.8-2,3: rs) lr) -of SET) -0(-2-60) -(@-1)=-@(1+9) Hence the differential equation in dimensionless form is de =-@(1+2 Fe 7 O1+9) c. This equation is solved to give ° a =-(1+Q)ffde or Ox ear This is just the dimensionless form of the solution given in the text. d, When the solution is written in the dimensionless form, it is easy to see that it satisfies the differential equation and initial condition. It is also evident that @—>0 (or T->T.), which is to be expected. \5-31 15C.1 Parallel-counterflow heat exchangers a. From Eq. 15.1-3, with only the enthalpy terms contribut- ing, and Eq. 9.3-8 with only the first term being important (and the heat capacity constant), we get for the region between a and b wyCya(Ts ~ Tra) WaCpa(TH ~Ta) =0 from which Eq. 15C.1-1 follows. b. Then the application of Eq. 15.4-4 gives, for the region between the dashed lines in Fig. 15C.1, three differential energy balances over the heat-transfer surface dA: WyCyadTh = du(rs -T))dA A 1 wyC,aTh = ~5U(To -TH)dA wyCypdTp = [duns -T,)+ du(ry = T.) fia Then introducing the ratio R, and the dimensionless differential area da (defined in the problem statement) leads directly to Eqs. 15C.1-2, 3,and 4. c. First we differentiate Eq. 15C.1-4 with respect to a to get Rdo® da 2\ da do 1d'Ty dT y {@ +)-0 Then using Eqs. 15C.1-2 and 3 this becomes 1@T, dT, 1 1 Raat ae g(t -Th)-4(TH -T,)=0 Next use Eq. 15C.1-1 to rewrite this as aT, dT, 1 Fatt R GE G(Ta-Tm)=0 or 15-32 In the second equation we have used the dimensionless temperature. This differential equation may be solved (see Eq. C.1-7a) as follows: e=c, exp[-3(R ~VR*=1)a +Cy exp[-3(R + VR? =1)al] = Ce" +Cye™* Application of the boundary conditions gives two equations for the integration constants: 1=C,+C, O=Cye™*" + Cye™er The integration constants are therefore: emer G=- 1 =~ Gar _ gia emer cn ear Lema = d. Next we obtain d0/da: do__me™* meme oe da 1_ VR Alar l-e an Then evaluate this at a =0 to get Lats) _ mm, 12 Ft lono 1 eV Her" 4 _ oR tar Ta Then eliminate the derivative by use of Eq. 15C.1-4 (also evaluated at a =() to obtain: \5-33 R m_ T-T,b Tm +4(Ta tT) ]= [elt * {oe Ra; o We now replace the denominator on the left side by using Eq. 15C.1- 1, and then we have to manipulate the left side in such a way as to obtain the ratio Y defined after Eq. 15C.1-8; thus the left side becomes: 3(Tar = Tor) +3 (Tao — Ta) _ (Tar = Tm) + 3(Pa2 Tm — Tar + To) Ta2-Ta Tya-Tn ~Ty-Ta , Ta -Tar With this substitution Eq. (*) gives ¥ as a function of a. But we would like to have ay as a function of ‘. That is, we have to solved the following equation for x: with A=m., B 0 3 But this is just a quadratic equation, which can be solved by the standard method, which yields two solutions: C-A =1 id x=— x=1 an ean The solution x=1 is physically uninteresting. The other solution gives ap (YY) +4)— me. _ (-24¥)-2%m__ 2-¥(1-2m_) Cy) (1-2m,) Jom, (-2+¥)-2¥m, 2—¥(—2m,) 2-¥(1-1+R-VR?4+1 © 2-¥(1-14R+VR? +1) VR oy e Taking the logarithm of this equation gives Eq. 15C.1-8. 15-34 15D.1 The macroscopic entropy balance a, First rewrite Eq. 11D.1-3 as (v-pvs) FlP-a)- F(t) ¥-ov8)-(v-ta)-Ze(a-v1)-4(x:vv) We now integrate this over the entire macroscopic flow system, which is presumed to have some moving parts in it so that the volume and surface of the system are time dependent: fest V=- Ae pSV - { (v-ta}v vit) vi -f Af(q-vinT)+(«:Vv)]av voT Since the last term is the integral of the entropy production terms in Eq. 11D.1-4, we can label this term gs... We may now use the Leibniz rule to move the time-derivative operator to the left of the integral sign and the Gauss divergence theorem to transform the volume integral into a surface integral: J 5eSt7= IoSv InorsSpis= SS fla-ovs}s t ve) When this is inserted into the preceding equation and the Gauss theorem applied to the integrals containing divergences, we get: d 3 1 ase ~Je -p(v-vs)5)As “S(a Aq\is + Sot We now divide the surface into four parts: S(t) = 5, + $, +5, +S, the cross-section of the entrance (1), the cross-section of the exit (2), the fixed solid surfaces (f, and the moving surfaces (mm). The first surface integral above will contribute only at I and 2: 15-35 ~ J(n-o(v—vs)8)}1S = 9,248,5; - p.028,5, = w,8, - 08, S(t) The second surface integral will, however, contribute at all four surfaces: Sere so) S82 ( ta) Wii __Waffr -B™_ f (n-oqlis=—h -eh_+Q, r, sls Ty pat, Pitt, Hence the macroscopic entropy balance becomes: a%-[s+aba}-(Seatg reese which is Eq. 15D.1-1. b. The macroscopic energy balances states that the increase in the total entropy of the system results from (1) the convection of entropy in and out of the system (w,5,-w,$,); (2) the entropy transport at the inlet and outlet of the system by heat conduction ((q:51/T;) ~ (q252/T))--presumably very small compared with the entropy convection terms; (3) the entropy transport through the walls of the system, and (4) the entropy production within the system, which consists of a term involving the heat flux and a term involving the momentum flux. Entropy is thus produced by the dissipative effects of heat conduction and viscous flow. These terms are positive for linear flux laws. c. The term in the entropy production term involving the stress tensor can be written 1 f(u:Vv)dv = ave Vib) if the temperature variation throughout the system is not too large with respect to the absolute temperature. Then the integral is indeed the energy dissipation by viscous heating divided by the average temperature. 15-36 15D.2 Derivation of the macroscopic energy balance When Eq. (N) of Table 11.4-1 is integrated over the entire volume of the flow system we get: J (boo? tel +p )av =- J (9. (¥-(J00? + pli +96)v)}av vit) - f(v-qyav - J(@-peav- J vba vo We now apply the Leibniz formula to the integral on the left side of the equation to get {(zee* sells pd)iv+ | J (n -(J90? + pli + pb)v.)ds a “ity 4 + Ure + Prop Jef or (jo0? «p+ 8) sv This is now substituted into the preceding equation (in which the volume integrals have been converted into surface integrals by using the Gauss divergence theorem) to get: 4 ag Reet + Usoe + Poe) = ~ J (0-(be0" + pli + pb)(v—vs))as -f(n ‘ads iin py)ds- Jt n-[t-v])dS S(t) We now divide the surface into four parts: S(t) =, +S, +S, +S, the cross-section of the entrance (1), the cross-section of the exit (2), the fixed solid surfaces (f, and the moving surfaces (m). The first surface integral above will contribute only at 1 and 2: “Se -(40v* + ptt + p@)(v—vs))as = (Sos(0t) + 01th, (2,) + 01, (0,))5; ~($2(v2) + Prtla(v2) + 02%, (02))S2 15-37 The contribution of the second surface integral at 1 and 2 is presumed to be negligible with respect to the convective energy transport, but will in general have a nonzero value on the surfaces; we call this Q. The third integral will contribute both at "1" and "2" and on the moving surfaces: ~ J(m-pv)ds = +p,(24)S; ~P2(02)S2+We? S(t) where W{) is the pressure contribution to the work done on the system by the via the moving surfaces. The fourth integral will be: = f(n-[e-v] sew so which is the stress contribution to the work done on the system by the via the moving surfaces. Of course there are also contributions at 1 and 2, but these are considered to be negligible to the pressure forces. When all of the above contributions are assembled, we get 4 (kg Ug +B) = tot (oslo? )+ pith (21) +14,(2,))S, - ($02(22) + aU (e2) + 02%2(09))S, +Q+P1(01)S1 — Po (2)S2 + Wh + Wi? which is the macroscopic energy balance given in Eq. 15.1-1. 15-38 15D.5 The classical Bernoulli equation For an isentropic flow, * A 1 “ a“ 1 dU =-pdV = -pd— or VU=-pVV=-pV— pav =— pd p ae Then Eq. (E) of Table 11.4-1, with the heat and momentum fluxes set equal to zero, becomes for a steady-state system p(v-V40") + p(v-(-pp))=-(V-pv) +p(v-8) Using g=—gVh and some standard vector identities p(v-V}0")+pp"(v- Vp) =—p(V-v)-(v-Vp)—p(v-Vh) The second term on the left and the second term on the right can be shown to be equal, using the equation of continuity (appropriately simplified for steady-state flow). Next, consider that the del operators are acting only along a streamline, so that V = (v/v)(d/ds), and hence we get 41g) a POA nye 4 ov .(0*)= pas? PPS gst When this equation is divided through by pu, we get Eq. 3.5-11. Then Eq. 3.5-12 follows at once. \5-39 16A.1 Approximation of a black body by a hole in a sphere Use Eq. 16.2-12 -— 2 fvote = FF Fle) or - (1- note) pote €) Into this we insert the values ¢ = 0.57 and ¢,.,. = 0.99 and get 0.57(1-0.99) _ 0.57(0.01) — 0.0057 0.99(1-0.57) 0.99(0.43) 0.4257 f = 0.01339 Then using the definition of f we write 2 Thole. 4n(3*) Then solving for the hole radius we get f= 0.01339 = Thote = 2V0.01339(3) = 0.69 in. 16-1 16A.2 Efficiency of a solar engine The area of the mirror is (aR*)=25zft?. Since the solar constant (heat flux entering the earth's atmosphere) is, according to Example 164-1, 430 Btu/hr-ft?, the energy input to the solar device is 430 Btu/hr -ft?)(252 ft?)(3.93x10 hp/Btu-hr*)= 13.3 hp ne -f2 Therefore the efficiency of the solar device is Efficiency = 3 = 0.150 = 15% \G-2 16A.3 Radiant heating requirements The heat required is the sum of the radiant heat-transfer rates betwen the floor and each of the other surfaces. Since no pertinent data are supplied for its estimation, the convective heat transfer will be neglected. It can be expected to be appreciable, however. The total radiant heat-transfer rate is then Qua = Ancor (Toor ~ Tats Fi Floor to ceiling: F, =0.49 Floor to large walls: F, =F, =0.17 Floor to small walls: F, =F, =0.075 Summing the contributions, we get: DF; = 0.49 +2(0.17) + 2(0.075) = 0.49 + 0.34 + 0.15 = 0.98 Alternatively, we may consider the floor to be completely surrounded by black surfaces. From that point of view we know that the sum of the F, should be 1.00. A cumulative error of 2% has thus resulted from considering each of the cold surfaces separately. Then we get Qua = (1-712 10” Btu/hr- ft?R*)(450ft”) -[(75 + 460)* - (-10 + 460)*]R* = (1.712 x10)(450)(8.19 x10” - 4.1010”) =3.15x10* Btu/hr Here we have used the value of the Stefan-Boltzmann constant given on p. 867. 163 16A.4 Steady-state temperature of a roof Since June 21 is (conveniently) very close to the summer solstice, the angle of incidence of the sun's rays on a flat roof may be calculated quite simply. We know that the earth's axis is tilted at an angle of about 23.5 degrees. Thus the angle of incidence of a flat roof at 45 degrees north latitude will be about 45-23.5=21.5 degrees, and the heat received by the roof will be given by the solar constant multiplied by the cosine of the angle of incidence and then further multiplied by the absorptivity of the surface: (430)(cos 21.5°)a. = (430)(0.9304)a., in units of Btu/hr.ft2, We now equate the radiant energy received from the sun by the roof to the radiant energy emitted by the roof plus the heat lost by convective heat transfer for the two cases given in parts (a) and (b): (430)(0.9304)a = h( Typos ~Tue) + O8T Sp a. For a perfectly black roof, we have (430)(0.9804)(1.00)= 2.0(T yy ~ 560) + (1.712 10" )(1.00)P8,, This equation may be solved by trial and error to get about 625°R or Dee tree eee eee 0.3 and e = 0.07 we get (430)(0.9304)(0.3) = 2.0(Tyaag ~ 560) +(1.71210")(0.07)T sy which may be solved by trial and error to give 610°R or 150°F. ib-4 16A.5 Radiation errors in temperature measurements Assume that the thermocouple behaves as a gray body in a large black enclosure, and equat the net radiation loss to the convective heat input: eo(Ti, - Than) = W( Taps -T x) in which T,, is the thermocouple junction temperature. For the conditions of this problem (0.8)(1.712 10 (960)* = (760)*}=(50)(Tyas 960) Solving this for T,,,, we get 705 _ oro ° Ty = 960+ 7 = 971°R = 514° F There is thus a 14°F difference between the calculated gas temperature and the thermocouple reading. 16A.6 Surface temperatures on Earth’s moon. (a) A quasi-steady-state energy balance on a lunar surface element that directly faces the sun gives Ina = co max in which Jp is the solar constant, T, max is the temperature of that surface element, and a and ¢ are its total absorptivity and emissivity. Setting a = ¢ for a gray surface, and using the value calculated in Example 16.4-1 for the solar constant, we obtain the quasi-steady-state estimate Tajmax = (Ioo)/* = (430/1.7124 x 107°)'/4 = 708°R. of the maximum temperature on the moon. (b) For a spherical lunar surface, receiving radiation from the sun only, the local intensity of incident radiation is _ fncosé, for0<$0< x/2; n= {i for r/2 T, The left side gives T =T, as t goes to infinity, and T=T, as f goes to zero. The left side is dimensionless, and the right side is also, as can be seen by using the table of notation given on pp. 872 et seq. Cer) nyo (Bee) = dimensionless Ibi 16B.6 Heat loss from an insulated pipe. (a) Eq. 10.6-29 gives for this problem, 2n(To Tle) For Taz Qk" n= with ro = 1.0335 in., ry = 1.0335 + 0.154 = 1.1875 in.and rz = 1.1875 +2 = 3.1875 in. With the given thermal conductivity values, we then obtain Qn) 1 = 2n(To - Te) TLIO) ISTEP) _ 2x(250 ~ Ta) ~ 0.0053 + 2.8 - {z Btu/hr-ft if Ty = 100°F; oO if T, = 250°F (b) The net radiative heat loss is given by Eq. 16.5-3. Setting = the aluminum foil, we get, 0.05 for QO /L = onD2(0.05)(T3 — Te) = 0.1712 x 107*x(2 x 3.1875)(0.05)(T¢ — 540") _ [23 Btu/hr-ft if Ty = 100°F = 560°R; * {290 Btu/hr-ft if Ty = 250°F = 710°R The free-conveetive heat loss is predictable as in §14.6. For T, = 100°F, Ex- ample 14.6-1 gives Qe") /L = 18 Btu/hr-ft For T, = 250°F, Eqs. 14.6-4,5 and Tables 14.6-1,2 give Nul™ = 0.772(0.515)(GrPr)!/* = 0.398(GrPr)!/* ‘The needed properties of air at Ty = (80 + 250)/2 = 165°F= 74°C are obtained from the ideal gas law, from Table 1.1-2, and from CRC Handbook of Chemistry and Physics, 81st Ed., 2001-2002, pp. 6-1, 6-2, and 6-185. # = 0.0206 mPa-s = 0.0498 Ib /ft-hr = pM/RT = 1.017 x 10~* g/cm* = 0.0634 Ib», /ft? Cy = 1.012 J/g/K = 0.2420 Btu/Ib» /edotR k = 29.5mW /m-K = 0.0170 Btu/hr-ft-F B=1/T, = (1/625)R™* 16-13 Hence, GrPr (2 x 3.1875/12)°(0.0634)?(4.17 x 10°)(170/625) \ / (0. ~ (0.0498)? = (2.76 x 107)(0.709) = 1.96 x 10” and Num = (0.398)(GrPr)'/4 (0.398)(1.96 x 107)'/4 = 26.5 ‘Therefore, QE) /L = hima D(To ~ Te) = (Numk/D)(#D)(Ty ~ Te) = Numtk(Ty — Ta) = (26.5)x(0.0170)(250 — 80) = 241 Btu/hr-ft giving Qbeona) — Qed) — Qleom) __( 334 — 23-18 = 293 Btu/hrft if Ty = 100°F; = {0-290 — 241 = -531 Btu/hr-ft if Ty = 250°F. (c) Linear interpolation to zero heat accumulation at the outer surface gives the steady-state values Ty = 100 + 293/824(250 — 100) = 153°F Qn) (L = Quad Hon) 17, — 334 + (293/824)(0 — 334) = 215 Btu/hr-ft [ola 17A.1 Prediction of a low-density binary diffusivity. (a) We begin by looking up the needed properties of the species from Table E.l: Species M,g/e-mol T., K Pe, atm A: CHy 16.04 191.1 45.8 B: CoH 30.07 305.4 48.2 Equation 17.2-1 then gives the following prediction of Day for methane-ethane (treated here as a nonpolar gas-pair) at p= 1 atm and T = 203K: . m0 (peg) Coareny eaten) (Ma + 1/8)” In od 1923 arta) + (45.8 x 48.2)'/3(191.1 x 305.4)°/12(1/16.04 + 1/30.07)"/?/1 atm = 0.152 em?/s Das = 2.745 x 107% ( (b) Equation 17.2-3 gives 1 (peapen)!® _ 6 (1 4 1)” @ecapen) (cDap)c = 2.96 x 10 (z (DeaTep)7? _ -e(_1_ 1)" (45.8 x 48.2)0/9 = 2.96 x 10° (aa +3007) Gora x 305.47 = 4.78 x 10~* g-mol/em-s ‘The reduced conditions for Fig. 17.2-1 for this problem, calculated as described on page 522, are Te or VTaTes V9.1 x3054 _ P _ 1.0 © VPeaPes 45.8 X 48.2 At this reduced state, Fig. 17.2-1 gives (cDaz)+ = 1.20. Hence, the predicted value of cDap is 1.20(cDan)e = 5.74 x 10-*. Dividing this result by the ideal-gas prediction ¢ = p/RT = 4.16 x 10-* g-mol/cm~* at this low-density condition gives Dap = 0.138 cm?/s. = 0.021 Pr (c) Equations 17.3-14 and 15 give the binary interaction parameters oap = (3.780 + 4.388)/2= 4.0844 and ean/x = VI54 x 232 = 189 K \T-\ when the Lennard-Jones parameters of Table E.1 are used for the individual species. Then, at KT/ep = 293/189 = 1.550, Table B.2 gives Qp,4p = 1-183, and Eq. 17.3- 12 gives the prediction 1 16.04 = 0.018583, /(293)* ( 1 ) (D@084)2(1-183) = 0.146 cm?/s (d) Use of Eqs. 1.4-1la,e with the combining rules of Eqs. 17.3-14,15 gives the estimates €aB/K = 0.77V 191.1 x 3054 = 186.0 K 1/3 ays . cap = 24 (#) eo (2) = 4.2224 2 Pca PeB Then at KT/eap = 293/186.0 = 1.575, Table B.2 gives p,a5 = 1-1755, whereupon Eq. 17.3-12 gives 1 1 1 , = hoo) — a aloe ae) ( 6.04 sai) (W4222)7(1.1755) = 0.138 em?/s 12 17A.2 Extrapolation of binary diffusivity to a very high temperature. (a) Equation 17.2-1 in the nonpolar form gives Daslisoox = DAB| 29K (1500/293)' "7? = 2.96 cm?/s (b) Equation 17.3-10, with n = p/KT, predicts Dag o T*/?, thus giving Davlsoox = DAB lagax (1500/298)" = 1.75 em?/s (c) Equation 17.3-12 and Table E.2, with ¢4m = 135.8 from Table E.1 and Eq. 17.3-15, gives 20,40 lassx jean PaBlisoox = PAB|yosx(1500/293)°/? (= ) = 2.51 cm?/s AB s00x ean ‘The superior agreement of Method (c) with the experimental value of 2.45 em?/s illustrates the wider range provided by the Chapman-Enskog theory when combined with the Lennard-Jones potential-energy model. (1-3 17A.3 Self-diffusion in liquid mercury. = \ 1A KT (Na Daw = BS rua Equation 17.45 gives or in egs units, (1.38066 x 10-!erg/K)(T,K) (oa x shectaisiemet) ae = EERE BAKE) | SRRETE x oie mot 2n(Ha,g/em's) (200.61g/g-mol)( Va, em*/g) Insertion of the values tabulated for this problem, with (ep) divided by 100 to get HA,g/cm-s, gives the following results: T(K) Daar,obs. Daar, pred. —_ Ratio, pred. /obs. 215.7 152x1075 1.2410" 0.82 289.6 168x10-° = 140x110" 0.84 364.2 257x10-° 2.16 x10" 0.84 The predicted self-diffusivities are about 5/6 of the measured values. 17A.4 Schmidt numbers for binary gas mixtures at low density. We begin by tabulating the needed molecular parameters for species A and B from Table E.1, and estimating the binary parameters ¢4p and ¢4p/K from Eqs. 17.3-14 and 15: Species M,g/g-mol 9, A e/K,K A: Hy 2.016 2.915 38.0 B: CChF2 120.92 5.116 280. AB: 4.0155 103.15 Equation 17.3-11 and Table E.2 then give the following prediction of cD4z for binary mixtures of H and Freon-12 at T = 25°C=298.15 K: 2646 x 1075 (3 +a) zy Dap = Ma * Mp) o%p90,a5 L 1 1 = 2.2646 x 10-5, /298.15 (<> + > )__7____ * (5 ore * mm) (£0155)°(0.9597) = 1.794 x 107 g-mol/em-/s With this prediction of cDag and the viscosity data of Problem 14.4, the Schmidt number can then be calculated as Se=—t_ = Hs pDaw- McDap (taMat+zpMp)cDap in accordance with Eqs. G and L of Table 17.7-1. Results are as follows: ta=rn, 0.00 0.25 0.50 0.75 1.00 gH, g/em-s 124.0 x 10~® 128.1 x 10-® 131.9 x 10-® 135.1 x 10-* 88.4 x 10-® M, g/g-mol 120.92 91.194 61.468 31.742 2.016 Se 0.057 0.078 0.120 0.237 2.44 We see that the Schmidt number depends strongly on the composition when Mg and Mg differ greatly. This fact is also illustrated in Table 17.3-1. VFS 17A.5 Estimation of diffusivity for a binary mixture at high density. ‘The following properties of Nz and C2Hs for this problem are obtained from Table El: Species M,g/g-mol Te, K Pe, atm, A: No 28.01 126.2 33.5 B: C2H¢ 30.07 305.4 48.2 (a) The measured value Dag = 0.148 cm?/s at T = 298.2 K permits calcula- tion of an experimentally-based value of (eD4x)- The reduced conditions for this measurement are T 298.2 rr VleaTen — V126.2X 3054 10 = 0.025 = a Pe reads 555 XABD On Fig. 17.2-1, this state lies essentially at the low-pressure limit, with (cDap)r = LAT. Accordingly expressing ¢ by the ideal gas law, we find Dap = ppPaw _ atm A ~ (206 x 208.2 cm®-atm/gmol 148 em? /s) = 6.05 x 10-* g-mol/em-s Hence, the critical Dag value is (cDaB)c = ¢Pan/(cDan)r = 6.05 x 10-°/1.47 = 4.12 x 107° g-mol/em-s Now, the reduced conditions for the desired prediction are Tr 28 T= eee = ee eee VTaTep 126.2 x 305. Pr 40 _ 0.995 _——————————— ~ VpeaPes 33.5 x 48.2 17.2-1 gives (CDag), = 1.42. The resulting prediction is then (cDan)e(cDap)r = (4.12 x 107*)(1.42) = 5.8 x 1078 g-mol/cm-s. (b) Equation 17.2-3 gives the predicted critical value (cDap)c = 2.96 x 10-* (ga = 3.78 x 10-6 g-mol/em-s. Multiplication by (Dap), = 1.42 as in (a) gives cDap = (3.78 x 107*)(1.42) = 5.4 x 107% g-mol/em-s. \T-& 17A.6 Diffusivity and Schmidt number for chlorine-air mixtures. (a) We begin by tabulating molecular parameters for chlorine and air from Table B.1, and estimating the binary parameters 24g and €4p/K from Eqs. 17.3- 14 and 15: Species M, g/mol 0, A elk, K A: Cl 70.91 4.115, 357. B: Air 28.97 3.617 97.0 AB: 3.866 186.1 Equation 17.3-12 and Table E.2 then give the following prediction of Dap for chlorine-air mixtures at T = 75°F = 23.89°C = 297.04 K: Dap = 0.0018583,/T? (Gr 1 1 Ma i) Po%p2D,aB 1 1 1 = 2 ——————— = 0.00185834 (297.04) (aa: Ta) eR TT = 0.120 em?/s (b) Equation 17.2-1 needs the following values from Table E.1: Component M, g/g-mol —T;, K Pe, atm A: Ch 70.91 417. 76.1 B: Air 28.97 132, 36.4 ‘The nonpolar version of Eq. 17.2-1 then gives the prediction 1.823 Dap = 2.745 x 10-* ( + (76.1 x 36.4)'/3(417. x 132.)°/12(1/70.91 + 1/28.97)!/?/1 atm = 0.123 em?/s (c) The result of (a), and the ideal gas expression for ¢, give Dap = pas = 4.92 x 107° g-mol/em-s With this prediction of cD4p and the viscosity predictions of Problem 14.4, the Schmidt number can be calculated as Se=—t— ee ——— pDas McDap (taMa+2pMp)cDap in accordance with Eqs. G and L of Table 17.7-1. Results are as follows: AI TO, 0.00 0.25 0.50 0.75 1.00 y,g/ems 0.000183 0.000164 0.000150 0.000139 0.000131 M, g/g-mol 28.97 39.455 49.94 60.425 70.91 Se 1.28 ost 0.61 0.47 0.375 We see that the Schmidt number depends strongly on the composition when Ma and Mg differ greatly. This fact is also illustrated in Table 17.3-1 and in Problem 17A.4. iT8 17A.7 The Schmidt number for self-diffusion (a) Equation 1.3-1b, written for non-tracer species A, gives 10 x 10-° MY pe THA Be = eA, for the critical viscosity in g/em-s. Here M, is in g/g-mol, pa in atm and T.4 in K. Eq. 17.2-2 gives (Darw)e =2.96 x 107° (41 4 4 7 ares coal Ma* Ma.) Peavea ‘The resulting critical Schmidt number for self-diffusion with M4 ~ Ma. is H 7.70 —#_). = 184 Gata), 2.962 (b) Figs. 1.3-1 and 17.2-1, with the result in (a) for Ma © Mas, give Se for self-diffusion as the following function of T, and p,? se(TrsPe) S . = 1.84-— “aa (Daae)e(ToPe) Calculations from this formula are summarized below: Phase Gas Gas Gas Liquid Gas Gas T, 0.7 1.0 5.0 0.7 1.0 2.0 Pr 0.0 0.0 0.0 sat. 10 1.0 Hr 0.32 0.45 1.62 78 1.00 0.94 (cDaae)r 0.72 1.01 3.65, 0.37 1.03 1.83 Seaas 0.82 0.82 0.82 39 179 0.94 17A.8 Correction of high-density diffusivity for temperature. ‘The following properties of CHy and CzHg for this problem are obtained from Table El: Species M,¢/gmol Te, K Pe, atm A: CHy 16.04 191.1 458 B: CoH, 30.07 305.4 48.2 ‘The reduced conditions (State 1) for the given Dap value, calculated as de- scribed on page 522, are 313 136 Sess F130; p= ae = 2.89 Vistrxasa 8% = agsxass and Fig. 17.2-1 gives (Dap), = 1.27 at this state. The reduced conditions (State 2) for the desired prediction are T, 351 136 T= 15, pp = =. Vioi.1 x30 = Tasexaea 8 and Fig. 17.2-1 gives (Dag), = 1.40 at this state. The prediction of cD4p is then obtained as follows: (Pr|gtate 2 (P)elstate 1 = 66x 10-* gmol/es (Dab) |state 2 = (Pa2)|state 1 ‘The observed cD4g value at State 2 is 6.3 x 10-® g-mol/cm:s, in fair agreement with this prediction, . iH10 17A.9 Prediction of critical Day values. (a) Equation 17A.9-1 gives KT/eaae = 1/0.77 = 1.2987 at T = Tea, and Table E.2 gives Qp,aa+ = 1.2746 at this argument value. Insertion of this result, along with Eq. 17A.9-2, into Eq. 17A.9-1 gives 2.2646 x 1075 1 1 1 (Dasede = OE [tea Gin tae) 1.01 Ma ~ Maz} (2.44(Tea/pea)/*)?(1.2746) . (Ay 1 yl ge = 2.955 x 10 (a +i ree which verifies Eq. 17.2-: from low-density self-diffu within the uncertainty of the coefficient 1.01 determined data. (b) Evaluation of the component parameters in Eqs. 17A.9-4,5 according to Eqs. 1.4-1la,c gives iF ie TesPen\ "8 cap = 244 (2) (72) =2.44 ( A 7) pea) \ pew PeaPes 48 (/Mg) = The derivation of Eq. (O) is done in like manner. b. To get Eq. (P’) from Eq. (P), 1-3 In going from line 1 to line 2, we used Eq. (M) and then Eq. Eq. (K). To get line 3, we used Eq. (K) again. The same method is used to get Eq. (Q) fro Eq. (Q). c. First we must write the binary equivalent of Eq. (N) in such a way that only one variable appears on the right side (we have to keep in mind that the two mass fractions cannot be varied independently): Os O, : Ma My = a _ =. __ Ma __ 0, , ©, A Oe On, M, Ms Ma, Mg Then to get Eq. (P’), we differentiate - MMs 7D A TO On Oe On On My, Mz My Ms The derivation of Eq. (Q’) from Eq. (O) proceeds similarly. Vo, 114 17B.2 Relations among the fluxes in multicomponent systems a. To verify Eq. (K) of Table 17.8-1, we proceed as follows: vooN N N Die= 2% Pa(Va-V)= 2 PaVa~ 2 Pa¥ ma a r= Ny N =pPLOw-V> Pa = a =pv-vp=0 In the second line, we have used Eq. (C) of Table 17.7-1, and to get the third line we used Eq. (B) of the same table, as well as Eq. (B) of Table 17.2-2. The proof of Eq. (0) of Table 17.8-1 proceeds analogously. b. The verification of Eq. (T) of Table 17.8-1 follows directly from the definitions of the fluxes: je =Pa(Va-¥) and Ng =PaVa Substitution of these definitions into Eq, (T) gives v Pa(¥a-¥)=Pa¥a~ On Pave = This can be rewritten as x Pa a ~ PaY = PaV a ~ 0a %(65/0)¥s = or, making use of Eq. (C) of Table 17.7-1, x —Pa¥ =—Pa YOpVg B= Then, use of Eq. (B) of Table 17.7-2 completes the verification. The verification of Eq. (X) of Table 17.8-1 may be done analogously. ihe 17B3 Relations between the fluxes in binary systems The expressions for the fluxes are ia =Pa(va-v) Ja=ca(va-v*) or MaJa=Pa(va-v") Next form the difference between the above expressions, and then repeat for species B ja~MaJa=Pa(v-v*) and in ~ MsJa = Po( ¥-v*) Then eliminate v—v* between these two equations to get ja _ Maa _ in Modo Ps Px PsP Then use Eqs. (K) and (O) of Table 17.8-1 to eliminat the fluxes with subscripts B and then rearrange to get i222) =7,(Mo+Me) or ifo+2)-n{2+ “lon 0s) “Ven” Ps “Vp, Ps) “Lea whence j (tactee) =y,( Sater} “CU pats ) “(cate where Eq, (H) of Table 17.7-1 has been used. Next use Eqs. (B), (C), (B), and (F) to get Eq. 17B.3-1. 17-16 17B.4 Equivalence of various forms of Fick's law for binary mixtures (Note: Problem 17B.3 should be worked prior to Problem 17B.4) a. To get Eq. (B ) of Table 17.8-2, we start by rewriting Eq. (A) by using Eq. 17B.3-1 and Eqs. (Q’) and (L) of Table 17.7-1: + POWs __ yy MaM 1 ceary PDA Bx, Next, use of Eq. (G) of Table 17.7-1 allows us to rewrite this as Vu = Sea 08g HAN AB vx, 0,0," (jer ‘Then we note that Eqs. (C), (F), and (H) of Table 17.7-1 give @, Cpa A so that . e(p 1 pl , --(2 tot pou MMs oy, = 89 430% ae ole M, cMz at 4 apenas Equation (D) of Table 17.8-2 follows at once from Eq. (B) of Table 17.8-2 and and Eq. (V) of Table 17.8-1: Ny =c,v*4J4 = Cav *—CD 5 V2 4 Equation (F) of Table 17.8-2 is obtained by writing Eq. (C) of the same table for both species A and B after using Eq. (I) of Table 178-1: ca(Va—V*)=—CB agVxq and Cy(Vp—V*)= CA ggVxp = +69 45 VE 4 These equations may be rewritten as eA and (vp-v*)=+ 29,04 Xp Subtraction of these equations eliminates the molar average velocity, and we get +2 Nouv, =-Savy, XB X4Xp b. To get the first equation, we use is Eq. (Q') of Table 17.7-1 . M4M, ja =-PBasV0 =-P® ania VX From this equation, we conclude that the expressions written in a mixture of molar and mass quantities are more complex than those written entirely in molar quantities alone or mass quantities alone. To get the second equation, we start with Eq. (B) of Table 17.8-2 (which we have seen in (a) can be obtained from Eq. (A)), written for species A and B J = ~€® gpV% 4 Jp =— CD gp Vp = +6D ppVXq Then we rewrite these equations in terms of the combined molar fluxes (using Eq. (V) of Table 17.8-1) Ny -cav* =-c® y,VX4 Ng ~cpV*= +O qgV4 We now multiply the first equation by x, and the second by x, to get xgN 4 — CX 4X gV* = —X5CD y,VX 4 x4Np—CX4XpV" = +2 4CDspVX 8 Subtracting the second equation from the first then gives Eq. 17B4-2 after use has been made of Eq. (J) of Table 17.7-1 x4Ng—X5Nq =CD nV, This equation contains no reference to the mass average velocity or the molar average velocity. That is true also of Eq. (E) of Table 17.8- 2, which can be derived in analogous fashion from Eq. (A) of the same table. c. We start by rewriting Eq. (F) in the following form 1 Xp +X, Vx, =-D45| “2774 |ux, XAXp XaXp Va-Vgp=—-Dap This may also be written as 3 \I9 17C.1 Mass flux with respect to the volume average velocity a. Using the definitions of jM and j, we get ve mc 4Vavg + CpVav9= oa¥,( )saavi( ies v) Using the relation just after Eq. 17C.1-3 the above result gives the relation between the volume average velocity and the mass average velocity m_yi{Va_ Va ovina) If we now subtract Eq. (E) of Table 17.8-1 from Eq. 17C.1-2 we get v, Vp -»)--o 2-2, Then, using the relation just after Eq. 17C.1-3 we get iM jn =-Ps , ¥, V; Vy V, in wis(2-u P+, B}- inex te + py Ve Ms and this leads to Eq. 17C.1-3 . V, Vs\_, 0 V; im =is(o» me tPa #) haha b. Starting from Eq. 17C.1-3 we have Vv, “08 von) 12 Mp 1 1 1-c,V, “rt a{ 200,040 aes) £8 u( Ave, +0402 pf ae 17-20 1 Pa Pa, Px \( 1=CaVan =-pDj4,| —Vp, -24-Vp || Pa +2 |] Sala p (3 Pap 0 i ht ce A =-B@pVPq~(B nV) LAE 8 Be LAs a) a Ca My Cp pew) fe) =-DasVen—(DasVPa (e pectve «iVa) {2 D40V(Pn *pe)}e%s =-D4pV0n +47 4D a5 VPa +65 508 Basho =-B43V04 +P, Dap(VaVen +V5VCp) Since the second term on the right side is zero, we get Eq. 17C.1-4. To get the equation just after Eq. 17C.1-4, we proceed thus. The definitions of the partial molar volumes enable us to write dV =V dn, +V dng Since the volume is a homogeneous function of order 1, we get from Euler's theorem: V=n,V, + 1gV 5,01 by dividing by V: 1=c,V,+cpV5 (#) Forming the differential of V from this equation gives dV =n,dV,+V dn, +ngdV, +Vpdny Comparing this with the first equation gives dV, + ngdV, = 0, or by dividing by V: c4dV, +c,dV,=0 (*) Differentiating (#) and subtracting (*) gives the desired result. \y-21 17C.2 Mass flux with respect to the solvent velocity a. First we rewrite Eq. 17C.2-1 as ja =Pa(Va-¥)~Pe(Vn -¥) Then we modify the terms on the right side as follows a Pa _. (Pw yi = Pa (Va ~¥)-| 2 low (vy -¥)= Ie -| 2 B= Pale -v)-(2¢)p(vn-¥)=Ie-( fin Here we have used the definition in Eq. H of Table 17.8-1. b. Application of Eq. 17C.2-2 to a binary system with N = B gives iN =i -(%}i Then we make use of Eq. (K) of Table 17.8-1 and Eq. (A) of Table 17.8- 2 to get i=in -(2}iad=n(2+24) Ps . (PatPs\_, (P p =j,{ Pate) =; (2)-[2\p9,,v0, { Ps } iff) (2) aoa c. For a very dilute solution of A in B, we know that p, =p, SO that i, =~PD4,VO4 7-2 17C.3 Determination of Lennard-Jones parameters from,diffusivity data for a binary mixture. (a) Computational software is preferable to graphical methods for treatment of this statistical problem. The results shown below were obtained by Dr. Mike Cara- cotsios, using Athena Visual Workbench, a Windows package for formulation and solution of chemical process models. The problem formulation and results are dis- played fully at www.AthenaVisual.com /transportphenomena, a website provided by Stewart & Associates Engineering Software, Inc. In the following table, PAR(1) (the first adjustable parameter), denotes 4p, and PAR(2) (the second adjustable parameter), denotes ¢4g/K. The values in the “Last PAR(I)” column, with status “Estimated”, are the recommended pa- rameter values in the sense of maximum posterior probability density based on the data provided. The individual uncertainty of each parameter is reported as “95% Marginal HPD Interval” (the interval of highest posterior density contain- ing 95% of the posterior probability distribution for that parameter; we prefer this Bayesian interpretation to the notion of a “confidence interfal”, which invokes a hypothetical population of alternative sets of data). The two columns labeled DY- DPAR(1:NPAR) are the parametric sensitivities of the predicted diffusivities to the individual parameters. Further documentation of this problem and the modeling capabilities of Athena Visual Workbench are provided at the website named above. NONLINEAR SINGLE-RESPONSE ESTIMATION STATISTICS SUMMARY ‘SUM OF SQUARES OF RESIDUALS + 3.15638D-05 RESIDUAL DEGREES OF FREEDOM. : 6 MODEL PARAMETERS ESTIMATED. 2 RMS VALUE OF WEIGHTED RESIDUALS. + 2.29361D-03 '7(0.025)-STUDENT DISTRIBUTION VALUE, ss 2.447 ADJ.PARS Last PAR(T) Status 95% Marginal HPD Interval PAR( 1) 3.313569E+00 Estimated 3.313569E+00 +- 4.080E-02 PAR( 2) 8.024473E+01 Estimated 8.024473E+01 + 1.205E+01 EVENT OBSERVED PREDICTED" RESIDUAL —_-ERROR(%) DYDPAR(1:NPAR) -> 1 4. 7000E-01 4.7273E-01 --2:7282E-03 © -0.58 -2.8533E-01 -1.22428-03 2 6.90008-01 6.9072E-01 -7.2080E-04 -0.10 —-4.1690E-01 -1.6487E-03 3 '9.40008-01 9.3925E-01 7.52178-04 0.08 «= -5.6691F-01 —-2. 1090-03 4a 1.2200E+00 1-2161E+00 3.8625E-03 0.32 -7.3404E-01 -2.6078E-03 5 1.5200E+00 1-5198E+00 2.24348-04 0.01 -9.1731E-01 ~3.1524E-03 6 1.8500E+00 1-84898+00 1.0664E-03 0.06 = -1.1160E+00 © -3.7483-03 7 2.20008+00 2.2026E+00 -2.6318E-03 -0.12 -1.3295E+00 --4.3978E-03, 8 2.5800E+00 2.5801E+00 — -5.1116E-05 0.00 -1.5573E+00 5. 1009E-03 NUMBER OF MODEL CALLS. 13. NUMBER OF ITERATIONS. 4 11-23 18A.1 Evaporation rate. Let A denote chloropicrin and B denote air; then Eq. 18.2-14 gives, for constant total pressure p and ideal gas behavior, Natlene, = oot n ( 222 sea Goa) en =P Das 4, () RT (22-2) PBL Inserting the values T = 15°C = 298.15K, pp2 = p = 770 mm Hg and pp = 770 — 23.81 = 746.19 mm Hg, we get Nal (770/760) atm (0.088 em?/s) In 770 Atle (62.06 x 298.15) cm*atm/g-mol (11.14 em) 746.19 = 1.03 x 1078 g-mol/em?-s Finally, the evaporation rate in g/hr is Wa =Na:MaS = (1.03 x 10-* g-mol/cm?-s)(164.4 g/g-mol)(2.29 cm?)(3600 s/hr) = 0.0139 g/hr [S-1 18A.2 Sublimation of small iodine spheres in still air. (a) From Table E.1 and Eqs. 17.3-14,15, we get the following values for the systern Ip-aii Species M oA elk, K Ak 253.81 4.982 550. B: air 28.97 3.617 97.0 AB 4.2995, 231.0 Thus, at T = 40°C = 313.15K, we get the argument value KT/e ap = 313.15/231.0 = 1.356, at which Table E.2 gives Qp,ap = 1.251. Equation 17.3-12 then gives 1 = 5 2 0.018583, /(313.15)' (aa 1 a) (747/760)(4.2995)?(1.251) = 0.0888 cm?/s (b) Equation 18.2-27, with rz + 00, gives = tg, PDAB =a ar G aa) _ (747/760 atm)(0.0888 cm?/s) = 44(0.5 cm) 9 06 x 313.15 cmPatm/g-mol) = 2.95 x 107® g-mol/s x 3600 s/hr = 1.06 x 1074 g-mol/hr \8-2 18A.3 Estimating the error in calculating the absorption rate. Equation 18.5-18 gives Wa = KesoVDan in which K is a product of known quantities. Then the error in W. resulting from small errors Acay and ADap is AWa= (#2) Acao + ( oWa ) ADap Acao ODaB. Keao =KVDapA AD, ap Acao + 5 re ADaw Division by W then gives the fractional error expression AWa _ Acao , 1ADan Wa ~ cao | 2 Dap Hence, the maximum absolute percentage error in the calculation of Ws under the given conditions is AWa Wa 100 =5%+ 300%) = 10% 13-3 18A.4 Chlorine absorption in a falling film. ‘The absorption rate is predicted by Eq. 18.5-18, which may be rewritten in terms of the average film velocity by use of Eq. 2.2-20: Wa = 2nRLeao\/ The solubility is cao = pwao/Ma % (0.998 g soln/em*)(0.00823 g Clz/g soln)/(70.91 g Clo/g-mol Cl) = 1.16 x 107* g-mol Cl;/em* Then the predicted absorption rate is Wa = 2a(1.4 x 13 cm?)(1.16 x 107* g-mol Clz/em*) 6( m?/s)(17 Js em) = 7.58 x 10° g-mol/s = 0.273 g-mol/hr 6 x 10 em/s) 18-4 18A.5 Measurement of diffusivity by the point-source method. (a) Directly downstream of the source, the distance s from the source reduces to z; hence, Eq. 18C.1-3 takes the form Wa Daze Therefore, the injection rate Wa required to produce a mole fraction 24 ~ 0.01 at p=1atm and T = 800°C= 1073K at a point 1 cm downstream of the source is Wa =4rDap(0.01e)z = 4xDa4p(0.01p/RT)z = 4x(5 cm?/s)(0.01){(1 atm)/(82.06 x 1073 cm*atm/g-mol)|(1 cm) = 7.1 x 107° g-mol/s cA= (b) Expansion of s in powers of r? at constant z gives, to second order in r, i and and 1p z r i= [-35+-] Equation 18C.1-3 then yields the following expansion for constant z, complete through order r?: eaters) = eA Fexpl-(00/2Dan\o— 2) =ea(0, ofa J fp-cor2D a0 [35 -- |+-] = ca(0,z) in = EE (1+ (2/2Dau)) +0 (3)| Thus, ca(r, 2) and x4(r, z) will be within 1% of their centerline values as long as the second-order term within the square brackets does not exceed 0.01. The deviation r, of the sample collector from the axis, at the given conditions, therefore must satisfy 227 1+ v92z/2DaB _ (0.02)(1 em?) ~ 1+ (50 cm/s)(1 cm)/(2 x 5 cm?/s) = 0.00333 cm? r, $ V0.00333 cm? = 0.058 cm r? < (0.01) Hence, 18-5 18A.6 Determination of diffusivity for ether-air system. (a) Equation 18.2-17 gives Dap RT (2-a) “p in@m/eni) AT (@-4) P In(p/(P — pawap)) for each finite increment At of time and —Az; of decrease in liquid height. Insertion of the given constants gives (0.712 g/em*) (0.2 2.06 x 295.15 cmPatm/g-mol) (74.12 g/g-mol) (Ats) (747/760 atm) (2 =A, am) tn(747/(747 — 480) __ (24642 em? /g-mol gas) _(0.2 em) (22 — 21, em) ~ (104.1 cm?/g-mol liquid A) (At, s) _In(747/267) (2-21, em) (i, s for 0.2 em level decrease) Dap, em?/s = + = 46.015: and the following calculated values for the six time intervals: (22 — 21), av., em 10,150 2058S At for Az; 0.2 cm 590 895 1185 1480 2055 2655 Dap, cm/s 0.0780 0.0771 0.0779 0.0777 0.0784 0.0780 ‘The average of these values is 0.07785 cm?/s. (b) Conversion of the above result to 760 mm Hg and 0°C, using Eq. 17.2-1 in its nonpolar form, gives Dap = 0.07785 x (747/760) x (273.15/295.15)!*" = 0.0664 cm?/s ‘This result appears preferable to the one reported by Jost. 18-6 18A.7 Mass flux from a circulating bubble. (a) With the data provided, Eq. 18.5-20 gives the surface-average mass flux 4Danve =D (4)(4.46 x 10-* em? /s)(22 cm/s) (Nabave = cao ).041 x 107 g-mol/em*) -17 x 10~8 g-mol/cm?-s (b) Equation 18.1-2, with the surface-averaged ke obtained by Hamerton and Garner, gives (Na)avg = Ke(Ca0 — C400) = (117 em/hr)(1 hr/3600 s)(0.041 x 10~* g-mol/em*) = 1.14 x 107* g-mol/em?-s 18-7 18B.1 Diffusion through a stagnant film--alternate derivation From Equation 18.2-1 we get 1 dx, __ Na Tx, dz Day or Which is in agreement with Eq, 18.2-14 13-8 18B.2 Error in neglecting the convection term in evaporation a. Without the convection term, Eq 18.2-1 and 4 become dx, na OKA Nye G* and A =0 Integration of the latter equation twice then gives x4 =Cz+C, Then application of the boundary conditions gives two equations: X=C\z,+C, and x =Cz,+C, These equation may be solved simultaneously to give XyrX tart C= 722 and C= x4, + 24ers, -% rh Therefore, in the approximation being considered here, the mole- fraction profile in the system is given by XX 27% Xa Xa 22-7 b. To get the result in (a) from Eq. 18.2-14, we can expand the latter in a Taylor series, as was done in getting Eq. 18.2-16. c. To get the solution of Example 18.2-2 by using the result in (a), we make the following calculation p,.-—NakT_NaRT(%-%)_ (7.2610 )(82.06)(273)(17-1) 8° pdx ,/d2) p(t - Xn) (755/760)[ (33/755) — 0] =0.0641 cm?/s Hence the error is 2:0641= 0.0636 , 199 - ,79% 0.0636 18B.3 Effect of mass transfer rate on the concentration profiles a. Rewrite Eq. 18.2-14 as cBag , 1-x, 1-x, Na. (z, Nae fBatintitee op IHD coup Nal B-4 Ix 1-ka CD a When this is substituted into the right side of Eq. 18.2-11, we get b. Starting with Eq. 18.2-1 and integrating directly we get 1 od__ Na Tox, dz Dag a and Nin Toag** = egal When the integrals are evaluated we get dex, _ Na (,_ Teg eB yg 2) Changing signs and taking the antilogarithm of both sides then gives the result in (a). c. Expanding the right side of Eq. 18B.3-1 in a Taylor series in the argument of the exponent, we get 1x4 4, New 1-x4 cDag (ea }r If we retain just two terms in the Taylor series, and bring the "1" on the right side over to the left side, we get Nas(l=3a)(,_ 2) Xa =X qr cD ay which is of the form x, = mz +b, that is, a straight line function. 18-10 18B.4 Absorption with chemical reaction a, Equation 18.4-8 remains valid, but now the boundary conditions are: at (=1, P=1; and at £=0, dl'/d¢=0. Hence the boundary conditions lead to a pair of simultaneous equations: 1=C,cosh+C,sinh@ and 0=04+C,9 from which it follows thatC, =0 and C, = 1/cosh@. Then the analog of Eq, 18.4-9 is T'=cosh ¢¢/cosh The mass flux at the liquid gas interface at z= L is then: deq| _ CyoDyn a =+6,, 464] = S400 aE iat sn Td 0D 45 dsinh 9¢| N, el L cosh¢ |,., which leads directly to the result in Eq. 18.4-12. b. We start with Eq. 18.4-7, the solution of which can be written in the form of a superposition of exponentials T=Cye™ +C,e-% which is to be solved with the boundary conditions given just above Eq. 18.4-8. This leads to the following équations for the constants of integration: 1=C,+C, and 0=C,ge® -C, ge" These two simultaneous equations can be solved to give 1 1+e? ew e GQ and 2 1se eh $e Therefore , the dimensionless concentration profile is \$-il 4[e? +e? ] cosh @ Thus we are led to the same result that we obtained in Eq. 184-9. c. Equation 184-12 can be written thus Therefore, for very large L we get (using an expansion appropriate for large values of the argument) es Nazleeg = ¢20 VD anki (1 2exp(-2k/1?/® 4p)+-~*) > Cao Danke” For very small values of L we get (using an expansion appropriate for very small values of the argument) mofE (4 URE Naam Bake SEE) Similarly, Eq. 18.4-10 can be written as ra aE rd £4. 2 cosh | He tanh AE sinh, [EC C40 ‘AB ‘AB Bay As L becomes infinite, this becomes ¢, & - a £4. cosh | He —sin [Ae ox(- 3] cao Das Be Bae As L becomes zero, the dimensionless concentration becomes unity. (8-12 18B.5 Absorption of chlorine by cyclohexene a. For a second-order reaction, Eq. 18.4-4 has to be replaced by 2 Hee pepe Dan dz with the same boundary conditions as before. Introduce the dimensionless variables P =c4/c4g and £= Kil, / 68 apz- Then the differential equation becomes aT oe ae =9 with boundary conditions [(0)=1 and T'(e)=0. We now let aV/al= p(T), so that d’P/dg? = dp/dg =(dp/dV)(aT/dg) = p(dp/ar). Then we obtain a differential equation that is first order and separable 4p _ 6p? & or Par This may be inegrated, and we use the boundary condition that at ¢=0, 7 =0 and also that dP'/df = p(T) =0: P, =-er Sopdp = f, °ar from which ar dp? =21° or Fra tar? Here we must choose the minus sign, since the slope of the concentration vs. distance curve is negative. Then using, the first boundary condition, we can integrate this equation to get: firPar=-ffag or rt =(1¥ey 18-13 Hence the final expression for the concentration profile is 2 Cao LAI £40 =[ 44. |k2Es0, C4 [ 6D 45 * b. From the result of (a) we get the absorption rate at the liquid gas interface: deg Nazleeg =~ San oT VRB an c. The equation to be solved is dc dy ¢, Bahr fles)=0 or p= Ma) where we have introduced the variable p as before. The resulting equation is integrated, as before, to give pd A second integration yields de, 2 pea ga = Gt ef seal 1 pea pe yn Bhs Ve, or CON _ 8 (2/9 an)fy S(Ea)aea Then we differentiate both sides with respect to z to get 1 de a0 ee =~(2/ Bas )fy""F(ca)dca (2/ Ban)" fEa)acn © This together with N4,|,_9 =-®4»(de,/dz),_, gives Eq. 18B.5-2. de tana 264! nek \$-14 18B.6 Two-bulb experiment for measuring gas diffusivity--quasi- steady-state analysis a. The molar shell balance on Az gives for species A SN gel, —SN seleyge =O lesz Division by Az and taking the limit as Az goes to zero gives dN 4,/dz=0 or N 4,= constant. b, Equation 18.0-1, for this problem, may be simplified thus: N g.= ~C® gp (dx 4/dz) +x 4(N a.+N p.)= ~CB gp (dz 4 /dz) since N ,,=—N ;,; this is true, because in a system at constant c for every molecule of A that moves to the right, a molecule of B must move to the left. c. The equation in (b), with N ,,= constant, then becomes dx ,/dz=-(N 4,/c4g), which when integrated becomes x yee +C, Day d. At z=L, x ,=x%, 80 that Am XA Nu j=-Ne1e¢, “Bas Subtracting the last two equations gives then N a: (L-z) Xa-XA= cDag e. At z=—L, we know that x ,= x, =1- x3, so that + N lz (1-x4)-*4 = (L-(CL) 4B from which Eq. 18B.6-2 follows directly. 18-15 f A mass balance over the right bulb states that the time rate of change of moles within V must exactly equal the rate at which moles enter V by diffusion at the end of the tube. That is (veri) =5N or veMHA = s(4—x5 Sa in which $ is the cross-section of the connecting, tube. g. The equation in (f) may be integrated The integration constant may be obtained by using the fact that at t = 0, we know that the mole fraction of A in the right bulb will be zero, or C, =~In(}-0). Therefore the slope will be the diffusivity, B45. 18-16 18B.7 Diffusion from a suspended droplet a. A mass balance on A over a spherical shell of thickness Ar is (in molar units) Ann? Nay], -4n(r+ Ar) -Nay|,,4, =0 near or, equivalently (427°N,,), ~ (Am? N4,)) near Now divide by 4zAr and take the limit as Ar goes to zero to get d a (P?-N4,)=0 This may be integrated to give r?N,,, = C,. We may use the boundary condition that N4,=N4, at r=, (the gas liquid interface) to evaluate the constant and obtain r?7N 4, = 1?Nan- b.. Equation 18.0-1, written for the radial component in spherical coordinates, is Na =D gy BA + 4(Nay +Ny) If gas B is not moving, then N,, may be set equal to zero and the equation may be solved for the molar flux of A: Dap AX, Na, = Ar“ 1=x, dr Multiplying by 7? and using the result obtained in (a), we get Eq. 19B,7-1: Das 2 ata 1-x,' dr 2N,. = Nan = c. Equation 19B.7-1 can be rearranged to give \8-17 dx, I-x, ay, ar _ Nantz = Bap Integration then gives Neo =-e8,s{-In(1-x,)] or 11 FN 4-2) = 408 lln(1~20)-In(t~Xa) 2 This may be rearranged to give FEN | 2) pn 2 a2 or, when solved for the molar flux of A Nop = Bae (2)n32 TAN) Xe When r, — © (and presumably also x, —> 0), this last result gives 8 Noy = 2248 nL n Xp 18-18 18B.8 Method for separating helium from natural gas Let A be helium and B be pyrex. Then a shell mass balance gives the following equation d apl'Nar)=0 Insertion of Eq. 18.0-1. with Ny, =0 and x4 <<1, gives for constant diffusivity ses) Integrating twice we get c,=C,Inr+C, The boundary conditions are: at r=R,, c,=Cq, and at r=R,, C4 =Cqp. Evaluation of the constants of integration gives for the concentration profile: C4~Cxy _ In(R,/r) Car-Caa In(R,/Ry) Then the molar rate of diffusion through the wall is N, =, Ha = t9a0(Ca1=Ca2) “ AP dr rin(R,/R,) and 2ALD 42(c, ) W, =2arL-N,.= 4B (Car ~ Caz 9B Nae Tal Ra/ Bi) is the molar flow rate of the helium through the pyrex tube. 18-14 18B.9 Rate of leaching a. The molar balance for substance A over a thin slab of thickness Az is Naal,5~ Naz 0 let Az S= Division by SAz and letting the slab thickness Az go to zero yields dN 4, a” Then inserting an approximate version of Fick's law gives de =-8,, 4 as= Dane which is good for a dilute solution of A in B. Thus the diffusion equation becomes ae, =0 dz? b. The above differential equation may be integrated to give C4 =Cyz+C, The constants are determined from the boundary conditions that Cy =C4o atz=0, and Cy =C4s at z= 6. The final expression is then 4-6, z 4-6, Zz ala 2 oF A fas Zz Cases 8 Crocus 8 c. The rate of leaching (per unit area) is then dc 1) _ Daa (Cao -¢, Nano * Banal = yl cao ~€4s)(~Z) = Pare —tas) 18-20 18B.10 Constant-evaporating mixtures. (a) For this one-dimensional, steady-state, nonreactive system, the species con- servation equations take the form dNa;/dz =0, and give Naz = const. for each species. ‘The coefficients cDag depend only on T in low-density systems, according to Eq. 17.3-16 and the corresponding formula of Mason and Monchick for polar gas mi: tures; thus, each of these coefficients is predicted to be constant over this isothermal system. Assuming insolubility of nitrogen (3) in the liquid mixture, we obtain Nz then Eq. 17.9-1 gives dyy Naz, Na dz Diy Das as the Maxwell-Stefan equation for ys. Integration and use of the boundary condi- tion at z = L give Ms Naz Ings = 1 = LCDs teas |e-9=4c-n whence ty = exp[A(z ~ D)] (18B.10~ 1) Equation 17.9-1 and the condition yi + y2 + vs = 1 then give _ WM: — vi Nos + ysNiz “Dn Dis taaws)Naz—uiNos | aNax Dia Dis Miz | Nit Naz =Mi , Ms i 1. at bay ( az, Ms ) Dyn" De Dir * Dis as the differential equation for the toluene mole fraction profile y:(z). Insertion of the notations B, C and D, and the solution for ys, gives \g-2! which has the form of Eq. C.1-2, with f(z) = —B and g(z) = C+ Dexp[A(z-L)}. Using the solution indicated there, we get tn = exp(Bz) [ fexn(-Bey(-c + Dexp[A(z — L]}d2 + K = KPC fePede Dede d-Betbe-tide Bs = KeP — cet 5 plead [ora c, D = KeP= 42g Pac Ke 454+ ae The boundary condition y; = 0 at z = L gives G = KBE ES Oa Ke + E+ 5-5 The resulting integration constant is ‘Thus, the toluene mole-fraction profile is D c 2) =P ey _ [ n@)= aoe [5 in agreement with Eq. 18B.10-1. (b) Numerical results of the suggested calculation procedure are shown in the following graph, prepared by Mike Caracotsios. Cubic spline interpolation was used to get equilibrium vapor compositions over the tabulated range of liquid composi- tions. The interfacial vapor composition yio calculated from the equilibrium data becomes equal to that calculated in step (iv) at a liquid composition x19 = 0.192. [8-22 0.254 0.204 0.15} Yio 0.10+-- 0.05-} 0.00 Interfacial Soterfeced ‘4 Vapor molar toluene fraction + Vapor molar toluene fraction from Diffusion from Equilibrium 0.0 O41 0.2 03 0.4 Liquid molar toluene fraction 18-23, 18B.11 Diffusion with fast second-order reaction a. A shell balance on species A plus Fick's first law of diffusion leads to oxy dz? 0 which has the solution xy=Cz+C, When the constants of integration are determined from the boundary conditions that x,(0)=x,9 and x,(5)=0, the mole-fraction profile becomes Then the rate of dissolution of A at the solid-liquid interface is Dash no Naclao=-eBaeege] =a leat “dz | b. For the system pictured in Fig. 18B.11, we have the following differential equations ax, a ax, dz? =0 (for 0 t)dV = S| -®,,—4| a wi ea ) (- 10a dé says nevs 4dS = ¢455— J a J oe nat The first term on the left can be neglected, because of the quasi- steady-state assumption. When the right side of this expression and the right side of the previous equation are equated (and divided through by S) we getEq. 19B.4-2. c. The unsteady molar balance on the silicon dioxide gives & feptes t)dV = S(kyc45) tie) This states that the mass of silicon oxide increases because of the surface reaction of oxygen with silicon and also because of the change in volume of the silicon oxide region. Once again the Leibniz formula is used to rewrite the left side as Hp = 0,540 = J Biv flo vsudS = e985 ~ dé vn where V, is the molar volume of the silicon dioxide. Equating the right sides of the last two equations gives Eq. 19B.4-3. d. Following the directions given in the text we get _£a0 aB| Cas) _ pm, ~KiCa5 Pid = Voki cas ~ and this may be rearranged to give Eq. 19B.4-4. e. The differential equation for the movement of the boundary is k]—_£a0__|__1 do N14(k/5/Bys)| Vy at or Ky6 ) dd Packt -(1+ $2) 8 AB This is a separable first-order differential equation, which may be solved with the initial condition that 5(0)=0: Vacaokifjdt = G(1+#2 5 and this yields kyo? 2D an Vocaok(t= 6+ which may be rearranged to give Eq. 19B.4-5. 19-1 19B.5 The Maxwell-Stefan equations for multicomponent gas mixtures a. If we start with the first form of the Maxwell-Stefan equations, we have Vx, =-7278(v, vp) AB From this, we get, by rearranging cf. elva —va) =a vx which is just Eq. (F) of Table 17.8-2. b. If we start with the second form of the Maxwell-Stefan equation we have Vig =-——(xpNq—x4Np) or XpNq—X4Ng =-C pV, 1 CB ag If now we add and subtract x,N., on the left side, we get (x4 +p )N4—Xa(Nq +Ng)=-cD spV24 or Na =Xa(Na+Ng)—cO45Vx4 which is just Eq. (D) of Table 17.8-2. 19-8 19B.6 Diffusion and chemical reaction in aliquid a. The differential equation for the steady-state diffusion from a sphere is : {(r a). kre, ea F ag Hl ag According to Eq. C.1-6b, the solution to this equation is or Pp = 4B ar" dr a )er=o qv , C. r=Ge% aoe é é Application of the boundary conditions that P(1)=1 and T(2)=0 determines the constants, and the final result is (cf. Eq. 19B.6-1) The mass flux at the sphere surface is then _ Suntan a le OR del, de, Nala =-Sa0 4 HazCa0 1+b) R (1+b) eR = and the total loss of A from the sphere in moles per unit time is = 4nR? Basten) k’R? W, = 4nr'( pee b. The unsteady-state mass balance on the dissolving sphere of species A is 4 pR3p9.) = 4m? DaBesoMa kyR? alta Pan) = 4R Fi it Ba Since we have used here the steady-state expression for the molar flux at the surface of the sphere, this is a quasi-steady-state treatment. The integration of this equation can be accomplished as follows: First we divide through by 47 p,,, to get Psp = pedR_SantaoMa of 1 4 {MR dt AB Next, put all the factors containing R on the left side R AR _ ByyCaoMy 1+(K/R? /®4n) at Pop Next we introduce a new dimensionless variable Y = Ryk/'7/D4g and write Yd _KibigMy 14Y¥ dt Pap Then we integrate y Y¥ KE soMa ct 7 ay —KitsoMa 1 ap Jato a and finally 14Y¥ __KiesMa 1+Y, Poor (Y-Y,)-In. (tt) or Ki” L+Vk7Ba—R _ Ke soMa TF ter) . Bg RRM EIB eR Pan From this one can get the dependence of the sphere radius on the time. 19-10 19B.7 Various forms of the species continuity equation a. To get Eq. (A) of Table 19.2-1 from Eq. 19.1-7, use Eq. (C) of Table 17.7-1, in the form p, = p@, on the left side of Eq. 19.1-7, and Eq. (S) of Table 178-1 on the right side. To get Eq. (b) of Table 19.2-3 from Eq. 19.1-7, move the term ~(V-p,v) to the left side of the equation, and then use Eq. 3.5-4 (with fidentified as @,,). b. Rearrange Eq. 19.1-11 to get He s(v-eqv*) (V-Ju)+R Then rewrite the divergence term on the left side by differentiating the product Se 4 e4(V-v")+(v*-Veq) (V-Ji,)+R, Then write c,, = cx,, and once again differentiate the products, thus: oe +2, SF 4 xge(V-v*)+o(v* Vg) 4 Q(v*-Ve)=—(V Te) # or, on rearranging Hy a ; { e+(v--78,)}+xa(Z4(v-e0")) =-(V-Ju)+R, Next we may make use of the overall equation of continuity in Eq. 19.1-12 to modify the second term on the left (with the prefactor x,) (V-Ju)+Ra This equation can be put into the form given in Eq. 19.1-15. No assumptions have been made in getting to this result from Eq. 19.1- 11. A {B+ (vevx,)) +80 SR = at fa (9-11 19C.1 Alternate form of the binary diffusion equation Equation 19.1-17, in the absence of chemical reactions, is Ox, A+ EA (v*Vx,)= Buty Equation 19C.1-1 can be written as 1 OM 1 Ht hw-vm)= Bul ¥- am) =84[ 59" M-s1.(vM-vM) | To compare this with Eq. 19.1-17, we multiply through by M and then replace M by its definition: M=x,M, +XsMg=x,M, +(1-x,)Mp =x, (M4 - Ms) + Mg. This leads to Bay (v-Vxq) = 4g V2 +B gy Maa Mo)(Wxa Vea) MeN, Vx) To get this result, we have also divided through by the factor ~ Mg (which is never exactly equal to zero). When this last equation is compared to the first equation above, we see that the two results are the same, if +(1/M)(Mq ~My) ® 4 V24 To show this we introduce the definitions of v and v*, and also Fick's first law in the form of Eq. (F) of Table 17.8-2. This yields 1 M,-™, Bava 1 (Matava + Myra) MA MO srg —¥5) 1 = pl Ma — Mass + Maks) av +(My ~ Mya +Ma%4)82¥5] =(YM)[(Maxa + Mote )xav4 +(Matp + Max, )*p¥0] Since this is an identity, the proof is completed. 19-12 19D.1 Derivation of the equation of continuity a. The equation of continuity for species A states that for an arbitrary fixed volume V the time rate of change of the total mass of species A must equal the net rate of addition of A over the bounding surface S (by convection and diffusion) plus the rate of production of species A by chemical reactions. This statement in integral form is given by Eq. 19D.1-1. The surface integral may be transformed into a volume integral by using the Gauss divergence theorem to give [So,av =-[(V-m,)av + fraav v v v Here we have also moved the time-derivative operator inside the integral, inasmuch as the volume is not changing with time. Since the volume is completely arbitrary, we may remove the integral signs to get the equation of motion in the form of Eq. 19.1-6: a Pa = (Vong) 4 b. For an arbitrarily chosen "blob" of fluid, contained within the volume V(t), whose boundaries are moving with the mass- average velocity, we may write the mass conservation statement as follows: the time rate of change of A within the volume equals the rate of addition of A by diffusion across the surface S(t), plus the rate of production of A by chemical reaction within V(t) : d . = fogdV=- f (n-j,)dS+ fradv at ye sto vo, Next we apply the Leibniz formula to the left side to get J Zo,dv + Jea(n-vsS=- f (n-j,)dS+ frdV vo si) st) vo This may be rearranged to give (9-13 a : f HP ad =~ J(m-(i4 +0av))AS+ fra vit) S(t) vi) where use has been made of the fact that the surface of the blob is everywhere moving with the mass-average velocity, ie., v; =v. Then, we make use of the Gauss divergence theorem for the surface integral, to obtain a . J Sppadv =~ [(V-Ga tPav) HV + [rad vay v(t) vi) Since the volume element V(t) was chosen arbitrarily, we may remove the integrals to get the species equation of continuity for A in the form of Eq. 19.1-7: Ba _ Pa = (Vin +04))+"4 19-14 19D.2 Derivation of the eqution of change for temperature for a multicomponent system a. Applying the chain rule for the functions related in Eq. 19D.2-1, we get (ae), BS), 5), A), a 7 , Ne 6 . an (2) oe __™ | 4H Ag), | Ey, (B,m,) _ st aft mg) 2) (5 ve) (for «#N) Oy (4) -5] (mE) 905), ( alti (=) Oy), bei Ap), Lamy), (Om J, Amn ) =m )(_™) , #5 )(_™). 4 - -n3{ 2) ms) -a~3f 2) ve )ea (for @=N) Subtraction of these two results gives Eq. 19D.2-4. b, Regard H as a function of p, T, and the first (N - 1) mass fractions, so that “ _( aH aH “aH aa -(22) »(F) ars 3 } dig ® Jr Tyo, KPa) 76, When this is applied to a fluid element moving with the fluid, Eq. 19D.2-5 results. Equations 19D.2-6 and 7 are standard thermo- dynamic results. c. Because of the relation m,=n,M,, the differential quotients in the last term in the last equation can be rewritten, with the help of Eq. 19D.2-4, in terms of partial molar quantities: 19-15 of) _( aH) (oH Iq), Ama), Ft Jn, -1(9H) _1/9H) _Fy Hy Ma \ arte Jy, ~My Oy) 5, Mz, My In Eq. 19D.2-5, we use Eq. (E) of Table 19.2-4 for the substantial derivative of the enthalpy, and we use Eq. 19.1-14 for the substantial derivative of the mole fractions. We also use Eqs. 19D.2-6 and 7 for the other differential quotients. Then Eq. 19D.2-5 becomes -g)—(t:vv) +22 =| 4-( ain ~(¥-q)-(e:Vv) +B =| (ser), d, Rearranging we get a DT alnV\) Dp = =-+(V-q)-(1:¥' =f Por Dp (Va) (Sarl. Dt a SFB \-i.)-re) The last term in this equation can be simplified as follows: “Ene £4(V-in)=Fa)-[ B((V-he)~re)] p= Fl (¥-Ia)~Re)+ Fu (9 In)-Ry) Hi, ((V-Ja)-Ra) 2 1 8 i] = This, then, leads to Eq. (F) of Table 19.2-4. 19-16 19D.3 Gas separation by atmolysis or "sweep diffusion" a. The Maxwell-Stefan equation for a three-component mixture are dx, 1 1 Tet saggy aN HON ae) ty —(BaNes He) dx 1 1 = app (aNae 8a) tg (taNex “4Ne) Bac We now make the substitution x- =1-x,~ xp in order to have just two independent variables. Then by introducing the indicated dimensionless variables, we get Eqs. 19D.3-1 and 2. b. Next we take the Laplace transform of Eqs. 19D.3-1 and 2: PR, ~Xa1 = Yaa¥a +Yan%n + PY 4 PRp~ Xp = Yaka + Yonts +P "Ys This set of equations can be solved simultaneously to give for ¥, g, - Xaltar-toiP) *” pp-P.)(p-P-) in which p, and p_ are given by Eq. 19D.3-4, and X4(X41/%m:P) by Eq. 19D.3-5. Similar equations can be given for X, by interchanging the subscripts A and B. The transformed expression X, can be inverted by using the Heaviside partial fractions expansion theorem if we exclude the relatively unimportant cases p, = p_ and p, (or p_)=0. Thus we get _Xeaetn 0) , Xen otnsrs)e | Xea kare. ers 3) p.(P.-P-) p-(P_~Ps) The expression for x, is obtained by interchanging all indices A and B. \9-17 c. Finally, at (= 1, x4 = x4, So that gg = UaueoriO) , XCar tersPs JOE, XCar tnrsP)eP PsP- P.(P.-p-) p-(p--P.) and a similar expression for x. We hence have a pair of equations giving the relations among X41, X42, Xp), and Xp, the dimensionless fluxes v,, and the dimensionless diffusivity ratios. Keys and Pigford go further and give a plot of the separation factor op = talon Xa/Xar for the special case of very small mole fractions. Note that the choice of notation in this problem has the advantage that there is symmetry between A and B, which makes it easy to give the results for both species after the result for one species has been worked out. 19-18 19D.4 Steady-state diffusion from a rotating disk a. Equation 19.1-16 for steady-state diffusion in the absence of chemical reactions is (v-Vp4)=D4sV70,(z) For the special case that p, depends on z alone, the diffusion equation simplifies further to ; nga = £0 v, tba = Suk a & att i in which the dimensionless coordinate ¢=z/Q/v has been introduced. b. The differential equation is solved by setting dp, /d¢ =p, so that we get the first-order separable equation dp/df =ScH(¢)p, which has the solution Inp=SefSH(So+InC, or Oa =p=C, exp(ScffH1(2)t2) A further integration gives Pa =Cafpex( Sef; HEHE JIE +c, The boundary conditions that p,(0)= Pay and p4(c2)=0 then give __fen(SeMEVE HE evp(asced*}C Pao *” Gooo( Sele H(e}E}E SRexp(Seat? ae the second expression being the high Schmidt number limit, ie., taking only the first term in the expansion for H. The integral in the denominator can be evaluated analytically 19-19 c. The mass flux in the z direction is then dq, R4Sca di jac =~ Ban BA = +0080 We ool sone) AE (8) in which d¢/dz = JQ/v. At the surface of the disk the mass flux is . ¥Sca /x(0.510) - ine? 2 BETO ns 3 rt = 0.620 4Dap(v/ Bay) OP v? which is the result in Eq. 19D.4-7. 19-20 20A.1 Measurement of diffusivity by unsteady-state evaporation. Assuming ideal gas behavior, and insolubility of species “B” in the liquid, we esti- mate the mole fraction of ethyl propionate in the interfacial vapor as PAyap 240 = Eq. 20.1-1-22 gives AV(t) = Szaov vt — ¥240] Linear interpolation to 249 = 0.0545 in Table 20.1-1 gives = 1.0235, whence Dap em?/s = 7 [3 Av(t) ]* ASraov) | Ve— 240, Ave) Vi = V240. with AV in cm® and ¢ in s. Application of this formula to the tabulated data gives the following results: vi 15.5 19.4 23.4 26.9 30.5 34.0 37.5 415 Dap 1.2 0.0281 0.0278 0.0272 0.0273 0.0270 0.0273 0.0269 = was [AO ‘The average of the last seven determinations of Dap is 0.0274 cm?/s. 20-\ 20A.2 Absorption of oxygen from a growing bubble. ‘The interfacial molar flux of oxygen into the liquid is given by Eq. 20.1-75 as 2n + 1)D, Naot) = cans) 22 * DPaw for a bubble with interfacial area S(t) = at" and interfacial liquid concentration cao, When a, n and cap are constants. The solubility w4o corresponds to a molar concentration cao = pwao/Ma = (1.0 g soln/em)(7.78 x 10~* g O2/g soln)/(32 g O2/g-mol O2) = 2.43 x 107° g-mol O2/cm* which is used here as the interfacial concentration of dissolved O2. (a) For constant growth rate of the bubble volume, r3 oc t, so that ry ox #/* and S(t) o r3 « #7/5, giving n = 2/3 in Eq. 20.1-75. Then at t = 2s, the interfacial molar flux of O2 into the liquid is Nao(t) = e404 ane) = (2.43 x 10° g-mol/em?) [Sioa gents =7.55 x 1078 g-mol/em?-s The total absorption rate in g/s is then walt) = 40r2(ONao(t)Ma = 4n(0.1/2 cm)*)(7.55 x 10~® g-mol/cm?-s)(32 g/g-mol) = 7.6 x 10-* g/s (b) For constant radial growth rate, ry ot and S(t) « #, giving n = 2 in Eq. 20.1-75. Then the interfacial flux at time t = 2 s is (4 + 1)D, Nao = cao) & a AB = (2.43 x 107% g-mol/em* = 1,11 x 1077 g-mol/em?-s and the total absorption rate in g/s is wa = 4ar2(t)\Nao(t)Ma = 4n(0.1/2 em)*(1.11 x 10-7 g-mol/em?-s)(32 g/g-mol) = 1.11 x 1077 g/s 20-2 20A.3 Rate of evaporation of n-octane. Table E-1 gives the following Lennard-Jones parameters: Species M oA eK A: nCsHis 114.23 7.035 361. B: No 28.013 3.667 99.8 Eqs. 17.3-14,15 then give the interaction parameters 74g = 5.351A and cap = 189.8 K, and Eq. 17.3-10 gives Dap, em?/s = 0.018583) | (293.15) ( 1 1 a) (, atm)(5.351)*(1-185) = 0.0580/(p, atm) (a) If p = 1 atm, then Dap = 0.0580 em?/s and x40 = 10.45/760 = 0.01375. Interpolation in Table 20.1-1 gives y = 0.00859, and Eq. 20.1-20 gives the volume of vapor produced in 24.5 hr as Va = SpV4Dant = (1.29 cm?)(0.00859) \/4(0.0580 em? /s)(24.5 x 3600 s) = 1.585 cm* ‘The mass of vapor produced in 94.5 hr is pVaMa RT _ (1 atm)(1.585 cm®)(114.23 g/g-mol) ~~ 82.0578 x 293.15 em® atm/g-mol = 0.0075 g ma (b) If p = 2 atm, then Day = 0.0200 cm?/s and xo = 0.01375/2 = 0.0688. Table 20.1-1 then gives y = 0.00430, whence Va = SpV4Dapt = (1.29 cm?)(0.00438) y/4(0.00290 em?/s)(24.5 x 3600 s) .561 em? and the mass of vapor produced in 24.5 hr is 4 = ¥aMa RT __ (2 atm)(0.561 em*)(114.23 g/g-mol) ~~ 82.0578 x 293.15 cm® atm/g-mol 0.0053 g 203 20A.4 Effect of bubble size on interfacial composition. Equations 20A.4-1 and 2 give 261 wa=H Po + | Assuming H to be independent of r,, the ratio of w4 to its value for a very large bubble is want) _ Poot 2a/rs _y , 20 wao(oo) Poo Pool s ‘Thus, the bubble radius corresponding to a 10% increase of wo over its value for a very large bubble is given by ez) moe =01 or ots Pols Poo For a gas bubble in water at 25°C, with pop = 1 atm, this gives the required bubble radius as (2)(72 dynes/em) ~ (0.1)(1.0133 x 10° dyne/em?) 0.00142 cm = 14 microns The (normally minor) dependence of liquid-phase free energy on total pressure has been neglected here. To include this effect, one would need partial molar volume data for the particular system. 20-4 20A.5 Absorption with rapid second-order reaction a. The first thing one has to do is to determine the parameter y from Eq. 20.1-37, using the concentrations and diffusivities given in Fig. 20.1-2. By a trial-and-error procedure we have found that y=4.9x10° mm?/s. We now verify this by substituting into Eq. 20.1-37. We first have to convert the diffusivities in ft?/hr into units of mm?/s: (12-2.54-10 mm/ft)? 3600 s/hr Bgs = 1.95 10" ft2/hr = 0.503x 10° mm?/s Djs = (3.9 10" ft? /hr)) = 1.00610 mm?/s Substituting into Eq. 20.1-37 now gives: = 1-ext,| 49*10° 0.503 x10 ) OS xt £9107. "006% 10°" Vi.006x 10> 1-erf 0.3121 = (2.828)(erf 0.2207)(0.9525) 4.9x10% __4.9x10% 1.006x10 0.50310 or When the error functions are evaluated, using a table, we get 1-0.341 = (2.828)(0.245)(0.9525) This gives 0.66 = 0.66. Therefore, zp (t)=/47# gives the location of the reaction zone as a function of t. Thus we get the following table of results that can be compared with Fig. 19.1-2: t Za (t)= y4yt 0.625 s 0.011 mm 25 0.022 10.0 0.044 Qo-5 This is not in very good agreement with the graph in Fig. 19.1-2. The reason for this may be that Eq. 415 on p. 336 of Absorption and Extraction by Sherwood and Pigford (which corresponds to our Eq. 20.1-37), contains two errors: the r on the left side should be Vr and the D,, in the argument of the second error function should be Ds. b. To get Nyo at 2.5 seconds, we use Eq. 20.1-38, as follows: _¢ Nao" oF 1/Das _ (1. gmol/dm?)(10 cm/dm © erf,|(4.9%10°)/(1.006 x10" 1000 3 2 =——_(1- 1 =4. O25" 13x10) = 4.61 gmol/cm?s 1.006 x 10° mm’ 159)(2.5 s)(100 mm?/cm?) 20A.6 Rapid forced-convection mass transfer into a laminar boundary layer. Equation (20.2-51) gives, for the conditions of this problem, (wo = Woo) nao/ (nao + RB0) — Wao ew Ry= x) Fig. 22.8-5 gives a dimensionless mass flux ¢ of 1.55 for mass transfer at Ry = 8.0 and Se = 2.0. With this result, Eq. 22.8-21 and Table 20.2-1 give KA Kx20 155= O,A,0) > 0.5072 so that K = 1.55 x 0.5972/2 = 0.463 ‘The definition of K in Eq. 20.2-48 then gives the total interfacial mass flux nao() + npo(t) = povo(z) = Kpvoo Woot = 0.33 / pvp] In this problem ngo(z) is stated to be zero, so the previous result reduces to nao(z) = 0.33y/pvecu/x 20-7 20A.7 Slow forced-convection mass transfer into a laminar boundary layer. (a) Equation (20.2-51) gives, for the conditions of this problem, the binary mass flux ratio _ (wo - we0) nao/(n40 + 2B0) — Yao = (005 50H) = 0.0421 Eq, (20.2-55) and Table 20.2-2, with Se= 0.6, then give R = aA72/s_ Be Ke aN TE ORS 0.0421 = 0.4642 x (0.6-7/). \—___—___. = 0.02¢ 1+ 0.768 x 0.0421 teed Equation 20.2-48, with the specification npo(r) = 0, then gives the evaporative mass flux v ngo(2) = povo(2) = Kprsey/ 5 = 0.0266 /pvaon/2e = 0.0188 /pv..n/e (b) Eq. 20.2-57 gives Bao = wa0(MAo+ BO), yy pop)Se2/? = 0.832, Poo ° ee Voot which gives, since no = 0, nigg = 0.332S0~2/9 240 — PAD” py. |_Y 1—wao Vet = 0.992(0.6)- 79 oO pvscnle = 0.0196/pocon/= (c) At the value K = 0.0266 found in (a), Table 20.2-1 gives the interpolated interfacial gradient I'(0,Sc, K) = 0.3797. Then Eq. 20.2-47, with nao = 0, gives the reference solution 9 ~ Wee II"(0,Se, K) 7 nigg = Tee SO pttoo l-wo Woot 0.05 — 0.01 0.3797 => Vpveot/22 1-005 06 = 0.0188, /pvan/r in excellent agreement with the truncated expansion used in part (a). 20-3 20B.1 Extension of the Arnold problem to account for interphase transfer of both species Equations 20.1-1, 2, and 3 are still valid if both species are crossing the interface. In Eq.20.1-3 we now eliminate c by using Eq. (D) of Table 17.8-2 (evaluated at the interface) to get . (Nazo+Neo) —_ ax4| — __ (ltr) ax,| * Naso = *a0(Nazo + Nexo) 9 eo 1X a0 F) & leo When Noio = 0 (or the ratio r= Np.o/N az 0S to zero), this equation simplifies to Eq. 20.1-4. Equation 20.1-5 is then replaced by OX, (l+r) dx, Ot \1=xg(1+r) az AB Oz \Ss- » Px, e-0, We now introduce the dimensionless variables II (defined in Eq. 20.1-23) and Z (defined just below Eq. 20.1-8); note that when X4. =0, IT is the same as -(X ~1). Then the combinarion of variables method gives (cf. 20.1-9) the ordinary differential equation oH og along with Pal X40 7X40 )(1+ 1) aT] 9(Xa0-7)=+ A (tan #e)(L47) al 2 1-xao(lt+r) aZleao This is Eq. 20.1-24. If r=0 and x,,, =0, it reduces to Eq. 20.1-10. The sign discrepancy comes about because (in the limit that x,.. is zero, IT is the same as -(X-1). The solution of the ordinary differential equation for II proceeds exactly as that for X in the text, and the final result is erf(Z-9) + erfg l= lierfp 20-9 for the concentration profile. If r=0 and x,.=0, this reduces exactly to Eq. 20.1-16. When this concentration profile is used to evaluate dII/dZ at Z=0, then we get an expression for 9(x49,1): 1 (X40 vet A eh XaoT)=+> (40/7) 2 ‘ao(itr) | Va 1+erfo This may be rearranged to give (X40 —X a0 (1 +7) 1—Fag(1 +r) Vi(1+erfg)pexp(+9") in agreement with Eq. 20.1-25. 20-10 20B.2 Extension of the Arnold problem to nonisothermal diffusion a, Equation (M) of Table 19.2-4 without the last two terms is: 3c, H, (vd ENT, 2) (v-KVT) Replacing the partial molar quantities by quantities per mole (see comment after Eq. 19.3-6), and assuming that k is constant then gives Br — 3c fig +(V- SNH, |=Kv°T am a Differentiating the products in the first and second terms allows us to rewrite this as Nc N _ XN - Ca Oe, $ Seite +D(V-Ng)Aa + DO(Na Vlg) = kT % at aa Ot at at In the second term of this equation, we replace the derivative of the concentration by using Eq. 19.1-10 (omitting the reaction-rate term), and then we see that the second and third terms just exactly cancel. Next, we replace the enthalpy by the heat capacity multiplied by a temperature difference in accordance with Eq. 9.8-8, to get ae °)4 SIN, ve 7 6a 5Cpa(T-T )+d(Ne “VEjq(T-T?)) = kV?T Me 8 For constant heat capacities this then becomes: N . ar Ne DeaCya Get DCpa(Na“VT)=kVT F=) a If all the heat capacities are alike, then they can be taken outside the summation to get ~ a =(% €, 246 EN. -vr) =kVT 20-11 Next we make use of cC, = pC,, and then divide the entire equation by pC,; this gives, with the help of Eq. (M) of Table 17.8-1 aT ((1s 2 ar 2 < 4|(-3N, |-VT|= av’ £+(v*.VT)=aV?T o((23: 3) } av’T or me y= a which is just the 3-dimensional version of Eq. 20B.2-1. b. When Eq. 20B.2-1 is rewritten in terms of the dimensionless temperature, we get ly ys Me ge a oe ae We now postulate that the dimensionless temperature is a function only of the variable Z; =z/V4at. This leads us to the following ordinary differential equation: @Uy _ r{E- AB “a +2(Z; - 07) mz =0 where 9r=0:)= 9-4 It was shown in Eq. 21.1-24 that @ is a function of the terminal mole fractions and the interfacial molar-fluxes, but not of time. Therefore, since Qr is just g multiplied by the square root of the Lewis number, it may be treated as a constant in the above ordinary differential equation. This equation may be solved by the same technique used in §20.1 to give ny a1-tretf(Zr = er) _ exf(Zr - or) + erfor TT lterfo, lterfo, c. To get the interfacial heat flux, we use Fourier's law: a} __K(T.-T) d She” ‘ Teron det (21 Pro 20-12. =- ~MEa=To) 2. exp[-(Z;-91)'] 1-92) l+erfo, Va _ KT. tet) 2. 2 “Trerfo, Va! P(-9 * a =~ __H(T=T.) Va(1+ erfg; Jexp(+9? Vat lz=0 ae Next we calculate the ratio in Eq. 20B.2-4: Nao + Noo cu; ___&rya/t Gep(To-T=) Ao) y(To-T=) 4o/Cp(To-F-) C, = CoV ie(1+ ext, exp( +03 Val] = ioe cabal 1+ exfg,)exp(+9?)] - at ateyes exp(+97) In the first step, we used Eq, (M) of Table 17.8-1, and later we used the relation eC, = p¢,. 20-13 20B.3 Stoichiometric boundary condition for rapid irreversible reaction We begin by rewriting Eq. 20B.3-1 as Mepl (emt) +(0t—0n)]=—feaom 02) +(0!-o4)] Then, using Fick's first law in the form of Eq. (B) of Table 17.8-2 (along with Eq. (1) of Table 17.8-1, we get 1 Ox, 1 . 7 Bas G+ ea(et —0R)= We now make use of the fact that the system has constant c so that oc, 1 a, . 1 1 58a Ge tp 8 Ge (0e Ma) ge 45) At the plane z= zg, there is no A or B present, so that the last term on the right side is zero. Therefore, we get co a dy -1» as Pe Str 1 ays Se for one of the boundary conditions at z = zp (cf. 20.1-31). 20-14 20B.4 Taylor dispersion in slit flow This problem can be solved by paralleling the treatment in §20.5 for circular tubes. We give here the intermediate results along with the corresponding equation numbers in the text. Some results from Problem 2B.3 are needed here. (@,4)= aN ogdx (20.5-3) a ae (4- 2) 2a where g=3 (20.5-7) O,= Pema (g 4g) ea), ,(0,2) (20.5-10) (on aa (Gg tonton eos @4-(@,4)= gees Hor) Hea) Bte?-484) (20.5-12) = -2pWp A@ad] 287(0.) : = 28D SM OSB, (20.5-13) The quantity in brackets is the Taylor dispersion coefficient. 20-\5 20B.5 Diffusion from an instantaneous point source a. The relevant diffusion equation is Dy G 2a) a “#e a or The time derivative of Eq. 20B.5-1 is a a 4 at * 7D at Next we build up the quantity on the right side of the diffusion equation by differentiating Eq. 20B.5-1, thus: Ps (__t 24s 4 (__? a PA 3m,8) 4d a = Pal Te 2(r Be) ee, we find that p, — 0, as it should. c. Integrating over all space in spherical coordinates, we get Ohh ae, Ga por 2/489 apt) rdrsin adOdg =4n “eae exp(-u? )wdu- (4.8 apt)” m, 9p =4, my NT 4D nt)? = Gap ae OS!) In going from the first to the second line, we have made a change of variable: 1/,/4D ,5f =u 20-16 d. Let n=(1/4® jst). Then Eq. 20B.5-1 becomes Pa =m, (n/2)"” exp(-nr?) The limit of this function as 1 © is py = ,6(x)5(y)5(z). That is we get a delta function in the three spatial variables. In other words, all the material is "piled up" at the origin. [For more on this, see R. B. Bird, C. F. Curtiss, R. C. Armstrong, and O. Hassager, Dynamics of Polymeric Liquids, Vol. 2, Kinetic Theory, Wiley-Interscience, New York (1987), SE.4, p. 405.] 20-17 20B.6 Unsteady diffusion with first-order chemical reaction (a) the thin disk catalyst particle ‘The reference problem is 20 a°o =< :0(0,n) = 1;0(£1) =0 or apo) (+1) ‘The problem of interest here is 30 _ Ho Oc On? Attempt a solution of the form Ko; o(0,n) = 1;@(+1) =0 @ = Ocxp(-Kt) Putting this trial solution into the differential equation for @ gives the original equation for ©, and the new equation also satisfies the initial an boundary conditions. It is our desired solution. (b) diffusion and reaction from a point source Again we may write the solution by inspection as st p= ge Here g is the normalized probability of finding a diffusing particle, originally at the coordinate origin, at any point and time, 20-13 20B.7 Simultaneous momentum, heat, and mass transfer: alternate boundary conditions. (a) With nao = 0, Eq. 20.2-51 gives the mass flux ratio 40 ~ WAco 1—wao Re Then with A = Se for binary diffusion, Eq. 20.2-52 gives the following implicit equation for the dimensionless mass flux, _ Ace 1 = BeOS.) which can be solved conveniently with Fig. 22.8-5, Eq. 22.8-21, and Table 20.2-1, in the manner of the solution given for Problem 20A.6. (b) With w4 =0 and ngo = —2n,49, Eq. 20.2-51 gives the mass flux ratio Then with A = Sc for diffusion in the binary gas phase, Eq. 20.2-52 gives the following implicit equation for the dimensionless mass flux, 1 K = Lepcll(0,8e,K) which ean be solved directly with the aid of Fig, 22.8-5, as noted in (a). (c) A steady-state energy balance from the inner to the outer boundary of the wall gives, for the region of laminar boundary layer, PovoGpTa = proCpTo + 40 whence - - G0 = poCp(Te — To) = na0Cp(Ta — To) for this one-component system. With this substitution, Eq. 20.2-50 gives the energy flux ratio Rp = Maelo = Tes) Then with A = Pr, Eq. 20.2-52 gives the following implicit equation for the dimen- sionless mass flux, 20-19 20B.8 Absorption from a pulsating bubble a. The essential point here is the calculation of the-integral in Eq. 20.1-72. Let the surface of the bubble oscillate between S, = 47R? and S, = 472R}, with S,/S, 21, so that s(t)=5, 2ns@tS2n+1 for n=0,1,2,- S(t)=S, 2n+1<@t<2n+2 for n=0,1,2,- We further let (S,/S,)?=r21. Then in each time region, we can calculate the value of 1/t times the integral in Eq. 20.1-72 as follows: 2 1p S O< ots: +2) df=1 1pve( $,) 1p (S 1(1 t dq= (2 St) at +(22) ae at), Oz (32), (3), 2.) ath, at), az), at 21), (32) =|—)] +} (F az é ) (3) -(2) { #4) _( aa du), \dz),’\ du? a2" where q is any scalar, here < >. Putting these results into Eq. 20.5-17 gives (52) -«(@58 at), au’), which is the desired result. aq _ ,8°q (b) Our differential equation is of the form <2 = KK at 62? where q is

and the proposed solution can be written as =A emp ( Eat ‘ q u Ne 0q/ dt =-—+ ~ a/ 2 gKE a u a __ 4, du 2Kt Sa_ 4, va du’ Kt 4 K*t? i du? Bt 4K The differential equation is satisfied. (©) Simple inspection shows that these conditions are met. 20-24 20C.1 Order-of-magnitude analysis of gas absorption from a growing bubble (a) The volumetric flow rate for this system is constant at a value The same volumetric flow must occur at any distance, and this constraint requires the suggested result. (b) This is obtained simply by putting the above result into the spherically symmetric continuity equation. (©) The expression (y-1) (A-¥/4,J° ‘The desired expression can be obtained simply by division, and it is also available in many standard tables, for example, H. B. Dwight, Tables of Integrals and Other Mathematical Data, Macmillan, #9.06. (@) Terms (5), (6) and (7) are second, third and forth order respectively. 20-25 20C.3 Absorption with chemical reaction in a semi-infinite medium a. First, introduce the following dimensionless variables: T=cy/C4o, € = (KD qpx, and t= k/t; also, define the combinations m=|(&/2Vz)-e] and p=[(é/2Vt) +7]. Then Eqs. 20C.3-1 and 2 are a _#r aa and 2P=e“erfom+eterfcp Then we calculate the derivatives as follows: 2 (et) (se) de” Ve (ae * Te) * le ae — an. eg tetertep = eer ar poten pet é 29D _ tent a Be teres SE gE (Sv er” eke? ober E tee ee een) When these contributions are substituted into the diffusion equation, it may be seen that the equation is satisfied. At f=0, we find that m=eo and p=, so that erfc m and erfc p are both equal to zero, and hence the initial condition is satsified. At x=0, P=3ferfc(-k/t)+erfe(+k/t)] which may also be written as 3[1—erf (kt) +1-erf(+k/t)]; however erf(—u) = -erf(u), so that we are left with }-2=1, which indicates that the boundary condition at x =0 is satisfied. At x=, erfcm and erfcp both go to zero, and e also. +eferfep— Hence we have to prove thate‘erfcp goes to zero: 20-26 lim ne{e/avi) _ jin Cove S/sn)(uevt) foe oo -e which shows that the boundary condition at x = is satisfied. b. To get the molar flux at the wall, we once again make use of dimensionless variables: Ky’ or Al Naglyep =-®anCao | =—Base ale ean |= Paseo ae g -é, fer 404 Bak -eteion-* Tz + eFerfep- 1 ma a = jean Bak -etel Fe) vent) -55] which is equivalent to Eq. 20C.3-3. Actually a neater form of the result is: Naxleeo = 40 Rae at ent it tet] In this form, the quantity in the brackets is the correction factor for the absorption rate because of the chemical reaction that is occurring. c. The total moles absorbed through an area A is <0 (B aah det A kit |aott = Ke lo = Boa VBankif" [eve + i Ma = Af" Natl Narlyeod™ We now perform the two integrations; in both of them we make the change of variable V7 =y: 20-2] ate’) 2 plat yz 7 fh Tat gh Me" dy =erf /kyt ait iat it 2 py st [ert Vrate =f (ertyaydy = "(Fle as) ayy = ELT (LE wae as = 2 are as in 2 (AR oat ge 2 oat Kite hy e as - eh se ds mm me 2 fit eR -s? =kitert ki -Z[-i« i +e «| P a J pa e = Kitert (RE + = ites — dent Jkt When these integrals are substituted into the expression forM,, we end up with Eq. 20C.3-4. d. For large values of k/t, the exponential in the last term goes to zero, the error function goes to unity, and the only remaining terms are those in the large parentheses in Eq. 20C.3-4. In this way, Eq. 20C.3-5 is obtained. 20-28 20C.4 Design of fluid control circuits ‘The response to a pulse input to a “long” tube can be expressed as (Est) = LA ep) C Ler Vere | GB) Here m; is the mass of solute “i” fed to a tube of length L and cross-sectional area A. This is just 20.5-18, modified to be explicit in time, and using a generalized dispersion coefficient E. For a short pulse most of the solute leaves near the mean residence time t=L/v where Vis the flow average velocity, . The term “E” is the sum of Taylor dispersion and molecular diffusivity: B=K+D, Ifwe then approximate ¢ as E the exit composition has the form of a normal probability distribution with variance * = 2Et /v® and O*is the standard deviation of the distribution. A reasonable set of design criteria is then that V2EF /v < 0.05t, L >> 2uR° /(3.8)°D,, (Eq. 20.5-4) In practice the “>>” requirement need not be very large. It is really only necessary that there not be much tailing. A discussion of this point is beyond the scope of the present discussion. In any event for most “long” ducts we may approximate “E” as the “K” of Eq. 20.5-15 K = R'v® / 48D,,; 2Rv/v < 2,100 For higher Re turbulence results in much smaller dispersion coefficients K ~ 10Rv* 20-24 We will not explore this turbulent region here. We thus find oe 182, A LR’ LR’ t > 8.338 ——;t, > 2.89, ° 0D," 1B, with the constraint on length introduced above. This in tum means that 2 < 0.05742 L 2 = 1.07R’ / 2D, 0 58 2, =2.7-10%em® /D,, Then for typical liquids, with diffusivities on the order of 10° cm’/s, 1, >> 2.7-10°s = 4.5min whereas for gases with diffusivities of, let us say 0.27, t, >> 0.01s 20-30 20C.5 Dissociation of a gas caused by a temperature gradient [The small Roman numerals after the equations correspond to the equations given by Dirac in his paper.] a. The equation of state It if assumed that the diffusing mixture is at constant temperature and that the ideal gas law is applicable, then we have p=(n,+1,)kT or (n, +1 )T = p/k = constant Here m, and 7; are the number densities of the single and double species at any point in the tube. Then if n? and n9 are the values of 1, and n, at the same pressure and temperature in a system with no temperature gradient, then we have also (n? +n) = p/K = constant Then Dirac defines a quantity o by the following: n-n?=o and 1, -n) =-0 @ Thus o gives the deviation from the equilibrium state, resulting from the temperature gradient. b. The equation of continuity for the mixture Equation 3.1-4 (or Eq. (D) of Table 19.2-1) gives, for steady state, d =P =0 or = constant dz Ps = 01 where v, is the mass-average velocity. Therefore, if we make use of Eq. (B) of Table 17.7-2, and if we make use of the fact that there is no net movement of mass across any plane z = constant, then we have P11 +20, =0 or NyM,V, + NyMV, = 0 where m, and m, are the masses of single and double molecules, respectively. Then nym, + My (2my Joy or mv, +2n,0, =0 (i) 20-31 c. The energy equation for the mixture For a steady-state system with all species subject to the same external force, the one-dimensional energy equation of Eq. (C) gives e, = E, a constant. Then we use the approximate expression for the energy flux (also used by Dirac) given in Eq. 19.3-6 KZ +(N,H,+N,H,)=E or 1D coh, +C,0,H, =E where the partial molar enthalpies have been replaced by the enthalpies per mole, as is normally done for ideal-gas systems. Then switching to quantities per molecule, we may write Zs moh +m, =E or 2 4n,04(2h =Iy)=E Then if we define the heat of dissociation by Mgisoc = 2H, — hy, the energy equation becomes 0 + m0, Aras =E (itl) d. The expression for the molar flux If we write Eq. 24.2-8 for species "1" in a 2-component mixture, it becomes: Si ( DL _DE)der dz Dy\ p,m) a& This equation is a generalization of Eq. (F) of Table 17.8-2, which takes into account the extra term needed to describe thermal diffusion. Since D? +Df =0 (see paragraph just above Eq. 24.2-3), the above equation may be written as 0-0, =-—4 1 - (iv) 20-32 This is exactly Dirac’s equation (iv) if - DJ /p is relabeled as D’. e. The equation of continuity for species "1" Equation 19.1-10 at steady state is d d - 0=-F-ei +R, or O=—-m0, + RN the second form being in molecular, rather than molar, quantities. Dirac then postulates that, for systems not too far removed from equilibrium, first order kinetics can be assumed so that d m smi = ko (v) The above set of equations (i to v) are the equations that Dirac derived, and these have to be solved simultaneously for 1, My, v1, 0, and T. for temperature dependent thermal conductivity, diffusivity, and thermal diffusion coefficient. First he shows how to reduce the set of five equations to a set of two equations. Then he resorts to numerical methods to complete the calculation for some sample values of the constants in the system. 20-33 20D.1 Two-bulb experiment for measuring gas diffusivities-- analytical solution The diffusion equation to be solved is a, ae “yD a0 at with the initial conditions that c,(z,0)=c} for 0ty h(Lyt) = = eap| Eas 9 > V4rkt 4Kt Itonly remains now to weight individual differential pulses, by fit’), each introduced at atime t’, and add them up by integration from time zero to observer time t. The response to an input of mass m; added over a cross-sectional area of column A, in a short time period is just (m, / A)h(L,t) ‘The mass of solute per unit area can in turn be written as dm, / A= p,|,, dt! = fat! for uniform concentration over the inlet cross-section. Summing, or more specifically integrating over the time interval of interest then leads directly to the solution suggested Leal lus “IEF fro FY (b) For the square pulse one need only replace f by f, = P. yg and change -ese2t!) dt! the lower limit of integration to “0 20-41 20D.5 Velocity divergence in interfacially embedded coordinates. (a) Integration of Eq. 204-7 over the boundary Sp of any closed domain D(u,w,y) in the boundary layer gives [fv ea80)= [, (woaso)+ f (Zee usp) (20D.5 — 1) (b) Since the third dot product is the outward normal component of the velocity of the boundary element dSp relative to stationary coordinates, its integral is the rate of change of the volume Vp of the domain D(u,w,y). With the aid of Eq. 20.4-3, we can rewrite this term as follows, (wnt!) as) = 4 | Jala,wuFidudwdy so at ad Jp a ALONE) 4) qudwdy (20.52) Ib using Leibniz’ rule to differentiate the second integral. (c) Application of Eq. A.5-1 (in which v is any differentiable vector function) to Eq. 20D.5-1, after use of Eq. 20D.5-2, gives the following equation in terms of volume integrals: (VeV)d¥p= | (Vev)d¥p +f Vi (us01¥8) audrwdy Vp Vo D ot Expressing each integral in the coordinates of Eq. 20.4-3, we then obtain f [ oV)-Vev)- i | Vali, wy, 8)dudwdy =0 I ‘o(u,w, ust) In order that this equation hold for every domain D of nonzero g(u,w,y,t), the quantity in square brackets must vanish identically in such regions, giving (VeV) = (Von) + vee w!) there in accordance with Eq. 20.4-8. 20-42 21A.1 Determination of eddy diffusivity In(sc4) = In, /42D4y)—(v9/2D4y (5-2) Assume that Weo, =0.001-W 4, (a) Begin by examining the behavior at r= 0, and note that xD* mms so that the volume fraction (equal to the mole fraction) of carbon dioxide L yD? —“o™ 20.001 Fong" 2K 40 where K sco, is the effective diffusivity of carbon dioxide in air. Here we have assumed the ideal gas law to hold. We must now estimate the volume fractions, and here they will be taken as 0.0105 and 0.007 for distances of 112.5 and 152.7 cm respectively. Solving ‘the above equation for the effective diffusivity then gives: K4-co, =18.6, 21.9 cm? /s respectively. abl 21A.2 Heat and mass transfer analogy Begin by defining the parameters =A Cay = Ed JsoR! Daz Yack?! Day The desired solution then follows directly 2 Day(Co=&) _ 9f Ymax f UOP dé FtoD ) S14 (VW vy(Se/ Se) 5 QN-2 21B.1 Wall mass flux for turbulent flow with no chemical reactions. a. The Blasius formula for turbulent flow in a tube may be found in Eq. 6.2-12: 0.0791 Re¥# fe The mass-transfer analog of Eq. 13.3-7 is 1 [F resgel =~ 4Re Sh 752 e Sc When these are combined, we get Sh =0.0114Re”* Sc¥3 b. If on the other hand, we use the mass-transfer analog of Eq. 13.4-20, 1s th = ——,|4 ReSc¥? large Se’ s sal es ) then we get Sh =0.0160Re”® Sc! which is in better agreement with experimental data. Q2\-3 21B.2 Alternate expressions for the turbulent mass flux. Seek an asymptotic expression for the turbulent mass flux for long circular tubes and a boundary condition of constant wall mass flux. Assume net mass transfer across the wall is negligible. ) Parallel the approach to laminar flow heat transfer in §10.8 to write TE.) = GS +T1.()+C, and show that q=4 Here 11 =~(0, ~@0)/joD! pDyw); § =7/ Ds § = (z/D)/ReSe and the subscript “0” indicates conditions at the wall. This is given in Section 10.8 b) Put these results into the species continuity equation to obtain gent 4|(), Sew) ate Edé ScO pp )? dé This equation is to be integrated with the boundary conditions AtE=1/2 The=0 and dIT/dé ) Integrate once with respect to & to obtain: 1_ if». 4j eae bey, +a eae] Sc we Here Se =p / pDYy. a4 21B.3 An asymptotic expression for the turbulent mass flux. Start with Eq, 21B.3-1 in the form dam, _ jdt 1 “Dae oe “DL ScoF 14.5v)'] ‘Making the suggested substitution yields amt _at se'"y" dy dn 14.5v so that ad i4sv 1 dy Dv,Se!? 147° and 14.5v T1() -11(0) = (co) — 1(0) = ecard ‘The integral is equal to a3 =73 __%? 1299 sin(z/3) 3 3 Tt follows that Sh=————_ = 0.057Re Se"? f/2 Ti) 11) eSNG If the Blasius expression is used for the friction factor we obtain Sh =0.0108Re”"* Se!’? which is within about 5% of the above value. This is good agreement. 2-5 21B 4: Deposition of silver from a turbulent stream. (a) We may write immediately from the results of the previous problem that dit 14.5 1 dn ReSe!? ff /21+7° where 1 is defined as in the last example. In our situation. However, for our initial purposes it is most convenient to write 2.54cm11.Acm/s ‘DaqvelV) = Ref f/2 = (agri) a 0.0101em"/s = 2,863 ‘The corresponding Schmidt number is 952 as stated in Fig.21B.3(b). The concentration gradient is then defined by a _ 1 n_d_ dn 1942° 147) and we are now able to calculate the concentration gradient explicitly in terms of n, ‘We may now calculate the concentration gradient, and the results are shown in Fig. 21B.3(b). However, the abscissa is written in terms of s'=sv./v where s is distance measured into the fluid from the surface under consideration. This is done to facilitate comparison with velocity profiles. We can calculate the concentration profile by integration of the concentration gradient with respect to s*, but this result is not shown on the figure. It rises so rapidly to a limiting value T(co) ~ 1(0) = 1.209/1942 = 6.2310 that the initial slope appears infinite on the scale of the figure, and the ordinate is 0.000623 for all readable positions. 2Q\-b 2864-9.84 1.209+14.5 polarized, i.e. the silver ion concentration there is essentially. This is also the situation of 1606 Under these conditions the cathode is completely ‘maximum silver flux and the corresponding maximum, or limiting current. ‘The corresponding Sherwood Number is and the mass transfer coefficient for Ag’ is D, 1078 k, =| 2 |p = LOGO orm 66 6.76107 cm/s D 254 5 ‘The corresponding silver ion flux N ggr = 6.710? 1979 SLA _ 6.7.97? $= US J s cm cms 21B.5 Mixing-length expression for the velocity profile. (a) For steadily driven turbulent flow in a tube, Eq. 5.5-3 gives d,/_ \_ r(Po—Pr) Lot.) = in which 7 = 7”) + 7, Integration gives - 1 (Po-Pu) Tay tO in which the integration constant Cy is zero because the other terms vanish at r = 0. Hence, 5, = 1Po= Pu) ee oT and the wall shear stress is R(Po Pr) 2 L Combining the last two equations, we get T= i pened =n (1-4) for0 =1.3e10* /18.5 ; Re~108 We are now ready to estimate ass transfer rates, and we begin by anticipating the results of Prob. 22B.8 by assuming that essentially all mass transfer resistance occurs in the water phase, The best available correlation is then that Sly ¥ 2+ 0.6ReM? Sc!’ = kom D/ Day and it remains to determine magnitudes of key system properties. Here we shall assume oxygen solubility to be given by 60, =12.17-0.0507T +5,604+10*T?](p/1atm) 22-16 where concentration is mmols/L, and temperature is in degrees centigrade [regression of data in J. E. Bailey and D. F, Ollis , Biochemical Engineering Fundamentals, 2cnd Ed, McGraw-Hill, 1986 ~ - p. 463]. Then for our circumstances, the entering oxygen concentration is about Co, © 1.38mmols/ L = 1.3810 g — mols/ em? For oxygen-water diffusivity we shall use Doyy ¥ (1.04 +0.053T)+10° em? /s Uhhis is a regression of data at 37 C by E. E. Spaeth and S. K. Friedlander, Biophys. J 1967, p.827 and data at 25 C reported in T. K. Sherwood, et al, Mass transfer 1975, Wiley] It follows that Dow © 2.1610 em? /s Then the Schmidt number Sc = 0.0100137/[2.1610°] = 476 and 2.1107 cm? /s 0.lem [2+0.6-108"'-476" | ~0.0106em/s ‘The inward directly oxygen flux at the bubble surface is then Now|, _p = keACo, = 0.010106+1.38+10°* gmoles lom?,s 202-1] 22B.6 Controlling diffusional resistances No first of al because there is no diffusional resistance in pure oxygen. However, the same answer would be true even if the gas phase were any nitrogen-oxygen mixture. ‘The gas phase contribution is truly negligible, so it is only necessary to recognize that the gas phase diffusivity is in the range of 0.1< Doy <1.0cm*/s where G is “any” gas in a gas containing “M” and “G” for these conditions. To show this begin by writing No.0 = Ket (Crs ~ C10) = keg (Ceo - Ca») = Ket (Cis — Se) Now define 1 (60 - ae) =—(C10 ~ Cre) m to write fan. keg Noting that Sh= Dk, Diy we may write reg Pow Sh, Ae = 4 Oo Ku Dos Shy and to an order of magnitude approximation (Sh, | Shg) << (Do,4! Dog)" where “‘n” is 1/3 for the situation being considered here. ‘Note that for our conditions latm, g = mole 10° mmols,cm* 82.05cm*-293.16 g—mole,L so that mx 0.03 Cg = p/RT = =41.6mmols/L 22-18 22B.7 Determination of diffusivity The basic problem here is to calculate the quantity y. this may be shown as follows: For the actual situation, SbDyalt whereas in the absence of convection a 7 v0 Pal 7 The ratio of these two quantities is then just y. Determination of y is difficult, as stated in Ex. 20.1-1, but the inverse is straightforward. Examples in the supplement attached show that significant corrections are-needed even for mole fractions as low as 0.03. These calculations were made using the approximation of the error function erf(x)*1-(attal? +af)e" 1 =1/1+0.47047x win a, = 0.34802 a, = 0.09587 a, = 0.74785 (Abramowitz and Stegun, 7.1.25. Mole fractions can be related to temperature using Fig. 228.6. Now just looking at Table 20.1-1 we see that y begins to increase very rapidly with interfagial water vapor mole fraction, and this means that heat supply can become 9 problem, QA Now the oxygen-air diffusivity is to the approximations of Chap. 17 equall to oxygen-air diffusivity, and from Table 17.1-1 this is 0.181 cm*/s. From Eq, 17.2-1 Do,q = (0.18)(293.2/273.2)'? =0.205cm* /s We then find that k 2.440%)? = << 0.03} ——__]_ =6.6+10° Ka 2.05610 which is truly negligible, Similar remarks hold or almost all systems of sparingly soluble gases, 22-20 22B.8 Marangoni effects in condensation of vapors We begin by calculating film thickness in the absence of surface tension gradients 5 =k/h=(0.73/5,000)m =1.46610-4m From Eq.2.2-19 = gd" /3v=24.0em/s and the mass flow rate per unit width p=24.000,96 =23.1g/om,s ‘We may now parallel the development of Ex. 2.2 and write Te = PEt, but now the integration constant is to be evaluated from the new BC so that pexte, =—pdv, Ide Integrating this equation with respect to x, and using the BC vfp=0 295? pevon| 28% (143-2) 3h 2 ped The term in x, represents the effect of surface tension gradients, and where this term is, yields small its denominator will be about that for no gradient, Now for our circumstances p86 =143dyne/ cm? , Surface tension effects will thus be small for systems like ours where surface tension increases downwards. In the reverse case however, even smpal| gradients can cause hydrodynamic instabilities and thus have major effects, 29-21 22B.9 and 228.10 Film models for spheres and cylinders. For all three of our basic coordinate systems we may write &, dr where r represents distance, y for rectangular coordinates and r for cylindrical and spherical. A subscript “r” is understood for each of the molar fluxes, Itis necessary for this development that the left side of the equation become independent of r, so we rewrite it as r'[Ny ~x4 (Ny +Nz)] where n = 0 for slabs, 2 for spheres and | for cylinders. Now for all three rN, = R"N,(R) a constant, and R is any convenient value of r. We may then write RNa 4 (Nan + Noe) cDyag diny a SA RNR tN, R'[Nan+ Nop al (ae Nae) with boundary conditions N4-X4(N4+Npg)=—cDyy n aX, cD yr ra X4=X4q atr= Ry X4=X45 atr=R; Here the subscript “R” denotes evaluation at r=. Separation of variables and integration gives R'Nag—Xyo Na+ Now) _ cx a +N 4x) dr 7 = Ne R" [Nae ~X40(Na + Nand] Dp or (N40 + Noo) Bo cp E Day Re R) 29-2 Note now that R cancels out on the left side of this equation. We can this eliminate it and, just for convenience, replace the “R” subscripts by “0”, referring to conditions at Ro. Moreover we can add 1 +-1 to the left side in the form of Ngo ~X40(N 40 + No) Nao ~*40(N40 + Noo) ‘We thus obtain 14 Naot No No40 — Xs) Ngo ~X40(N 40 + N50) as required. Moreover, E can be expressed as E=exp[ (No + Noo)! ke.oe | to complete the development. Remember that we are using the mass transfer coefficient for negligible convection, so that 40(N40 + Ngo) << Nao To show this for spheres, start with a spherical shell and write For the above boundary conditions this yields £4- Xap B=R(,_%) X45 7 X40 Rs r R, 1 Nyo=[eDyp (Sats 10-15) = Faso) The cylindrical result is obtained similarly, and the heat transfer results can be obtained directly by analogy: 22-1 Ng NACo4 cDyz ak x,>T T10 > % Moreover, one can extend this result to variable properties by including the transport properties inside the integral Qa 24 22C.1 Caleulation of ultrafiltration rates The calculational technique is described in detail on pages 714 and 715, and the results are shown in Fig. 22.8-9. 2-25 Prob. 23.1 Expansion of a gas mixture: very slow reaction rate We first the gas composition from Eqs. 23.5-56 through 59: _ Gp Neo) _ eo? -( Xo ) "in co,) Ceo, (05-xeo, 0.5VK oI VK =10°5? =0.841; 0.5-0.841 Yeo =p =p gay 20228 Xp, =X co, =0.5—0.228 = 0.272 It follows that, Dx,M, = 0.228(28.01+18.02) +0.272(2.016 + 44.01) = 23.01 Lx,Cpy = 0.228(7.93 + 9.86) + 0.272(7.217 + 12.995) =9.554cal/ g— mol,’ K Substitution of this last result in Eq, 23.5-54 yields RiExt, Ty = T,(py! py) = 1000(1/1.5)"°"5 = 920K ‘Substitution of the above values in Eq. 23.5-55 , using Table F3.3, yields Saw |” x; ¥, =4F, -h Ex, =| 2(1000-920) 23.01 =2.78-10°cm/s =1726 ft/s 9.554-4.1840-107 | ‘The speed of sound can be determined from Eq. 9.4-4 as = 6.48-10° cm/s = 2126 ft/s ‘The flow is therefore subsonic, and the assumption that pr is equal to the ambient pressure is justified 23-2 23A.2 Height of a packed-tower absorber Begin by completing the mass balance as shown on the accompanying figure. ‘The cyclohexane free gas stream rate in Ib-moles/hr We, = (363, /min)(60min/ hr) Moses eal) -0.99 303 359% = 56.8/b — moles /hr and Yyq =0.01/0.99 = 0.0101 ‘The corresponding exit mole ratio is Yip, = 0.0101-0.1=0.00101 ‘The exit liquid mole ratio is now obtained from the macroscopic mass balance Xo = Xin —(-56.8/19.94)(0.00909) = 0.0289 More generally we may write Y =0.00101+ (19.94/56.8)(X — 0.003) =0.35LX¥ — 0.00043 This is the answer to part (a). We now begin with Raoult’s law in the form Pop 121 par Pu? 105-760 Yo ‘X= 0.1516x5 This result can be used along with the definition of the mole ratio to obtain an equilibrium expression in terms of mole ratio: Y= yMl-y)=ax/[l-ax]; x= X(1+X) where ot equals 0.1516. This expression is also drawn on the accompanying figure, and it may be seen to effectively straight, because all solutions are so dilute. This is the solution to part (b), We may now determine the interface conditions from the relation Yoto _ (22) =-0.0225 X-X, (142 ‘Since both the equilibrium and operating lines are linear to the accuracy of the technique wwe can simplify this expression to 23-3 Y= 0.15X,3; ¥=0.351X - 0.00043 ‘We thus find _ ¥+0.0225xX © 0.15 + 0.0225 % = 0.15X, ‘These results are tabulated on the attached spread sheet, and they include the answer to part (). We may next calculate the tower height from the expression z--Wo_h}_a (35); S(kya),¥-¥%) (2-142 Numerical integration gives about 62 ft, which is the answer to part (d). Use of the log mean approximation yields 0 _=%) YY n with — ¥,-¥; = 0.00909; (Y—¥),, = 0.0003 and a packed height of 60 ft. I-4+ x = 0.003 X= 0.003 Y = 0.00101 20(0.997) Ib mols/hr = 19.94mols/hr 1 X = 0.0289 y = 0.01 = 0.0101 363(0.99) cu ft/min = 56.8 mols/hr 23A.2: Mass balances 23-5 Gas mole ratio 0.012 0.01 0.008 0.006 0.004 0.002 } A Packed-tower Absorber Solution to 23A.2 | operating | line 0 equilibrium! line - 0.01 0.02 Liquid mole ratio 0.03 23-6 23A.2 Graphical constructions x ° 0.001 0.002 0.003 0.004 0.005 0.006 0.007 0.008 0.009 0.01 0.011 0.012 0.013 0.014 0.015 0.016 0.017 0.018 0.019 0.02 0.021 0.022 0.023 0.024 0.025 0.026 0.027 0.028 0.029 0.03 Y -0.00043 -T.9E-05 0.000272 0.000623 0.000974 0.001325 0.001676 0.002027 0.002378 0.002729 0.00308 0.003431 0.003782 0.004133 0.004484 0.004835 0.005186 0.005537 0.005888 0.006239 0.00859 0.006041 0.007292 0.007643 0.007994 0.008345 0.008696 0.009047 0.009308 0.009749 0.0101 x y 0 0.000999 0.001998 0.002991 0.003984 0.004975 0.005964 0.008951 0.007937 0.00892 0.009901 0.01088 0.011858 0.012833 0.013807 0.014778 0.015748 0.016716 0.017682 0.018646 0.019608 0.020568 0.021526 0.022483 0.023438 0.02439 0.025341 0.02629 0.027237 0.028183 0.029126 ve 0 0.001514 0.0003028 0.0004534 0.000604 0.0007542 0.000042 0.0010538 0.0012032 0.0013522 0.001501, 0.0016495 0.0017976 0.0019455 0.0020931 0.022404 0.0023874 0.0025341 0.0026806 0.0028267 0.0029725 0.031181 0.0032634 0.0034084 0.0035531 0.0036976 0.0038417 0.0039856 0.0041292 0.0042725 0.0044155 0 0.000151 0.000303 0.000454 0.000604 0.000755 0.000905 0.001055 0.001205 0.001354 0.001503 0.001652 0.001801 0.001949 0.002097 0.002245 0.002303 0.002541 0.002688 0.002835 0.002981 0.003128 0.003274 0.00342 0.003566 0.003711 0.003857 0.004002 0.004146 0.004291 0.004435 xi0 0 0 0 0.004003 0.006168 0.008333 0.010499 0.012664 0.014829 0.016994 0.019159 0.021325 0.02349 0.025655 0.02782 0.029986 0.032151 0.034316 0.036481 0.038646 0.040812 0.042977 0.045142 0.047307 0.049472 0.051638 0.053803 0.055968 0.058133 0.060209 0.082464 Yio 0.0006 0.000925 0.00125 0.001575 0.0019 0.002224 0.002549 0.002874 0.003199 0.003523 0.003848 0.004173 0.004498 0.004823 0.005147 0.005472 0.005797 0.006122 0.006447 0.008771, 0.007096 0.007421 0.007746 0.00807 0.008395 0.00872 0.009045 0.00937 23-7 23B.1: Effective average driving forces in a gas absorber. This problem has really been solved in the above discussion of Prob. 23.2, but perhaps a more detailed analysis is in order. (a) We therefore begin by writing Y=mX +b, Y=mX+b, It follows that ¥,-Y=(m,-m,)X +(b,-5,) which is the required linear relation. It follows directly that, Y-¥%=GY+C, () We now write directly from Eq, 23.5-21 Y-Y=m(X- Xp) and K=mX,+6, It follows that y-yam [toh 4] mm, which is the required linear relation between Y and Yo. It follows that there is also a linear relation between (Y-Yo) and Y so that Y-%H=GY+C, where C; and C; are constants, obtainable from the above relations. (c) We now begin with the relation zu j ay S(k,a)i, GY +C, For C; non-zero and a finite denominator A We_y[Gh+C, G Sha) LOK +C, 23-8 CY, +C, = (¥ Yodo Now OK += -%), ¢, -W-%a- Oh = (=X) and Z= Woh) [2] 50,4) Y-%).-Y-K |W-%), We_W~%) S(k,a) (Y—Yo)in This result also holds for the degenerate of C3 equal to zero where (Y-Yo)a becomes or ‘equal to the driving force at any position in the column. The corresponding expression in terms of (K,a) follows by analogy. 3-9 23B.2 Expansion of a gas mixture: very fast reaction rate. Begin by computing the temperature distribution of as follows: 900K 950K 1,000K VK 0.676 0.754 0.841 Xz, =Xco, = 0.5/ VK 0.298 0.285 0.272 Xco =X = 9-5~ Xp, 0.202 0215 0.228 Ay, = 6340 + 7.217(T — 900) CD 6701 7062 Aco, =-83,242 +12.995(T —900) | #3242 | -82.592.— | -81,942 Aco =—16,636 + 7.932(T — 900) | “16:36 | -16,239 | 15,843 Ayo = ~49,378+9.861(T -900) | 49378 | 48,885 | -48,392 Hf =33x,H,,cal/ g—mole 36.252 “35,631 -35,013 AH/AT 12.42 12:36 The proposed value of 12.4 is clearly adequate. This is the answer to part (a). (b) From Eq, 23.5-53, which holds for constant mean molecular weight we get in| 2 )_1987,, i) =-0.065 7) 1240 \is so that T, =1,000-e°°* = 937K (©) For consistency with (b) we do not interpolate for AZ. Rather we set Ai, — H, ~12.4(937 -1,000) = -781cal / g - mole 3-10

You might also like